Sunteți pe pagina 1din 331

37

100. A 30-year-old was involved in a rapid deceleration motor vehicle crash at street speed. He was
gripping the steering wheel tightly, and complains of pain at the MCP joint of the right thumb. On
exam there is tenderness and mild swelling of the joint on the ulnar side. X-rays are negative.
Which of the following is correct regarding the evaluation and treatment of this injury?
a. If the ulnar collateral ligament demonstrates more than 35o of laxity when compared to the
uninjured side, place a thumb spica splint and refer to orthopedics for probable surgical
management.
b. If the radial collateral ligament demonstrates any laxity, refer to orthopedics for surgical repair.
c. All gamekeepers thumbs need to be repaired surgically.
d. The absence of a fracture indicates that no splinting will be necessary.
e. If the ulnar collateral ligament demonstrates less than 20o laxity, apply a universal wrist splint,
and refer for surgical repair.

101. This x-ray shows a:


a. dorsal dislocation of the lunate.
b. volar dislocation of the lunate.
c. dorsal perilunate dislocation.
d. volar perilunate dislocation. Rosen 5th,
e. scapholunate dissociation. Figure 44-11

102. Elbow dislocations are associated most commonly with concomitant injuries to the:
a. ulnar nerve and radial artery.
b. ulnar nerve and brachial artery.
c. radial nerve and ulnar artery.
d. radial nerve and brachial artery.
e. median nerve and radial artery.

103. This child fell while rollerblading and has a tender


elbow. This x-ray (figure 169-15) is most
consistent with:
a. nursemaids elbow.
b. radial head fracture.
c. proximal ulnar fracture. Rosens 5th,
d. elbow dislocation. Figure 169-15
e. supracondylar fracture

104. Monteggias injury is a :


a. fracture of the ulnar shaft with radial head dislocation.
b. fracture of the ulnar shaft with distal radioulnar dislocation.
c. fracture of both the radius and ulna.
d. distal radial fracture with radioulnar dislocation.
e. distal radial fracture with radial head dislocation

105. This x-ray (Figure 44-24) demonstrates a:


a. Colles fracture.
b. Monteggia fracture.
c. Galleazzi fracture.
d. Bennett fracture. Rosen 5th,
e. Barton fracture. Figure 44-24
38

100. a Injury to the ulnar collateral ligament (UCL) was first described as an occupational
th
Rosen 5 , hazard of Scottish gamekeepers who damaged their thumbs by repeatedly twisting the
Chapter 43, necks of hares. Skiing is now the most common cause of both acute and chronic injury
p. 520 to the UCL. UCL rupture occurs ten times more often than radial collateral ligament
injury. The mechanism of injury is forced radial deviation (abduction), and the
subsequent tear usually occurs at the insertion into the proximal phalanx. Complete
and partial ruptures can usually be differentiated by clinical examination. Valgus
stress testing of the ulnar collateral ligaments is required and should be performed in
full extension and in 30% flexion to avoid the stabilizing effect of the volar plate.
More than 35% of joint laxity or 15% more laxity than is present in the uninjured
thumb is consistent with complete UCL rupture. Acute partial ruptures of the UCL can
be effectively treated by a 4-week period of immobilization in a thumb spica cast, and
full recovery is the rule. Complete ligament tear requires surgical repair because a
high number have associated soft tissue interposition from the adductor aponeurosis
(Steners lesion) with limited predicted healing potential.
101. c A perilunate dislocation is best seen on the lateral view of the wrist. Although the
Rosen 5th, lunate remains in position relative to the distal radius, the capitate is dorsally
Chapter 44, dislocated. The PA view shows overlap of the distal and proximal carpal rows and
p. 541 may also demonstrate an associated scaphoid fracture or subluxation.
102. b In patients with elbow dislocations, neurovascular complications occur in 8 to 21%, the
Rosen 5th, most frequent being injury to the ulnar nerve. Vascular complications occur in 5 to
Chapter 45, 13% of elbow dislocations, with brachial artery injury the most common.
p. 572
103. e Supracondylar injuries are extra-articular fractures which occur most commonly in
th
Rosen 5 , children; 95% are displaced posteriorly as a result of an extension force. The patient
Chapter 45, will have significant swelling and tenderness at the elbow. Radiographs may reveal a
p. 564 fat-pad sign in undisplaced fractures due to visualization of fat from the olecranon
fossa (posterior fat pad) as it is displaced by the hemarthrosis. In some undisplaced
fractures, the fracture line may not be seen, with the fat-pad sign being the only
evidence of injury. Treatment should be initiated as though a fracture were identified,
with splint immobilization and orthopedic consultation.
104. a Fracture of the ulnar shaft with a radial head dislocation is often referred to as
Rosen 5th, Monteggia fracture-dislocation. It is typically a diaphyseal fracture in the proximal
Chapter 44, third of the ulna with an anterior dislocation of the radial head (60% of cases).
p. 551 Anterolateral and posterolateral dislocation of the radial head or a metaphyseal ulna
fracture are other possibilities. As a rule, the radial head normally points to the
capitellum in all radiographic views of the elbow. In a Monteggia fracture, the apex of
the ulna fracture points in the direction of the radial head dislocation.
105. c A Galeazzi fracture involves the junction of the middle and distal thirds of the radius,
Rosen 5th, with an associated dislocation or subluxation of the distal radial-ulnar joint (DRUJ).
Chapter 44, This rare injury accounts for only 7% of all fractures of the forearm. It occurs as a
p. 552 result of a fall on the outstretched hand, with the wrist in extension and the forearm
forcibly pronated. The radial fracture causes obvious swelling and deformity on the
radial side of the forearm, and pain is localized over the fracture site. In addition, the
DRUJ is swollen and painful on palpation, and the ulnar head appears prominent when
compared with the unaffected wrist.
39

106. A 16 year old boy complains of pain and swelling in his right hand after hitting a wall with a closed
fist. You know that:
a. the neck of the metacarpal is the most frequently fractured segment.
b. metacarpal fractures of the long and index finger do not need to be reduced if there is less than
30o of angulation.
c. metacarpal fractures of the ring and small fingers do not need to be reduced if there is less than
20o of angulation.
d. most metacarpal fractures angle in a volar direction.
e. a boxers fracture refers to metacarpal neck fractures of the long and sometimes ring fingers.

107. A 5 year old boy falls and fractures his radius.


a. Salter-Harris Type I fractures are the most common type
b. The picture shows a Salter-Harris Type II fracture.
c. Salter-Harris Type III fractures involve the epiphyseal plate and
have a widened space between epiphysis and metaphysis.
d. Salter-Harris Type IV fractures are difficult to diagnose using
standard radiographs.
e. Salter-Harris Type I, II, and III injuries have poor prognosis despite
proper reduction and immobilization.

108. A 29 year-old man is tackled while playing football and suffers what appears to be a shoulder
dislocation. You know that:
a. the most common direction of dislocation is posterior.
b. anterior-posterior (AP) and lateral radiographs are the best views to confirm the diagnosis.
c. after reduction he should be placed in a shoulder immobilizer for 2 months.
d. infraglenoid dislocations are associated with seizures.
e. inadequate muscle relaxation is the most common reason for failed reduction.

109. A 43-year-old softball player pulled up lame stretching a single into a double. You suspect a
ruptured Achilles tendon, since you know:
a. when the soleus and gastrocnemius muscles contract, the Achilles tendon pulls up the calcaneus,
dorsiflexing the foot.
b. rupture often occurs in professional sports settings, especially in over-conditioned athletes.
c. the patient hears a popping sound and then has difficulty walking.
d. the Thompson-Doherty test is performed by squeezing the midportion of the calf while the patient
is lying in the prone position; an intact Achilles tendon is demonstrated by dorsiflexion of the
foot.
e. unbearable pain is a cardinal symptom.

110. A 23-year-old hairdresser complains of two days of increasing pain and swelling in her right index
finger pad. She has no fever. Her finger pad is tense, red, warm, and very tender. Her nail and
paronychial region are not affected. You should:
a. place her on amoxicillin / clavulanate (Augmentin), elevation, and warm soaks.
b. perform a through-and-through incision of the pad volar to the neurovascular bundle, then
amoxicillin / clavulanate (Augmentin), elevation, and warm soaks.
c. perform a fish mouth incision and refer to a hand surgeon.
d. perform a unilateral longitudinal incision on the ulnar side dorsal to the neurovascular bundle,
amoxicillin / clavulanate (Augmentin), elevation, warm soaks, pain medicine, recheck in two
days.
e. remove the nail, incise the eponychium, hot soaks, pain meds, and recheck in two days.
40

106. d Fractures of the metacarpal neck are among the most common fractures in the hand.
th
Rosen 5 , They are usually due to a direct impaction force. A fracture of the fifth metacarpal
Chapter 43, neck is often referred to as a boxers fracture. These fractures are usually unstable
p. 513 with volar angulation. Angulation of less than 20 in the fourth and 40 in the fifth
metacarpal will not result in functional impairment. In the second and third
metacarpal, angulation of <15 is acceptable. These fractures should be splinted with
the wrist in 20 extension and the MP flexed at 90. Fractures of the second or third
metacarpal that are significantly displaced or angulated require anatomic reduction and
surgical fixation.
107. c
Rosen 5th, Rosen 5th, I II III IV V
Chapter 169, Table 169-1
p. 2373

108. e Dislocation of the glenohumeral joint is the most common major joint dislocation.
th
Rosen 5 , Anterior dislocations are by far the most common. Although the anteroposterior
Chapter 46, radiograph will reveal the dislocation, the scapular Y radiograph will indicate the
p. 592-594 direction of dislocation: anterior or posterior. Since severe pain and muscle spasms are
the norm, muscle relaxation and analgesia are paramount. Posterior glenohumeral
dislocations may occur during a fall or from violent muscle contraction from a seizure
or electric shock. In uncomplicated cases the shoulder is immobilized for 3 to 6 weeks
in younger patients and 1 to 2 weeks in older (over age 40 years) individuals.
109. c Achilles tendon rupture usually occurs in middle-age persons during sporadic or
Rosen 5th, intermittent involvement in recreational sports. Injury mechanism is sudden
Chapter 51 unexpected dorsiflexion, forced dorsiflexion of a plantarflexed foot, and strong push-
p.716 off of the foot with simultaneous knee extension and calf contraction (e.g., a runner
accelerating from the starting position). The diagnosis is primarily clinical. Patients
usually describe a sudden onset of pain at the back of the ankle associated with an
audible pop or snap. Although the pain may resolve rapidly, weakness in
plantarflexion persists. The classic maneuver to assess the integrity of the Achilles
tendon is the Thompson test. This is performed with the patient prone and the knee
flexed at 90o. Alternatively, the patient may kneel on a chair with both knees flexed at
90o and the feet dangling over the edge. Squeezing the calf muscles in these two
positions should cause passive plantarflexion of the foot. Absence of this motion or a
weakened response compared with the uninjured side suggests complete rupture.
110. d A felon is a subcutaneous pyogenic infection of the pulp space of the distal finger or
Rosen 5th, thumb. The septa of the finger pad produce multiple individual compartments and
Chapter 43, confine the infection under pressure. The patient presents with throbbing pain and a
p. 529-530 red, tense distal pulp space. Staphylococcus aureus is the most common organism. If
the finger pad is swollen and tense, or if there is any palpable fluctuance, drainage
must be undertaken. A unilateral longitudinal approach spares the sensate volar pad
and achieves adequate drainage. Introduce a #11 blade lateral to the paronychium and
direct it in a volar direction until you find pus. Extend the incision ensure adequate
drainage, but not to the distal interphalangeal (DIP) flexor crease.
41

111. Which of the following is true of this injury? Rosen 5th,


a. This is a common wrist fracture. Figure 44-10
b. The lunate requires immediate reduction.
c. The distal radial-ulnar ligament is disrupted and will
require surgical repair.
d. The clenched fist AP view may accentuate this finding.
e. None of the above this is a normal x-ray.

112. A 38-year-old man developed severe lower back pain that shoots down his right leg as he was
lifting a large box at work. He has no previous history of back problems. He has no bowel or
bladder symptoms, and denies genital numbness. On exam he has no back deformity or palpable
pain, but when he raises his leg off of the bed, a severe pain shoots down his leg to his great toe.
He has some numbness of the lower leg and web space next to the great toe. He has 4/5 strength
with dorsiflexion of the foot. His deep tendon reflexes are normal. He most likely has:
a. herniated nucleus pulposus (HNP) of L1-L2.
b. HNP of L3-L4.
c. HNP of L5-S1.
d. severe lumbar strain.
e. drug seeking behavior, since the findings are non-anatomic.

113. A 34-year-old man is ejected from a car. He has no head


injury, but complains of severe pain in his pelvis. His Rosen 5th, Figure 48-6
airway and breathing are fine, but he is hypotensive BP
80/40 mm Hg, heart rate 120. His workup shows multiple
abrasions and pelvic pain. His pelvis x-ray is shown. His
abdominal CT shows no intra-abdominal injury. After 4
liters of IV fluid, his blood pressure is 95/60. Of the choices
given below, your most appropriate would be to:
a. re-examine by compression and distraction of pelvic
wings.
b. transfuse fresh frozen plasma to control bleeding.
c. obtain stat vascular consult for intraoperative exploration and cauterization of vessels.
d. apply pneumatic antishock garment (PASG) to act as splint and compressive dressing.
e. arrange for hanging traction with at least 20 pounds of weights.

114. A 22-year-old man was in a head-on motor vehicle crash. His right leg is in adduction, flexion, and
internal rotation. Which is the correct statement about this injury?
a. This is likely a femoral neck fracture, so x-rays and pain relief should be given.
b. After sufficient analgesia and sedation, gentle flexion to 90o with upward traction should be
performed as soon as possible.
c. The most likely associated injury is to the femoral nerve.
d. The risk of associated acetabular fracture is minimal.
e. The risk of avascular necrosis is >50%.

115. Ankle sprains:


a. are common in young children and toddlers.
b. most commonly involve the anterior talofibular ligament.
c. most commonly occur on the medial side of the ankle.
d. require radiography to diagnosis a third-degree injury.
e. involving isolated deltoid ligament injuries are very common.
42

111. e A stage I injury, or scapholunate dissociation, results in a characteristic widening of the


th
Rosen 5 , scapholunate joint on the PA view, which has been called the Terry-Thomas sign, after
Chapter 44, the British comedian with a gap between his front teeth. When a scapholunate
p. 538 ligament injury is suspected clinically, additional stress views should be obtained.
Views taken with a clenched fist and ulnar deviation (the clenched fist AP view) will
accentuate widening of the scapholunate joint.
112. c The most important test for back pain is the straight leg raise. This is done with the
Rosen 5th, patient supine and the legs extended, or with both legs slightly flexed at the hips and
Chapter 47, knees. The symptomatic leg is passively raised, with the knee fully extended. Pain
p. 617 caused by a disk herniation in L4-L5-S1 usually results in increased pain in the leg at
anywhere from 30 to 70o of elevation, as the affected nerve root is stretched. It is
important to note that back pain, probably secondary to stretching of the paravertebral
muscles as the leg is elevated, indicates absolutely nothing about radiculopathy. A
straight leg raise test is positive only if lifting the affected leg reproduces or worsens
the pain in that leg. The pain of radiculopathy is usually worse in the leg than in the
back and almost always radiates past the knee. Ninety-five percent of disk herniations
occur at L4L5 or L5S1. Without complaint of sciatica, or pain below the knee in a
dermatomal distribution, the chance of a herniated nucleus pulposus is 1 in 1000.
113. d When attempts to stabilize the patient hemodynamically with crystalloid infusion and
Rosen 5th, transfusion of blood products are unsuccessful, application of a pneumatic anti-shock
Chapter 48, garment should be discussed with orthopedic and general surgical consultants.
p. 640 Although controversial, PASG can temporarily splint the fracture and tamponade
pelvic bleeding until definitive stabilization and angiography with embolization can be
performed..
114. b The Allis maneuver (Figure 49-27) should be attempted in order to reduce a [posterior
Rosen 5th, hip dislocation as quickly as possible. The patient is placed supine the knee is
Chapter 49, flexed and steady traction is applied in line with the deformity the hip is brought
pages listed slowly to 90o of flexion after reduction, the hip is extended while maintaining
in answer traction (p. 664). A patient with a femoral neck fracture presents with leg externally
rotated and shortened. A patient with a posterior hip dislocation typically holds the hip
flexed, adducted, and internally rotated (p. 662). Sciatic palsies, especially the
peroneal nerve branch, occur in 10% of patients with posterior hip dislocation (p. 662).
Acetabular fractures are common (p. 662). Avascular necrosis is reported to occur in
1% to 17% of patients with this injury (p. 665).
115. b Most ankle sprains occur from extreme inversion and plantarflexion. Approximately
Rosen 5th, two thirds of ankle sprains are isolated anterior talofibular ligament injuries, whereas
Chapter 51 20% involve both anterior talofibular and calcaneofibular ligament injuries. Isolated
p.714 injury of the deltoid ligament occurs in fewer than 5% of ankle sprains. A grade III
sprain involves a complete tear of the ligament with marked joint instability and severe
edema and ecchymosis, and is a clinical diagnosis.
43

116. A 75-year-old woman fell at a local flower shop and complains of


severe left hip pain. Her left leg is shortened and externally
rotated. She has strong pedal pulses and sensation is intact.
Thorough head-to-toe examination reveals no other injuries. Her
left hip x-ray is shown (Figure 49-17), so you arrange for the
patients admission to the orthopedic doctor on call. The patients
daughter, a pediatrician, wants to know about potential
complications from this injury. You tell her that, just as in
children, the most common complication is:
a. avascular necrosis.
b. hemorrhage. Rosen 5th,
c. osteomyelitis. Figure 49-17
d. fat embolism.
e. femoral nerve injury.

117. A 24 year-old basketball player severely twisted his left knee several hours ago and is unable to
bear weight. There is a large joint effusion. While examining his knee, you see that valgus stress at
30o flexion causes more than a centimeter of give without a firm endpoint. The knee is stable in
extension with valgus stress. This suggests:
a. an isolated medial collateral ligament disruption.
b. an isolated lateral collateral ligament disruption.
c. a medial collateral ligament tear with probable involvement of the cruciates and posterior capsule.
d. a lateral collateral ligament rupture with posterior cruciate tear.
e. probable anterior cruciate rupture with meniscal tear.

118. A 32-year-old man swung at a pitch while playing softball, then


collapsed due to severe knee pain. His injured leg looks like this. Your
treatment should be:
a. immobilization, x-ray, pain medicine, and examination for probable
peroneal nerve injury.
b. needle arthrocentesis of the prepatellar bursa, then ice and elevation.
c. orthopedic consult for repair of the patellar tendon.
d. gentle lateral pressure on the patella with the knee in extension.
e. examine for possible popliteal artery disruption.

119. A 35-year-old male skier fell and twisted his knee. Within five minutes, he developed severe pain
and a large joint effusion. When testing his knee stability, you know that:
a. the anterior drawer sign is highly accurate in diagnosing acute ACL injuries.
b. abduction, flexion, and internal rotation of the femur on the tibia produce injuries to the lateral
side of the knee.
c. adduction, flexion, and external rotation of the femur on the tibia produce injuries to the medial
side of the knee.
d. lateral knee injuries are far more common than medial knee injuries.
e. a negative Lachmans test rules out ACL injury with >90% accuracy.

120. Metabolic acidosis without an anion gap is caused by:


a. cyanide poisoning.
b. renal tubular acidosis.
c. lactic acidosis.
d. ketoacidosis.
e. salicylate ingestion.
44

116. a In a patient with femoral neck fracture, avascular necrosis is the most common
th
Rosen 5 , complication, despite optimal treatment, because of the complex arterial anatomy. The
Chapter 49, mortality rate during the first year after a femoral neck fracture is 14%, compared with
p. 656 9% for the control population. Deep infection, in the form of osteomyelitis or septic
arthritis, is more common with femoral neck fractures because the fracture line extends
into the joint. Pulmonary embolism is the leading cause of death at 7 days post-
fracture in all orthopedic patients.
117. a Abduction or valgus pressure applied to a knee in approximately 30 of flexion
Rosen 5th, determines the integrity of the medial capsular and ligamentous structures. If there is a
Chapter 50, demonstrated laxity of greater than 1 cm without a firm end-point as compared to the
p. 678 other knee, there is a complete rupture of the medial collateral ligament. If there is
laxity with a firm end-point or a laxity of less than 1 cm, an incomplete or partial tear
is present. If there is no demonstrated instability but there is pain, the patient has
suffered a strain. The patient who is unstable with the valgus test performed with 30
of flexion should be brought into full extension, if possible, and similar maneuvers
carried out. Medial instability in full extension indicates a severe lesion involving the
cruciate ligaments and posterior capsule along with the medial ligaments.
118. d Patellar dislocation may occur from an athletic injury caused by valgus stress
Rosen 5th, combined with flexion and external rotation. Patients may complain of the knee giving
Chapter 50 out accompanied by pain and swelling. Inability to bear weight or to flex the knee is a
p.688 common complaint. Examination shows an anterior defect with the patella deviated
laterally. Closed reduction is accomplished through force or pressure directed
anteromedially on the lateral patellar margin while simultaneously gently extending the
leg.
119. e A tear in the anterior cruciate ligament or intra-articular fracture usually results in an
Rosen 5th, immediate hemarthrosis and inability to continue activity. A torn meniscus may cause
Chapter 50 an acutely locked knee but more commonly has delayed onset of swelling over 12 to
p.677 24 hours and intermittent locking associated with joint line pain. Lachmans test is
currently the single best clinical test for determining the integrity of the ACL and one
of the only reliably performed tests in a patient with an acute hemarthrosis. Accuracy
in diagnosing ACL injury increases from 70% to 99% using Lachmans test rather than
the anterior drawer test. The anterior drawer test is not reliable and is of little value in
diagnosing acute ACL injuries.
120. b Elevated Anion Gap Normal Anion Gap
th
Rosen 5 , (CAT MUD PILES) Gastrointestinal loss of HCO3
Chapter 118 Carbon monoxide / Cyanide exposure Diarrhea
p.1719 Acute alcohol intoxication / Alcoholic ketoacidosis Enterostomy
Toluene exposure Ureteroenterostomy
Methanol intoxication Renal loss of HCO3
Uremia Renal tubular acidosis
Diabetic ketoacidosis Acetazolamide
Paraldehyde ingestion Hyperalimentation
Isoniazid (INH) / Iron intoxication
Lactic acidosis
Ethylene glycol intoxication
Salicylate intoxication
45

121. Which statement is correct concerning the entity shown in this x-ray?
a. This is a common foot injury, usually caused by inversion.
b. Associated fractures are quite common, and usually involve the base
of the 2nd metatarsal.
c. This is a pseudo-pseudo-Jones fracture.
d. Rest, ice, elevation, and ambulation as tolerated give the best long-
term results.
e. You can easily reduce this injury with gentle axial traction and plantar
flexion. Rosen 5th, Figure 51-24

122. A 35-year-old man accidentally chopped off his left index finger at the PIP joint about six hours
ago while cutting firewood at a campsite in a local state park. He stuck the amputated finger in his
pocket, hiked out of the woods, and drove to your Emergency Department. You should:
a. clean the amputated finger with Betadine and place it in the refrigerator.
b. clean the amputated finger with hydrogen peroxide, wrapped in saline gauze, and place it on ice.
c. irrigate the amputated finger with saline, wrap it in sterile dry gauze and place it in a plastic bag,
which should be immersed in ice.
d. Irrigate the amputated finger with saline, wrap it in saline-moistened gauze and place it in a
plastic bag, which should be immersed in ice-water slurry.
e. There is no need to care for the amputated extremity as the ischemic time is too long and no
reimplantation will be possible

123. A 20-year-old college student has redness, tenderness, and swelling on the plantar surface of his
foot. Two days ago, while wearing sneakers, he stepped on a nail. You should prescribe:
a. cephalexin.
b. amoxicillin/clavulanate.
c. ciprofloxacin.
d. tetracycline.
e. erythromycin.

124. Antibiotics are clearly indicated for:


a. all dog bites.
b. all cat bites.
c. all human bites.
d. all foot puncture wounds.
e. all intraoral lacerations.

125. The acute life threats in adrenal inufficiency are:


a. hypertension and hyperglycemia.
b. hyperthermia and hypoglycemia.
c. hypothermia and hyperglycemia.
d. hypotension and hypoglycemia.
e. hyperthermia and hypertension.

126. A patient being treated for diabetic ketoacidosis develops weakness and paresthesias. The most
likely metabolic abnormality is:
a. acidosis.
b. hypokalemia.
c. hypophosphatemia.
d. hypocalcemia.
e. hypoglycemia.
46

121. b The six-bone tarsometatarsal complex is known as the Lisfranc joint. Up to 20% of
th
Rosen 5 , fractures at this joint are missed in the ED. The force required and the mechanisms of
Chapter 51, injury are varied and can range from a seemingly minor rotational force to severe axial
p. 726 load as seen in an automobile accident. The great majority of injuries to the Lisfranc
joint are associated with fractures, usually of the metatarsals, the cuboid, or the
cuneiforms. A fracture of the base of the second metatarsal is pathognomonic of a
disruption of the ligamentous complex. Injuries to the Lisfranc joint frequently require
open reduction and fixation or percutaneous placement of Kirschner wires and non-
weight-bearing for several weeks.
122. d The time that an amputated part can survive before reimplantation has not been
Rosen 5th, determined. As a general rule, the more proximal the amputation, the less ischemic
Chapter 43, time the amputated part can tolerate. Warm ischemia may be tolerated for 6 to 8 hours,
p. 527 but there are reports of successful replantation of digits after 30 hours of warm
ischemia. The amputated part requires minimal handling and should be cooled as soon
as possible. After wrapping in saline-moistened gauze, the part is sealed in a dry
plastic bag and placed in ice water. Ice should not come in direct contact with the
tissue because this can cause local damage. Local antiseptics, especially hydrogen
peroxide or alcohol, should not be used because they may damage viable tissues.
123. c The infection rate for foot puncture wounds has been reported to be as high as 15%.
Rosen 5th, Most infections are due to gram-positive organisms, with Staphylococcus aureus
Chapter 52, predominating, followed by other staphylococcal and streptococcal species.
p. 748 Pseudomonas aeruginosa is the most frequent pathogen isolated post-puncture wound
osteomyelitis, particularly when foreign-body penetration occurs through the sole of an
athletic shoe. Because this organism is not detected in new shoes, it has been
postulated that the foam rubber material becomes colonized in the warm, humid
summer months. The only oral agents consistently effective against Pseudomonas are
the fluoroquinolones, including ciprofloxacin and levofloxacin.
124. b Antibiotic prophylaxis is required for all patients with cat bites. Antibiotic prophylaxis
Rosen 5th, for dog bites is more controversial, and seven of eight randomized trails of dog bite
Chapter 52, wounds show no benefits with antibiotics. While human bites of the hand are at a high
p. 748 risk for infection, human bites of other locations appear to have no worse infection rate
than other wounds. No data suggest a benefit from prophylactic antibiotics in foot
puncture wounds. Little data suggest a clear indication for prophylactic antibiotics in
patients with intraoral lacerations, but through-and-through wounds have an infection
rate twice as high as simple mucosal injuries.
125. e The acute life threats in adrenal insufficiency are hypotension and hypoglycemia.
Rosen 5th, Hypotension responds well to glucocorticoid replacement with IV hydration, and
Chapter 122, hypoglycemia to IV administration of D5W.
p. 1781
126. b Potassium replacement is invariably needed in DKA. The initial potassium level is
th
Rosen 5 , often normal or high despite a large deficit because of severe acidosis. Potassium
Chapter 120, levels often plummet with correction of acidosis and administration of insulin.
p. 1753-1754 Potassium should be administered with the fluids while the laboratory value is in the
upper half of the normal range.
47

127. According to his father, this 11-year-old boy has complained about a vague hip,
thigh, and knee pain for the past 2 weeks. He recalls no injury. There has been
no fever. The child walks with a limp and winces when his right foot hits the
ground. His x-ray demonstrates:
a. osteomalacia femoralis.
b. Osgood-Schlatter disease.
c. Legg-Calv-Perthes disease. Rosen 5th,
d. septic synovitis. Figure 49-34
e. slipped capital femoral epiphysis.

128. In a patient with primary adrenal insufficiency you would expect to see:
a. hypernatremia.
b. hypokalemia.
c. hypoglycemia.
d. azotemia.
e. acidosis.

129. The most common cause of a markedly elevated serum potassium is:
a. laboratory error.
b. renal failure.
c. Addisonian crisis.
d. rhabdomyolysis.
e. acidosis.

130. Hirschsprung's disease:


a. is typically diagnosed in infancy.
b. almost always is associated with enterocolitis.
c. is more frequent in females.
d. can be diagnosed by ultrasound.
e. can best be managed pharmacologically.

131. You deliver a child at 36-weeks gestation and intestines are protruding from a defect in the
abdominal wall. On inspection, you note that the intestines lack a peritoneal covering.
a. A gastroschisis is a defect in the umbilical ring that allows the intestines to protrude out of the
abdominal cavity in a sac.
b. An omphalocele is a defect in the abdominal wall that allows the evisceration of abdominal
structures without a sac being present.
c. Gastroschisis and omphalocele are invariably fatal within hours.
d. You are twice as likely to encounter gastroschisis you are to see omphalocele.
e. Emergency department treatment gastroschisis involves gastric decompression; this same
treatment is contraindicated in omphalocele.

132. A 13-year-old male complains of acute, severe pain and swelling in his left testicle. He says that
the pain woke him from sleep. He had something silimar a week or two ago, but it got better
spontaneously. Suspecting torsion testicle, you know that:
a. absence of the cremasteric reflex is not a useful physical exam finding.
b. if this patient's symptoms have only been present for 1 hour, Doppler imaging of the testicles can
be obtained before obtaining surgical consultation.
c. relief of this pain with elevation of the scrotum indicates that epididymitis is the diagnosis.
d. torsion of the right testicle is more common due to a longer spermatic cord.
e. testicular pain may be constant or intermittent, but it is not positional.
48

127. e Children with a stable slipped capital femoral epiphysis have symptoms of intermittent
th
Rosen 5 , limp and pain for several weeks to months. Stable slips make up approximately 90%
Chapter 169 of all cases. The pain may be localized to the hip but more commonly is poorly
p.2387 localized to the thigh, groin, or knee. On AP radiographs, signs of slippage include
Kleins line drawn along the superior margin of the femoral neck. With a normal hip,
the line intersects or falls within the epiphysis, whereas in a hip with a slipped
epiphysis, the line does not come in contact with the epiphysis.
128. c Two thirds of patients with adrenal failure have associated hypoglycemia. The
Rosen 5th, symptoms are characteristic of hypoglycemia: perspiration, tachycardia, weakness,
Chapter 122 nausea, vomiting, headache, convulsions, and coma. The pathophysiology is decreased
p.1781 gluconeogenesis and increased peripheral glucose use secondary to lipolysis.
Electrolyte abnormalities are common. Hyponatremia is present in 88% of cases,
hyperkalemia in 64%, either hyponatremia or hyperkalemia in 92%, and hypercalcemia
in 6% to 33%.
129. a When faced with a report of a high serum potassium level, the Emergency Physician
Rosen 5th, should first consider the possibility of laboratory error, the most common cause of
Chapter 119, hyperkalemia. Hemolysis during phlebotomy, as may occur when blood is obtained
p. 1730 with a small needle or sampled in a high-vacuum tube, releases potassium into the
sample and causes a spuriously high potassium level to be measured.
130. a Hirschsprungs disease accounts for 20% of partial intestinal obstruction early in
Rosen 5th, infancy. It is 4 to 5 times more common in boys and can be associated with Down
Chapter 165, syndrome or a variety of other anomalies of the GI, GU, or neurologic systems.
p. 2305-2306 Neonates often present in the nursery with a failure to pass meconium. Diagnosis is
suspected on barium enema, and confirmed by biopsy. Treatment in surgical.
131. d Gastroschisis is a defect in the abdominal wall that allows the antenatal evisceration of
Rosen 5th, abdominal structures without a sac being present. Omphalocele is a defect in the
Chapter 9, umbilical ring that allows the intestines to protrude out of the abdominal cavity in a
p. 103 sac. Gastroschisis occurs twice as frequently as omphalocele. ED treatment involved
gastric decompression and placing an occlusive plastic covering to prevent heat and
water loss.
132. e Manual detorsion is never curative but should be attempted in most cases while
Rosen 5th, awaiting surgical treatment. Prehn's sign, or relief of pain with elevation of the
Chapter 94 scrotum, was historically thought to be associated with epididymitis, but this
p.1423 association is false and should not be used to distinguish torsion from epididymitis.
The cremasteric reflex is almost always absent in testicular torsion. The salvage rate in
testicular torsion remains high if pain has lasted less than 6 hours, but a urology
consult should be obtained as soon as the diagnosis seems likely clinically.
49

133. Concerning intraosseous lines:


a. Fat embolism is a common complication, especially in adult patients.
b. In adults, the tibia is technically easier than in children because of relative proximity to the
surface.
c. The preferred insertion site in the adult femur is the lateral surface of the trochanter.
d. Infection rates approach 20% in both adults and children.
e. Any drug which can be given intravenously can also be delivered through an intra-osseous line.

134. A 3-month-old girl is irritable, feeds poorly, and breaks into a profuse sweat whenever she takes the
nipple. She is tachypneic and pale. Her heart rate is 280 /
minute. Chest x-ray shows a large heart with alveolar
infiltrates. A rhythm strip is shown. You know that:
a. jugular venous distension and peripheral edema are
commonly found in infants with this condition.
b. volume replacement is the most essential treatment
c. heart rate is the least malleable of the cardiovascular parameters.
d. cardioversion with 0.01 watt-sec/kg is indicated for profound shock
e. a stable patient can be treated with vagal maneuvers

135. A 16-month-old child is brought to the emergency department immobilized in cervical spine
precautions. The child was an unrestrained passenger in a moderate-speed rapid-deceleration motor
vehicle collision. You are concerned about possible neck injury, keeping in mind that:
a. pseudosubluxation of C3 on C4 is common in children.
b. the small neck muscles make fractures more common than ligamentous injury.
c. the predental space should not exceed 4 to 5 mm in children younger than 10.
d. anatomic features of the cervical spine approach adult patterns at around 12 years of age.
e. if the child was ambulatory at the scene, spinal precautions are unnecessary.

136. A 2-year-old child is brought to the emergency department because of fever and pulling at his left
ear. Otitis media is best confirmed by:
a. decreased mobility of his tympanic membrane and loss of normal landmarks.
b. hyperemia of his tympanic membrane in the absence of crying.
c. retraction of his tympanic membrane accompanied by upper respiratory infection symptoms.
d. loss of the tympanic light reflex.
e. the presence of otalgia and upper respiratory infection symptoms.

137. An 18-month-old boy has fever and poor feeding. His ears look normal, but he has an exudative
pharyngitis and enlarged cervical lymph nodes. A rapid antigen detection test for Group A beta-
hemolytic streptococcus (GABHS) is negative.
a. Rapid streptococcal tests are highly sensitive, so it is highly unlikely that this child has acute
GAS pharyngitis.
b. While GABHS is unlikely at this age, detection is important because of the significant risk of
acute rheumatic fever in untreated patients.
c. If the test had been positive, appropriate treatment would include a dose of intramuscular
procaine penicillin.
d. If the test had been positive, early institution of appropriate antibiotics would likely have
shortened the clinical course of the illness
e. Infectious mononucleosis is not a consideration because of the childs age.
50

133. e For pediatric patients up to 5 years of age, the tibia is the preferred site. In adults, the
th
Rosen 5 , most commonly used site is the medial malleolus. Although complications such as fat
Chapter 8, and marrow emboli to the lungs have been reported, recent data suggest that the risk is
p. 84 not increased by the use of IO access for emergent fluid and drug administration. The
tibia is technically more difficult in adults than in children because the adult bone is
thicker and the needle tends to slip off. The incidence of infection, including both
cellulitis and osteomyelitis, is less than 1%, and the potential for infection can be
minimized by limiting the duration of intraosseous infusion and avoiding hypertonic
solutions.
134. e The predominant symptoms of congenital heart disease include poor feeding, excessive
Rosen 5th, diaphoresis, irritability or lethargy with feeding, weak cry and, in severe cases,
Chapter 164 grunting and nasal flaring. Tachypnea is a cardinal sign. Since feeding is the infants
p. 2290-2292 primary form of exertion, dyspnea and sweating during feeding can often be elicited in
the history. Peripheral edema, jugular venous distention, and rales are unusual and late
signs in infants. Heart rate is the most malleable of the cardiac physiologic parameters.
The common pediatric dysrhythmia is paroxysmal supraventricular tachycardia. Initial
management of unstable patients with narrow complex tachycardia consists of
immediate synchronized cardioversion at 0.5 J/kg with increases in power output to 2
J/kg as needed. In the stable patient, vagal maneuvers are the intervention of choice. If
vagal maneuvers are not successful in the stable patient, IV adenosine (0.1 mg/kg
maximum first dose 6mg) followed by (0.2mg/kg maximum 12mg) can be used.
135. c On a lateral cervical spine view the distance between the anterior aspect of the
Rosen 5th, odontoid process and the posterior aspect of the anterior ring of C1, the so-called
Chapter 32, predental space, should not exceed 5 mm in a child. Pseudosubluxation of C2 on C3
p. 274-276 occurs in approximately 40% of children up to adolescence. Anatomic features of the
cervical spine approach adult patterns between the ages of 8 and 10 years. Compared
to adults, the child has relatively underdeveloped neck musculature and a head that is
disproportionately large and heavy compared to the body, leading to fewer fractures
and more ligamentous injuries.
136. a The most significant sign of acute otitis media (AOM) is the loss of or decrease in
Rosen 5th, mobility of the tympanic membrane. The light reflex is of no diagnostic value. The
Chapter 67, normal eardrum is translucent and pearly gray but may become reddened with crying.
p. 928-931 The TM of AOM is usually opaque, hyperemic, and sometimes bulging, and bony
landmarks (long and short process of the malleus) are not easily discernible.
137. d Early antibiotic treatment of streptococcal pharyngitis leads to a 13% earlier resolution
Rosen 5th, of symptoms and shortens the course of illness by about 1 day. GABHS is primarily a
Chapter 70, disease of children 5 to 15 years old and occurs in winter and early spring. It is
p. 969-972 responsible for less than 15% of pharyngitis in patients older than 15 years of age and
is rare in patients less than 3 years old. Rapid strep tests have reported specificity of
70% to 100% (with most being >95%) and sensitivity of 31% to 100% (with most
being 60% to 95%). The incidence of rheumatic fever parallels that of GABHS, with
the peak incidence in children 5 to 15 years old, less common in adults, and rare in
children less than 3 years of age. Monospot tests are positive in about 30% of children
0 to 20 months with mononucleosis.
51

138. A 3-day-old baby presents to the emergency department with purulent conjunctivitis. His mother
has a history of syphilis treated prior to this pregnancy. She had no prenatal care, but delivered in
the hospital. The infant received topical erythromycin prophylaxis in the nursery. The most likely
cause of this childs illness is:
a. syphilis.
b. Neisseria gonorrhoeae.
c. group B streptococcus.
d. Herpes simplex.
e. Chlamydia trachomatis.

139. A 2-year-old patient has screamed inconsolably for four hours and is brought to the ED by his
frazzled parents. He is afebrile and non-toxic in appearance; initial exam shows nothing but mild
left conjunctival erythema without discharge. Your next step should be:
a. lumbar puncture.
b. Schiotz tonometry.
c. fluorescein staining of the cornea.
d. urinalysis.
e. stool for occult blood.

140. Parents bring a 6-week-old infant to the ED because of 3 days of vomiting, occurring immediately
after eager feedings. The baby is fussy, but alert. His mucous membranes are tacky. When you
feed him, he eagerly takes the bottle, but then promptly vomits. Except for the dehydration, you
find nothing else on the exam. This child will require:
a. surgical intervention.
b. intravenous hydration and observation.
c. prompt parenteral antibiotic therapy.
d. discharge with BRAT diet and parental reassurance.
e. change to a soybean-based formula.

141. Cyanosis frequently occurs with:


a. ventricular septal defect.
b. atrial septal defect.
c. patent ductus arteriosus.
d. tetralogy of Fallot.
e. hypertrophic cardiomyopathy.

142. Regarding ear infections:


a. The tympanic membrane should always appear normal in acute otitis externa.
b. Normal tympanic membrane mobility rules out acute otitis media.
c. Children with acute otitis media who are compliant with appropriate antibiotic regimens do not
require any follow-up.
d. An intense worsening of the pain usually indicates sudden tympanic membrane perforation.
e. Topical antibiotic therapy for otitis externa is ineffective unless the canal is free of debris and
cerumen.
52

138. e Neonatal conjunctivitis (ophthalmia neonatorum) occurs within the first month of life.
th
Rosen 5 , The chemical irritation from antimicrobial prophylaxis against bacterial infection
Chapter 66, occurs within 1-2 days of birth. Other causes include Chlamydia trachomatis, H.
p. 917 influenzae, and Streptococcus pneumoniae. Due to mandated use of erythromycin
ocular prophylaxis, Neisseria gonorrhoeae is not a major cause of neonatal
conjunctivitis in the United States. This drug, however, has not been proven effective
against chlamydia infections. Gonococcal conjunctivitis generally has its peak time of
onset between 3 and 5 days after birth. By the end of first week of life and throughout
the first month of life, chlamydia becomes the most frequent cause of conjunctivitis.
139. c Your next step should be a fluorescein staining of the reddened eye, followed by
Rosen 5th, examination with a cobalt blue light. Corneal abrasion is a common cause of
Chapter 66 inconsolable crying in an otherwise normal infant.
140. a The infant with a history of nonbilious projectile vomiting must be considered to have
th
Rosen 5 , pyloric stenosis. Physical examination usually demonstrates a hungry infant who has
Chapter 165, failed to gain weight over the past several weeks or has lost weight. Surgery is the
p. 2297-2301 treatment of choice although there are reports of success with IV and oral atropine, a
141. d Tetralogy of Fallot is one example of cyanotic heart disease associated with decreased
Rosen 5th, pulmonary blood flow. Transposition of the great vessels exemplifies cyanotic heart
Chapter 164, disease with increased pulmonary blood flow. Other congenital cardiac lesions
p.2283-2287 causing cyanosis are truncus arteriosus; tricuspid atresia; and total anomalous venous
return. They are known collectively as the terrible Ts.
142. e A thorough and atraumatic cleansing of the ear canal is the most important part of
Rosen 5th, therapy. For mild infections, dry mopping using a small tuft of cotton attached to a
Chapter 67, wire applicator is sufficient and may be curative. The tympanic membrane of a child
p. 931-932 with otitis externa may be as red and distorted as that of a child with otitis media,
although mobility of the tympanic membrane is normal or only slightly decreased in
otitis externa. Visualization of the tympanic membrane may be difficult because of
edema of the external auditory canal.
53

143. Which statement is true regarding Group A beta-hemolytic streptococcal (GABHS) pharyngitis:
a. Experienced clinicians can accurately exclude GABHS pharyngitis with the history and physical
exam alone.
b. Overuse of penicillin has resulted in emergence of resistant strains of GABHS requiring a change
in recommended first line antibiotics.
c. It is inappropriate to treat sore throat with antibiotics unless there is culture-proven GABHS.
d. Throat culture for GABHS has very few (less than 1%) false-negatives.
e. Treatment of streptococcal pharyngitis significantly shortens the course of the disease.

144. You are evaluating a 6 week-old infant for irritability as reported by her mother. You find an
alert but cranky infant with a rectal temperature of 95oF. The left eardrum is a little red. The baby
is fussy when held, but relatively quiet when left on the bed. Which statement is correct?
a. This infant needs lumbar puncture.
b. Since you found an otitis media, you have an explanation for the infants illness and an LP is not
necessary.
c. Given a reliable mother, antibiotic therapy and close outpatient follow-up is appropriate
d. It is almost impossible for an infant this young to have an otitis media.
e. If the fontanelles are flat, you can reliably rule out meningitis.

145. In a child with a chest x-ray that shows pneumonia, the most common physical finding would be:
a. rales.
b. rhonchi.
c. tachypnea.
d. asymmetric breath sounds.
e. wheezes.

146. A 4-year-old girl presents with six days of fever. You find erythema of her hands and feet,
conjunctival injection, cervical adenopathy, strawberry tongue, dry lips, red throat, and a
morbilliform rash. You should treat this girl with:
a. penicillin.
b. aspirin.
c. ceftriaxone.
d. amoxicillin.
e. dexamethasone.

147. A 3-year-old girl has a plastic bead stuck in her ear. She kicks and screams when you try to remove
the bead and, even after sedation, you are unsuccessful in your attempts to remove the foreign body.
Not wanting to cause injury, you give up and call the ENT surgeon, who tells you not to worry
about it; send the patient home and have Mom call the next day for an appointment. You should:
a. follow the consultants instructions.
b. insist that the consultant come in and see the patient.
c. call a different consultant.
d. sedate the patient even more deeply to allow successful foreign body removal.
e. turn the child upside-down and shake her vigorously.

148. Which electrical injury is correctly paired with its resultant complication?
a. low-voltage alternating current (AC) ventricular fibrillation
b. lightning ventricular fibrillation
c. high-voltage AC superficial burns
d. lightning compartment syndrome
e. high-voltage AC tetanic contraction
54

143. e Research has clearly demonstrated the beneficial effects of early antibiotic therapy on
th
Rosen 5 , reduction of signs and symptoms of GABHS pharyngitis. Diagnostic accuracy on the
Chapter 70, basis of clinical findings alone is reported at about 50 to 75% for children thought to
p. 971-972 have GABHS and 75 to 85% for children thought not to have GABHS. There is
general agreement that clinical diagnosis alone would result in an unacceptably high
rate of misdiagnosis. GABHS is highly sensitive to penicillin, and there has been no
evidence of development of resistance in vitro despite decades of use. The rate of
false-negative results from single throat culture is about 10%. Because clinical
judgment is insufficient and rapid diagnostic tests are not always accurate and diagnose
only GABHS, this disease process is often treated empirically.
144. a The symptoms and signs suggestive of bacteremia and serious bacterial illness most
Rosen 5th, frequently produce an overall ill appearance. Parents may note poor feeding,
Chapter 168, decreased responsiveness, or irritability in response to attempts to console. Physical
p. 2344-2350 examination findings suggestive of an ill appearance include poor eye contact and
muscle tone, including weak suck, poor head control, and indifferent response to
stimuli. The most important historical and physical findings are related to overall
appearance. Bulging fontanelle, a hallmark of an increased intracranial pressure, is
present in approximately 15% of neonatal acute bacterial meningitis.
145. c The best physical examination finding for ruling out pneumonia in an infant or child is
Rosen 5th, the absence of tachypnea. However, tachypnea is a nonspecific symptom and may
Chapter 163, occur secondary to fever, anxiety, metabolic disease, cardiac disease, or other
p. 2268-2270 respiratory problems. Fever can increase an infants respiratory rate by 10 breaths per
minute for each degree centigrade of elevation. Auscultation of the lungs may reveal
localized rales, wheezing, and decreased air entry in the affected area. However,
auscultatory findings may not be reliable in children. In younger children, decreased
breath sounds, rather than rales, are often heard, since the involved areas tend to be
ventilated poorly.
146. b Kawasaki syndrome is a vasculitis that causes a prolonged fever (more than 5 days),
Rosen 5th, usually exceeding 39.5-40C (104 F). Clinical findings include conjunctival injection
Chapter 161 without exudate; red, peeling lips; strawberry tongue; and pharyngitis. Early there may
p. 2238-2239 be painful swelling of the hands and feet; peeling of the skin of the fingers occurs late
in the course. Cervical adenopathy in excess of 5 cm is common. Treatment is
directed toward the amelioration of symptoms and the prevention of coronary
aneurysms. Gamma-globulin 2 gm/kg intravenously should be administered over 12
hours, followed by high-dose aspirin therapy (100 mg/kg/24 hr PO given in divided
doses every 6 hours for 14 days).
147. a If routine methods are unsuccessful or if the patient, is uncooperative or in too much
Rosen 5th, distress, the emergency physician should cease removal efforts and refer the patient to
Chapter 53 an otolaryngologist. Primary operative intervention is frequently indicated in very
p.754-756 young children and in those in whom the EP and the ENT specialist believe
nonsurgical attempts unlikely to be successful. Inappropriately prolonged efforts at
foreign object removal can result in wasted time, unnecessary patient discomfort, and
high potential for complications.
148. a Low-voltage alternating current generally produces ventricular fibrillation. High-
Rosen 5th, voltage AC that causes greater current intensity is more likely to produce asystole.
Chapter 136, Direct current is also more likely to cause asystole. The most common arrhythmia
p. 2010-2014 encountered in victims who sustain cardiac arrest from electrical injury is ventricular
fibrillation. The immediate cause of death due to a lightning strike is usually direct
current depolarization of the myocardium and sustained cardiac asystole.
55

149. A 4-year-old non-immunized child presents with sudden sore throat, stridor, drooling, and fever.
He looks sick and is sitting forward on the litter, but is alert. Pulse oximetry is 99% saturation on
room air. Your next step is to:
a. attempt to visualize his epiglottis.
b. send him to x-ray for lateral neck radiographs.
c. immediately begin bag-valve-mask ventilation.
d. immediately intubate the child nasotracheally.
e. none of the above.

150. A 3-year-old boy is brought to the emergency department after choking on a watch battery. X-rays
show that the battery is in his stomach. Appropriate management involves:
a. parental reassurance and home observation.
b. ipecac to induce expulsion of the battery.
c. cathartics to speed transit through the gastrointestinal tract.
d. admission to the hospital for observation.
e. laparotomy if the battery is still in the small bowel at 48 hours.

151. Which of the following fractures is most suggestive of child abuse?


a. 19-month-old with a supracondylar fracture after falling off a chair.
b. 22-month-old with a spiral femur fracture from a fall while running.
c. 10-month-old with linear skull fracture from a fall down stairs in an infant walker.
d. 4-month-old with a transverse fracture of the humerus from a fall out of bed.
e. 3-year-old with a tuft fracture of the distal phalanx from the finger being closed in a door.

152. A 10-year-old boy complains of mild intermittent abdominal pain and a rash. His parents have not
noted any fever. Physical examination shows a well-appearing boy with a raised purpuric rash,
most prominent on the legs. His abdominal exam is normal. Temperature = 99.8oF. The other
vital signs are normal. What is the most appropriate next step in the management of this patient?
a. Obtain blood for culture and start antibiotics.
b. Obtain abdominal radiographs.
c. Obtain a urinalysis, BUN, and creatinine.
d. Perform a skin biopsy.
e. Order liver function tests.

153. Concerning cold exposure injuries:


a. chilblains (pernio) is a painless macular rash.
b. in frostbite, the zone of stasis is most severely injured.
c. early surgical intervention is recommended for severe frostbite.
d. a body part previously affected by cold injury is immune from reinjury.
e. trench foot may result in irreversible damage.

154. The most common finding in a patient with a brown recluse spider bite is:
a. vomiting.
b. severe muscle cramps.
c. anaphylaxis.
d. local tissue necrosis.
e. respiratory failure.
56

149. e The ideal approach is to take any patient with suspected epiglottitis to the operating
th
Rosen 5 , room, administer anesthesia, and examine the airway with a laryngoscope while the
Chapter 162 patient is anesthetized. If the diagnosis of epiglottitis is made, the patient can be
intubated. If it is ruled out, the patient can be returned to the ward or the emergency
department to continue the workup, secure in the knowledge that epiglottitis is not
present.
150. a Button batteries that have passed the esophagus need not be retrieved in the
Rosen 5th, asymptomatic patient unless the cell is not passing through the pylorus after 48 hours
Chapter 147, of observation, in which case endoscopic retrieval is the preferred option. Ipecac has
p. 2118 no place in the management of button battery ingestion.
151. d Fractures suggestive of child abuse include spiral fractures caused by torsion (twisting)
th
Rosen 5 , of a long bone, and metaphyseal chip fractures, especially when present in infants less
Chapter 169, than 6 months of age. While this is the most suspicious of the fractures described, it
p. 2379-2381 must be emphasized that any of these fractures could result from abuse. Abuse must
always be considered as a possibility with injuries involving young children.
152. c Henoch-Schnlein Purpura is an immunoglobulin A mediated systemic vasculitis
Rosen 5th, involving the small blood vessels supplying the skin, gastrointestinal tract, and joints.
Chapter 167 The hallmark is a palpable, purpuric, or petechial rash most prominent on the lower
extremities and extending to the buttocks, which is the presenting complaint in ~50%
of patients. GI complaints are present in ~65% of patients, especially periumbilical,
dull pain resulting from bleeding into the intestinal wall. 25% - 50% of children
develop a self-limiting glomerulonephritis manifested by hematuria. There are no
specific tests to confirm HSP, and the diagnosis can be difficult if the classic rash is
absent. Screening tests such as urinalysis, BUN and creatinine, CBC, and coagulation
studies may be needed to rule out other pathologic diseases.
A helpful mnemonic is ARENA:
A = Abdominal pain
R = Rash
E = Edema
N = Nephritis
A = Arthralgias / arthritis
153. e Trench foot develops slowly over hours to days and is initially reversible but if allowed
Rosen 5th, to progress will become irreversible. Chilblains, or pernio, is characterized by mild but
Chapter 133, uncomfortable inflammatory lesions of the skin of bared extremities caused by chronic
p. 1974 intermittent exposure to damp, nonfreezing ambient temperatures. The zone of stasis
is the middle ground and is characterized by severe, but possibly reversible, cell
damage. It is here that treatment is directed. Early surgical intervention is not
indicated in the management of frostbite. Premature surgery has been an important
contributor to unnecessary tissue loss and poor results in the past. Refreezing
previously frozen tissue worsens the prognosis.
154. d Most victims of bites by the brown recluse spider do not even know they were bitten.
Rosen 5th, Pain usually develops within 3 to 4 hours, and a white area of vasoconstriction begins
Chapter 55 to surround the bite. A bleb then forms in the center of this area, and an erythematous
p.796 ring arises on the periphery. The lesion at this stage resembles a bulls-eye. The bleb
darkens, necroses over the next several hours to days, and continues to spread slowly
and gravitationally, involving skin and subcutaneous fat.
57

155. A 25-year-old man was playing catch with his pet rattlesnake. He now complains of pain and
swelling in his hand and forearm, with perioral numbness, and vomiting. His blood pressure is
90/60 mmHg. He will require:
a. fluid resuscitation with fresh frozen plasma.
b. administration of 2 to 4 vials of antivenin.
c. measurement of coagulation factors and platelets.
d. immediate forearm fasciotomy.
e. non-narcotic pain medication, to avoid masking of respiratory symptoms.

156. Acetazolamide acts by inhibiting the enzyme carbonic anhydrase. This in turn:
a. reduces reabsorption of bicarbonate in the kidney, leading to bicarbonate diuresis and metabolic
acidosis, which produces compensatory hyperventilation.
b. increases the blood carbon dioxide level, allowing a respiratory acidosis and compensatory
diuresis.
c. slows the sodium-potassium ATPase pump, causing a natriuretic compensation for the leftward-
shifting oxygen dissociation curve.
d. causes the kidney to reabsorb more bicarbonate ions, leading to a metabolic alkalosis, which
shifts the oxygen dissociation curve to the right to compensate for lower atmospheric pressures.
e. prevents acute mountain sickness by an unknown mechanism.

157. A 55-year-old male diver complains of back pain and urinary retention which started about one
hour after ascent from a dive. He most likely has:
a. bladder barotrauma.
b. lumbar strain.
c. envenomation by coelenterate species.
d. nitrogen narcosis.
e. decompression sickness.

158. Concerning High Altitude Pulmonary Edema (HAPE):


a. it is the most lethal of the altitude illnesses.
b. women are more susceptible than men.
c. salt deprivation is a risk factor; use of a sleeping medication is somewhat protective.
d. early symptoms include a moist cough with blood-tinged sputum; rales are universally present.
e. the condition typically improves at night.

159. Choose the correct statement concerning electrical burns.


a. When tissue carbonization occurs, resistance to current flow decreases even further.
b. Contact with low voltage long distance communications lines and telephone can cause death in
certain circumstances.
c. A very narrow range exists between the threshold of perception of current and the level above
which a person becomes unable to release the current source because of muscular tetany.
d. The foot is the most common body part involved in alternating current injuries.
e. Sweating increases resistance against electricity.

160. Which of these patients requires admission to a burn-care facility?


a. A 35-year-old man with extensive partial-thickness burns on the back, shoulders, and buttocks.
b. A 60-year-old diabetic with a full-thickness burn of the entire forearm.
c. A 25-year-old woman with full-thickness burns of both hands and lower arms.
d. A 40-year-old house-fire victim with multiple, small partial-thickness burns and wheezing.
e. All of the above
58

155. c Patients admitted to the hospital after poisonous snakebite should have serial
th
Rosen 5 , determinations of platelets, prothrombin time, and urinalysis to check for myoglobin
Chapter 55, and hemoglobin. Daily comprehensive laboratory tests should be performed.
Fasciotomy is not usually indicated unless compartment pressures are elevated.
Antivenin (Crotalidae) polyvalent is the mainstay of therapy for poisonous snakebite;
usually 10 vials or more are required. Debridement should probably not be performed
earlier than 3 days after the bite, until the coagulopathy has resolved.
156. a Acetazolamide is a carbonic anhydrase inhibitor that induces a renal bicarbonate
Rosen 5th, diuresis causing a metabolic acidosis, thereby increasing ventilation and arterial
Chapter 138, oxygenation. The diuretic effects may be of benefit for the fluid retention common in
p. 2041 AMS. The drug also lowers CSF volume and pressure, which may play an additional
role in its therapeutic and prophylactic use.
157. e The central nervous system is particularly susceptible to decompression illness because
Rosen 5th, of its high lipid content. The spinal cord, especially the upper lumbar area, is more
Chapter 137, often involved than cerebral tissue. Symptoms of spinal DCS include limb weakness
p. 2024 or paralysis, paresthesias, numbness, and low back and abdominal pain. Bladder
symptoms, such as urinary retention, can occur, as well as fecal incontinence and
priapism.
158. a High Altitude Pulmonary Edema (HAPE) is the most common fatal manifestation of
Rosen 5th, severe high-altitude illness. It can occur, and even be fatal, at altitudes as low as 8000
Chapter 138, feet. Many patients have a single episode of HAPE and subsequently are able to return
p. 2042 to high altitude without a recurrence. Others who have had uneventful high-altitude
exposures may have HAPE develop on a future ascent.
159. c A very narrow range exists between the threshold of perception of current (0.2 to 0.4
Rosen 5th, mA) and the let-go current (6 to 9 mA), the level above which a person becomes
Chapter 136, unable to release the current source because of muscular tetany. When tissue
p. carbonization occurs, resistance to current flow increases. No deaths are recorded
from contact with low voltages associated with long distance communications lines
(24V) or telephone lines (65V). The hand is the most common site of contact via a
tool that is in contact with an AC electric source. Sweating can reduce the skins
resistance from 10 40,000 ohms to 2500-300 ohms.
160. e 1. Partial- or full-thickness burns involving >10% of body surface area (BSA) in
Rosen 5th, patients under 10 or over 50 years of age.
Chapter 56, 2. Partial- or full-thickness burns of >20% of BSA in other age groups.
p. 812, Box 3. Partial- or full-thickness burns with the threat of functional or cosmetic impairment
56-2 that involve face, hands, feet, genitalia, perineum, or major joints.
4. Full-thickness burns of >5% of BSA in any age group.
5. Electrical burns, including lightning injury.
6. Chemical burns with the threat of functional or cosmetic importance.
7. Inhalation injury with burns.
8. Circumferential burns of the extremities or chest.
9. Burn injury in patients with preexisting medical disorders that could complicate
management, prolong recovery, or affect mortality.
10. Any burn patient with concomitant trauma, such as fracture.
59

161. Concerning victims of near-drowning:


a. dry drowning without aspiration results from laryngospasm and glottal closure.
b. electrolyte abnormalities contribute significantly to mortality.
c. hemolysis resulting in anemia is found in more than half of fresh-water victims.
d. disseminated intravascular coagulation is a common finding.
e. postural drainage or the abdominal thrust (Heimlich maneuver) is recommended by most experts
as a way to remove water from the lungs and improve oxygenation.

162. After radiation exposure, the best predictor of hematopoietic involvement is the:
a. Absolute neutrophil count at 24 hours.
b. Absolute neutrophil count at 48 hours.
c. Absolute lymphocyte count at 24 hours.
d. Absolute lymphocyte count at 48 hours.
e. Absolute platelet count at 24 hours.

163. A 35-year-old man presents complaining of headache, weakness, nausea, and vomiting after
working with paint remover in an enclosed space. You know that:
a. a special antidote kit is required.
b. treatment must continue longer in patients with this exposure than from other sources.
c. the patient's oxygen-hemoglobin dissociation curve is shifted to the right.
d. severe metabolic acidosis may be present.
e. methylene blue may be required.

164. A 25-year-old photographer is found slumped over his workbench in the back of his store. Minutes
earlier he had complained of headache, dizziness, and weakness. Presently the patient is comatose
and apneic with a thready pulse. Cyanosis is absent. A venous blood sample drawn by paramedics
in the field is noted to be very red. Laboratory results reveal a high anion gap acidosis. Blood gas
determinations on arterial and venous blood samples reveal nearly identical pO2 values.
Carboxyhemoglobin is 5%. Although the patient is intubated and being hyperventilated with 100%
oxygen, the acidosis persists. What is the proper management?
a. administer methylene blue IV
b. administer physostigmine and hydroxocobalamin (vitamin B12a)
c. administer 2-PAM
d. administer sodium nitrite IV, then infuse sodium thiosulfate
e. transfer the patient to a facility with a hyperbaric chamber

165. In a mass casualty situation involving a large number of victims of a dirty bomb, the highest
priority patients are those:
a. who are critically injured and contaminated.
b. with external local body radiation only.
c. with external total body radiation only.
d. with internal contamination.
e. with external contamination.

166. A 24-year-old man was bitten on the arm by his girlfriends cat six hours ago. He now has warmth
and erythema at the site. The infecting organism is most sensitive to:
a. gentamicin.
b. metronidazole.
c. clindamycin.
d. penicillin.
e. fluconazole.
60

161. a In a small number of submersion victims, severe laryngospasm causes hypoxia,


th
Rosen 5 , convulsions, and death in the absence of active aspiration (dry drowning). Most
Chapter 139, drowning victims do not aspirate enough fluid to cause life-threatening changes in
p. 2051 blood volume or serum electrolyte concentrations. Although DIC has been reported, it
is uncommon. Procedures to drain fluid from the lungs are ineffective and potentially
dangerous because of the increased risk of vomiting and aspiration. If particulate
material is obstructing the airway, the Heimlich maneuver may be of benefit but it is
not indicated for the removal of fluid.
162. d The absolute lymphocyte count 48 hours after exposure is a good predictor of
Rosen 5th, hematopoietic involvement. If the absolute lymphocyte count is greater than 1200, it is
Chapter 140, unlikely that the patient has received a fatal dose of radiation. If the absolute
p. 2058-2059 lymphocyte count falls between 300 and 1200 at 48 hours, the possibility of exposure
to a lethal dose of radiation should be suspected. A level in this range is an indication
for hospitalization. Levels less than 300 are critical, and heroic procedures such as
bone marrow transplant and the use of hematopoietic growth factors may be
considered in individual cases.
163. b Carbon monoxide (CO) toxicity can develop when inhaled methylene chloride vapor in
Rosen 5th, paint strippers or from leaking "bubble" electric Christmas tree lights is slowly
Chapter 152, metabolized to produce CO. The elimination half-life of CO from methylene chloride
p. 2159-2162 is about twice that of inhaled CO because it is stored in tissues and gradually released.
The binding of CO to hemoglobin transforms the oxyhemoglobin dissociation curve
from a sigmoid shape to an asymptotic shape, increasing the ability of HbCO to hold
on to oxygen at the remaining heme moiety sites. In CO toxicity, both the reduced
oxygen carriage and the transformation of oxyhemoglobin dissociation curve impair
tissue oxygen delivery. In effect, high HbCO imposes the equivalent of a sudden
chemical anemia in the patient.
164. d Cyanide in its salt form (e.g., sodium or potassium) is important in the metallurgic
Rosen 5th, (e.g., jewelry) and photographic industries. Cyanide salts do not have an odor under
Chapter 153, dry conditions. Since cyanide prevents tissue extraction of oxygen from the blood, the
p. 2166-2168 oxygen content of venous blood approaches that of arterial blood. Clinically this may
appear as the arterialization, or brightening, of venous blood to resemble arterial
blood. Although the exact mechanism of the cyanide antidote kit is controversial, the
accepted goal of therapy is to reactivate the cytochrome oxidase system by providing
an alternative, high-affinity source of ferric ions (Fe3+) for cyanide to bind. Sodium
nitrite and sodium thiosulfate are the active ingredients in the kit.
165. a If treatment of great numbers of radiation exposed and contaminated patients is
Rosen 5th, necessary, resuscitation and stabilization always takes precedence over
Chapter 140, decontamination. However, health care workers must not expose themselves.
p. 2060-2062 Protection of health care workers from injury takes priority over the patient's ABCs'.
166. d Pasteurella multocida is the major pathogen found in infected cat bite wounds, isolated
th
Rosen 5 , in up to 80%. Infection is characterized by a rapidly developing, intense inflammatory
Chapter 54, response, often within a few hours and rarely more than 24 hours after the bite. Pain
p. 775-776 and swelling are prominent. The antibiotic best used to treat the major organism in this
presentation is penicillin.
61

167. Your friend has just stepped on a jellyfish while swimming. He is tearful because of the pain. You
can offer temporary relief by:
a. drying the area.
b. rinsing the site with ethanol.
c. rinsing the site with fresh water.
d. rinsing the site with milk.
e. rinsing the site with salt water.

168. The vast majority of mushroom-related fatalities in North America are due to:
a. coprine-induced disulfiram-like reaction when consumed with ethanol.
b. cyclopeptide-induced liver and renal failure.
c. monomethylhydrazine-induced seizures and hepatorenal failure.
d. muscarine-induced "SLUDGE" syndrome.
e. psilocybin-induced hallucinations and seizures.

169. A 16-year-old girl ingested an unknown plant an hour ago. Now she is tachycardic with mydriasis,
decreased bowel sounds, altered vision, and abnormal mental status. After ensuring the basic
ABCs, your next action should be:
a. intravenous atropine.
b. GI decontamination.
c. isotonic intravenous fluid administration.
d. subcutaneous physostigmine.
e. intramuscular glucagon.

170. A hiker on the Appalachian Trail is bitten on the hand by a snake, which was positively identified
as having red bands next to yellow bands. She describes mild pain at the site of the bite, but is
otherwise asymptomatic. Your recommended treatment is:
a. admission to the hospital for observation only.
b. discharge after extended observation.
c. discharge after routine wound care.
d. administration of antivenin if the patient becomes symptomatic.
e. immediate administration of antivenin.

171. Which is the typical sequence of cardiac arrhythmic deterioration in a patient whose core
temperature is below 30oC (86oF).
a. atrial fibrillationventricular fibrillationpulseless electrical activityasystole
b. sinus bradycardiaventricular tachycardiaventricular fibrillationasystole
c. sinus bradycardiaatrial fibrillationventricular fibrillationasystole
d. atrial fibrillationatrial flutterventricular fibrillationasystole
e. sinus bradycardiaatrial flutteratrial fibrillationfine ventricular fibrillation

172. A 25-year-old man notices burning pain in his arm while retrieving logs from a woodpile. Within
six hours, he has developed a necrotic area surrounded by an erythematous ring. He then notices
fever, myalgias, nausea and generalized weakness. Of those listed, the most likely cause is:
a. acute contact dermatitis.
b. brown recluse spider bite.
c. splinter from maple bark.
d. deer tick bite.
e. black widow spider bite.
62

167. e Jellyfish envenomation sites should be washed off with salt water from the ocean, as
th
Rosen 5 , fresh water is reported to worsen the pain.
Chapter 55,
p. 797-798
168. b Cyclopeptides such as amatoxin, found in certain amanita species, produce a three-
th
Rosen 5 , phase pattern of toxicity with early GI effects followed by a quiescent phase during
Chapter 144, which the patient may be released from medical care. The devastating third phase
p. 2203-2205 results in severe hepatic and sometimes renal toxicity. Ninety-five percent of North
American deaths from mushroom poison are due to these toxins. While
monomethylhydrazine poisoning from the Gyromitra species may have 10 40%
mortality, this poisoning is much less common and thus yields fewer fatalities.
Poisoning due to mushrooms containing muscarine, psilocybin, or coprine generally
responds well to supportive measures and antidotal therapy.
169. b Anticholinergic toxicity commonly occurs with jimsonweed ingestion. This plant
Rosen 5th, contains belladonna alkaloids such as atropine, scopolamine, and ecgonine. After
Chapter 144, assuring the ABC's, treatment includes GI decontamination with emesis or lavage,
p. 2081-2081 activated charcoal, and supportive care (IV fluids, external cooling, and restraints for
Chapter 158, patient protection). GI decontamination may be useful for up to 48 hours after
p. 2199-2201 ingestion if the patient remains symptomatic. Isotonic fluid administration is the first
line treatment for hypotension. Many authorities recommend treatment of "full-blown"
anticholinergic syndrome with physostigmine, but reports of asystole, ventricular
arrhythmia, hypotension, bronchospasm, and seizures have occurred.
170. e Red on yellow, kill a fellow; red on black, venom lack. This applies to women
Rosen 5th, victims, too. Anyone bitten by the Eastern coral snake (Micrurus fulvius) should be
Chapter 55, given the antivenin even before any symptoms develop. The toxicity of this venom
p. 789-792 has a rapid onset, and once the symptoms develop, it may be too late to reverse the
effects with antivenin. The recommended dose is three to five vials in 300 to 500 ml
of normal saline.
171. c Patients are at risk for dysrhythmias at body temperatures below 30C (86F); the risk
Rosen 5th, increases as body temperature decreases. Although various dysrhythmias may occur at
Chapter 134, any time, the typical sequence is a progression from sinus bradycardia to atrial
p. 1989-1990 fibrillation with a slow ventricular response, to ventricular fibrillation, and ultimately,
to asystole.
172. b As the name implies, the brown recluse spider is a reclusive organism. Humans
Rosen 5th, encounter this spider in attics, storage sheds, crawl spaces, and woodpiles. Encounters
Chapter 55, with humans are uncommon. Brown recluse spiders are not aggressive and bite
p. 795-796 humans in self-defense. The venom results in epidermal and subcutaneous necrosis.
This reaction is a local process around the bite site. About 6 to 8 h after the bite, pain
associated with a red-to-violaceous discoloration develops.
63

173. When resuscitating a hypothermic patient whose core temperature is <85oF you should:
a. administer prophylactic lidocaine.
b. move the patient as little as possible.
c. pronounce the patient dead if there is no cardiac response after rewarming to 90oF.
d. use Ringer's lactate as the intravenous fluid of choice.
e. massage extremities and apply heating blanket.

174. A diver was stung by a stingray and has severe pain, which you can relieve by:
a. applying ice packs.
b. applying vinegar.
c. immersing his leg in hot water.
d. urinating on his leg.
e. using a topical slurry of sodium bicarbonate.

175. The delayed complications of fetal hemorrhage into maternal circulation are of most concern in a
woman:
a. taking steroids.
b. who is Rh-negative.
c. with beta-thalassemia.
d. with alpha-thalassemia.
e. with sickle trait.

176. A 36-week pregnant woman presents in shock with painful vaginal bleeding and a hard, tender
uterus. She most likely has:
a. eclampsia.
b. endometritis.
c. placenta previa.
d. threatened abortion.
e. abruptio placenta.

177. A 28-year-old woman is brought to your Emergency Department by ambulance in cardiopulmonary


arrest after an automobile collision. Cardiopulmonary resuscitation was started approximately 4
minutes prior to arrival. Her husband is in the next room, crying hysterically. Shes due with our
first baby next month. Judging from her fundal height you estimate that the fetus is at 34-weeks
gestational age. You are the only physician in the hospital, covering a Level II trauma center. Your
most appropriate next action is:
a. page the obstetrician.
b. start a tocolytic agent and page the obstetrician.
c. pronounce the patient dead.
d. continue ACLS and page the trauma surgeon.
e. perform perimortom cesarean delivery.

178. The earliest that a serum beta-hCG test can detect pregnancy is:
a. shortly before a missed period.
b. during the time of an expected period.
c. 2-weeks after a missed period.
d. 4-weeks after a missed period.
e. 6-weeks after a missed period.
64

173. b The hypothermic myocardium is extremely irritable, and ventricular fibrillation may be
th
Rosen 5 , induced by a variety of manipulations and interventions that stimulate the heart,
Chapter 134, including rough handling of the patient. The activity of antiarrhythmic and
p. 1989-1994 cardioactive drugs is unpredictable in hypothermia, and the hypothermic heart is
relatively resistant to atropine, pacing, and countershock.
174. c The success of therapy for stings from marine animal spines depends on rapid
Rosen 5th, initiation. Treatment is directed at combating the effects of the venom, alleviating
Chapter 55, pain, and preventing infection. The wound should be irrigated immediately, and any
p. 797-799 visible pieces of the spine or integumentary sheath should be removed. As soon as
possible, the wound should be immersed in hot water to tolerance [45C (113F)] for
30 to 90 min or until there is pain relief. During the hot-water soak, the wound should
be explored and foreign material removed.
175. b Rh immunization occurs when an Rh-negative female is exposed to Rh-positive blood
Rosen 5th, during pregnancy or delivery. The incidence of sensitization is about 16% when the
Chapter 172, Rh-negative woman and Rh-positive fetus are ABO-compatible and 1% to 2% if they
p. 2424-2425 are ABO-incompatible. Sensitization occurs most commonly at the time of delivery,
but transplacental hemorrhage can occur during threatened miscarriage (even without
fetal loss), spontaneous miscarriage, surgery for ectopic pregnancy, and amniocentesis.
Anti-D immune globulin (RhoGAM) should be administered when these events occur.
RhoGAM is routinely administered to Rh-negative mothers (if the father is Rh-positive
or his status is unknown) at about the 28th week of gestation to protect the mother
from spontaneous sensitization, which occurs during the third trimester. RhoGAM has
also been used to prevent further deterioration of fetal anemia in Rh-D immunizations
if started before severe fetal anemia and imminent hydrops fetalis arise. Minidose
RhoGAM has been used in some patients with early miscarriage or trauma.
176. e Abruptio placentae, or separation of the placenta from the uterine wall, accounts for
Rosen 5th, about 30% of episodes of bleeding during the second half of pregnancy. Vaginal
Chapter 172 bleeding occurs in 80% of patients with abruptio placentae. Blood is characteristically
p. 2420 dark and the amount is often insignificant, although the mother may have
hemodynamic evidence of significant blood loss. Uterine tenderness or pain is seen in
about two thirds of women; uterine irritability or contractions are seen in one third.
With significant placental separation, fetal distress occurs and the maternal coagulation
cascade may be triggered causing DIC.
177. e The need to perform perimortem cesarean delivery in cases of maternal cardiac arrest
Rosen 5th, arises extremely infrequently. The time to delivery from the onset of maternal arrest
Chapter 31, was found to be critical to fetal survival with good neurologic outcome. Excellent
p. 264 outcomes were reported when delivery took place within 5 min of maternal death.
Survival was unlikely if delivery occurred after 20 min of maternal arrest.
Consideration of perimortem cesarean delivery must be made only after immediate and
optimal advanced maternal cardiopulmonary resuscitative measures have been
instituted. Successful maternal revival following fetal delivery has been reported.
Improved venous return to the central circulation, increased maternal oxygen delivery
following removal of the high uterine demand, and decreased pooling of blood in the
uteroplacental circulation have all been suggested explanations.
178. a The quantitative beta-hCG analysis may be very helpful. It is first detectable as early
Rosen 5th, as 9 to 11 days following ovulation (usually 24 days after the last menstrual period)
Chapter 171, and reaches 200 IU/mL at the expected time of menses.
p. 2404-2405
65

179. A 20-year-old woman is 14 weeks pregnant. She has had vaginal spotting for eight hours. Her
vital signs are normal, including orthostatic readings. Her vaginal vault is normal; her cervical os is
closed. Because she is obese, you cannot estimate uterine size or adnexal structures, but feel that
there is no tenderness. Fetal heart tones are heard with a Doppler stethoscope at a rate of 150 beats
per minute. Her hematocrit is 35%. Optimal management includes:
a. conjugated estrogen 20 mg IM and observation for eight hours.
b. emergent gynecologic consultation to admit the patient for bed rest and fetal monitoring.
c. transvaginal ultrasound followed by fetal monitoring for 12 hours.
d. type and screen, administer Rho (D) Immune Globulin if Rh-negative.
e. discharge home on bed rest with OB follow-up in 24 hours.

180. A 23-year-old G3P2 patient who is 34 weeks by dates presents with acute abdominal pain and heavy
dark vaginal bleeding. Her blood pressure is 78/32 mmHg. When the nurse draws blood and starts
an intravenous line, the patient oozes blood from the IV site. She also has a nosebleed. A DIC
panel will probably show:
a. diminished fibrin split products.
b. negative d-dimer.
c. prolonged prothrombin time.
d. elevated fibrinogen levels.
e. normal platelets.

181. A 28-year-old female insulin-dependant diabetic who is 16 weeks pregnant complains of 2 days of
vomiting, shortness of breath, and high blood sugar. She has no known allergies. Her examination
shows oral temperature 100.0oF, heart rate 130 beats / minute, respiratory rate 28 / minute, and
blood pressure 100/70 mmHg. Her breath smells like fingernail polish, and her urine dips strongly
positive for ketones. Her bedside fingerstick glucose is 287 mg/dl. You suspect diabetic
ketoacidosis, knowing that in pregnancy:
a. DKA may develop at lower glucose values.
b. hyperemesis is seldom a precipitant.
c. maternal insulin and counter-regulatory hormones cross the placenta.
d. it is more frequently seen in patients with gestational diabetes than in those with juvenile
diabetes.
e. serum pH may be deceptively low.

182. A 15-year-old female presents by ambulance, complaining of sudden abdominal pain. She is pale
and diaphoretic. She is initially afebrile with heart rate 140 beats / minute, respiratory rate 20 /
minute, and blood pressure 60/0 mmHg. Her lungs are clear, and her heart is tachycardic. Her
abdomen is soft with bowel sounds present and no masses palpable. Her stool is brown stool and
negative for blood. Pelvic exam shows blood in the vault. She has a nonfocal neurologic exam, but
she loses consciousness as you examine her. Bedside glucose is 180 mg/dl. You assure the ABCs
and start two large bore antecubital intravenous lines. She does not respond to 0.8 mg of naloxone.
Your next step is:
a. call general surgeon, transfer to OR with probable ruptured appendix.
b. call obstetrics for emergent transfer to OR, type and screen.
c. send patient to radiology for abdominal and vaginal ultrasound.
d. await results of serum pregnancy and quantitative beta-hCG.
e. stat CT scan of abdomen and pelvis.
66

179. d In a woman with first trimester pregnancy and vaginal bleeding, you must rule out
th
Rosen 5 , ectopic pregnancy and threatened miscarriage. A transvaginal ultrasound is not
Chapter 171, indicated because the presence of fetal heart tones confirms the dates. Emergent
p. 2403-2406 obstetric consultation is not indicated because she is hemodynamically stable.
Chapter 172, Estrogens are not appropriate in pregnancy. It is important to assess the woman's Rh
p. 2417-2419 status and give RhoGAM if appropriate.
180. c MOST USEFUL HELPFUL
Rosen 5th, Prothrombin time Prolonged aPTT Usually prolonged
Chapter 116, Platelet count Usually low Thrombin clot time Prolonged
p.1698, Fibrinogen level Low Fragmented RBCs Should be present
Table 116-3 Feibrin split products Elevated
D-dimers Elevated
181. a About 10% of insulin-dependent diabetics will develop ketoacidosis at some point
Rosen 5th, during pregnancy. DKA occurs more rapidly and at lower glucose levels in pregnant
Chapter 173, patients as compared with nonpregnant patients. Hyperemesis and noncompliance or
p. 2440-2441 errors in insulin dosage are the most common precipitants. Admission is often
indicated to correct dehydration and more carefully adjust glucose control. The serum
pH may be deceptively normal in the pregnant patient, because the initial pH tends to
be higher in pregnancy due to physiologic hyperventilation. DKA is rare in patients
with gestational diabetes, and insulin and counter-regulatory hormones do not cross the
placenta.
182. b Unstable patients suspected of ectopic pregnancy should receive resuscitation, urgent
Rosen 5th, consultation, and operative intervention. Surgery may be both diagnostic and
Chapter 172, therapeutic if an EP is found or may reveal another cause for the patients condition.
p. 2416-2419 When bedside ED sonography is available, it may be valuable even in unstable patients
if it does not interfere with resuscitation, consultation, and rapid transfer to the
operating room.
67

183. Concerning the diagnosis of appendicitis during pregnancy:


a. diagnosis of appendicitis is delayed because x-rays are not taken.
b. positive urinary WBC esterase and bacterial nitrite are usually seen in appendicitis.
c. the appendix rotates from McBurney's point as pregnancy progresses, ultimately lying near the
midline in the upper abdomen.
d. the incidence of appendicitis is increased in pregnancy.
e. the incidence of perforated appendix is increased in pregnancy.

184. A woman who is 30-weeks pregnant is involved in a motor vehicle crash and sustains a spiral
fracture of the right humerus. She is hemodynamically stable and has an otherwise normal physical
exam. In addition to treating her orthopedic injury, you know she needs:
a. abdominal CT scan
b. ECG
c. fetal monitoring
d. urine pregnancy test
e. urine toxicology screen

185. A 26-year-old woman has dysuria for one week. She denies fever, nausea, vomiting, and back pain.
She does admit to having a new sexual partner over the past month. Her pelvic examination shows
a small amount of whitish discharge from the cervical os but is otherwise normal. Urinalysis shows
10 WBCs, 0 RBCs, no epithelial cells, and no bacteria. Her most likely diagnosis is:
a. pyelonephritis secondary to E. coli infection.
b. cystitis secondary to E. coli infection.
c. urethritis secondary to Chlamydia infection.
d. cervicitis secondary to gonorrheal infection.
e. vaginitis secondary to candida infection.

186. You are using serial quantitative beta-hCG tests to assess the health of a fetus in a patient with first
trimester threatened abortion. You know that in a healthy pregnancy with a normal fetus the level:
a. doubles every two to three days.
b. doubles every seven days.
c. triples every two to three days.
d. triples every seven days.
e. will not change by more than 10% during the first 12 weeks of pregnancy.

187. You are arranging transportation for a 23-year-old woman who fell down some stairs and fractured
her pelvis. She is 28 weeks pregnant by bedside ultrasound. Her blood pressure is 64 palpable.
The position best for her to travel is:
a. 60o Trendelenberg.
b. tilted 15o to the left.
c. tilted 45o to the right.
d. with her head elevated 45o.
e. lying on her right side.

188. The most common manifestation of gonococcal infection in children is:


a. salpingitis.
b. cervicitis.
c. pharyngitis.
d. vaginitis.
e. conjunctivitis.
68

183. e Appendicitis is the most common surgical emergency in pregnancy. Appendectomy is


th
Rosen 5 , one of the most common laparoscopic procedures performed during pregnancy, second
Chapter 172, only to cholecystectomy. The incidence of appendicitis in pregnancy is unchanged
p. 2425-2427 from the nonpregnant state, but delays in diagnosis contribute to an increased rate of
perforation. During the first half of pregnancy, diagnostic findings are usually similar
to those in the nonpregnant female, but the clinical picture becomes less classic during
the second half. During pregnancy the appendix is displaced counterclockwise out of
the right lower quadrant so that by the last trimester, the appendix is located deep in the
right upper quadrant, superior to the iliac crest. Proximity to the ureter, and during
later pregnancy to the kidney itself, leads to an increased incidence of sterile pyuria in
the pregnant patient who has appendicitis, further confounding the diagnosis.
184. c Fetal evaluation in the secondary survey focuses on the fetal heart rate and noting any
Rosen 5th, fetal movement. Once the presence of fetal heart tones has been confirmed,
Chapter 31, intermittent monitoring of fetal heart rate is sufficient for the pre-viable fetus. If the
p. 262-264 fetus is viable (i.e., 24 weeks or more), continuous external monitoring should be
initiated quickly and maintained throughout all diagnostic and therapeutic procedures.
Such monitoring can also benefit the mother because fetal hemodynamics are more
sensitive to decreases in maternal blood flow and oxygenation than are most maternal
measures. Fetal distress can be a sign of occult maternal distress. Signs of fetal
distress include an abnormal baseline rate, decreased variability of heart rate, and fetal
decelerations after contractions.
185. c Chlamydial urethritis may be present in up to 20% of women with dysuria. The
Rosen 5th, history of a new sexual partner, the gradual onset of symptoms, the discharge from the
Chapter 94, os, and the sterile pyuria are all consistent with chlamydial urethritis. The lack of
p. 1395-1396 fever, vomiting, and back pain make pyelonephritis less likely.
186. a Serum beta-HCG levels normally double every 1.8 to 3 days for the first 6 to 7 weeks
Rosen 5th, of pregnancy, beginning 8 to 9 days after ovulation.
Chapter 172,
p. 2418
187. b For pregnant patients beyond 20 weeks of gestation who must be transported in the
th
Rosen 5 , supine position or in whom spinal immobilization is indicated, a wedge should be
Chapter 31, placed under the right hip area tilting the patient toward her left side to avoid
p. 256 & 264 hypotension from inferior vena cava compression by the gravid uterus.
188. d Vaginitis is the most common gonococcal infection in children. The thin, friable
th
Rosen 5 , genital mucosa of the prepubescent girl is susceptible to gonococcal infection, resulting
Chapter 93, in copious, purulent vaginal discharge. Obtain specimens from the child at the vaginal
p. 1395 introitus. Vaginitis, scalp infection, bacteremia, arthritis, meningitis, and endocarditis
are manifestations of gonorrhea in the newborn.
69

189. HELLP stands for:


a. hepatitis elevated lipids low platelets
b. hemolysis elevated lipids low platelets
c. hepatitis elevated liver enzymes low platelets
d. hemolysis elevated liver enzymes low platelets
e. herpes extrophic labia lumbar pernio

190. In a patient with genital herpes:


a. HSV-1 accounts for up to 50% of the cases of genital herpes.
b. systemic acyclovir decreases the frequency of recurrences.
c. systemic symptoms are common with the initial presentation of genital herpes.
d. Tzanck smears are positive in the majority of cases.
e. cultures of fluid obtained from herpes vesicles are positive only a third of the time.

191. In the patient with sinusitis:


a. CT scans of the sinuses are the "gold standard" for diagnosing the disease.
b. ethmoid sinusitis pain is made worse when the patient sits up.
c. maxillary sinusitis pain is made worse by placing the patient supine.
d. phenylephrine or oxymetazoline decongestants should be avoided, as they can irritate the nasal
mucosa.
e. the Water's view is most helpful in finding ethmoid sinusitis.

192. The most common agent causing infectious pharyngitis in an adult is:
a. Chlamydia trachomatis.
b. group A beta-hemolytic streptococcus.
c. Neisseria gonorrhoeae.
d. viral.
e. pneumococcus.

193. A 65-year-old woman has swelling at the base of her tongue and a "bull neck". She is an insulin-
dependent diabetic. You find brawny induration of the floor of the mouth. She is ill appearing but
is not in any respiratory distress. You should:
a. incise and drain the area of swelling.
b. admit the patient to the ICU for close monitoring of possible airway obstruction.
c. start IV antibiotics in the ED and admit her to the general medical floor.
d. discharge home on outpatient antibiotics with ENT follow-up the next day
e. consult a dentist for probable tooth extraction.

194. Mnires disease is characterized by vertigo and:


a. fever.
b. hearing loss.
c. headache.
d. blurred vision.
e. facial nerve paralysis.

195. Concerning nosebleeds:


a. anterior epistaxis accounts for 75% of nosebleeds.
b. anterior epistaxis requires immediate consultation with an otolaryngologist.
c. anterior epistaxis usually originates from the turbinates of the lateral wall of the nasal cavity.
d. posterior epistaxis is less likely to require hospitalization than anterior epistaxis.
e. posterior epistaxis usually occurs from the posterior branch of the sphenopalatine artery.
70

189. d The HELLP syndrome (an acronym for hemolysis, elevated liver enzymes, and low
th
Rosen 5 , platelets) is an important clinical variant of preeclampsia that has a predilection for the
Chapter 172, multigravid patient, in contrast to the primigravida, in whom preeclampsia is more
p. 2422-2423 common. In the HELLP syndrome, the blood pressure is variable and may not be
elevated initially. This fact, combined with the usual complaint of epigastric or right
upper quadrant pain, makes it easy to mistake the HELLP syndrome for other causes of
abdominal pain, such as gastroenteritis, hepatitis, pancreatitis, cholecystitis, or
pyelonephritis.
190. c Overall, 85 to 90% of genital herpes infections are caused by HSV-2. There may be
Rosen 5th, both local and systemic manifestations. Cultures are positive 85 to 95% of the time.
Chapter 93, Usually, initial infection is more severe and lasts longer than do subsequent
p. 1392 recurrences. A Tzanck smear stained with either Wright or Giemsa stain is positive in
up to 50% of cases. Systemic antiviral agents provide partial control of the signs and
symptoms and accelerate healing of the lesions, but do not affect the frequency or
severity of recurrences.
191. a The best method for imaging the sinuses is to perform a CT. The Waters view is a
Rosen 5th, plain x-ray that best shows the maxillary sinuses and can provide good views of the
Chapter 70, frontal sinuses. Maxillary sinusitis is usually made worse when the patient leans
p. 981-984 forward, while assuming the supine position exacerbates ethmoid sinusitis.
Decongestants are an important part of treatment, allowing drainage of the sinuses.
Prolonged use of these agents can cause a rebound inflammation on cessation.
192. d Causal agents of pharyngitis include viruses, bacteria, fungi, and parasites. Most often,
Rosen 5th, viruses are the culprits. Rhinovirus and adenovirus are the most common, but Epstein-
Chapter 70, Barr virus, herpes simplex virus, influenzavirus, parainfluenzavirus, and coronavirus
p. 969-971. are responsible for about 5% of the infections.
193. b Cellulitis of bilateral submandibular spaces and the lingual space is called Ludwigs
Rosen 5th, angina and is potentially life threatening. It is a rapidly spreading cellulitis that results
Chapter 65, in brawny induration of the suprahyoid region and elevation of the tongue.
895-896 Involvement of the floor of the mouth pushes the tongue posteriorly. Epiglottic
Chapter 70, involvement is not uncommon. As a result, airway compromise is the immediate
p. 977-978 primary concern. The primary focus of initial management is maintenance of a patent
airway. Timely administration of high- dose penicillin and metronidazole or cefoxitin
is essential. Immediate oral and maxillofacial surgical consultation and hospitalization
for incision and drainage and intubation as indicated are necessary.
194. b With Mnires disease the onset of vertigo is usually sudden, and duration ranges
Rosen 5th, from 20 minutes to 12 hours. It is associated with nausea, vomiting, and diaphoresis.
Chapter 13, The frequency of attacks can vary from several times per week to several times per
p. 125-127 month. Between attacks, the patient is usually well, although deafness may persist.
Other hallmarks of the diagnosis include associated symptoms such as roaring tinnitus,
diminished hearing, and fullness in one ear.
195. e Anterior epistaxis is responsible for 90% of nosebleeds. It usually arises from the
Rosen 5th, anterior-inferior nasal septum from vessels known as Kiesselbach's plexus. Although
Chapter 67, posterior epistaxis requires consultation with an otolaryngologist and hospitalization,
p. 933-935 most anterior bleeds can be managed on an outpatient basis without immediate
consultation of a specialist. The most common site of posterior bleeds is the posterior
branch of the sphenopalatine artery.
71

196. Necrotizing external otitis is almost always caused by:


a. group A beta-hemolytic streptococcus.
b. Staphylococcus aureus.
c. E. coli.
d. Pseudomonas aeruginosa.
e. aspergillus.

197. Central retinal vein occlusion:


a. causes brief, transient blindness or flickering vision.
b. causes a pale retina and optic disc with boxcar segmentation of the retinal veins.
c. is secondary to embolic phenomenon.
d. is treated with eye massage, acetazolamide, timolol, and increasing the inhaled pCO2.
e. has a wide range of clinical appearances.

198. A 4-year-old child has purulent nasal discharge from the left nostril for 4 days. She flails her arms
and shakes her head vigorously when you attempt to examine her, but you notice a bead from a toy
necklace partially occluding the left nostril. You should:
a. send her home on oral antibiotics with assurance to mom that the object will dislodge itself.
b. instill topical vasoconstrictor into the nostril and attempt blind removal with a right angle hook.
c. obtain an X-ray of the nasal bones.
d. under conscious sedation, grasp the object with a suction catheter taking precautions against
pushing the foreign body further back into the nasopharynx.
e. consult ENT for referral to the operating room.

199. An 18-year-old man presents with right ear pain and discharge. Temperature 39oC (102.2oF). You
find a perforated right tympanic membrane with purulent drainage, and a tender swelling over the
mastoid area. The most appropriate treatment is:
a. admit, culture drainage, start IV antibiotics, obtain urgent otolaryngology consultation.
b. admit patient on antibiotic ear drops with routine consult otolaryngology.
c. discharge on oral antibiotics to follow-up with an otolaryngologist in one week.
d. incise and drain mastoid area, then discharge on oral antibiotics.
e. topical sulfacetamide drops for 7 days.

200. A 32-year-old woman is struck in the left eye with a tennis ball. She complains of pain but denies
flashes of light, floaters, diplopia, or a decrease in vision. On initial evaluation her visual acuity is
20/20 vision in each eye, she has orbital emphysema, and her extraocular muscles and globe are
intact. Shortly after arrival she complains of suddenly decreased visual acuity. You should now:
a. ballotte the globe in an attempt to dislodge the clot causing the central retinal artery occlusion.
b. intraorbital needle aspiration or lateral canthotomy with cantholysis to release pressure under the
orbit.
c. ophthalmologic consult for traumatic retinal tear with vitreous hemorrhage.
d. topical cycloplegics (5% homatropine) to the affected eye for treatment of traumatic iridocyclitis
with an ophthalmologic follow-up.
e. anterior chamber paracentesis.
72

196. d Previously known as malignant otitis externa because of its high mortality, necrotizing
th
Rosen 5 , external otitis is an extremely aggressive form of otitis externa. It occurs primarily in
Chapter 67, adults with diabetes mellitus but has also been seen rarely in immunocompromised
p. 932 children. Pseudomonas is the predominant pathogen, but S. aureus, S. epidermidis,
Proteus mirabilis, Klebsiella, Aspergillus, and Salmonella have all been described.
197. e Central retinal vein occlusion (CRVO) symptoms are similar to those of central retinal
Rosen 5th, artery occlusion (CRAO) in that there is a painless loss of vision. Brief transient
Chapter 66, blindness is uncommon. A pale retina and optic disc with boxcar segmentation are
p. 921 seen in CRAO. The degree of vision loss depends on the degree of ischemia and
ranges from mild to severe. CRAO results from embolic phenomenon, but not CRVO.
No treatment is necessary acutely for CRVO other than watching for elevation of
intraocular pressure consistent with neovascular glaucoma. Eye massage,
acetazolamide, timolol, and increasing the PCO2 are all indicated treatments for
CRAO.
198. d With conscious sedation, a majority of nasal foreign bodies can be removed. If
Rosen 5th, attempts are unsuccessful, the patient should follow-up with an otolaryngologist within
Chapter 53, one to two days.
p. 756-757
199. a Mastoiditis requires admission for antibiotic therapy. Antibiotic choices include
th
Rosen 5 , semisynthetic penicillin combined with chloramphenicol, or a third-generation
Chapter 67, cephalosporin such as cefuroxime (50 to 150 mg/kg/day), or ceftriaxone (50 to 75
p. 932-933 mg/kg/day), usually for 1 week. Surgical procedures may range from myringotomy
drainage and tympanostomy tube placement to mastoidectomy and drainage for more
extensive disease progression. Mastoidectomy is required in approximately half of
mastoiditis cases.
200. b Orbital emphysema associated with orbital fractures is usually a benign, self-limited
Rosen 5th, condition. This patient however, is complaining of a sudden decrease in visual acuity
Chapter 66, in the traumatized eye. The emergency physician must consider that air may have built
p. 909 up under pressure in the orbit, causing cessation of blood flow in the central retinal
artery. The air must be released immediately or the patient may lose her vision. This
is done by performing a lateral canthotomy with cantholysis or intraorbital needle
aspiration of the trapped air. Ballottement of the globe will not benefit this patient
because the etiology of her visual loss is not embolic occlusion of the retinal artery.
Iridocyclitis following trauma is painful but should not be associated with loss of
visual acuity.
73

201. The most common complication of acute otitis media is:


a. brain abscess.
b. facial nerve paralysis.
c. hearing impairment.
d. labyrinthitis.
e. meningitis.

202. In a patient with suspected retropharyngeal abscess, the most appropriate initial
diagnostic study is:
a. magnetic resonance angiography of the neck.
b. direct laryngoscopy.
c. indirect laryngoscopy.
d. soft tissue lateral x-ray.
e. barium swallow.

203. A 23-year-old woman noticed that her right eye was red as she was putting on her make-
up. She has no pain and denies any trauma. Her past medical history is unremarkable.
Her visual acuity is 20/20 in each eye. Her pupils are reactive, and the extraocular
muscles are intact. There are several patches of bright red blood scattered across the
sclera. Funduscopic exam is normal. You know that this patient needs:
a. reassurance.
b. outpatient workup for von Willebrands disease.
c. STAT ophthalmologic referral.
d. inpatient stay for trabeculoplasty.
e. referral to a hematologist.

204. A 15-year-old boy was punched in the right eye during a brawl at a local basketball court.
He complains of photophobia and a dull ache in the injured eye. Physical examination
shows visual acuity of 20/80 that corrects to 20/40 with a pinhole. The right pupil is dilated
and sluggishly reactive, but extraocular movements are intact. There is scleral and ciliary
injection. Slit lamp exam shows moderate cell and flare in the anterior chamber. He has:
a. traumatic cranial nerve III palsy.
b. conjunctivitis with traumatic lens dislocation.
c. conjunctivitis with iris sphincter rupture.
d. conjunctivitis with ruptured globe.
e. traumatic iridocyclitis.

205. The 39-year-old woman shown in this picture was struck in the eye while playing
racquetball. She complains of mild pain and
decreased vision. Her visual acuity is 20/100 with
pinhole. Her eye is shown. You need to treat her
with:
a. bed rest, head elevation, avoid reading.
b. pilocarpine 1% eye drops every 4 hours for 48
hours.
c. fresh frozen plasma.
d. aminocaproic acid. Rosen 5th Edition, Figure 66-9
e. surgical wash out of the anterior chamber.
74

201. c Almost all children with otitis media will have temporary conductive hearing loss.
th
Rosen 5 , Sensorineural hearing loss is less common, but may contribute to the association of otitis media
Chapter 67, p. with delayed speech, language, or cognitive development. Labyrinthitis, facial nerve paralysis,
930 meningitis, and extradural abscesses are also complications, but are far less common,
particularly in the postantibiotic era.
202. d Radiographs and CT scans are helpful diagnostic tools in assessing a patient for retropharyngeal
Rosen 5th, abscess. A lateral soft-tissue radiograph of the neck taken during inspiration with moderate
Chapter 70, p. cervical extension demonstrates thickening and protrusion of the retropharyngeal wall.
979
203. a The fragile conjunctival vessels can rupture from trauma, sudden Valsalva pressure spikes
th
Rosen 5 , (sneezing, coughing, vomiting, straining), hypertension, or spontaneously with no discernible
Chapter 66, p. etiology. No treatment is necessary, and the hemorrhage usually resolves within 2 weeks. If
912 multiple recurrent episodes occur, coagulation studies and further investigation are warranted.

204. e Blunt injury of the globe may contuse and inflame the iris and ciliary body, resulting in ciliary
th
Rosen 5 , spasm. Patients complain of photophobia and deep aching eye pain. Examination reveals
Chapter 66, p. perilimbal conjunctival injection (ciliary flush), cells and flare in the anterior chamber
913 (representing white blood cells and protein), and a small, poorly dilating pupil.

205. a Management of hyphema must be individualized for a given patient. Selected low-grade
th
Rosen 5 , hyphemas in reliable patients may be managed on an outpatient basis. General therapy
Chapter 66, p. includes elevating the bed 30 to 45 degrees, bed rest, and limiting eye movements such as
912-913 reading. Specific treatment for hyphema with miotics, mydriatics, cycloplegics, steroids, and
antifibrinolytics (such as aminocaproic acid) will vary depending on the specific clinical
situation, and is best left to the ophthalmologist.
75

206. Regarding acute traumatic retinal detachment:


a. most detachments can be visualized on standard funduscopy.
b. examination may reveal the typical blood and thunder retinal bleeding pattern.
c. the patient usually reports flashing lights which are due to retinal neuron stimulation.
d. visual outcome depends on the extent of involvement of the optic disc.
e. sudden onset of pain is a prominent feature.

207. A 62-year-old female with a history of hypertension, diabetes, and glaucoma complains of
sudden, painless loss of vision in her right eye. Visual acuity shows only hand-motion
recognition on the affected side. There is an afferent papillary defect and absent red reflex
on the right. You are unable to visualize the fundus. Slit lamp examination is normal. Her
most likely diagnosis is:
a. acute open-angle glaucoma.
b. central retinal artery occlusion.
c. central retinal vein occlusion.
d. optic neuritis.
e. vitreous hemorrhage.

208. The patient in question #207 is best treated with:


a. anterior chamber paracentesis.
b. expectant management with delayed phototherapy.
c. intravenous prednisolone.
d. ocular massage.
e. pilocarpine, intravenous acetazolamide, and urgent laser iridectomy.

209. A 19-year-old man was assaulted and robbed outside the baseball stadium after bat day.
He has an obvious mid-face fracture and unstable mandible. His left eye is mildly
proptotic with severe conjunctival swelling and a large subconjunctival hemorrhage.
Visual acuity is limited to counting fingers. His pupil is fixed and mid-point. You must now:
a. begin therapy with mydriatics and cycloplegics.
b. avoid sedation and analgesia, so as not to mask intracranial injuries.
c. begin therapy with local fibrinolytic injection.
d. apply a firm occlusive eye patch and arrange outpatient follow-up.
e. perform emergent lateral canthotomy.

210. A 75-year-old woman with diabetes and hypertension complains of abrupt onset of right
eye pain, blurred vision, unilateral headache, and mild nausea. Her visual acuity is 20/200
in the affected eye, which does not correct with pinhole. She has conjunctival injection
and a cloudy, edematous cornea. Her pupil is mid-point and nonreactive to light. Slit lamp
examination shows no corneal staining. You should now:
a. order erythrocyte sedimentation rate (ESR).
b. dilate her pupil and perform direct funduscopy.
c. order orbital CT with 3-mm cuts.
d. perform tonometry.
e. order an intraocular ultrasound.
76

206. c Retinal tears and detachments from blunt eye trauma are common. Symptoms include floaters
th
Rosen 5 , from bleeding, flashing lights from stimulation of retinal neurons, and visual field cuts or
Chapter 66, p. decreased visual acuity. Retinal tears or detachments do not cause pain. Examination may
914 reveal the hazy gray membrane of the retina billowing forward, but many tears are located
peripherally and cannot be seen with direct ophthalmoscopy. Visual acuity will be normal
unless the macula is involved.
207. e Vitreous hemorrhage results from bleeding into the preretinal space or vitreous cavity. The
Rosen 5th, most common causes are diabetic retinopathy and retinal tears. Symptoms begin as floaters or
Chapter 66, p. cobwebs in the vision and may progress to severe painless vision loss over a few hours.
922 Direct ophthalmoscopy reveals reddish haze in mild cases to a black reflex in severe cases.
Vitreous hemorrhage by itself will not cause an afferent pupillary defect, but this finding will be
208. b present if there is a retinal detachment behind the vitreous hemorrhage. Initial therapy consists
Rosen 5th, of bed rest with elevation of the head of the bed. Vascular retinopathy if treated with laser
Chapter 66, p. photocoagulopathy or cryotherapy.
922
209. e The patient has a retrobulbar hemorrhage which is impinging on the optic nerve and retinal
th
Rosen 5 , circulation. Immediate ophthalmologic consultation is warranted. If intraocular pressure is
Chapter 66, p. increased, begin treatment with carbonic anhydrase inhibitors, topical beta blockers, and
910 intravenous mannitol. A lateral canthotomy can be done in the ED as a temporizing measure
before definitive decompression.
210. d Attacks of primary angle closure glaucoma are precipitated by pupillary dilatation, which
Rosen 5th, increases the degree of pupillary block and leads to an accumulation of aqueous humor in the
Chapter 66, p. posterior chamber. Symptoms are abrupt in onset and include severe eye pain, blurred vision,
919-920 headache, nausea, vomiting, and occasionally abdominal pain. Lab studies are not necessary,
and dilating the eye with a mydriatic agent will make the problem even worse.
77

211. The most appropriate initial therapy for the patient in question #210 is:
a. parenteral steroid therapy.
b. anterior chamber paracentesis.
c. beta-blocker ophthalmologic drops.
d. tropicamide ophthalmologic drops.
e. phenylephrine ophthalmologic drops.

212. A 73-year-old man has fevers to 103oF, with a stiff neck, headache, and slurred speech.
Your exam shows a sleepy patient who arouses to sternal rub. His right hand strength is
weak. His wife mentions that he had an endoscopic sinus polypectomy three days ago.
a. The patient almost certainly has meningitis; he requires an immediate lumbar puncture.
b. The patient almost certainly has encephalitis; he requires an immediate lumbar
puncture.
c. Since most strokes are not associated with alterations in mental status, this patient
requires an immediate lumbar puncture.
d. The patient probably has a brain abscess and will require neurosurgery.
e. He requires immediate treatment with steroids, mannitol, and antibiotics.

213. A complication from using phenobarbital to control status epilepticus is:


a. an anion gap metabolic acidosis.
b. a prolonged QT segment.
c. paradoxical elevation of blood pressure.
d. poor tolerance in pediatric patients.
e. prolonged obtundation.

214. You are evaluating a 29 year-old man with a history of AIDS and HIV encephalopathy for
deterioration in his level of function. He complains of a headache, but his vital signs are
normal. He is lethargic but easily awakens when his name is called. Appropriate
management should include:
a. routine laboratory blood work; if the cell count and chemistries are normal, he can go
home.
b. brain computerized tomography (CT) scan without contrast; if normal he can go home.
c. discharge home after advising family that HIV encephalopathy is a progressive disease
and nothing can be done to alter its course.
d. brain computerized tomography (CT) scan with contrast, lumbar puncture, admit to
hospital.
e. admit on high-dose steroid therapy.

215. Which statement concerning myasthenia gravis is true?


a. Most patients have general weakness, especially of the distal extremity small muscle
groups.
b. Most patients have general weakness, especially of the neck flexors.
c. Ptosis and diplopia are the most common presenting symptoms.
d. Symptoms can fluctuate throughout the day, usually improving toward evening.
e. Sensory, reflex, or cerebellar abnormalities are also common.

216. Guillain-Barr syndrome:


a. is an autoimmune disorder of the neuromuscular junction.
b. causes impaired cough reflex, leading to frequent aspiration.
c. leads to respiratory complications in 50% of patients.
78

d. can cause respiratory muscle compromise even with mild peripheral muscle
involvement.
e. is easily treated with steroids.
79

211. c Treatment for acute glaucoma involves a three-pronged attack: block aqueous production
th
Rosen 5 , (topical beta-adrenergic antagonist, oral carbonic anhydrase inhibitor, topical alpha-2 agonist),
Chapter 66, reduce vitreous volume (systemic hyperosmotic agent such as oral glycerol, oral isosorbide, or
p. 920 intravenous mannitol), and facilitate aqueous outflow (pilocarpine drops to pull the iris from the
iridocorneal angle). Topically administered timolol 0.5% decreases intraocular pressure in 30
to 60 minutes.
212. d CNS abscesses may occur at any age and any time of the year, and are more common in men
Rosen 5th, than in women. CNS abscesses are associated with both local contiguous and remote systemic
Chapter 103, infections, intravenous drug use, neurologic surgery, and cranial trauma. Brain abscess
p. 1528 secondary to otitis media most often presents in pediatric or older adult populations. When
associated with sinusitis, it most often presents in young adults.
213. e Anticonvulsant therapy with phenobarbital is derived primarily from studies in the pediatric
Rosen 5th, populations. Phenobarbital is a CNS depressant that decreases both ictal and physiologic
Chapter 96, p. cortical activity. Sedation and depression of the respiratory drive must be anticipated,
1453 especially if this drug is combined with a benzodiazepine.

214. d Cryptococcus neoformans may be seen in up to 10% of patients with AIDS and may cause
Rosen 5th, either focal cerebral lesions or diffuse meningoencephalitis. The most common initial
Chapter 126, symptoms are fever and headache. The diagnosis depends on identifying organisms in the CSF.
p. 1849 Cryptococcal antigen in the CSF is nearly 100% sensitive and specific; less definitive are India
ink staining (60 to 80% sensitive), fungal culture (95% sensitive), and serum cryptococcal
antigen (95% sensitive). Treatment involves admission for intravenous amphotericin or other
anti-fungals.
215. c Most myasthenia gravis patients have general weakness, especially of the proximal extremities
Rosen 5th, muscle groups, neck extensors, and facial or bulbar muscles. Ptosis and diplopia are the most
Chapter 102, common presenting symptoms, but oropharyngeal symptoms (dysphagia and dysarthria) and
p. 1523 limb weakness also can be seen. These symptoms can fluctuate throughout the day, usually
worsening as the day progresses.
216. d In most cases, GBS is caused by an autoimmune attack on myelinated motor nerves.
Rosen 5th, Sometimes the patient first notices numbness and tingling of the lower extremities, followed by
Chapter 101, weakness of thighs, legs, and then arms. In classic cases, there is symmetric extremity
p. 1508-1509 weakness, more pronounced in the legs. Despite subjective sensory disturbances, sensation on
examination is usually normal. The hallmark finding in GBS is the lack of deep tendon
reflexes. There may be a marked limb ataxia, and the patient may not be able to walk or stand
despite reasonable strength. In all forms of the disease, there is a chance of developing
respiratory failure and lethal autonomic fluctuations.
80

217. A 4-year-old boy has acute fever, severe headache, and projectile vomiting. He is
stuporous and resists neck flexion. On physical exam you find papilledema and a
petechial rash. Suspecting meningococcal disease, you prioritize your workup and
treatment in this order:
a. LP CT head antibiotics steroids
b. LP antibiotics CT head steroids
c. steroids antibiotics CT head LP
d. CT head LP antibiotics steroids
e. antibiotics steroids CT head LP

218. A 23-year-old woman with AIDS is brought by her mother after she
has a seizure. She is waking up, but complains of a headache.
Prior to performing a spinal tap, you do a head CT which looks like
this. Your next step is to:
a. start anti-parasitic therapy, such as pyrimethamine plus
sulfadiazine (Daraprin).
b. start anti-fungal therapy, such as fluconazole or amphotericin.
c. start broad-spectrum antibiotic coverage with a third-generation
fluoroquinolone.
d. arrange for imaging-guided drainage of the abscess.
e. arrange for ventriculoperitoneal shunt placement.

219. A 50-year-old woman complains of fatigue, cold intolerance, and weight gain.
Temperature 94.5oF, heart rate 52 beats / minute, blood pressure 82/38 mmHg. She has
dry skin, generalized edema, and diminished reflxes. Her most immediate life threats
include:
a. metabolic acidosis, hyperglycemia, and hyperkalemia.
b. hepatorenal syndrome.
c. respiratory insufficiency, hypotension, and hypoglycemia.
d. myocardial infarction and stroke.
e. hypertension and intracranial hemorrhage.

220. A 42-year-old woman with adult T-cell lymphoma-leukemia complains of back pain,
abdominal pain, and confusion. Laboratory evaluation shows a total calcium of 15.8
mg/dl. Appropriate management of this patient should include:
a. plasmapheresis.
b. intravenous bicarbonate.
c. intravenous hypertonic saline and oral Kayexalate.
d. intravenous normal saline and intravenous furosemide.
e. glucagon.

221. A 25-year-old insulin dependent diabetic complains of nausea, vomiting and abdominal
pain for two days. His bedside glucose is 565, and his serum bicarbonate is 8 mEq. The
anion gap is 34. Your initial treatment should include:
a. insulin by intravenous drip.
b. intravenous bicarbonate until serum pH is normalized.
c. adequate phosphorus repletion in the form of potassium phosphate.
d. judicious intravenous fluids to avoid volume overload.
e. glucagon by intravenous drip.
81

217. e If a patient's presentation is an acute, fulminating, febrile illness and bacterial meningitis is the
th
Rosen 5 , concerning diagnosis, early initiation of antimicrobial therapy is mandatory. The role of
Chapter 103, dexamethasone therapy for acute bacterial meningitis has long been the focus of clinical
p. 2349 interest, and considerable controversy exists among individuals who view the same data
differently. Consensus opinion suggests that for infants beyond 8 weeks old, dexamethasone
may improve some neurologic sequelae, particularly hearing loss with meningitis cause by H.
influenzae. Beneficial effect, if any, of dexamethasone on morbidity and mortality caused by
other bacteria is less well defined.
218. a Toxoplasma gondii is the most common cause of focal encephalitis in patients with AIDS.
Rosen 5th, Common symptoms include fever, headache, altered mental status, and seizures. Serologic
Chapter 126, testing is not useful as the antibody is present in the general population. Diagnosis of
p. 1849 toxoplasmosis is most often made by the presence of multiple subcortical lesions on CT scan.
Noncontrast CT is the initial study of choice, as addition of contrast has been shown to be of
marginal value in patients with completely normal noncontrast CT scans. In the presence of
contrast, toxoplasmosis lesions are ring-enhancing with surrounding areas of edema.
219. c Hypoventilation and hypoglycemia are the two immediately serious metabolic abnormalities of
Rosen 5th, myxedema. ABGs may be the only indication that significant hypercapnea and respiratory
Chapter 122, acidosis exist. An elevated serum carbon dioxide partial pressure may be seen in nearly one
p. 1778 third of patients with myxedema coma, and ventilator support can immediately reverse this
cause of coma. Serious hypoglycemia is unusual and is less characteristic of primary
hypothyroidism than of secondary hypothyroidism. If present, hypoglycemia can contribute to
coma, although seizures may be a more likely outcome. Patients should receive 5% dextrose in
water (D5W), and serum glucose should be monitored.
220. d Patients with severe hypercalcemia (>14 mg/dl) require immediate treatment regardless of
Rosen 5th, symptoms. The four basic goals of therapy are (1) restore intravascular volume, (2) enhance
Chapter 119, renal calcium elimination, (3) reduce osteoclastic activity, and (4) treat primary disorder.
p. 1735 Isotonic saline is the first step. Once volume has been restored, the calcium will usually have
decreased by 1.6 to 2.4 mg/dl, but hydration alone rarely leads to complete normalization.
Loop diuretics such as furosemide inhibit the resorption of calcium in the thick ascending loop
of Henle, increasing the calciuric effect of hydration. Volume expansion must precede the
administration of furosemide, however, because the drugs effect depends on the delivery of
calcium to the distal nephron.
221. a Rapid fluid administration is the single most important initial step in the treatment of DKA.
Rosen 5th, Fluid restores intravascular volume and normal tonicity, perfuse vital organs, improves
Chapter 120, glomerular filtration rate (GFR), and lowers serum glucose and ketones. The average adult
p. 1753 patient has a water deficit of 100 mL/kg (5 to 10 L) and a sodium deficit of 7 to 10 mEq/kg. It
is generally accepted that the ideal way to administer insulin is by continuous infusion of
small doses of regular insulin through an infusion pump. This approach appears to be more
physiologic, helps produce a more linear fall in serum glucose and ketone bodies, and is
associated with less-severe metabolic complications (hypoglycemia, hypokalemia, and
hypophosphatemia).
82

222. Euvolemic hyponatremia is found in patients with:


a. diabetes insipidus.
b. vomiting.
c. laxative abuse.
d. SIADH.
e. decreased water intake.

223. A 27-year-old alcoholic with insulin dependent diabetes is lethargic and vomiting. He
smells heavily of alcohol. Labs show Na+ 143, K+ 4.2, Cl- 107, HC03 18, glucose 305. His
serum ethanol level is 0.128 (128 mg%). His urine dips positive for ketones. What is the
likely cause of his ketoacidosis?
a. alcoholic ketoacidosis, anion gap 36
b. diabetic ketoacidosis, anion gap 18
c. alcoholic ketoacidosis, anion gap 18
d. diabetic ketoacidosis, anion gap 36
e. diabetic ketoacidosis, anion gap 22

224. A common finding in myxedema is:


a. hypocalcemia.
b. hyponatremia.
c. hyperglycemia.
d. hypocholesterolemia.
e. hypokalemia.

225. A 48-year-old woman with a history of Graves disease complains of nervousness and
palpitations. Temperature 39.5oC; pulse 150 / minute; respirations 36 / minute; blood
pressure 150/70 mmHg. She is thin and very anxious. The first medication you should
give is:
a. propylthiouracil.
b. dexamethasone.
c. potassium iodide (SSKI).
d. methimazole.
e. propranolol.

226. A 48-year-old woman with a history of rheumatoid arthritis complains of profound


weakness and diffuse abdominal pain. She just returned from a two week cruise and
admits that she forgot to take her prednisone. You suspect adrenal insufficiency and
begin her treatment with:
a. potassium supplementation.
b. rapid volume replacement.
c. intramuscular mineralocorticoid.
d. subcutaneous insulin.
e. colloid volume expanders.

227. A 73-year-old man with a history of noninsulin-dependent diabetes mellitus is being


evaluated for altered mental status. Physical exam is remarkable for signs of severe
dehydration. His serum glucose level is 1283 mg/dl. You also expect to find:
a. BUN : creatinine ratio less than 20:1.
b. markedly elevated serum ketones.
c. elevated troponin.
d. hyponatremia (uncorrected).
83

e. profound acidosis.
84

222. d The many causes of euvolemic hyponatremia include the syndrome of inappropriate secretion
th
Rosen 5 , of ADH (SIADH), defined as the secretion of ADH in the absence of an appropriate
Chapter 119, physiologic stimulus. Hypovolemic hyponatremia results from the loss of water and sodium
p. 1724-1725 with a greater relative loss of sodium. Typical causes include vomiting, diarrhea,
gastrointestinal (GI) suction or drainage tubes, fistulas, and third spacing of fluids (e.g.,
burns, intra-abdominal sepsis, bowel obstruction, pancreatitis).
223. b The difference between the serum sodium (the contribution of potassium, largely an
Rosen 5th, intracellular ion, is usually ignored) and the sum of serum chloride and bicarbonate equals the
Chapter 179, concentration of the unmeasured anions, or the "anion gap." Almost all patients with DKA
p. 2524 present with blood glucose greater than 300 mg/dL, but patients who present just after receiving
insulin or who have impaired gluconeogenesis (e.g., in alcohol abuse or liver failure) may have
lower initial serum glucose levels. In alcoholic ketoacidosis, alcohol levels are usually low or
undetectable at the time of presentation, but some patients may present with an elevated blood
alcohol level, making diagnosis more challenging. Serum glucose levels are usually less than
200 mg/dl.
224. b In the myxedematous patient, hyponatremia occurs often and is usually mild. The mechanism
Rosen 5th, is thought to be syndrome of inappropriate secretion of antidiuretic hormone (SIADH), and
Chapter 122, thyroid replacement therapy reverses the abnormality. Hypoglycemia is unusual and typically
p. 1776, Box mild; its correction usually does not materially affect the clinical symptoms. Hypercalcemia is
122-9 rare, mild when present, and of uncertain cause. Cholesterol levels are typically elevated, are
rarely less than 250 mg/dl, and in 86% of cases are greater than 290 mg/dl. A mild normocytic,
normochromic anemia without reticulocytosis may be present.
225. e Treatment of thyroid storm has five goals: (1) inhibit hormone synthesis, (2) block hormone
Rosen 5th, release, (3) prevent peripheral conversion of T4 to T3, (4) block the peripheral effects of thyroid
Chapter 122 hormone, and (5) provide general support. Blockade of the peripheral adrenergic hyperactivity
p.1774 of thyroid crisis may be the most important factor in reducing morbidity and mortality.

226. b Treatment of adrenal crisis includes replacement of fluids and sodium, administration of
Rosen 5th, glucocorticoid, correction of hypotension and hypoglycemia, reduction of hyperkalemia, and
Chapter 122, identification and treatment of a precipitating cause of the crisis. A rapid infusion of 5%
p. 1781-1782 dextrose and isotonic saline should be started immediately. This acts to correct dehydration,
hypotension, hyponatremia, and hypoglycemia.
227. d Patients in a hyperglycemic hyperosmolar nonketotic coma have a blood glucose level greater
Rosen 5th, than 600 mg/dl and serum osmolarity greater than 350 mOsm/L. The BUN concentration is
Chapter 120, invariably elevated. Although patients with HHNC do not have a ketoacidosis caused by
p. 1755 diabetes, they may have a metabolic acidosis secondary to some combination of lactic acidosis,
starvation ketosis, and retention of inorganic acids attributable to renal hypoperfusion. They
typically manifest more profound electrolyte imbalance than with DKA. Initial serum sodium
readings will be inaccurately low because of hyperglycemia.
85

228. A 25-year-old with hemophilia A has a painful swollen ankle but no history of trauma.
Appropriate treatment includes ice, immobilization, and a Factor VIII infusion of:
a. 12.5 units/kg.
b. 25 units/kg.
c. 40 units/kg.
d. 50 units/kg.
e. 100 units/kg.

229. A 4-year-old boy has fever and headache. Temperature 104F, pulse 168 / minute,
respiratory rate 42 / minute, blood pressure 58/35 mmHg. He is lethargic and irritable, and
has a purpuric rash. You start antibiotics and steroids and perform a lumbar puncture,
which shows Gram-negative diplococci. One hour later, the nurse notes that there is
oozing from his IV site. He then has a large bloody stool and passes dark red-brown urine
which tests strongly positive for blood. You expect lab studies to show:
a. elevated fibrinogen.
b. elevated fibrin degradation products (FDP) and d-dimers.
c. diminished INR.
d. elevated platelet count.
e. shortened thrombin clot time.

230. A 72-year-old woman with a past history of congestive heart failure complains of epigastric
pain, black bowel movements, and vomiting some coffee-ground material. Her
hemoglobin is 6.5 mg/dL, so you order two units of packed red blood cells for transfusion.
The first unit is transfused uneventfully over a 45 minute period. Thirty minutes after the
second unit of packed red blood cells is started, she complains of a headache and
shortness of breath. Her lung exam now shows bilateral crackles. You tell the nurse to:
a. give a bolus of intravenous saline.
b. use a leukocyte reduction filter.
c. administer diphenhydramine.
d. slow the transfusion and begin diuresis.
e. stop the transfusion and report a transfusion reaction.

231. A healthy 12-year-old African-American female complains of weakness and fatigue 3 days
after starting a course of trimethoprim-sulfamethoxasole and pyridium for a urinary tract
infection. Her hemoglobin is 4.8 mg/dl, and her urine is tea-colored, but you see no red
blood cells on microscopic exam. She probably has undiagnosed:
a. hemolytic uremic syndrome.
b. G6PD deficiency.
c. idiopathic thrombocytopenic purpura.
d. thrombotic thrombocytopenic purpura.
e. sickle cell disease.

232. In methanol poisoning the most likely cause of toxicity is:


a. acetaldehyde.
b. formic acid.
c. folate.
d. ethyl methanol.
e. oxalic acid
86

228. b Treatment of nontraumatic hemarthrosis in a hemophiliac patient begins DDAVP. If not


th
Rosen 5 , available, Factor VIII 12.5 U/kg as a single dose is given for early or mild bleeds; most
Chapter 116, hemophiliacs usually require 25 U/kg every 24 hours for 2 or 3 days.
p.
229. b Tests for DIC Results Pathophysiology
th
Rosen 5 ,
Low platelets, schistocytes, RBCs fragment on fibrin strands
Chapter 116, Peripheral smear
RBC fragments (schiztocytes not always seen)
p. 1698, Table
116-3 Consumed in clotting, reflected in
Platelet count Low usually <10,000/mm3
bleeding time
Prothrombin time Prolonged Factors II and IV consumed
Activated partial
Prolonged Decreased Factor II
thromboplastin time
Fibrinogen level Low Factor II consumed
Dependent on secondary
FDP / d-dimer Zero to large
fibrinolysis
230. d Chronically anemic, normovolemic elderly patients are at greatest risk for developing
Rosen 5th, congestive heart failure with the rapid infusion of blood. Taking four hours to infuse a unit and
Chapter 5, p. using diuretics (if needed) should prevent this complication.
51
231. b Deficiency of the red blood cell (RBC) enzyme glucose-6- phosphate dehydrogenase (G-6-PD)
th
Rosen 5 , is the most common human enzyme defect, affecting nearly one-tenth of the worlds
Chapter 115, population. The RBC is unable to protect itself against oxidant stress. Acute hemolytic crises
p. 1676-1677 occur that are incited by bacterial and viral infections, exposure to oxidant drugs, metabolic
acidosis (such as diabetic ketoacidosis), and ingestion of fava beans in some patients. Within 1
to 3 days following oxidant stress, the patient can develop hemoglobinuria and the potential for
vascular collapse. These hemolytic crises are generally well tolerated and self-limited because
only the older RBCs will hemolyze. The drugs most commonly associated with oxidant stress
are sulfa drugs, antimalarials, phenazopyridine, and nitrofurantoin.
232. b Methanol itself has little toxicity, producing less CNS depression and inebriation than does
Rosen 5th, ethanol. Metabolites of the parent compound are, however, very toxic. Although small
Chapter 149, amounts of methanol are eliminated via renal and pulmonary routes, 90% is hepatically
p. 2127 metabolized. Methanol is oxidized by alcohol dehydrogenase to formaldehyde, which is then
rapidly converted by aldehyde dehydrogenase to formic acid, which is the primary toxicant and
accounts for much of the anion gap metabolic acidosis ocular toxicity peculiar to methanol
ingestion.
87

233. A 5-year-old girl complains of weakness and fatigue. Two weeks ago she saw her family
doctor and was diagnosed with an upper respiratory infection. She received no medicine
at that time. Physical examination shows only a scattered petechial rash located in areas
where her clothing is snug against her skin, such as underwear elastic lines. Laboratory
studies show a white blood cell count 11,000/mm3 , Hgb 10.5 mg/dL, and platelet count of
16,000/mm3. Appropriate management of this patient should be:
a. platelet concentrate transfusion.
b. discharge home with instructions to limit contact sports.
c. admit for salicylate therapy.
d. admit for splenectomy.
e. admit for observation.

234. A 38-year-old woman has von Willebrands disease, Type I. She complains of blood-
tinged emesis and epigastric pain. Her stool tests weakly positive for blood. Appropriate
initial therapy includes:
a. vitamin K.
b. 6 units platelet concentrate.
c. factor IX concentrate.
d. desmopressin (DDAVP).
e. plasmapheresis.

235. A 71-year-old woman takes warfarin for chronic atrial fibrillation. She complains of bloody
bowel movements. Her INR is 16.0 and her hemoglobin is 8.0 gm/dl. The first thing you
should administer is:
a. heparin.
b. fresh frozen plasma.
c. cryoprecipitate.
d. fibrinogen.
e. desmopressin (DDAVP).

236. A 32-year-old man complains of shortness of breath. He has just started trimethoprim-
sulfamethoxazole for chronic sinusitis. He is cyanotic. His pulse oximetry is 85% and
does not improve despite 100% oxygen. His lungs are clear. Room air arterial blood gas
shows a PaO2 of 94. Appropriate therapy for this patient would be:
a. subcutaneous epinephrine 1:1000.
b. intravenous diphenhydramine.
c. rapid sequence intubation.
d. albuterol nebulization.
e. intravenous methylene blue.

237. A 15-year-old boy with hemophilia A complains of a diffuse headache which started two
hours after he hit his head on a kitchen cabinet at home. He has a 3 cm contusion on his
left parietal scalp, but otherwise appears normal. Your first step is to:
a. give intranasal desmopressin (DDAVP).
b. give factor VIII replacement followed by brain CT.
c. order a brain CT and discharge home if normal.
d. observe for six hours in Emergency Department.
e. do a thorough neurologic exam and, if normal, discharge home.
88

233. e Acute idiopathic thrombocytopenic purpura (ITP) is seen most often in children 2 to 6 years
th
Rosen 5 , old. A viral prodrome is common, usually within 3 weeks of the onset. The platelet count falls,
Chapter 116, usually to less than 20,000/mm3. The course is self-limited, with a greater than 90% rate of
p. 1693 spontaneous remission. Morbidity and mortality are low, although full recovery may take
several weeks. Treatment is supportive, and steroid therapy does not alter the disease course.
234. d The treatment of von Willebrands disease depends on the type of disease that is present and the
Rosen 5th, severity of bleeding. Desmopressin (DDAVP) treatment has benefit in patients with mild to
Chapter 116, moderately severe von Willebrands disease, but should be given in consultation with a
p. 1697 hematologist. Factor VIII (cryoprecipitate) or fresh frozen plasma may be used in patients with
severe bleeding.
235. b Warfarin has a half-life of 2.5 days in patients with normal hepatic function. Patients with
Rosen 5th, bleeding complications can be treated with fresh frozen plasma (FFP) or vitamin K1
Chapter 5, p. (intravenous, intramuscular, or subcutaneous) each has advantages and disadvantages.
50 Infusion of FFP can result in the rapid repletion of coagulation factors and control of
hemorrhage, but carries some risk of viral transmission and volume overload. Parenteral
vitamin K1 will reverse the warfarin effect in 12 to 24 hours. Intravenous vitamin K1 can cause
hypersensitive anaphylactic reactions, although usually 1 mg can be given intravenously safely.
Intramuscular or subcutaneous vitamin K1 is typically given in doses of 5 to 10 mg daily in
states of coagulation factor deficiency.
236. e Most cyanotic patients improve with oxygen, whereas patients with methemoglobinemia do
Rosen 5th, not. Agents causing methemoglobinemia include nitrites, dapsone, lidocaine, aniline dyes,
Chapter 180, sulfonamides, phenacetin, and the fluoroquinolones. Methylene blue is the treatment of choice
p. 2540 for patients who are symptomatic or have levels above 30%. This substance acts as a cofactor
that accelerates the reduction of methemoglobin.
237. b Any patient with hemophilia who complains of a new headache or any neurologic symptoms
Rosen 5th, requires immediate factor VIII replacement therapy followed by immediate computed
Chapter 116, tomographic (CT) scanning of the head.
p. 1695
89

238. A 40-year-old woman has a four day history of progressive confusion, fever, and mild
jaundice. Laboratory tests show a severe anemia with schistocytes and
thrombocytopenia. Her prothrombin time (PT), partial thromboplastin time (PTT),
fibrinogen, and fibrin split products are all normal. Her most likely diagnosis is:
a. acute idiopathic thrombocytopenic purpura.
b. severe autoimmune hemolytic anemia.
c. disseminated intravascular coagulopathy.
d. G6PD deficiency.
e. thrombotic thrombocytopenic purpura.

239. A 68-year-old man complains of headache, dizziness, and blurred vision. His blood
pressure is 190/118 mmHg. He has a florid face, normal fundi, and marked splenomegaly.
His hematocrit is 67%. Reasonable therapy includes:
a. 250cc salt-poor albumin.
b. intravenous nitroprusside.
c. phlebotomy.
d. plasmapheresis.
e. sublingual nifedipine.

240. As an antidote, pralidoxime (2-PAM) is:


a. used in lieu of atropine.
b. employed in poisoning due to chlorinated hydrocarbon insecticides.
c. necessary to treat carbamate insecticide toxicity.
d. effective against many of the same toxins as physostigmine.
e. necessary to reverse nicotinic effects of insecticide poisoning.

241. A 2-year-old child is found comatose in his grandfathers barn with muscle twitching,
vomiting, diarrhea, and wheezing. He probably got into some:
a. parathion.
b. jimson weed.
c. DDT.
d. nitrate fertilizers.
e. antifreeze.

242. An 18-year-old hair salon stylist complains of abdominal pain and nausea for several
hours. She seems confused and intoxicated, but her breath does not smell of alcohol.
You begin intravenous fluids and send some lab work. One hour later, her blood pressure
drops to 60 palpable and she becomes apneic. You intubate her just as the lab results
come back. The most important clue that this may be isopropyl alcohol poisoning is:
a. bradycardia.
b. elevated creatinine with high serum acetone level.
c. fever greater than 39C.
d. hypertension.
e. profound metabolic acidosis with elevated osmolar gap, but no anion gap.
90

238. e TTP is a heterogeneous clinical syndrome characterized by this classic pentad of symptoms and
th
Rosen 5 , signs (all five are present only in 40%):
Chapter 116, 1. Microangiopathic hemolytic anemia (MAHA) with schistocytes on the blood smear and a
p. 1693 reticulocytosis
2. Thrombocytopenia: platelet counts range from 5000 100,000 / mL
3. Renal abnormalities: azotemia, proteinuria, or hematuria
4. Fever (90%)
5. Neurologic abnormalities including headache, confusion, cranial nerve palsies, seizures, or
coma
The disease may affect patients of any age or sex, but the majority are 10 to 40 years old, and
60% of cases occur in women.
239. c Emergency treatment of any form of symptomatic polycythemia is phlebotomy. Usually not
Rosen 5th, more than 500 ml of blood is slowly removed as the volume is replaced with a comparable
Chapter 115, amount of normal saline.
p. 1683
240. e The definitive treatment of acetylcholinesterase inhibition starts with the administration of
th
Rosen 5 , atropine. The second part of acetylcholinesterase inhibition treatment is the use of pralidoxime
Chapter 157, (Protopam, 2-PAM) to break up the organophosphate-acetylcholinesterase complex and restore
p. 2189 cholinesterase activity at both muscarinic and nicotinic sites. The use of 2-PAM in carbamate
poisoning is controversial.
241. a Commonly used organophosphate poisons include diazinon, orthene, malathion, parathion, and
Rosen 5th, chlorpyrifos. The accumulation of acetylcholine results in the classic cholinergic syndrome
Chapter 157, called the DUMBELS syndrome. This stands for diarrhea, urination, meiosis, bronchorrhea,
p. 2187 emesis, lacrimation, and salivation. The mnemonic SLUDGE ignores bronchorrhea, the
principle cause of morbidity and mortality. This represents the general hypersecretion seen in
poisoning with this class of pesticide. Later, the effect of nicotinic hyperstimulation of skeletal
muscle determines the ultimate morbidity and mortality of acetylcholinesterase inhibitors.
Signs of skeletal muscle hyperactivity include involuntary twitches, fasciculations, and
hyperactive reflexes.
242. b One early clue to the diagnosis of isopropanol ingestion is pseudo renal failure or isolated
Rosen 5th, false elevation of creatinine with a normal BUN. This results from interference of acetone and
Chapter 149, acetoacetate by the colorimetric method of creatinine determination. The most common
p. 2134 laboratory abnormality is is ketosis with little or no acidosis and normal blood glucose levels.
The ketosis is from the metabolite acetone.
91

243. A 27-year-old unemployed man drank a bottle of Liquid Drano in a suicide attempt. He
appears uncomfortable but is drooling. You know that:
a. activated charcoal will be a helpful adjunct in his treatment.
b. forcing fluids is never indicated..
c. he requires immediate decontamination with a large-bore nasogastric tube.
d. neutralization with a weak acid will be helpful.
e. barium swallow will help delineate the depth of involvement.

244. A teenage girl ingests a handful of over-the-counter pills. She is tachycardic,


hypotensive, and hyperthermic. Her pupils are dilated and she appears to be
hallucinating. The antidote most likely to be effective in treating her symptoms is:
a. atropine.
b. naloxone.
c. physostigmine.
d. pralidoxime.
e. propranolol.

245. A 40-year-old woman is found comatose with pulse 80 / minute, respirations 8 / minute,
and blood pressure 90/60 mmHg. An empty butalbital bottle is found nearby. Treatment
includes:
a. respiratory support, vasopressors, forced alkaline diuresis.
b. respiratory support, fluids.
c. respiratory support, immediate hemodialysis.
d. respiratory support, immediate hemoperfusion.
e. respiratory support, acid diuresis.

246. Ethylene glycol toxicity is characterized by:


a. high anion gap acidosis, osmolal gap, and elevated serum calcium.
b. high anion gap acidosis, osmolal gap, and crystals in urine.
c. high anion gap acidosis, retinal hyperemia, and crystals in urine.
d. osmolal gap, distinctive breath odor, and visual changes.
e. distinct CNS depression with acidosis, but no osmolal gap.

247. A 74-year-old man has apparently been despondent since his cardiac bypass surgery. He
took a whole bottle of his blood pressure pills. He is bradycardic, hypotensive, and
seizing. Bedside glucose is 270 mg/dl. The most effective treatment would be:
a. lidocaine 100 mg.
b. digoxin 0.25 mg.
c. dopamine 5-10 mcg/kg/min.
d. isoproterenol 0.2 mg.
e. glucagon 10 mg.

248. A 2-year-old child mistook a bottle of her uncles cancer vitamins for candy. She is
comatose, hypotensive, and bradycardic, but her skin is pink. Antidotal therapy includes:
a. sodium nitrite to induce methemoglobinemia.
b. methylene blue to induce methemoglobinemia.
c. sodium thiosulfate to produce cyanomethemoglobin.
d. sodium nitrite to bind cyanide.
e. amyl nitrite to induce a methemoglobinemia of 50%.
92

243. b Emesis induction and activated charcoal administration have no place in the treatment of a
th
Rosen 5 , caustic ingestion, and forcing fluids is never indicated. Gastric and esophageal lavage is
Chapter 147, usually not warranted. Use of an acid neutralizing agent will cause an exothermic chemical
p. 2117 reaction and possible severe burns. Noninvasive techniques such as barium swallow do not
gauge the depth of involvement. Corticosteroid therapy remains controversial.
244. c The patient exhibits classic anticholinergic toxidrome (Blind as a Bat, Hot as a Hare, Red as a
Rosen 5th, Beet, Dry as a Bone, Mad as a Hen). Physostigmine reverses delirium in 96% of patients who
Chapter 144, receive it as a first-line therapy, and has been used successfully to treat intractable seizures,
p. 2085 coma, severe agitation, and ventricular tachycardia. Because of potential toxicity,
physostigmine should be infused slowly in an initial dose of 1 to 2 mg for adults to control
severe anticholinergic manifestations.
245. b The management of barbiturate intoxication is based on meticulously supporting cardiovascular
Rosen 5th, and respiratory systems, maintaining adequate renal function, and avoiding unnecessary
Chapter 159, aggressive interventions. Gastric lavage, charcoal hemoperfusion, and hemodialysis are used in
p. 2209 only a small minority of cases. Alkalinizing the urine with NaHCO 3 will increase renal
clearance of long-acting phenobarbital, but does not effect clearance of short-acting barbiturates
such as butalbital.
246. b Like methanol, ethylene glycol often causes a profound anion gap metabolic acidosis when the
Rosen 5th, metabolites glycolic acid and glyoxylic acid (and, to some extent, lactic acid) accumulate. As
Chapter 149, with methanol toxicity, an elevated osmolal gap as measured by freezing point depression is a
p. 2131 clue to the diagnosis of ethylene glycol toxicity.

247. e Toxicity with overdose of both beta-blockers and calcium channel antagonists causes lethargy,
th
Rosen 5 , confusion, and coma. Unlike beta-blockers, calcium antagonists rarely induce seizures.
Chapter 146, Pulmonary effects of both include noncardiogenic pulmonary edema. While atropine can be
p. 2112 used to treat both hypotension and bradycardia, its effect are disappointing and short-lived.
Isoproterenol or dobutamine along may not reverse (or may even exacerbate) peripheral
vasodilation, so it is logical to add a vasopressor such as norepinephrine, metaraminol, or
phenylephrine.
248. a Amygdalin is a cyanogenic glycoside that is found in particularly high concentrations in apricot
Rosen 5th, pits and bitter almonds. It is the principal constituent of Laetrile, a compound popular for
Chapter 153, nontraditional cancer therapies in the late 1970s. The clinical signs and symptoms of cyanide
p. 2166 poisoning mimic those of hypoxia, with one exception: unless respiratory arrest has occurred,
patients are not cyanotic.
93

249. A standard 0.4 mg dose of naloxone should be sufficient to reverse an overdose from:
a. dextromethorphan.
b. fentanyl.
c. morphine.
d. pentazocine.
e. propoxyphene.

250. The diagnosis of methemoglobinemia is consistent with:


a. chocolate-colored blood which turns red upon exposure to air.
b. a normal pO2.
c. a bitter-almond breath odor.
d. a decreased calculated O2 saturation.
e. urine which turns red when left exposed to air.

251. A 60-year-old man has been taking 20 or more aspirin a day for two weeks for pain relief.
He complains of ringing in his ears, confusion and vomiting. You draw and send an
arterial blood gas, expecting to find:
a. primary respiratory alkalosis.
b. mixed respiratory alkalosis and metabolic acidosis.
c. mixed respiratory acidosis and metabolic acidosis.
d. mixed respiratory acidosis and metabolic alkalosis.
e. mixed respiratory alkalosis and metabolic alkalosis.

252. You wish to use ketamine for an adult patient who requires Procedural Sedation and
Analgesia (PSAA). You are concerned about the possible occurrence of reimergence
phenomenon, so you also give him:
a. midazolam.
b. morphine sulfate.
c. fentanyl
d. ketorolac.
e. nitrous oxide.

253. A 26-year-old man had an unknown ingestion in an apparent suicide attempt. He is


unresponsive with evidence of emesis. Heart rate 130 / minute; respiratory rate 26 /
minute; blood pressure: 110/70 mmHg. You intubate him for airway protection and start
intravenous fluids. The secondary exam is remarkable only for normal pupils, clinical
evidence of dehydration, active bowel sounds, and bibasilar rales. CBC is normal. Na+
142; K+ 3.8; Cl- 96; CO2 16; BUN 58. ABG shows pH 7.14; pO2 198; pCO2 30; HCO3 12,
O2sat 99% on 100% oxygen. Chest x-ray shows pulmonary edema with a normal-sized
heart. After unremarkable gastric large and activated charcoal administration, this
patient's toxicology screen returns positive only for a salicylate level of 145 mgldl. Your
next treatment is:
a. correction of serum potassium.
b. forced diuresis.
c. hemodialysis.
d. repeat activated charcoal administration.
e. urine alkalinization using intravenous sodium bicarbonate.
94

249. c Exposures to propoxyphene, fentanyl, pentazocine, or dextromethorphan may require large


th
Rosen 5 , doses of naloxone. Standard dose should be sufficient to reverse morphine intoxication.
Chapter 156,
p. 2184
250. a The diagnosis of methemoglobinemia should be considered in all patients who present with
th
Rosen 5 , cyanosis, and is particularly suspect in those whose cyanosis does not improve with
Chapter 180, supplemental oxygen. The blood of these patients has a characteristic chocolate brown color,
p. 2067, Table analogous to that seen in the chocolate agar used to plate gonococcus (which contains
141-3 methemoglobin).

251. b The initial manifestations of acute salicylate toxicity include tinnitus and impaired hearing.
th
Rosen 5 , More clinically significant toxic manifestations include hyperventilation, vomiting,
Chapter 143, dehydration, hyperthermia, and central nervous system (CNS) disturbances. Salicylate-induced
p. 2077 hyperpnea may manifest as increased respiratory depth without commensurate increase in rate
(panting dog respiration). Vomiting may occur 3 to 8 hours after ingestion. Signs and
symptoms of chronic intoxication include hyperventilation, tremor, papilledema, agitation,
paranoia, bizarre behavior, memory deficits, confusion, and stupor. Neurologic abnormalities
are much more common in chronic salicylate poisoning and often mislead physicians. Serious
dehydration may occur from hyperpnea, vomiting, and hyperthermia. Most often, respiratory
alkalosis is followed by metabolic acidosis.
252. e Emergence phenomena manifested by hallucinations and nightmares occur in up to 50% of
Rosen 5th, adults (and 10% of children) receiving ketamine. A study of adult patients requiring PSAA to
Chapter 183, facilitate a painful procedure demonstrated that a combination of midazolam and ketamine
p. 2582 produced safe and effective sedation and analgesia with a low incidence of emergence
phenomena.
254. e Urinary alkalinization should be considered in patients with salicylate levels greater than 35
Rosen 5th, mg/dl, acid-base abnormalities, or rapidly increasing salicylate levels. A urine pH of 7.5 to 8.0
Chapter 143, is necessary to increase excretion. ABG values will help determine acid-base status. Sodium
p. 2078 bicarbonate (1 to 2 mEq/kg) may be administered over 1 to 2 hours, with subsequent dosage
adjustment determined by urinary and serum pH.
95

254. A tobacco harvester complains of nausea, vomiting, diarrhea, and abdominal pain. His
plants have recently been sprayed with Sevin, a carbamate insecticide. Appropriate
management would include:
a. physostigmine intravenously if seizures occur.
b. pralidoxime chloride (2-PAM) 1 gram IV.
c. atropine 0.4 to 2.0 mg IV as needed until atropinization is achieved.
d. gastrointestinal decontamination.
e. intravenous rehydration and antiemetics.

255. A 25-year-old man complains of chest pain six hours after smoking crack cocaine. An
ECG shows ST elevation in leads II, III and aVF. A drug you should avoid in treating him
is:
a. diazepam.
b. morphine.
c. nitrates.
d. propranolol.
e. sodium bicarbonate.

256. A 2-year-old girl ingested an unknown liquid in her neighbors garage. She is comatose
and has nystagmus. Physical exam is otherwise remarkable only for the smell of alcohol.
Basic metabolic panel: Na+ 140; K+ 4.2; Cl- 107; HCO3 24; BUN 7, creatinine 0.6, glucose
92. Her urine dips positive for ketones, but negative for glucose. She probably drank
some:
a. ethanol.
b. ethylene glycol.
c. gasoline.
d. isopropanol.
e. methanol.

257. Drugs which can be dialyzed usually have:


a. high molecular weight.
b. high protein binding.
c. large volume of distribution.
d. renal metabolism.
e. water solubility.

258. Choose the correct statement.


a. Opioid agonist / antagonists do not precipitate withdrawal in the dependent patient.
b. Opioid withdrawal in adults does not cause seizures.
c. Propoxyphene overdose may result in seizures.
d. The duration of the effects of naloxone is longer than that of methadone.
e. The primary cause of death from heroin overdose is myocardial depression.

259. Choose the correct statement concerning the stages of acute iron poisoning:
a. Stage I of iron poisoning is considered the corrosive or GI stage.
b. Intestinal obstruction is the clinical sign most commonly associated with Stage II iron
poisoning.
c. Systemic toxicity characterizes stage III of iron toxicity, including shock and lactic
acidosis, with grossly elevated serum iron levels.
d. Stage IV of iron poisoning includes delayed sequelae such as gastric outlet obstruction
e. Stage V, or the hepatic stage of iron poisoning, develops 4 to 6 weeks after ingestion.
96

253. e Symptoms of nicotine toxicity begin shortly after absorption. Nausea, vomiting, salivation,
th
Rosen 5 , lacrimation, diarrhea, hypertension, tachycardia, diaphoresis, agitation, and fasciculation are
Chapter 158, seen initially. More severe toxicity is manifested by seizures, respiratory depression (muscle
p. 2200 weakness), and hyperthermia. Treatment for ingestion is initially aimed at limiting absorption
with activated charcoal. Gastric lavage is of questionable value. Other treatments are
supportive in nature, because no specific antidote for nicotine is available. Benzodiazepines
can be used for seizures and agitation. If severe salivation and lacrimation occur, atropine may
be useful.
255. d Beta-adrenergic antagonist therapy such as propranolol is absolutely contraindicated because
Rosen 5th, unopposed stimulation of a-adrenergic receptors may worsen coronary and peripheral
Chapter 148, vasoconstriction, hypertension, and possibly ischemia. Although labetalol (a mixed alpha- and
p. 2122-2123 beta-adrenergic antagonist) has been suggested, labetalol increased seizures and mortality in an
animal model of cocaine toxicity and does not decrease cocaine-induced coronary
vasoconstriction in humans.
256. d The clinical features of isopropanol intoxication are similar to those seen with ethanol
Rosen 5th, intoxication except that the duration of symptoms and signs is longer and central nervous
Chapter 149, system depression may be more profound because of the formation of acetone. Nystagmus is
p. 2134 usually present. Severe poisoning is marked by early onset of coma, respiratory depression, and
hypotension. Hypoglycemia may occur secondary to depressed gluconeogenesis. Isopropanol
poisoning should be suspected when the smell of rubbing alcohol is present on the breath, when
there is acidosis associated with ketonuria and ketonemia without glycosuria or hyperglycemia,
and in the presence of an elevated osmolal gap.
257. e Lithium and the toxic alcohols are readily dialyzable because of their small molecular weight,
Rosen 5th, water solubility, and lack of protein binding.
Chapter 141,
p. 2173
258. c Propoxyphene causes sodium channel blockade similar to that of type IA antidysrhythmic
th
Rosen 5 , agents; dysrhythmias caused by propoxyphene overdose can be treated with sodium bicarbonate
Chapter 156, therapy. Opioid withdrawal occurs in tolerant individuals when opioid exposure is
p. 2185 discontinued or an antagonist is administered. Increased sympathetic discharge and adrenergic
hyperactivity are responsible for the neurologic manifestations so prominent in opioid
withdrawal, including restlessness, agitation, anxiety, and seizures. Naloxone has a half-life of
elimination of 1.1 hours, whereas methadones half-life of elimination varies from 15 to 40
hours. Opioids decrease both respiratory rate and tidal volume in a dose-dependent manner by
suppression of the sensitivity of the medullary respiratory center to hypercapnea.
259. a Stage I reflects the corrosive effects of iron on the gut. Vomiting occurs within 80 minutes of
Rosen 5th, ingestion and GI bleeding is common.
Chapter 151, Stage II represents an apparent (but not complete) recovery that lasts less than 24 hours but can
p. 2151 extend up to 2 days.
Stage III is characterized by recurrent GI bleeding, severe lethargy or coma, cardiovascular
collapse, anion gap metabolic acidosis, leukocytosis, and coagulopathy. Acute renal failure
may develop.
Stage IV, 2 to 5 days after ingestion, is characterized by acute hepatic failure, with jaundice,
hepatic coma, elevated transaminase and ammonia levels, hypoglycemia, and coagulation
defects. Fulminant hepatic failure generally is fatal.
Stage V is characterized by pyloric or proximal bowel scarring, sometimes associated with
obstruction.
97

260. A 33-year-old man ingested 50 of his grandfather's benztropine tablets. Rectal


temperature: 39oC; heart rate: 130 / minute; respiratory rate: 20 / minute; blood pressure:
100/60 mmHg. His skin is dry and his pupils are dilated. Since arriving, he has become
more delirious and lethargic. You consider giving an antidote, but are especially worried
about the occurrence of:
a. diarrhea.
b. hyperthermia.
c. respiratory depression.
d. seizures.
e. torsade de pointes.

261. A 29-year-old woman ingested what she thought was a Valium tablet at a party. She
complains that she cant swallow. Her eyes are deviated upwards and to the right, and
she cannot voluntarily move them from this position. She probably was given:
a. atropine eye drops.
b. GHB (gamma-hydroxybutyrate).
c. haloperidol (Haldol).
d. methyl scopolamine.
e. flunitrazepam (Rohypnol)

262. You should treat the patient in question #261 with:


a. benztropine (Cogentin).
b. flumazenil (Romazicon).
c. lorazepam (Ativan).
d. naloxone (Narcan).
e. physostigmine (Antilirium).

263. A potentially lethal medication combination would be:


a. phenelzine (Nardil) and ketamine.
b. imipramine (Tofranil) and bronchodilators.
c. lithium and acetazolamide (Diamox).
d. theophylline and high dose steroids.
e. thioridazine (Mellaril) and cimetidine (Tagamet).

264. You would expect bradycardia in a patient who has overdosed on:
a. angiotensin converting enzyme inhibitor.
b. hydralazine.
c. nifedipine.
d. dietary supplements.
e. verapamil.

265. A 3-year-old complains of headache and dizziness. His 5-year-old sister says he was
eating flowers in the back yard. The cardiac monitor shows frequent PVCs. Shortly after
arrival he has a generalized seizure. You subsequently identify the flowers as oleander.
Treatment should be started with:
a. activated charcoal.
b. digoxin-specific Fab fragment.
c. phenytoin.
d. hemodialysis.
e. sodium bicarbonate
98

260. d Benztropine (Cogentin) is an anticholinergic. Physostigmine in the management of


th
Rosen 5 , anticholinergic overdoses is controversial. In the absence of anticholinergic blockade,
Chapter 144, physostigmine itself has significant toxicity. It causes seizures, muscle weakness, bradycardia,
p. 2085 lacrimation, salivation, bronchorrhea, and diarrhea, and it exacerbates asthma. Even in
documented anticholinergic toxicity, seizures have been reported after its rapid administration.
261. c Acute dystonia, the most common adverse effect seen with neuroleptic agents, occurs in 1% to
Rosen 5th, 5% of patients. Reactions can occur at any point during long-term therapy and up to 48 hours
Chapter 104, after administration of a neuroleptic agent, such as haloperidol. The patient may have
p. 1546 protrusion of the tongue (buccolingual crisis), deviation of the head to one side (acute
torticollis), sustained upward deviation of the eyes (oculogyric crisis), extreme arching of the
back (opisthotonos), or rarely laryngospasm. Dystonic reactions should be treated with IM or
262. a IV benztropine (Cogentin), 1 to 2 mg, or diphenhydramine (Benadryl), 25 to 50 mg.
Rosen 5th, Intravenous administration usually results in an almost immediate reversal of symptoms.
Chapter 104, Patients should receive oral therapy with the same medication for 48 to 72 hours to prevent
p. 1546 recurrent symptoms.
263. a Five drugs with significant monoamine oxidase (MAO) inhibition are marketed in the United
Rosen 5th, States: the antidepressants phenelzine and tranylcypromine, the antiparkinsonian agent
Chapter 145, selegiline, the antimicrobial furazolidone, and the antineoplastic procarbazine. MAOI-drug
p. 2099-2101 interaction signs and symptoms are sympathomimetic storms or the serotonin syndrome. Most
MAOI-drug interactions occur in patients who are regularly taking MAOIs long term and ingest
incompatible drugs, such as indirect-acting sympathomimetics (e.g. ketamine) , mixed-acting
(direct/indirect-acting) sympathomimetics, methylxanthines, antidepressants, opioids (e.g.,
meperidine), and other drugs that can cause the serotonin syndrome. These drugs produce
excessive concentrations of endogenous biogenic amines that are not degraded because of
MAO inhibition.
264. e Verapamil generally causes hypotension and bradycardia as early symptoms. Other rhythm
Rosen 5th, disturbances include AV block of all degrees, sinus arrest, AV dissociation, junctional rhythm,
Chapter 146, and asystole. Nifedipine, overdose more commonly causes reflex sinus tachycardia from
p. 2111 peripheral vasodilation. Calcium channel blockade has little effect on ventricular conduction,
so QRS widening is not seen early .
265. b Nerium oleander is one of many plants that contain toxic cardiac glycosides structurally similar
Rosen 5th, to digoxin. Small ingestions (up to several leaves) are unlikely to cause serious symptoms.
Chapter 158, However, large exposures from suicide attempts or misidentification of plants used for teas or
p. 2200 herbal products can lead to severe toxicity or death. The cardiac glycosides are potent sodium-
potassium adenosine triphosphatase (ATPase) inhibitors, and the symptoms they cause are
similar to those of digoxin poisoning. Measurement of an abnormal digoxin level is only
qualitative proof of exposure, because the absolute digoxin level may not correlate with serum
levels. Negative digoxin measurements do not rule out exposures, because the level of cross-
reactivity with different assays varies. Treatment for oleander toxicity involves digoxin-
specific Fab antibodies; however, larger doses of Fab fragments will be needed than for
comparable digoxin poisonings. Initial empiric doses of 10 to 20 vials of digoxin-specific Fab
fragments have been suggested.
99

266. Which statement is true regarding the stages of ethylene glycol toxicity?
a. Stage I, the acute neurologic phase: inebriation, convulsions, decreased pupillary
reflexes, and papilledema, occurring 24 to 48 hours after ingestion.
b. Stage II, the cardiopulmonary stage: hypertension and tachycardia with development of
pulmonary infiltrates.
c. Stage III, the renal stage: hematuria, proteinuria, and dihydrate crystalluria occurring 24
to 72 hours postingestion.
d. Stage IV, the delayed neurologic sequelae stage: usually manifests as a central cord
syndrome with a stocking-glove peripheral neuritis.
e. Stage V, the hepatic stage: can first present 6 to 12 days after ingestion with fulminant
jaundice and hyperammonemia.

267. In a patient with acute salicylate toxicity, the MOST LIKELY finding will be:
a. metabolic alkalosis.
b. respiratory acidosis, metabolic alkalosis.
c. respiratory acidosis , metabolic acidosis.
d. respiratory alkalosis, metabolic acidosis.
e. respiratory alkalosis, metabolic alkalosis.

268. Of the conditions listed, the one most closely resembling strychnine poisoning is:
a. botulism poisoning.
b. puffer fish poisoning.
c. rat poison ingestion.
d. organophosphate poisoning.
e. tetanus infection.

269. Clonidine toxicity is reported to be reversed by:


a. digoxin Fab fragments.
b. calcium chloride.
c. naloxone.
d. physostigmine.
e. hypertonic saline solution.

270. Body packers :


a.are individuals who suddenly swallow the evidence.(i.e. drugs) in imminent law
enforcement presence.
b.are at greater risk of developing toxicity from ingested drug than body stuffers.
c. have gastrointestinal placement of drugs intentionally prepared for safe swallowing and
transportation.
d.are less likely to have positive x-rays than body stuffers.
e.are more likely to be harboring cannabinoids or amphetamines than cocaine or heroin.

271. Glucagon is indicated in the treatment of overdose caused by:


a. iron.
b. nicotine.
c. organophosphates.
d. tricyclic antidepressants.
e. verapamil.
100

266. b Stage I, the acute neurologic stage, occurs 30 minutes to 12 hours after ingestion. Stage II, the
th
Rosen 5 , cardiopulmonary stage, occurs 12 to 24 hours after ingestion. Often the patient exhibits mild
Chapter 149, hypertension and tachycardia. Stage III consists of elements of renal failure typically occurring
p. 2130-2131 24 to 72 hours postingestion. Stage IV, the delayed neurologic sequelae stage, occurs 6 to 12
days after ingestion and typically manifests as cranial neuropathy (facial auditory nerve
oxalosis). No Stage V has been described.
267. d Salicylates stimulate the medullary respiratory center and inhibit Krebs cycle dehydrogenases.
Rosen 5th Most frequently reparatory alkalosis is followed by metabolic acidosis.
Chapter 143,
p. 2076-2077
268. e Strychnine poisoning is similar to tetanus in that the patient develops opisthotonos while
th
Rosen 5 , remaining awake. Strychnine is suggested by a history of toxin ingestion and by muscle
Chapter 123, p rigidity alternating with periods of relaxation. Also in strychnine poisoning, trismus is a later
1791-1793 finding. Trismus is the presenting symptom in most patients with tetanus. Treatment of the
two conditions is similar, and toxicologic studies can confirm the presence of strychnine.
269. c The clinical response to naloxone is not pathognomonic of opioid intoxication; other
Rosen 5th, intoxications may improve with naloxone therapy as well, including valproic acid, clonidine,
Chapter 156, tramadol, captopril, and ethanol. Naloxone has been given to patients who have ingested these
p. 2184-2185 agents either because of a presentation similar to opioid intoxication or suspicion of a mixed
exposure that included opioids. The mechanism of these responses to naloxone is not
established. Some of these drugs may have activity at opioid receptors.
270. c Body packing, which entails systematic GI tract placement of previously prepared drug
Rosen 5th, packages, should be clinically differentiated from body stuffing, which denotes hurried
Chapter 53, p ingestion of hastily prepared packages in the face of imminent police presence. Body stuffers
766 are more likely to suffer toxicity due to the poor packaging of drugs and are less likely to have
positive plain radiography findings. Drugs most often seen with body packing or body stuffing
are cocaine and heroin, with amphetamines and cannabinoids seen less frequently.
271. e Glucagon can be used as an antidote in patients with beta-blocker or calcium channel blocker
Rosen 5th, overdose. Recommended dose is 510 mg in adults, then infusion of same dose per hour.
Chapter 141, p
2067
101

272. A 2-year-old girl is transferred from a local Day Care


Center where she had two seizures shortly after her
mother dropped her off this morning. The Day Care
worker cant get in touch with mom, but knows the child
was recently adopted from China. She was given 10 mg
diazepam IV in the ambulance, but had a third seizure
just prior to arrival. She arrives in status epilepticus.
Temperature 100oF; heart rate 140 / minute; respiratory
rate 16 / minute; blood pressure 98/65 mmHg. WBC
18,000; hemoglobin 16; Na+ 140; K+ 4.9; Cl- 100; HCO3- 7;
Rosen 5th, Figure 129-3
glucose 120; pH 6.99; pO2 112; pCO2 45. Her chest x-
ray, done to check tube placement, is shown. You now
know that you should attempt to stop the seizures using intravenous:
a. diazepam (Valium) 10 mg.
b. fosphenytoin (Cerebyx) 15-20 mg/kg.
c. phenobarbital 10-20 mg/kg.
d. pyridoxine 5 g.
e. vecuronium 0.1 mg/kg.

273. The generally recognized toxic dose of acetaminophen in adults is:


a. 70 mg/kg.
b. 100 mg/kg.
c. 140 mg/kg.
d. 180 mg/kg.
e. 220 mg/kg.

274. Acetaminophen is toxic in overdose situations because


a. renal excretion is overwhelmed increasing liver excretion.
b. the toxic metabolite n-acetyl-p-benzoquinoneimine (NAPQI) exceeds glutathione supply
and binds to hepatocyte intracellular proteins.
c. it is directly toxic to the liver.
d. the cytochrome p450 system is overwhelmed and there is increased metabolism through
sulfation and glucuronidation which cause hepatotoxicity.
e. once glutathione stores are depleted, the native acetaminophen compound builds up
and causes toxicity.

275. Trans-abdominal ultrasonography can detect a normal intrauterine pregnancy at about:


a. 2 weeks gestation.
b. 4 weeks gestation.
c. 6 weeks gestation.
d. 8 weeks gestation.
Rosen 5th, Figure 85-7
e. 10 weeks gestation.

276. The use of ultrasound evaluation is currently increasing in


emergency departments. This ultrasound (Figure 85-7)
demonstrates:
a. cholelithiasis.
b. ruptured abdominal aortic aneurysm.
c. ectopic pregnancy.
d. pericardial effusion.
e. hydronephrosis.
102

272. d Patients with seizures refractory to benzodiazepines should be screened for isoniazid overdose,
th
Rosen 5 , since the only effective pharmacological treatment is pyridoxine. A dose of 1 mg IV of
Chapter 16, p. pyridoxine for each 1 mg of isoniazid ingested should be administered, or if an unknown
147; Chapter quantity was ingested 5 mg IV should be given.
129, p. 1917;
Chapter 168,
p. 2359
273. c In adults, indications for laboratory evaluation include ingestion of >4 grams (in children >75
Rosen 5th, mg/kg) acetaminophen in a 24 hour period associated with malnourishment, chronic alcohol
Chapter 142, consumption, or chronic use of medications that induce the cytochrome P450 system.
p. 2071 Laboratory evaluation should also be performed for adults who ingest >7.5 grams (in children
>150mg/kg) in a 24 hour period.
274. b Once acetaminophen (APAP) is absorbed, hepatic metabolism normally accounts for up to 90%
Rosen 5th, of its elimination. APAP is primarily metabolized in the liver through three routes: (1)
Chapter 142, p. conjugation with glucuronide (40% to 67%), (2) conjugation with sulfate (20% to 46%), or (3)
2069-2070 oxidation via the cytochrome P450 (CYP450) mixed-function oxidase system with subsequent
fig 142-1 conjugation. The oxidation of APAP by CYP450 subfamilies results in the formation of highly
reactive NAPQI. NAPQI rapidly combines with glutathione and other thiol-containing
compounds forming nontoxic conjugates, which are eliminated in the urine. When NAPQI
formation exceeds glutathione supply, free NAPQI binds to hepatocyte intracellular proteins,
causing toxicity.
275. d At 6 weeks of gestation, the gestational sac can be visualized with transabdominal sonography
Rosen 5th, (TAS). At 8 weeks, the fetal pole and fetal heart activity can be visualized using
Chapter 171, p. transabdominal sonography (TAS). Using transvaginal sonography (TVS) an intrauterine
2405 gestational sac can be seen at 5 weeks gestation and this correlates with an HCG of more than
1,800 IU/L. Fetal heart motion can be detected using transvaginal sonography (TVS) at 6
weeks of gestation and correlates with an HCG level of 6,770 IU/L.
276. a Ultrasound is the most useful test for evaluating gallbladder disease in the emergency
Rosen 5th, department It is the procedure of choice in evaluating for gallstones. Visualization of the
Chapter 85, p. gallbladder without identification of stones has an extremely high negative predictive value for
1266-1267 cholecystitis, whereas the presence of stones, a thickened gallbladder wall, and pericholecystic
fluid has a positive predictive value in excess of 90%.
103

277. The FAST ultrasound exam:


a. is for the evaluation of ectopic pregnancy.
b. has the primary role of detecting free retroperitoneal blood after blunt trauma.
c. is sensitive for detecting subcapsular splenic injury.
d. may be false positive in a patient with ascites.
e. .is sensitive for bowel perforations.

278. Usually, the minimal amount of intraperitoneal fluid that ultrasound (US) easily detects is:
a. 10 ml.
b. 50 ml.
c. 500 ml.
d. 1,000 ml.
e. 1,500 ml.

279. An anatomic area not well visualized by CT scan is the:


a. ureter.
b. retroperitoneum.
c. aorta.
d. bladder.
e. posterior intracranial fossa.

280. You are called to the radiology suite when an outpatient having an IVP done suddenly
becomes short of breath, tachycardic, and complains of chest pain. His face is red, you
hear wheezing as you walk into the room, and the nurse tells you his blood pressure is 76
palpable. The most important treatment is:
a. diphenhydramine (Benadryl) 50 mg IV.
b. cimetidine (Tagamet) 300 mg IV.
c. methylprednisolone (Solu-Medrol) 125 mg IV.
d. epinephrine 10ml of 1:100,000 dilution IV over 10 minutes.
e. albuterol by nebulizer.

281. The procedure of choice to evaluate possible aortic dissection in the unstable patient is:
a. magnetic resonance imaging. (MRI).
b. transthoracic echocardiography (TTE).
c. transesophageal echocardiography(TEE).
d. computed tomography (CT).
e. aortography.

282. MRI is the procedure of choice in the acute evaluation of:


a. spinal epidural hematoma.
b. periorbital/globe injury.
c. bladder injury.
d. traumatic bowel perforation.
e. testicular torsion.

283. A 8-month-old girl has scabies. The preferred treatment is:


a. crotamiton (Eurax).
b. Diphenhydramine (Benadryl).
c. lindane (Kwell).
d. permethrin (Elimite).
e. topical hydrocortisone.
104

277. d Focused assessment with sonography for trauma (FAST) exams primary role is detecting free
th
Rosen 5 , intraperitoneal blood after trauma. . It cannot distinguish blood from ascites. False-negative
Chapter 30, FAST scans commonly involve bowel perforations or solid parenchymal damage if free
p. 252-253, intraperitoneal blood is absent as in subcapsular splenic injury.
Chapter 39,
p. 423-424
278. c Sensitivity in detecting as little as 100 ml and, more typically, 500 ml of intraperitoneal fluid
th
Rosen 5 , ranges from 60% to 95% using ultrasound.
Chapter 39, p.
423
279. e MRI is better than a head CT in detecting posttraumatic ischemic infarctions, subacute
th
Rosen 5 , nonhemorrhagic lesions and contusions, and axonal shear injury, and lesions in the brainstem or
Chapter 34, p. posterior intracranial fossa. MRI is not as useful as CT in detecting bony injuries or hyperacute
296 bleeding.

280. d Epinephrine, with its combined - and - adrenergic agonist actions, is the first drug of choice
th
Rosen 5 , in the treatment of anaphylaxis. The route of administration chosen depends of the severity of
Chapter 113, the clinical presentation. Subcutaneous epinephrine is usually effective in those situations in
p. 1632 which the clinical manifestations are mild and the patient is normotensive. In the patient with
diffuse, generalized uriticaria, subcutaneous absorption of epinephrine may be slow and
unpredictable, so the intramuscular route should be used. If the patient demonstrates severe
upper airway obstruction, acute respiratory failure, or shock (systolic blood pressure less than
80 mm Hg, not associated with a ventricular tachydysrhythmia), intravenous epinephrine should
be administered. The risk of supraventricular, accelerated idioventricular, and ventricular
tachydysrhythmias; accelerated hypertension; and myocardial ischemia, including the stunned
heart syndrome, is increased by using intravenous route of epinephrine. Because of these risks,
dilution and slow administration is recommended.
281. c While CT, MRI, and aortography have all been used to diagnose aortic dissections, they all
Rosen 5th, require the patient to leave the ED. In contrast transesophageal echocardiography (TEE) may
Chapter 80, p. be done at the bedside and thus when available is the primary diagnostic method for evaluating
1173-1174 for possible aortic dissection in the unstable patient. Transthoracic echocardiography (TTE) has
not been useful in detecting aortic dissection.
282. a The primary advantages of MRI over other imaging modalities lie in its ability to directly image
Rosen 5th, nonosseous structures of the spinal complex. An MR scan allows noninvasive direct
Chapter 36, visualization of intramedullary and extramedullary spinal abnormalities that potentially cause
p. 360,363, fig neurologic deficit. Its major impact has therefore been in demonstrating lesions that are
36-40 potentially correctable surgically, including acute disk herniation, ligamentous injury, bony
Chapter 100, compression, epidural and subdural hemorrhage or abscess, and vertebral artery occlusion.
p. 1502-1503
283. d Treatment options for scabies include crotamiton (Eurax) lotion and cream, or permethrin 5%
th
Rosen 5 , cream (Elimite). Lindane is no longer the preferred treatment. Patients in whom the former
Chapter 114, treatment fails may respond to the latter. Permethrin 5% cream applied overnight once weekly
p. 1651 for 2 weeks over the entire body is the treatment of choice for infants and small children. It is
more effective than crotamiton in eliminating the mite, in reducing secondary bacterial
infection, and in reducing pruritus.
105

284. A 20-year-old woman complains of painful, swollen right and left knees , low grade fever,
and a pustular rash on her hands. Your next step in her evaluation is:
a. ASO titer.
b. pelvic exam.
c. knee ultrasound.
d. rheumatoid factor.
e. urinalysis.

285. A 57-year-old female complains of oozing lesions with mild swelling and slight erythema of
her face and hands after working in her garden 2 days prior. The appropriate treatment of
moderate to severe dermatitis on the face and hands from a known poison ivy exposure is:
a. dexamethasone (Decadron) injection 0.01 mg/kg.
b. intravenous hydrocortisone (Solu-Cortef).
c. prednisone 10 mg daily for 10 days.
d. prednisone tapered over 21 days, starting with relatively high dose.
e. topical calamine lotion.

286. A 54-year-old man has had a burning pain in


his chest for 5 days. He has no shortness of
breath, and there is no radiation. Today he
noticed this rash, which alarmed him and his
wife. You can tell them that this rash is
associated with:
a. a high mortality rate.
b. arthralgia.
c. neuralgia.
d. pruritus.
e. resolution in 48-72 hours. Rosen 5th Figure 114-29
287. Nikolsky's sign is characteristically seen in:
a. toxic shock syndrome.
b. pemphigus vulgaris.
c. Rocky Mountain spotted fever.
d. scarlet fever.
e. erythema nodosum

288. A 17-year-old female complains of large raw, denuded areas of her skin. She also has low
grade fever and generalized malaise. She does recall seeing her doctor recently for an
upper respiratory infection. Toxic epidermal necrolysis (TEN):
a. has no association with immunizations.
b. has mucus membrane involvement .
c. has a low mortality rate, generally less than 2%.
d. is rarely caused by medications.
e. causes bullae along the cleavage plane between the dermis and the subcutaneous
tissue.

289. The treatment of contact dermatitis includes:


a. cool wet compresses of Burows solution.
b. topical corticosteroid ointment.
c. systemic steroids.
d. systemic antihistamines.
106

e. all of the above.


107

284. b Gonococcal arthritis begins with fever, chills, and a migratory tenosynovitis and arthralgias that
th
Rosen 5 , progress to arthritis, predominantly in the knee, ankle, or wrist. A characteristic rash in two
Chapter 110, thirds of patients accompanies the tenosynovitis and arthritisa countable number of
p. 1593 hemorrhagic necrotic pustules that typically first appear on the distal extremities, including the
sides of the fingers. Rarely does the patient complain of cervicitis or urethritis. Cervical,
urethral, rectal, and pharyngeal cultures are positive in 75% of cases. For best yield, all
possible sites (synovium, blood, cervix, urethra, rectum, pharynx, and skin lesions) should be
cultured. The initial diagnosis is a clinical one; it is the diagnosis to be excluded in any young
patient with a fever, migratory polyarthritis, and polytendinitis.
285. d Oozing or vesiculated lesions caused by the Rhus species (poison ivy ,oak, and sumac) should
Rosen 5th, be treated with cool wet compresses of Burows solution. Topical baths, available over the
Chapter 114, counter, may also be comforting. A course of systemic corticosteroids is often necessary.
p. 1647-1648 Prednisone in a dosage of 30 to 80 mg/day (depending on the severity of involvement) should
be prescribed initially. This should be tapered over at least 10 to 14 days, and 21 days for
poison ivy.
286. c Herpes zoster has a very low mortality rate and is rarely life threatening even when
Rosen 5th, dissemination to the visceral organs occurs. The vesicles progress to scab then crust over 10 to
Chapter 114, 12 days then the scab falls off in 2 to 3 weeks. Pain is the symptom typically reported.
p. 1656 Complications include CNS involvement, ocular infection, and neuralgia.
Meningoencephalitis, myelitis, and peripheral neuropathy have been reported. Postherpetic
neuralgia, pain that persists after the lesions have healed, occurs more commonly in the elderly
and immunosuppressed patients. It may last a number of months and is often resistant to
treatment with standard analgesics.
287. b Nikolskys sign is the easy separation of the outer portion of the epidermis from the basal layer
Rosen 5th, (desquamation) when pressure is exerted. It is present in and characteristic of pemphigus
Chapter 114, vulgaris. It is also found in .toxic epidermal necrolysis (TEN) and staphylococcal scalded skin
p. 1643,1653 syndrome.
Chapter 131,
p. 1946
288. b The main feature of nonstaphylococcal-induced toxic epidermal necrolysis (TEN), or Lyells
th
Rosen 5 , disease, is the separation of large sheets of epidermis from underlying dermis. The full
Chapter 114, thickness of epidermis is involved. Drugs including the long-acting sulfa drugs, penicillin,
p. 1643 aspirin, barbiturates, phenytoin, carbamazepine, allopurinol, and nonsteroidal anti-inflammatory
drugs are an important cause of TEN. TEN has occurred after vaccination and immunization
against polio, measles, smallpox, diphtheria, and tetnus. A mortality rate of 15% to 20% is
expected with this condition. Mucus membrane involvement is the rule.
289. e Treatment of contact dermatitis includes avoidance of the irritant or allergen and treatment of
Rosen 5th, secondary bacterial infection. Oozing or vesiculated lesions should be treated with cool wet
Chapter 114, compresses of Burows solution applied for 15 minutes three or four times a day. Topical
p. 1647-1648 baths, available over the counter, may also be comforting. A course of systemic corticosteroids
is often necessary. Prednisone in a dosage of 30 to 80 mg/day (depending on the severity of
involvement) should be prescribed initially. This should be tapered over at least 10-14 days,
and 21 days for poison ivy. The long, slow taper is needed to prevent rebound of the disease.
Systemic antihistamines, such as hydroxyzine and diphenhydramine, may help control pruritis.
The patient should also be counseled to wash all clothes that might have contacted the plant.
108

290. A 27-year-old male presents with the lesions


shown, but does not recall any pertinent historical
information. A linear vesicular eruption on
exposed area of the arms or legs is most likely
secondary to:
a.diabetic neuropathy.
b.mycobacterium.
c.parasite.
d.poison ivy.
e.spirochete. Rosen 5th Fig 114-12

291. A Tzanck smear can establish a diagnosis of:


a. gonorrhea.
b. herpes.
c. syphilis.
d. staphylococcal cellulites.
e. candida.

292. A 4-year-old child presents with a patch of scaly alopecia for two weeks duration.
Recommended treatment would be:
a. clotrimazole (Lotrimin) cream.
b. griseofulvin oral suspension.
c. mupirocin (Bactroban) ointment.
d. systemic steroids.
e. cephalexin.

293. A 65-year old woman complains of blisters on her


face over the last two days which are quite painful.
She has no other complaints. Physical examination
reveals the rash shown here. Which of the following
examinations is also essential for appropriate
evaluation?
a. sedimentation rate
b. complete blood count
c. sinus MRI
d. lumbar puncture
e. slit lamp examination

294. Appropriate treatment of neuroleptic malignant syndrome includes:


a. antihistamines (both H1 and H2).
b. anticholinergics.
c. high-dose steroids.
d. dantrolene.
e. physostigmine.

295. Which statement about suicide is true?


a. Attempted suicide rates are equal in both sexes.
b. Completed suicide rates are equal in both sexes.
c. Marital status does not seem to have an effect on successful completion rates.
d. Older individuals are more likely to complete or succeed.
109

e. Wrist cutting is the most common attempted method of suicide.


110

290. d The primary lesions of contact dermatitis are papules, vesicles, or bullae on an erythematous
th
Rosen 5 , bed. Of the allergens, Rhus species are the most likely to cause bullous eruptions. Oozing,
Chapter 114, crusting, scaling, and fissuring may be found, along with lichenification in chronic lesions. The
p. 1647 distribution of the eruption depends on the specific contactant and may be localized,
asymmetric linear, or unilateral.
291. b HSV infections are often obvious, but if tehre is doubt or a need to confirm the diagnosis, a
Rosen 5th, simple Tzanck slide preparation helps reveal the characteristic multinucleated giant cells
Chapter 93, p. approximately 50% of the time.
1392
292. b Tinea capitis is a dermatophyte infection of the scalp. It is most commonly seen in children,
th
Rosen 5 , particularly African-American children. Clinically one sees areas of alopecia with broken-off
Chapter 114, hairs and scale at the periphery. The alopecia is patchy and usually nonscarring. Occasionally,
p. 1635-1636 tinea capitis is associated with an intense inflammatory response. After a diagnosis is
established, the current first-line therapy is griseofulvin. Topical treatment alone is not
effective.
293. e Herpes zoster, or shingles, is an infection caused by the varicella-zoster virus. It occurs
Rosen 5th, exclusively in individuals who have previously had chicken pox. Ocular complications occur in
Chapter 114, 20% to 70% of the cases involving the ophthalmic division of the trigeminal nerve. Eye
p. 1656 involvement produces anterior uveitis, secondary glaucoma, and corneal scarring. There is a
close correlation between eye involvement and vesicles located at the tip of the nose
(Hutchinsons sign).
294. d Treatment of neuroleptic malignant syndrome revolves around supportive care and
Rosen 5th, discontinuation of the offending medication. Agitation, psychomotor hyperactivity, and
Chapter 155, muscular rigidity should be treated aggressively with intravenous benzodiazepines. Dantrolene,
p. 2177-2178 which inhibits the release of calcium from the sarcoplasmic reticulum, has been advocated as an
adjunctive therapy for NMS, along with the dopamine antagonists bromocriptine and
amantadine. There is no proven benefit to any of these agents.
295. d Suicides are highest among older individuals, particularly elderly white men. White men
Rosen 5th, commit 73% of all suicides in the United States. Whites and Native Americans are much more
Chapter 109, likely to commit suicide than African Americans, Hispanics, or Asians. Marriage decreases the
p. 1576 likelihood of suicide, but separated or divorced persons have a higher rate of suicide than those
who never had a close relationship. Women attempt suicide three to four times more often than
men, whereas men are three to four times more likely to kill themselves. Men have a higher
incidence of alcoholism and tend to use more lethal methods, such as firearms. Most completed
suicides involve firearms (70%), whereas most attempted suicides involve the ingestion of
drugs or poisons (72%).
111

296. A 70-year-old male complaints of difficulty sleeping followed by periods of sleeping for
over 20 hours daily, fatigue, difficulty concentrating and forgetfulness. He denies weight
loss or new weaknesses. Upon further questioning, you find that his wife of 45 years
passed away 4 months ago. The most likely cause of these symptoms is:
a. cancer.
b. dementia.
c. depression.
d. hypothyroidism.
e. stroke.

297. Symptoms of major depression include:


a. sleep disturbances.
b. dysphoria.
c. agitation.
d. lack of concern about personal hygiene.
e. all of the above.

298. A 52-year-old schizophrenic woman has been treated with fluphenazine (Prolixin) for
many years. Today she complains of uncontrollable movements of her face and tongue.
She is probably suffering from:
a. acute psychotic break.
b. akathisia.
c. dystonia.
d. neuroleptic malignant syndrome.
e. tardive dyskinesia.

299. The most appropriate management of a comatose patient who overdosed on his lithium is:
a. activated charcoal.
b. adenylate cyclase stimulators.
c. chelation with EDTA.
d. hemodialysis.
e. aggressive fluid replacement.

300. A 44-year-old young woman complains of the sudden onset of blindness. She was visiting
her debilitated father in the hospital. Her husband confides that she is distraught by her
fathers diagnosis of metastatic colon cancer and the sight of him distresses her greatly.
Her physical exam, including pupillary response and retinal exam is completely normal.
Which statement is true regarding her likely diagnosis?
a. The disorder usually presents with multiple complaints.
b. It is more prevalent in well educated and higher socioeconomic social status.
c. It usually involves orthopedic or neurologic complaints.
d. The patient is typically older.
e. Patients are typically very distressed by their symptoms.
112

296. c The essential features of major depression are a persistent dysphoric (sad) mood or pervasive
loss of interest in usual activities, lasting for at least two weeks. Vegetative symptoms
297. e involve physiologic functioning and include loss of appetite and weight, sleep disturbance,
Rosen 5th, fatigue, inability to concentrate, and psychomotor agitation or retardation. The depression may
Chapter 105, begin gradually or rapidly but usually will have been present for several weeks before the
p. 1550-1551 patient comes for treatment.
298. e Tardive dyskinesia is an involuntary movement disorder that is idiosyncratically associated with
Rosen 5th, chronic therapy with the typical antipsychotic agents. Symptoms classically include
Chapter 155, involuntary, repetitive, choreoathetoid movements of the face, mouth, and tongue, and may be
p. 2175 mild or severe and debilitating; uncommonly, the limbs and trunk are also affected. Elderly
patients, especially those with diabetes mellitus and organic brain damage, seem to be at
increased risk. The overall incidence is approximately 5% per year of exposure to the
offending agent.
299. d Severely lithium-poisoned patients in coma or with seizures may require urgent airway
Rosen 5th, interventions and anticonvulsant therapy with benzodiazepines, phenobarbital, and propofol as
Chapter 154, necessary. Many of these patients will be volume depleted. Although frank hypotension is
p. 2173 unusual, fluid resuscitation is needed and should include normal saline. The goal of saline
administration is to restore glomerular filtration, normalize urine output, and enhance lithium
clearance. Forced diuresis does not assist in lithium clearance. Dialysis may be useful for
patients with an acute overdose who have decreased level of consciousness, seizures, or levels
greater than 4.0 mEq/L, although asymptomatic patients with higher levels have been managed
without dialysis. For chronic toxicity, dialysis is helpful in symptomatic patients with levels
greater than 2.5 mEq/L.
300. c Also known as hysterical neurosis, conversion type, the rare conversion disorder is
Rosen 5th, characterized by the sudden onset and dramatic presentation of a single symptom, typically
Chapter 107, simulating some nonpainful neurologic disorder for which there is no pathophysiologic or
p. 1566 anatomic explanation. The most common ages of presentation are adolescence or early
childhood, although other age groups are also affected. The symptoms, generally conforming
to the patients own idiosyncratic ideas about illness, are not under the patients voluntary
control. The most common presentations to the ED include pseudoseizures, syncope or coma,
and paralysis or other movement disorders. Most patients are women, except for military
service and industrial accidents. Symptoms tend to be of sudden onset, waxing and waning in
response to environmental stresses. Patients may describe their conversion symptoms with a
lack of appropriate concern about their profound bodily dysfunction (la belle indiffrence).
113

301. You are treating a 2-year-old boy who is in cardiac arrest. Your boss borrowed the Broselow tape
for a paramedic lecture, but thats okay because you know that:
a. atropine is the first drug given for bradycardia in all age groups.
b. the minimum dose of atropine is 0.1 mg.
c. the initial defibrillation energy is 5 j/kg.
d. the curved Macintosh blade will be more effective than the straight Miller when you are
intubating.
e. the tidal volume necessary to ventilate children is 5 to 8 ml/kg.

302. Which statement about appendicitis in the pediatric age group is true?
a. Perforation and peritonitis are uncommon in younger children.
b. The presence of diarrhea effectively excludes appendicitis.
c. The condition most commonly mistaken for acute appendicitis is mesenteric adenitis.
d. Most abdominal x-rays in patients with appendicitis reveal a fecalith
e. The peak age of incidence in appendicitis is 6 to 10 years of age.

303. An Apt test is apt to:


a. differentiate central causes of blindness from peripheral causes.
b. confirm knee stability.
c. differentiate seizures from pseudo-seizures.
d. differentiate maternal from fetal blood.
e. determine the presence of pathogenic bacteria in stool samples.

304. A 7-year-old boy has fever, abdominal pain, and vomiting over the past 18 hours. He has pain in
his right lower abdomen when he coughs, and you find McBurneys point to be exquisitely tender.
Suspicious that he has a perforated appendix, you page the surgeon and instruct the nurse to give
the patient:
a. ceftriaxone (Rocephin).
b. ampicillin and gentamicin.
c. ampicillin, gentamicin, and metronidazole (Flagyl.
d. a macrolide and a fluoroquinolone.
e. levofloxacin (Levaquin).

305. Hirschsprung's disease:


a. accounts for 50% of intestinal obstruction in young infants.
b. requires barium enema to make the diagnosis.
c. is more frequently found in females.
d. is usually diagnosed early in infancy.
e. usually presents with a massive GI bleed.

306. The most common cause of intestinal obstruction between 3 months and 6 years of age is:
a. cystic fibrosis.
b. Hirschsprungs disease
c. intussusception
d. Meckels diverticulum.
e. natal adhesions.
114

301. b The recommended dose of atropine is 0.02 mg/kg IV, with a minimum dose of 0.1 mg
th
Rosen 5 , to prevent paradoxical bradycardia (p. 89). Initially, 2 joules (watts per second)/kg
Chapter 8, should be used for defibrillation (p. 97). Epinephrine is indicated in all cardiac arrest
pages listed settings, including asystole, pulseless electrical activity, and ventricular fibrillation
in text (p.84). Because of the superior and anterior position of the tracheal opening, as well as
the large tongue and relatively large floppy epiglottis, visualization of the glottic
opening can be obstructed in children less than 4 years of age if a curved blade is used.
The straight blade lifts these tissues so the glottic opening can be recognized (p. 91).
The tidal volume necessary to ventilate children is the same as that for adults: 10 to 15
mL/kg (p. 93).
302. c The condition most commonly mistaken for acute appendicitis in children is
Rosen 5th, mesenteric adenitis. The rate of appendiceal perforation before surgery varies from
Chapter 165, 17% to 40% and is inversely related to age with higher rates in younger age groups.
p. 2311-2312 The presence of diarrhea does not rule out appendicitis. An appendicolith is found in
about 10% of plain films. The peak age of incidence in appendicitis is 9 to 12 years of
age.
303. d In young neonates an Apt test can be performed to determine if a bloody stool contains
Rosen 5th, maternal or fetal blood. Add 1% NaOH to the bloody stool. Fetal hemoglobin resists
Chapter 165, oxidation and remains pinkish-red, whereas maternal hemoglobin changes to a dark
p. 2307 brown color.
304. c Once the decision has been made to operate, prophylactic antibiotics should be given
Rosen 5th, to cover gram-negative and anaerobic organisms. Intravenous second-generation
Chapter 88, cephalosporins, such as cefotetan of cefoxitin, provide good coverage. If the appendix
p. 1298 has already perforated, more complete coverage with ampicillin, gentamicin,
clindamycin, or advanced penicillin combinations such as piperacillin and tazobactam
may be preferred.
305. d Hirschsprung's disease is usually diagnosed in infancy. Although a barium enema can
Rosen 5th, reveal a narrowed aganglionic segment with proximal dilation, which is highly
Chapter 165, suggestive of Hirschsprungs disease, the diagnosis is confirmed by biopsy or
p. 2305-2306 manometry. It accounts for approximately 20% of cases of partial intestinal
obstruction early in infancy. It occurs at a rate of 1 in 5,000 live births and is 4 to 5
times more common in boys. Definitive therapy is surgical resection of the
aganglionic segments.
306. c Intussusception is the most common cause of intestinal obstruction in children less
Rosen 5th, than 2 years of age, and occurs most commonly in infants between 5 and 12 months of
Chapter 165, life. There is an estimated incidence of 1 per 2000 children under the age of 15 years
p. 2304 with a male predominance. Siblings of affected children have a relative risk 15 to 20
times higher than the general population. Mortality for untreated intussusception is
high.
115

307. In an infant with newly diagnosed pyloric stenosis, the classic electrolyte findings of patients with
severe symptoms is:
a. hyperchloremic, hypokalemic alkalosis.
b. hyperkalemic, hypernatremic acidosis.
c. hypernatremic, hypokalemic acidosis.
d. hypochloremic, hypokalemic alkalosis.
e. hypochloremic, hyponatremic acidosis.

308. Which of the following is correct regarding developmental milestones in children?


a. 1 month: holds head in midline
b. 4 months: rolls
c. 6 months: uses pincer response
d. 9 months: walks alone
e. 12 months: creeps up stairs

309. Sudden infant death syndrome (SIDS):


a. is most common in the neonatal period, peaking at 2 months of life, and then tapering off by 4
months.
b. is more common in countries where infants sleep supine.
c. is primarily a cardiac event.
d. does not occur more frequently in infants with apnea of prematurity.
e. means a higher risk for similar occurrence in siblings.

310. A clean wound located in a highly vascular area, such as the face, can safely be closed if the patient
presents within a maximum of:
a. 6 hours.
b. 10 hours.
c. 18 hours.
d. 24 hours.
e. 48 hours.

311. A 34-year-old man has extensive lacerations of his left ear after he disagreed with a local bartender
over the accuracy of his account. In preparing to reassemble his damaged ear, you tell the patient
that you will numb his entire ear by:
a. injecting lidocaine without epinephrine and raising a wheal about the entire base of the ear.
b. injecting lidocaine with epinephrine directly into the wound.
c. injecting lidocaine without epinephrine directly into the wound.
d. doing a hemi-facial nerve block.
e. performing conscious sedation, as there is no good way to regionally block an ear.

312. The best way to control the pain from a femoral shaft fracture is by using:
a. Bucks traction.
b. a femoral nerve block.
c. a combination of injectable opioid and non-narcotic pain medicine.
d. a steroid injection of the trochanteric bursa.
e. high-dose intravenous narcotic injection.
116

307. d Extensive and protracted vomiting in pyloric stenosis may lead to hypokalemia and
th
Rosen 5 , hyponatremia. More striking decreases occur in chloride concentration and an increase
Chapter 165, in pH and carbon dioxide content. This constitutes the characteristic changes of
p. 2297-2301 hypochloremic alkalosis.
308. b A one month old can raise his head only slightly from the prone position. A 2-month
th
Rosen 5 , old can hold his head in midline and lift his chest off the table. A 4 month old can
Chapter 160, roll,. At 6-months an infant can sit unsupported, but it is not until 9 months that the
p. 2228 pincer grasp begins and the infant starts to cruise and crawl. Walking alone typically
occurs at 1 year of age and creeping up stairs occurs at 15 months of age.
309. d Apnea of prematurity is not a risk factor for SIDS. SIDS is rare in the first month of
Rosen 5th, life, probably because the neonate has a better anaerobic capacity for survival, and with
Chapter 170, a gasp may be able to raise his or her arterial PO2 over 20 mmHg and continue
p. 2392-2396 breathing. Victims range in age from 1 month to 1 year, with peaks at 2 months and
at 4 months. Epidemiologic studies indicate that the incidence of SIDS is lower in
countries where infants sleep supine or in the side-down position. Death is a
respiratory rather than cardiac event, and some potential SIDS victims may be
successfully resuscitated with ventilation alone. Factors that do not appear to increase
the risk of SIDS include maternal alcohol use, recent diphtheria-pertussis-tetanus
(DPT) or other vaccine administration, being a sibling of a SIDS victim, sickle cell
disease, and bronchopulmonary dysplasia.
310. d Although the incidence of bacterial contamination and the risk of infection increase
Rosen 5th, with injuries more than 4 hours old, clean wounds in highly vascular areas (scalp, face,
Chapter 35, and neck) have an incidence of infection so low that primary closure may be possible
p. 318 even up to 24 hours after injury.
311. a Anesthesia of the ear is best achieved by raising a wheal with lidocaine with no
th
Rosen 5 , epinephrine about the entire base of the ear. This anesthetizes all but the external canal
Chapter 35, and concha, which, if involved with the injury, require direct infiltration with plain
p. 321 lidocaine.
312. b Femoral nerve block is an effective technique for relieving pain of a femoral shaft
Rosen 5th, fracture and is useful in the multitrauma patient when minimizing narcotics is
Chapter 49, important. The femoral nerve is lateral to the femoral artery at the inguinal ligament
p. 655 and innervates the anterior thigh, the periosteum of the femur, and the knee joint.
117

313. A 74-year-old retired staff surgeon slipped on some urine while visiting his
wife, who recently had a mastectomy. His right leg is externally rotated and Rosen 5th, Figure
49-14
shortened, and his x-ray is shown (Figure 49-14). He demands to see the
orthopedic surgeon who performed the original procedure, but you tell him
that the orthopedic doctor recently moved to Arizona because of the cost of
his malpractice insurance. He reluctantly allows you to attempt reduction.
You know the best way to get the prosthesis back is through:
a. longitudinal traction with simultaneous flexion and internal rotation, then
abduction.
b. longitudinal traction with simultaneous flexion and external rotation, then
adduction.
c. longitudinal compression with simultaneous extension and internal
rotation, then abduction.
d. longitudinal traction with simultaneous extension and external rotation, then abduction.
e. longitudinal compression with simultaneous extension and external rotation, then adduction.

314. Herpetic whitlow:


a. is usually caused by HSV-2 virus, especially in health-care workers.
b. heals more rapidly after incision and drainage.
c. is frequently painful and accompanied by axillary adenopathy.
d. usually involves several digits simultaneously.
e. clears rapidly with topical acyclovir.

315. A 27-year-old woman suffered a deep laceration of her left calf. You cannot see the base of the
wound. She tells you it feels like theres something in there. You know that:
a. patients who report the sensation of something in the wound should be taken seriously, as their
perceptions are usually correct.
b. ultrasound is the gold standard for finding foreign bodies.
c. glass must be leaded for standard radiographs to find.
d. plain radiographs find less than 50% of foreign bodies.
e. it is not essential to find foreign bodies, as they inevitably work their way out.

316. The drug of choice to treat Rocky Mountain Spotted Fever in a 6 year-old girl is:
a. azithromycin.
b. chloramphenicol.
c. doxycycline.
d. cephalexin.
e. linezolid.

317. Most cases of infant botulism are caused by ingestion of:


a. well-water.
b. canned baby food.
c. inadequately prepared instant formula.
d. raw chicken.
e. raw honey.
118

313. a Treatment for the dislocation is early closed reduction, usually under conscious
th
Rosen 5 , sedation. Strong, in-line traction is done with simultaneous flexion and internal
Chapter 49, rotation. Finally, the hip is abducted once the head clears the rim of the acetabulum.
p. 665-666
314. c Herpetic whitlow is a primary or recurrent HSV infection of the finger. HSV-1 is seen
th
Rosen 5 , in health care workers who are exposed to infected oral secretions. HSV-2 is more
Chapter 43, common among adults due to digital/genital contact in the community. The disease is
p. 530-531 usually limited to a single digit. Herpetic whitlow is frequently painful and
accompanied by axillary adenopathy. Vesicles, which may be recognizable early in
the course of the disease, coalesce and may appear to contain pus, but actually contain
necrotic epithelial cells causing the purulent appearance. Whitlow may be
misdiagnosed as a paronychia and incised, which may delay healing or allow a
secondary infection to occur. For patients with frequent painful recurrences
suppressive therapy with systemic acyclovir may be effective. Topical acyclovir has
not been shown to be effective in either the treatment or prophylaxis of this disorder.
315. a If a foreign body is suspected but not found during exploration of a wound, a plain film
Rosen 5th, should be ordered first, since plain radiography will detect as many as 80 to 90% of all
Chapter 53, foreign bodies. When a foreign body is suspected but cannot be palpated or visualized,
p. 770-771 plain films should be performed in an attempt to determine the location, size, and
number of foreign objects present. Even when the foreign body is not visualized,
radiographs may show secondary changes (e.g., pulmonary air trapping) providing
clues to foreign body presence. To assist in the localization of the object, two views
anteroposterior (AP) and lateralare usually necessary. Objects such as metal or
gravel, which are denser than the tissue in which they are embedded, are usually easy
to visualize on plain radiographs. However, organic material, which has a density
similar to that of human tissue, may not be seen on plain films. Besides plain
radiographs, other techniques may be useful for ED workup of foreign bodies. These
modalities include fluoroscopy, contrast studies, computed tomography (CT) scanning,
magnetic resonance imaging (MRI), and ultrasound (US).
316. b Tetracycline or doxycycline is the antibiotic of choice to treat Rocky Mountain Spotted
Rosen 5th, Fever, but chloramphenicol can be used for patients allergic to tetracycline and in
Chapter 128, children younger than 9 years of age. A usual course is 6 to 10 days and should
p. 1895 continue for 72 hours after defervescence. Sulfa drugs should be avoided, as they can
exacerbate the illness. Rickettsiae are routinely resistant to penicillins, cephalosporins,
aminoglycosides, and erythromycin.
317. e Adults can get botulism not by bacterial infestation but by ingestion of botulinum toxin
Rosen 5th, through exposure to foods such as home-canned vegetables. Unlike adults, infants,
Chapter 123, whose guts are not colonized fully, are susceptible to infection with viable bacteria that
p. 1795 elaborate toxin. Parents of infants should be questioned for possible ingestion of C.
botulinum spores, commonly transmitted by feeding honey, but occasionally reported
with corn syrup.
119

318. Synchronized intermittent ventilation (SIMV) prevents:


a. decreased renal blood flow.
b. decreased venous return to the heart and decreased cardiac output.
c. nosocomial infections of the lungs and sinuses.
d. respiratory alkalosis.
e. stacking breaths, which can lead to barotrauma.

319. A majority of acute hemolytic transfusion reactions result from:


a. antibodies in the donor's type O serum against the recipient's A or B antigen.
b. antigen-antibody reaction to the Rh factor.
c. emergent use of uncrossed matched type O-negative blood.
d. transfusion of the wrong unit of blood due to clerical error.
e. undetected leukocyte antibodies.

320. The safest place in the body for hydrocarbons is the:


a. stomach.
b. colon.
c. duodenum.
d. jejunum.
e. ileum.

321. Rhabdomyolysis is defined by:


a. end organ damage.
b. total serum creatine kinase more than five times normal.
c. total serum creatine kinase higher than 1000.
d. total serum myoglobin more than three times normal.
e. total plasma myoglobin higher than 1 mg/dl.

322. Eikenella corrodens is a worrisome bacteria found most commonly in injuries caused by:
a. cat bites.
b. foot punctures through an old sneaker.
c. lionfish spines.
d. human bites.
e. camel bites.

323. Twenty-four hours after she was exposed to an unknown amount of radiation, a 45-year-old woman
has an absolute lymphocyte count of 1500. This means she:
a. will almost certainly die, no matter what treatment you provide.
b. has a 50:50 chance of survival.
c. can survive, but only after aggressive therapy.
d. had exposure in excess of 2000 rad.
e. requires no special care, as she is expected to survive without complications.

324. An example of an ester local anesthetic is:


a. lidocaine.
b. tetracaine.
c. mepivacaine.
d. bupivacaine.
e. prilocaine.
120

318. e Synchronized intermittent ventilation (SIMV) supports a patient's spontaneous breath


th
Rosen 5 , at a preset rate that prevents a mechanical breath being delivered at the same time as a
Chapter 2, spontaneous breath (stacking). The other choices are all potential adverse effects of
p. 22 positive-pressure ventilation.
319. d Acute intravascular hemolytic transfusion reactions are immediate and life threatening.
th
Rosen 5 , They are usually the result of ABO incompatibility, most often due to clerical error
Chapter 5, (e.g., mislabeling of blood product, misidentification of patient). Because RBCs of
p. 50 type O do not have A or B antigens on their surface, they are not agglutinated or
hemolyzed by anti-A or anti-B antibodies. Less severe and more common febrile
transfusion reactions are due to antileukocyte and antiplatelet antibodies, seen in
multiply transfused patients.
320. a The aphorism, the safest place in the body for hydrocarbons is the duodenum, holds
Rosen 5th, true for most hydrocarbons regardless of the volume ingested. There is a high toxicity
Chapter 152, to the lungs, but low systemic toxicity to the gastrointestinal track, with low potential
p. 2162 for GI absorption.
321. b A specific creatine kinase level cannot define rhabdomyolysis. In general, however, a
th
Rosen 5 , creatine kinase level greater than five times normal is diagnostic, but levels as high as
Chapter 121, several hundred thousand have been reported.
p. 1766
322. d Eikenella corrodens, a facultatively anaerobic gram-negative rod harbored in human
th
Rosen 5 , dental plaque, is found in 25% to 29% of human closed fist injury infections. It acts
Chapter 54, synergistically with aerobic organisms, most frequently streptococci, and is thought to
p. 781 account for greater morbidity in these wounds. E. corrodens is susceptible to
penicillin, ampicillin, second- and third-generation cephalosporins, carbenicillin,
tetracycline, and the fluoroquinolones.
323. e The lymphocyte count 24 hours postexposure is useful in predicting the patients
Rosen 5th, clinical course. If the lymphocyte count is maintained above 1200/mL, it is unlikely
Chapter 140, the patient has received a lethal dose of radiation and no clinical support is required. If
p. 2059-2060 the absolute lymphocyte count falls between 300 and 1200 at 48 hours, the possibility
of exposure to a lethal dose of radiation should be suspected. Levels less than 300 are
critical and require heroic measures.
324. b Local anesthetic agents are chemical compounds that consist of an aromatic and an
Rosen 5th, amine group separated by an intermediate chain. The class that has an ester link
Chapter 182 between the intermediate chain and aromatic portion are called amino esters; procaine,
p.2571 chloroprocaine, and tetracaine are the only ones in use today. Amides have an amide
link and are more commonly used; lidocaine, mepivacaine, prilocaine, bupivacaine,
and etidocaine are examples (Notice that they all have two is). Esters are unstable in
solution and are metabolized in the body by the plasma enzyme cholinesterase.
Enzymes in the liver destroy the amides after absorption.
121

325. The LD50 from exposure to ionizing radiation is defined as the dose of penetrating ionizing
radiation that will result in the deaths of 50% of the exposed population within 60 days. In humans
who receive aggressive post-exposure therapy, this number is generally accepted to be:
a. 0.5 Gy (50 rad).
b. 1.0 Gy (100 rad)
c. 4.5 Gy (450 rad).
d. 6.0 Gy (600 rad)
e. 10.0 Gy (1000 rad).

326. Hyperbaric oxygen therapy is contraindicated in the patient with:


a. acute sinusitis.
b. pregnancy.
c. disulfiram (Antabuse) therapy.
d. epilepsy.
e. metallic prostheses.

327. Chondromalacia patellae:


a. is a traumatic syndrome of the patella ligament.
b. is caused by patellofemoral malalignment, leading to excessive lateral pressure on the articular
cartilage.
c. is most common in adolescent male athletes
d. causes joint effusion and generalized knee pain.
e. is made worse with descending stairs and squatting down.
Rosen 5th, Figure 50-9
328. Several young men coveted the jacket previously worn by the
patient whose x-ray is shown here (Figure 50-9). He refused to
relinquish this valued possession, having worked long and hard
at a menial position in order to earn the money to honestly
purchase it. After forcefully removing the aforementioned
jacket, the assailants jumped up and down on his leg a few times.
The nerve most likely to be affected is the:
a. peroneal.
b. superficial femoral.
c. deep femoral.
d. tibial.
e. sciatic.

329. A normal reticulocyte count is:


a. 0.5 1.5%
b. 2.0 5.0%
c. 6.0 7.5%
d. 7.5 10.0%
e. >10%
122

325. c The median lethal whole body dose (i.e., the dose that is lethal for 50% of test
th
Rosen 5 , subjects)for human beings is estimated to be about 4.5 Gy if victims are hospitalized
Chapter 140, and receive aggressive care. With whole-body exposure of more than 10 Gy, mortality
p. 2059 is nearly 100% unless bone marrow transplantation is performed.
326. c Very few absolute contraindications to HBO therapy exist. Treatment with
th
Rosen 5 , doxorubicin, cisplatin, bleomycin, disulfiram, and mafenide acetate is considered a
Chapter 194, contraindication to HBO therapy because of potentially toxic effects when combined
p. 2700 with HBO. An untreated pneumothorax is an absolute contraindication because of the
concern that it may progress to a tension pneumothorax, particularly during
decompression.
327. b The patellofemoral pain syndrome refers to the clinical presentation of anterior knee
Rosen 5th, pain related to changes in the patellofemoral articulation. The term chondromalacia
Chapter 50, has been used imprecisely to define this syndrome, but this is a pathologic term that
p. 688-689 refers to softening of the articular cartilage. A typical presentation is nonspecific
anterior knee discomfort that is nonradiating and occurs in a teenage girl. The knee is
more painful with prolonged flexion, and the discomfort is typically accentuated by
stair climbing and kneeling.
328. a The popliteal neurovascular bundle, composed of the popliteal artery, popliteal vein,
Rosen 5th, and common peroneal nerve, runs posteriorly behind all bony and ligamentous
Chapter 50, structures in the popliteal fossa. The popliteal artery is fixed in the fibrous tunnel of
p. 689 the adductor magnus hiatus proximally and traverses the fibrous arch of the soleus and
interosseous membrane distally. In essence, it is tethered to both the femur and tibia,
and its inherent immobility renders it susceptible to injury during dislocation. At that
time, the popliteal artery may be stretched, lacerated, or contused. Because of the
parallel course of the popliteal vein and peroneal nerve, they are vulnerable to a similar
injury.
329. a When the late normoblast extrudes its nucleus, it still contains a ribosomal network,
Rosen 5th, which identifies the reticulocyte. The reticulocyte retains its ribosomal network for
Chapter 115, about 4 days, of which 3 are spent in the bone marrow and 1 in the peripheral
p. 1665 circulation. The red blood cell matures as the reticulocyte loses its ribosomal network
and circulates for 110 to 120 days. The erythrocyte is then removed by macrophages
that detect senescent signals. Thus at any given time, approximately 1% of circulating
erythrocytes can be identified as reticulocytes.
123

330. A 56-year-old man was found unconscious in a nearby wooded area. Medics report that his dog
was lying dead next to him. As you undress him for
examination, you see these skin findings and realize
the cause of his coma is exposure to:
a. cyanide.
b. hydrofluoric acid.
c. herbicide aerial spray.
d. lightning.
e. carbon tetrachloride.

331. Metformin:
a. is more appropriately termed an antihyperglycemic than a hypoglycemic agent.
b. decreases peripheral sensitivity to insulin.
c. suppresses exogenous glucose uptake.
d. stimulates insulin secretion.
e. causes profound hypoglycemia if taken in error by a non-diabetic.

332. A 63-year-old woman developed acute left-sided weakness and garbled speech about 40 minutes
prior to arrival. Her head CT is unremarkable and she has no contraindications, so you choose to
treat her with:
a. low-molecular weight heparin, 1 unit/kg subcutaneously.
b. recombinant tissue plasminogen activator, 0.9 mg/kg, maximum 90 mg; first 10% administered as
bolus, with remaining amount infused over 60 minutes.
c. recombinant tissue plasminogen activator 90 mg IV bolus.
d. streptokinase IV.
e. nothing has been shown effective in this scenario.

333. A comatose 19-year-old man is brought from his college dormitory room. His roommate found
him at the end of classes today, but did not accompany him to the hospital. Fortunately he is
wearing a Medic Alert bracelet that identifies him as diabetic. He also smells like fingernail polish
remover. His bedside glucose is too high to be read, so you empirically start an intravenous with
normal saline solution after blood work is sent to the laboratory. You get a call several minutes
later with some panic values: Na+: 129, K+: 3.8; HCO3- : 12, glucose: 745. You know that the
corrected sodium is actually:
a. 119
b. 129
c. 134
d. 139
e. 145

334. A 47-year-old diabetic woman complains of a severe headache, fever, and a green-black nasal
discharge. She has generalized left facial swelling and redness, but does not have proptosis. You
see necrosis of the inferior turbinate, and a black spot on the soft palate. You know that she will
require aggressive treatment with intravenous:
a. vancomycin.
b. gentamicin.
c. cyclosporine.
d. dexamethasone.
e. amphotericin B.
124

330. d Feathering burns are not true burns because there is no damage to the skin itself. They
th
Rosen 5 , seem to be caused by electron showers induced by the lightning that make a fern
Chapter 136, pattern on the skin. These fern patterns are called Lichtenberg figures and are
p. 2014 pathognomonic for lightning.
331. a Metformin works by decreasing hepatic glucose output, leading to decreased insulin
th
Rosen 5 , resistance and lower blood glucose. Used alone, metformin does not cause
Chapter 120, hypoglycemia; it is contraindicated in patients with renal insufficiency and metabolic
p. 1759 acidosis, and must be used with caution in patients with hypoxemia, liver compromise,
and alcohol abuse.
332. b To date, the Food and Drug Administration (FDA) have approved only the use of
Rosen 5th, intravenous tPA for treatment of patients with acute ischemic stroke. These
Chapter 95, recommendations were based on the results of the National Institutes of Neurological
p. 1440-1441 Disorders and Stroke (NINDS) trial. The total dose of rt-PA is 0.9 mg/kg, with a
maximum dose of 90 mg; 10% of the dose is administered as a bolus, with the
remaining amount infused over 60 minutes. Heparin is frequently prescribed for
patients with acute ischemic stroke or TIAs, but its value is unproved. Streptokinase is
not recommended for use in patients with acute ischemic strokes.
333. d 745 100 = 645 / 1.6 = 10.3 + 129 = 139.
Rosen 5th, Redistributive hyponatremia is caused by osmotically active solutes in the extracellular
Chapter 119, space that draw water from the cell, diluting the serum sodium concentration.
p. 1724 Common situations causing such hyperosmolar states include hyperglycemia (e.g.,
diabetic ketoacidosis) and parenteral administration of mannitol or glycerol for the
management of intracranial hypertension or glaucoma. The measured serum sodium in
patients with hyperglycemia can be corrected by adding approximately 1.6 mEq/L for
every 100-mg/dl rise in the serum glucose over 100 mg/dl.
334. e Invasive fungal sinusitis is an aggressive opportunistic rhinocerebral infection that
Rosen 5th, affects immunocompromised hosts. Mucormycosis (Rhizopus) is usually associated
Chapter 70, with fever, localized nasal pain, and cloudy rhinorrhea. On examination, the affected
p. 982, 983 tissue (usually the turbinates) appears gray, friable, anesthetic, and nonbleeding
because of infarction caused by mucormycotic angioinvasion. In advanced cases the
tissues are necrotic and black, and the infection spreads beyond the sinus. Acute
fulminant fungal sinusitis requires emergent consultation and admission for IV
antifungal therapy and aggressive surgical debridement.
125

335. This 56-year-old woman was started on a new medication Rosen 5th, Figure 114-8
last week after she visited a walk-in clinic. Today she is
toxic in appearance, and large sheets of skin are peeling
from her back (Figure 114-8). The old chart is not
available, and the patient only remembers that her face
hurt. You recognize that the most likely culprit is:
a. prednisone for Bells palsy.
b. doxycycline for Lyme-induced Bells palsy.
c. clindamycin for dental abscess.
d. carbamazepine for trigeminal neuralgia.
e. cephalexin for erysipelas.
126

335. d The main feature of nonstaphylococcal-induced toxic epidermal necrolysis (TEN), or


th
Rosen 5 , Lyells disease, is the separation of large sheets of epidermis from underlying dermis.
Chapter 114, The full thickness of epidermis is involved. The two conditions are easily
p. 1643 histologically distinguishable with a skin biopsy. A mortality rate of 15% to 20% is
expected with this condition. Drugs including the long-acting sulfa drugs, penicillin,
aspirin, barbiturates, phenytoin, carbamazepine, allopurinol, and nonsteroidal anti-
inflammatory drugs are an important cause of TEN.
127

Abdominal and Gastrointestinal Disorders


336. In adults the most common source of upper gastrointestinal bleeding is:
a. Mallory-Weiss tear.
b. peptic ulcer disease.
c. esophageal varices.
d. esophagitis.
e. gastric erosions.

337. Melana is:


a. present in 95% of upper gastrointestinal bleeds.
b. from blood that has been in the GI tract for at least 24 hours.
c. present in approximately one-third of lower gastrointestinal bleeds.
d. may occur from as little as 10 mls of blood in the GI tract.
e. stool that will remain black and tarry for no more than 24 hours after bleeding stops.

338. Antiemetic drugs work by:


a. suppressing the vomiting center in the brain.
b. depressing the chemoreceptor trigger zone.
c. inhibiting impulses to the chemoreceptor trigger zone from the vestibular apparatus.
d. inhibiting impulses to the vomiting center from peripheral receptors.
e. all of the above.

339. In adults the most common cause of nausea and vomiting is:
a. cholecystitis.
b. medication.
c. kidney stones.
d. migraine headache.
e. myocardial infarction.

340. A patient is sent to the emergency department by his primary doctor to rule out gastric outlet
obstruction. After talking to him, you also believe that gastric outlet obstruction is the most likely
diagnosis given his symptom of:
a. emesis that is both bilious and bloody.
b. abdominal distention with normoactive bowel sounds.
c. vomiting during meals.
d. vomiting made worse by lying supine.
e. vomiting of material eaten more than 12 hours previously.

341. The most common cause of neuromuscular swallowing dysfunction is:


a. stroke.
b. myasthenia gravis.
c. poliomyelitis.
d. polymyositis.
e. tetanus.
128

336. b In adults the most common causes in descending order of upper gastrointestinal
th
Rosen 5 , bleed are: peptic ulcer disease , gastric erosions, varices, and Mallory-Weiss tears.
Chapter 23, The top three account for approximately three fourths of adult patients with upper GI
p. 194-195 bleed..
337. c Melena, or black tarry stool, occurs from approximately 150-200 ml of blood in the GI
th
Rosen 5 , tract. Melena is present in approximately 70% of upper GI bleeds and 33% of lower
Chapter 23, GI bleeds. Black nontarry stool may result from as little as 60ml of blood. Blood from
p. 195 the jejunum or duodenum must remain in the GI tract for at least 8 hours before turning
black. Stool may remain black and tarry for several days after bleeding has stopped.
338. e The chemoreceptor trigger zone area is rich in dopamine D2 receptors, which are
Rosen 5th, antagonized by drugs such as prochlorperazine, metoclopramide, and droperidol. The
Chapter 21, serotonin receptor has been found widely in the area postrema and the GI tract. It may
p. 179 act both directly and through the release of dopamine. Serotonin receptor antagonists
(ondansetron, alosetron, granisetron) have been shown to be effective in preventing
chemotherapy-induced nausea and vomiting. Concentrations of cholinergic and
histamine receptors are found in the lateral vestibular nucleus and are important in
motion sickness. Meclizine, diphenhydramine, and scopolamine act by antagonizing
these receptors.
339. b In adult medicine, nausea and vomiting are most often caused by medications. The
Rosen 5th, most common causes of nausea and vomiting are acute gastroenteritis, febrile systemic
Chapter 21 illnesses, and drug effects. Acute viral gastroenteritis is the most common GI disease
p. 178 in the United States.
340. e Vomiting of material eaten more than 12 hours previously is pathognomonic for outlet
th
Rosen 5 , obstruction. Bilious emesis essentially rules out a gastric outlet obstruction. High-
Chapter 21, pitched bowel sounds suggest a gastric outlet obstruction.
p. 180-181
341. a Neuromuscular diseases cause approximately 80 % of oropharyngeal dysphagias.
th
Rosen 5 , Strokes are probably the most common cause of neuromuscular dysphagia, and
Chapter 84, pharyngeal muscle weakness is often the mechanism. The second most common cause
p. 1245 of neuromuscular dysphagia is polymyositis or dermatomyositis. These disorders are
characterized by inflammatory and degenerative changes that produce weakness of the
palate, pharynx, and upper esophagus.
129

342. A 25-year-old man complains of a severe intermittent substernal chest pain. It started suddenly
while he was lying in bed. He admits to several months of difficulty swallowing solids and liquids,
and he occasionally has to stand when he eats to help food pass into his stomach. Physical
examination reveals an underweight male with no abnormal findings. His 12-lead ECG is normal.
His probable diagnosis is:
a. gastroesophageal reflux.
b. achalasia.
c. nutcracker esophagus.
d. spontaneous esophageal perforation (Boerhaaves syndrome).
e. duodenal ulcer.

343. In distinguishing chest pain caused by gastroesophageal disease from that caused by coronary
disease, the most helpful historical point is probably:
a. pain made worse by postural changes and swallowing.
b. pain precipitated by high emotional state.
c. pain which is almost always relieved by antacids.
d. pain precipitated by exercise and relieved by rest.
e. radiation of pain into the neck or the jaw.

344. The majority of esophageal perforations are:


a. spontaneous.
b. iatrogenic.
c. traumatic.
d. caused by caustic ingestion.
e. idiopathic.

345. One of the most common etiologies of peptic ulcer disease is:
a. dietary indiscretion.
b. Helicobacter pylori infection.
c. idiopathic.
d. increased parietal cell mass.
e. Zollinger-Ellison syndrome.

346. The description of abdominal pain may be helpful in its evaluation. For instance:
a. pain from ulcer disease is often described as colicky.
b. acute gastric volvulus is likely to present with mild pain.
c. peptic ulcer pain is classically described as a burning or gnawing quality.
d. pain from iatrogenic esophageal perforation always occurs immediately after the procedure.
e. pain from a perforated duodenal ulcer is usually appreciated in the epigastrium only and rarely
becomes generalized.

347. Treatment of choice for acute gastric volvulus is:


a. watchful waiting.
b. endoscopic decompression.
c. combined Trendelenberg / left lateral decubitus position.
d. attempted reduction by barium swallow.
e. surgery.
130

342. b Achalasia is a disorder in which there is a marked increase in the resting pressure of
th
Rosen 5 , the lower esophageal sphincter and absent peristalsis in the body of the esophagus.
Chapter 84, Dysphagia is common. Odynophagia, from esophageal spasm may be seen early in the
p. 1246 course of the disease. Symptoms often worsen with rapid eating and stress. A dilated
esophagus with air-fluid levels may be seen on chest x-ray.
343. a Esophageal pain is more likely to be positional and related to swallowing. Radiation
Rosen 5th, of pain is an inconstant finding in both esophageal and cardiac chest pain.
Chapter 84, Precipitation of pain by exercise and relief by rest may occur in pain from reflux or
p. 1238, 1240 ischemic heart disease. Emotional precipitation of pain occurs in reflux, although it is
also seen in coronary artery disease. Relief of chest pain from reflux by antacids is a
key point in the history, however the emergency physician should not place too much
weight upon this point as evidence against a cardiac etiology. The relief is often short
lived, and may recur in a short time.
344. b Spontaneous esophageal perforation accounts for only 15% of cases, with iatrogenic
Rosen 5th, injuries accounting for most of the remainder. These usually occur as a complication
Chapter 38, of upper endoscopy, dilation, sclerotherapy, or other GI procedures.
p. 406-409
Chapter 84,
p. 1236
345. b Peptic ulcer disease is considered to have two main etiologies: Helicobacter pylori
th
Rosen 5 , infection and NSAID use. Only about 1% of PUD is caused by increased levels of
Chapter 84, circulating gastrin from gastrin-secreting tumors (Zollinger-Ellison syndrome). These
p. 1241 patients have increased parietal cell mass and hypersecretion of acid.
346. c Colicky pain is rarely gastric or duodenal in origin. Classically ulcer pain is described
th
Rosen 5 , as nonradiating epigastric pain of a burning or gnawing quality. Primary volvulus may
Chapter 84, present with the sudden onset of severe pain. Pain from an iatrogenic perforation may
p. 1236, not appear until several hours after the procedure. Pain from a perforated duodenal
1242-1244 ulcer is usually appreciated first in the epigastrium but becomes generalized within a
short time
347. e Treatment of an acute gastric volvulus is surgical. Acutely, one should attempt
Rosen 5th, passage of a nasogastric tube, as this may occasionally reduce the volvulus. There
Chapter 84, have also been reports of reductions of volvulus using endoscopy, but this is best
p. 1244 reserved for patients who cannot tolerate surgery or who have no evidence of vascular
compromise.
131

348. A patient with a known exposure to hepatitis B presents with right upper quadrant abdominal pain
and scleral icterus. You know that her serum bilirubin must be at least:
a. 1.5 mg/dl.
b. 2.0 mg/dl.
c. 2.5 mg/dl.
d. 3.0 mg/dl.
e. 3.5 mg dl.

349. Of the tests listed, the one MOST ACCURATE in predicting a fulminant course of disease in a
patient with hepatitis is:
a. prolonged prothrombin time.
b. bilirubin higher than 10 mg/dl.
c. AST several times higher than ALT.
d. alkaline phosphatase more than 5 times normal.
e. lactate dehydrogenase more than 10 times normal.

350. A newly hired housekeeper is stuck with a hollow needle while picking up trash in a room recently
vacated by a patient known to have chronic hepatitis B. She has not yet started her primary
hepatitis prophylaxis series. You decide to:
a. send antibody levels (HBSAg) on the employee and determine treatment based on the results.
b. give her hepatitis B immune globulin (HBIG) 0.06 ml/kg and start the hepatitis B vaccine series,
with the first shot in the buttock.
c. give her hepatitis B immune globulin (HBIG) 0.06 ml/kg and start the hepatitis B vaccine series,
with the first shot in the deltoid.
d. give her hepatitis B immune globulin (HBIG) 0.6 ml/kg and start the hepatitis B vaccine series,
with the first shot in the buttock.
e. give her hepatitis B immune globulin (HBIG) 0.6 ml/kg and start the hepatitis B vaccine series,
with the first shot in the deltoid.

351. Ascites occurs as a consequence of:


a. portal hypertension.
b. impaired hepatic lymph flow.
c. hypoalbuminemia.
d. renal salt retention.
e. all of the above.

352. The organism which causes the majority of spontaneous bacterial peritonitis is:
a. Streptococcus pneumoniae.
b. Escherichia coli.
c. Klebsiella pneumoniae.
d. Staphylococcus aureus.
e. Pseudomonas abdominus

353. In children, the majority of acute esophageal obstructions are caused by:
a. balloons.
b. coins.
c. food.
d. strictures.
e. surgical procedures.
132

348. c Scleral icterus usually is not clinically apparent, even to the most astute observers,
th
Rosen 5 , until serum bilirubin is above 2.5 mg/dl. Muddy sclera, commonly found among
Chapter 85, African-American patients, may obscure or confuse this finding. An alternative in this
p. 1253 setting is examination of sublingual or subungual surfaces.
349. a Prothrombin time (PT) is a useful test to assess the degree of hepatic synthetic
th
Rosen 5 , dysfunction. Elevation of the PT may be the first clue of a complicated course. The
Chapter 85, white blood count (WBC) generally is not useful because values range from low
p. 1253 overall counts with a lymphocytic predominance to marked polymorphonuclear
leukocytosis. The typical case is associated with elevations (tenfold to 100-fold) of
serum aspartate aminotransferase (AST) and alanine aminotransferase (ALT), with
ALT generally elevated in excess of AST. Bilirubin may be moderately increased (5 to
10 mg/dl) and occasionally is markedly elevated (15 to 25 mg/dl). Alkaline
phosphatase and lactate dehydrogenase may be elevated but are rarely more than two
to three times normal.
350. c HBV immune globulin (HBIG) is recommended for immediate passive immunization
Rosen 5th, in individuals not previously immunized but exposed to potentially infective material.
Chapter 85, HBIG alone diminishes the risk of HBV infection by 75%. Unvaccinated, exposed
p. 1256 people should receive HBIG 0.06 ml/kg intramuscularly (IM) in addition to the HB
vaccine. Centers for Disease Control and Prevention (CDC) data suggest that optimal
immunologic response results from deltoid injection.
351. e Ascites occurs as a consequence of portal hypertension, impaired hepatic lymph flow,
Rosen 5th, hypoalbuminemia, and renal salt retention. Although ascites generally causes little
Chapter 85, more than unsightly abdominal distention and discomfort, it can become massive and
p. 1259 lead to respiratory compromise.
352.b E. coli account for approximately 47% to 55% of spontaneous bacterial peritonitis
Rosen 5th, infections, Streptococcus sp 18%-26%, Klebsilla sp 11%, and Streptococcus
Chapter 85, pneumoniae 8%-26%. Polymicrobial and anaerobic infections have been reported but
p, 1261 are not common. A third-generation cephalosporin such as cefotaxime is considered to
be a drug of choice with a demonstrated cure rate of 85%. An alternative would be an
ampicillin sulbactam combination. Ampicillin and an aminoglycoside is a rational and
effective combination but somewhat less desirable due to the risk of renal toxicity.
353. b Eighty percent of foreign body ingestions occur in the pediatric age group, with coins
Rosen 5th, being the most commonly impacted object. In adults, most impactions are due to
Chapter 84, pieces of food, particularly meat and bones.
p. 1234
133

354. Drug-induced liver injury usually causes damage by hepatocellular necrosis or cholestasis. A
commonly used drug which causes hepatonecrotic changes is:
a. amiodarone.
b. anabolic steroids.
c. chlorpromazine.
d. erythromycin estolate.
e. haloperidol.

355. The most likely cause of massive lower GI bleed in a child under two years of age is :
a. duodenal atresia.
b. gastric ulcer.
c. Mallory-Weiss tears.
d. Meckels diverticulum.
e. variceal bleeding secondary to portal hypertension.

356. The most common cause of massive lower GI bleed in adults is:
a. diverticular disease.
b. inflammatory bowel disease.
c. internal hemorrhoids.
d. left colon carcinoma.
e. right colon carcinoma.

357. Post-transfusion hepatitis is usually due to:


a. hepatitis A.
b. hepatitis B.
c. hepatitis C.
d. hepatitis D.
e. hepatitis E.

358. An ill-appearing, 47-year-old man complains of severe epigastric pain and vomiting. Lab studies
show a markedly elevated lipase. He is not a drinker. The cause is probably:
a. biliary tract disease.
b. cimetidine therapy.
c. hyperlipidemia.
d. indomethacin therapy.
e. infection.

359. The second most common cause of small bowel obstruction in the United States:
a. abscess.
b. congenital abnormality.
c. Crohns disease.
d. hernia.
e. regional enteritis.

360. Epigastric pain which is relieved by food, but which returns between meals is typical for:
a. sliding hiatal hernia.
b. esophagitis.
c. peptic ulcer.
d. pancreatitis.
e. Zollinger-Ellison syndrome.
134

353. a Although specific agents tend to cause damage characterized by a particular pattern of
th
Rosen 5 , injury, there is considerable overlap. Cellular necrosis is commonly associated with
Chapter 85, anesthetic agents (e.g., halothane), the antifungals amphotericin and ketoconazole, or
p. 1261-1262 the antidysrhythmic amiodarone. A cholestatic picture is characteristic of
chlorpromazine, haloperidol, anabolic or oral contraceptive steroids, and erythromycin
estolate.
355. d Esophagitis, gastritis, peptic ulcer disease, esophageal varices, and Mallory-Weiss tears
Rosen 5th, are common causes of upper GI bleeding in children . In children under the age of 2
Chapter 23, years, massive LGIB is most often a result of Meckels diverticulum or
p. 194-195 intussusception. Anal fissure, infectious colitis, inflammatory bowel disease, and
polyps are other causes of LGIB in children.
356. a At all ages rectal abnormalities are the most common cause of minor LGIB The most
Rosen 5th, common etiology of significant LGIB in adults is diverticulosis followed by
Chapter 23, angiodysplasia .
p. 194-195
357. c Although hepatitis C has been most prominently associated with transfusions, only
th
Rosen 5 , 10% of patients with this disease report a previous history of having received blood or
Chapter 85, blood products. The historic risk of hepatitis in patients receiving blood transfusions
p. 1252 was about 0.45% per unit transfused. The screening of donor blood for surrogate
markers (aminotransferases) and antibody to hepatitis C has decreased this risk to
0.03% per unit.
358. a Eighty percent of pancreatitis is caused by either gallstones (about 45%) or alcoholism
Rosen 5th, (about 35%). The exact mechanism of biliary pancreatitis is not clear. Either a stone
Chapter 86, within the bile duct applies transmural pressure on the pancreatic duct or a stone in the
p. 1273 common channel of the pancreatic duct and common bile duct causes obstruction.
Obstruction or pressure on the pancreatic duct causes bile reflux or increased pressure
of pancreatic secretions. Either mechanism leads to the activation of pancreatic
enzymes setting off the cascade of pancreatitis. Many cases that were presumed to be
idiopathic are actually due to small stones, sludge, or crystals that are too small to be
seen by ultrasound examination but may be noted on endoscopic retrograde
cholangiopancreatography (ERCP).
359. d In developed countries, postoperative adhesions are responsible for more than 50% of
Rosen 5th, all small bowel obstruction. It is estimated that as many as 15% of abdominal
Chapter 87, surgeries will eventually result in small bowel obstruction from adhesion. A
p. 1284 particularly high incidence of small bowel obstruction is found after gynecologic or
intestinal surgeries, as well as in those patients who have previously undergone surgery
in the presence of peritonitis or significant abdominal trauma. Other important causes
of small bowel obstruction include hernias and neoplasms, each with an incidence of
approximately 15%. Hernias can be either external or internal.
360. c The most common symptom of PUD is abdominal pain, occurring in 94% of patients
Rosen 5th, with an ulcer. Classically, ulcer pain is described as nonradiating epigastric pain of a
Chapter 84, burning or gnawing quality, although pain in the chest, back, or other areas of the
p. 1242 abdomen may be noted. Relief of pain after eating is a feature of gastric or duodenal
ulcer. Pain usually occurs 2 to 3 hours following a meal or at night.
135

361. You are evaluating a 57-year-old woman with epigastric discomfort, and suspect pancreatitis. You
know that:
a. endocrine products produced by the pancreas include amylase, lipase, trypsin, and other enzymes.
b. in patients with chronic pancreatitis, the degree of pain diminishes with the decreasing amount of
remaining pancreatic tissue.
c. the bile duct, portal vein, splenic vein, vena cava, aorta, and superior mesenteric artery all lie
posterior to the pancreas.
d. pancreatic tumors are usually painful and rapidly progressive.
e. the most common pancreatic disorder is pancreatitis.

362. The pain of uncomplicated small bowel obstruction is frequently:


a. a poorly localized, crampy abdominal pain.
b. a constant and severe abdominal pain.
c. associated with peritoneal signs.
d. mild in proximal intestinal obstructions.
e. associated with explosive profuse diarrhea.

363. A 6-year-old boy is brought by his frantic mother because she found blood on the toilet paper. You
know that the most common cause of this complaint in this age group is:
a. anal fissure.
b. cryptitis.
c. external hemorrhoid.
d. internal hemorrhoid.
e. perirectal abscess.

364. Which of these conditions may be drained appropriately in the emergency department?
a. horseshoe abscess.
b. intersphincteric abscess.
c. postanal abscess.
d. perianal abscess.
e. supralevator abscess.

365. Painful anorectal vesicles are most commonly caused by:


a. Chlamydia trachomatis.
b. Herpes simplex.
c. Neisseria gonorrhea.
d. papillomavirus.
e. Treponema pallidum.

366. The most prevalent parasite in the United States is:


a. Cyclospora cayetanensis.
b. Entamoeba histolytica.
c. Enterobius vermicularis.
d. Giardia lamblia.
e. Plasmodium vivax.
136

361. c The pancreas has both essential exocrine and endocrine functions. Exocrine products
th
Rosen 5 , include amylase, lipase, trypsin, chymotrypsin, elastase, carboxypeptidase,
Chapter 84, phospholipase, and other enzymes. The endocrine functions of the pancreas are
p. 1272, 1281 managed by insulin, glucagon, pancreatic polypeptide, and somatostatin. Advances in
care have decreased hospital mortality from pancreatitis from 25% to 30% to as low as
6% to 10% over the past 30 years. Anterior to the pancreas from right to left is the
transverse colon, the lesser sac of the omentum, and the stomach. Posteriorly lies the
bile duct, portal vein, splenic vein, vena cava, aorta, and superior mesenteric artery.
The presentation of pancreatic cancer is variable because progression of the disease is
indolent. Diabetes is the most common disorder of the pancreas, followed by
pancreatitis.
362. a Patients with small bowel obstruction typically complain of regularly recurrent bouts
Rosen 5th, of poorly localized abdominal pain lasting from seconds to minutes. Several hours of
Chapter 87, severe colicky pain in association with bilious vomiting and mild abdominal distension
p. 1285 is typical of proximal intestinal obstruction, whereas a day or two of progressively
worsening pain and more prominent abdominal distention is typical of distal
obstruction. A change in the description of the pain from intermittent and colicky to
constant and severe may signal the development of complications, such as intestinal
ischemia or perforation. The presence of peritoneal signs usually indicates late
obstruction with complications including strangulation. Patients with complete
obstruction eventually develop obstipation, while those with early or partial
obstruction may continue to pass stool or flatus.
363. a The most common cause of lower GI bleeding in children is an anal fissure. It is also
Rosen 5th, the most common anorectal problem in the pediatric patient.
Chapter 23,
p. 195
Chapter 91,
p.1347
364. d Perirectal and perianal abscesses are the most common (40% - 45%) and can be incised
th
Rosen 5 , and drained in the Emergency Department in the absence of complicating factors.
Chapter 9,
p. 1348-1349
365. b In herpes proctitis single or coalesced vesicles and ulcerations occur in the perianal
th
Rosen 5 , area and rectum. Syphilis (Treponema pallidum) may form a perianal ulcer or chancre.
Chapter 91, Chlamydia may form a perianal ulceration .Gonorrhea may form proctitis with a thick
p. 1353-1354 discharge. HPV (human papillomavirus)may form perianal warts.
366. c Enterobius vermicularis, also known as pinworm or seatworm, is perhaps the most
th
Rosen 5 , prevalent parasite in the United States. It is estimated that 20% to 30% of all children
Chapter 89, are infected with pinworms, and a total of 30 to 40 million persons are infected.
p. 1318
137

367. In differentiating a patient with cholangitis from one with simple cholecystitis, the most helpful
finding is:
a. fever.
b. hyperlipasemia.
c. jaundice.
d. Murphys sign.
e. elevated serum aminotransferases.

368. A 27 year-old man complains of fever and right upper quadrant pain. He is not jaundiced, and his
urine shows only trace bilirubin when dipped. Both ALT and AST are 50 to 60 times the norm.
You suspect:
a. hemolysis.
b. pancreatic disease.
c. biliary disease.
d. liver disease.
e. stomach cancer.

369. The most sensitive and specific test for diagnosing acute cholecystitis is:
a. Nuclear scintigraphy with technetium-99m-labeled IDA (iminodiacetic acid)
b. ultrasound.
c. oral cholecystogram.
d. non-contrast CAT scan of abdomen.
e. oral and intravenous contrast CAT scan of abdomen.

370. The test of choice for detecting the presence of gallstones is:
a. IDA scan.
b. non-contrast CAT scan of abdomen.
c. oral and intravenous contrast CAT scan of abdomen.
d. oral cholecystogram.(OCG)
e. ultrasound.(US)

371. Lipase:
a. hydrolyzes triglycerides.
b. exists only in the pancreas.
c. in acute pancreatitis peaks in 3 hours and levels fall over 24-48 hours.
d. degree of elevation can be used as a marker of pancreatitis disease severity.
e. is definitely an inferior test to amylase levels for pancreatitis.

372. This process in a 2-year-old child may be associated with:


a. celiac sprue.
b. Crohns disease.
c. cystic fibrosis.
d. Hirschsprungs disease.
e. intussusception.

373. The predominant symptom in a patient with diverticulitis is:


a. abdominal pain. Rosen 5th,
b. bloody stools. Figure 91-13
c. constipation.
d. diarrhea.
e. tenesmus.
138

367. c Although patients with cholangitis , in general, will have a higher fever and appear
th
Rosen 5 , more ill than those with cholecystitis, there can be considerable variability and overlap.
Chapter 85, The presence of jaundice is the clinical sign most helpful in differentiating these two
p.1267, 1269 disorders. An elevated bilirubin is characteristic of cholangitis and uncommon in
cholecystitis. Elevated serum aminotransferases may be found in both conditions.
368. d The typical patient with hepatitis has elevations (tenfold to 100-fold) of serum
Rosen 5th, aspartate aminotransferase (AST) and alanine aminotransferase (ALT), with ALT
Chapter 85, generally elevated in excess of AST. Bilirubin may be moderately increased, but
p. 1253 typically emerges several days to a week or more after the onset of clinical symptoms.
Alkaline phosphatase and lactate dehydrogenase may be elevated but are rarely more
than two to three times normal.
369. a Nuclear scintigraphy using technetium-99m-labeled iminodiacetic acid (IDA) is
Rosen 5th, generally considered the most sensitive and specific test for cholecystitis. Ultrasound
Chapter 85, imaging is the most useful test in the ED setting.
p. 1267
370. e Ultrasound is the procedure of choice for investigating the gallbladder. US imaging
th
Rosen 5 , can be performed rapidly, does not require the overnight delay necessary with an oral
Chapter 85, cholecystogram, is at least as sensitive, and provides the added use of permitting
p. 1266 evaluation of surrounding structures. Oral cholecystogram remains an option for the
patient with clinical suspicion of cholelithiasis in the uncommon circumstance when
US imaging has failed to identify the gallbladder.
371. a Lipase is a pancreatic enzyme that that hydrolyzes triglycerides. Lipase, like amylase,
Rosen 5th, exists in other tissue. In the presence of pancreatic inflammation it peaks at 24 hours
Chapter 86, and the levels fall over 8 to 14 days. The degree of elevation of amylase or lipase is
p. 1276 not a marker of disease severity. Several expert authors recommend using lipase over
amylase when seeking the diagnosis of pancreatitis.
372. c Rectal prolapse, or procidentia, is a disease of the extremes of age. In children,
Rosen 5th, procidentia may herald the presence of malnutrition or cystic fibrosis, and occurs
Chapter 91, during the first 2 years of life. Boys are more commonly affected than girls. Children
p. 1356 usually have a mucosal prolapse. The parent reports protrusion during defecation with
small amounts of mucus or blood. In adults complete procidentia is most common in
older women with a history of excessive straining. The cause is a laxity of attachment
structures and it is often accompanied by uterine prolapse or a cystocele.
373. a The predominant symptom in patients with classical sigmoid diverticulitis is persistent
Rosen 5th, abdominal pain . Initially the pain may be vague and generalized, but it quickly
Chapter 90, becomes well localized to the left lower quadrant. Low grade fever, malaise, and a
p. 1330 change in bowel habits, usually constipation, are common.
139

374. A 48 year-old man has severe abdominal pain that began suddenly as he was driving to work this
morning. He also complains of vomiting, which you see is not bilious. He has diffuse tenderness
on exam. His WBC count is 14,300 / mm. You suspect this is a surgical process rather than
medical because:
a. he has a fever.
b. his pain is diffuse.
c. his white blood cell count is elevated.
d. the pain started before the vomiting.
e. the vomiting started before the pain.

375. A 22 year-old homosexual male complains of rectal itching and a purulent discharge. Anoscopy
shows a proctitis. You suspect the causative organism to be:
a. Campylobacter.
b. Chlamydia.
c. Giardia.
d. N. gonorrhoeae.
e. Herpes simplex.

376. Which statement is correct concerning the patient with a small bowel obstruction?
a. increase in intraluminal pressure result in capillary and lymphatic obstruction.
b. early bowel distention decreases epithelial secretory activity and decreases the amount of fluid in
the bowel lumen.
c. even with aggressive treatment, there is little change in the mortality of patients with small bowel
obstruction.
d. peristalsis decreases during the early stages of the disease process.
e. all complete small bowel obstructions require emergent surgery.

377. The most common clinical manifestation of cholelithiasis is:


a. gallbladder carcinoma.
b. biliary colic.
c. cholecystitis.
d. cholangitis.
e. pancreatitis.

378. Travelers diarrhea is most commonly caused by :


a. Shigella.
b. Campylobacter.
c. Giardia lamblia.
d. rotavirus.
e. enterotoxigenic E. coli.

379. Acute mesenteric ischemia is most commonly caused by:


a. arterial thrombosis.
b. arterial embolus.
c. venous occlusion.
d. hypercoagulable state.
e. nonocclusive vascular disease.
140

374. d Surgical causes of abdominal pain are more likely to present with pain first, followed
th
Rosen 5 , by nausea and vomiting, rather than the converse. Diffuse pain is generally
Chapter 22, nonsurgical, but may represent the early visceral component of a surgical process.
p. 192 Fever does not accurately predict significant surgical pathology. Serial white blood
cell counts have failed to discriminate surgical versus nonsurgical conditions.
375. d Gonorrhea causes proctitis that results from anal intercourse or autoinoculation from
Rosen 5th, vaginal secretions after a 5- to 7-day incubation period. Proctitis may cause bloody or
Chapter 91, - purulent rectal discharge , tenesmus and pruritus ani.
p. 1353
376. a As intraluminal pressure increases it causes capillary and lymphatic obstruction with
th
Rosen 5 , subsequent bowel wall edema. Early bowel distension stimulates epithelial cell
Chapter 87, secretory activity. Mechanical small bowel obstruction stimulates peristalsis.
p. 1283, Aggressive treatment has decreased mortality from approximately 60% in 1900 to less
1284, & 1287 than 5 % today. Up to 35 % to 50% of patients with complete obstruction will have
resolution of symptoms with non-surgical intervention.
377. b Although cholelithiasis is associated with cholecystitis, cholangitis, gallbladder
Rosen 5th, carcinoma, and pancreatitis, its most common clinical manifestation is biliary colic.
Chapter 85,
p. 1265
378. e Approximately 80 % to 85% of travelers diarrhea is caused by bacteria. An estimated
Rosen 5th, 45%-50% is caused by enterotoxigenic E. coli, 8%-12% by Shigella and 7%-9% by
Chapter 89, Campylobacter. Rotavirus accounts for 5%-10% and Giardia lamblia accounts for
p. 1321,1323 4%-5%.
379. b Although acute mesenteric ischemia may be caused by mesenteric arterial thrombosis,
Rosen 5th, nonocclusive mesenteric ischemia, mesenteric venous thrombosis and hypercoagulable
Chapter 87, states (causing a venous thrombosis), the most common cause is an arterial embolus.
p. 1288-1289
141

380. The most predictive finding for appendicitis is:


a. pain for more than 48 hours.
b. migration of initial periumbilical pain to the right lower quadrant.
c. diffuse abdominal pain.
d. lack of abdominal rigidity.
e. history of similar episodes.

381. Cecal volvulus is most common in:


a. infants.
b. young children.
c. institutionalized teenagers.
d. age 25 to 35 years.
e. the elderly.

382. Regarding nasogastric(NG) tubes and lavage:


a. NG tubes do not aggravate hemorrhage from varices or Mallory-Weiss tears.
b. Iced water lavage is recommended in patients with upper GI bleeding.
c. Sterile fluid must be used as an irrigant.
d. Gastric lavage has been shown to reduce blood loss in patients with upper GI bleeding.
e. Gastric lavage has been shown to be completely safe with no serious complications reported.

383. Vomiting secondarily to Bacillus cereus is almost always from eating contaminated:
a. raw oysters.
b. poultry.
c. dairy products.
d. raw fish.
e. fried rice.

384. Meperidine (Demerol) :


a. has a longer duration of action than morphine.
b. has a metabolite that may cause CNS toxicity.
c. has no significant interaction with MAO inhibitors.
d. has been conclusively shown to be superior to morphine for treating biliary colic.
e. can be used repeatedly in the elderly without undue concern.

385. When considering abdominal pain:


a. somatic pain is dull, constant and poorly localized.
b. somatic pain is intense, intermittent and poorly localized.
c. somatic pain is intense, constant and well localized.
d. visceral pain is dull, intermittent and well localized.
e. visceral pain is intense, constant and well localized.

386. In children, gastroenteritis accompanied by seizures should suggest infection with:


a. Campylobacter.
b. E. coli 0157:H7.
c. Giardia.
d. Salmonella.
e. Shigella.
142

380. b Three findings that are highly predictive of appendicitis: right lower quadrant pain,
th
Rosen 5 , rigidity, and migration of initial periumbilical pain to the right lower quadrant.
Chapter 88,
p. 1294
381. d Volvulus of the cecum occurs in all ages but is most common in persons 25 to 35 years
th
Rosen 5 , of age. Unlike in sigmoid volvulus, severe, chronic constipation is not an underlying
Chapter 90, factor, and there is no association with psychiatric or neurological diseases.
p. 1335
382. a No evidence exists that gastric tube placement aggravates hemorrhage from varices or
th
Rosen 5 , Mallory-Weis tears. Gastric lavage does not reduce blood loss in patients with UGIB,
Chapter 23, and iced lavage is not recommended. The irrigant need not be sterile. Gastric tubes are
p. 197 safe in most patients but pharyngeal and esophageal perforation, cardiac arrest,
ethmoid sinus fracture with brain trauma and bronchial intubation have all been
reported.
383. e The emetic form of Bacillus cerus illness is almost always caused by the ingestion of
Rosen 5th, contaminated fried rice.
Chapter 89,
p. 1309
384. b Meperidines duration of action is no longer than morphines and most studies have
th
Rosen 5 , not shown it to be of a marked advantage for use in biliary tract disease or pancreatitis.
Chapter 182, It may have a fatal interaction with MAO inhibitors. It has a CNS toxic metabolite that
p. 2562-2563 is renally excreted and that also has a longer half-life in the elderly.
385. c Visceral pain results from stimulating autonomic nerves invested in the visceral
Rosen 5th, peritoneum surrounding internal organs. It is often the earliest manifestation of a
Chapter 22, particular disease process This discomfort is poorly characterized and difficult to
p. 190 localize. If the involved organ is affected by peristalsis, the pain is often described as
intermittent, crampy, or colicky. Somatic pain occurs with irritation of the parietal
peritoneum. Somatic pain is often described as intense ,constant and better localized
386. e In children it is common for shigellosis to cause convulsions.
Rosen 5th,
Chapter 89,
p. 1304-1305
143

387. You have just seen 4 adolescents from the same 7th grade class school with apparent appendicitis
right lower abdominal pain, anorexia, nausea, and low-grade fever. You suspect they may actually
be infected with:
a. Bacillus cereus.
b. E. coli 0157:H7.
c. Isosporidiosis.
d. epidemic Bunyavirus.
e. Yersinia enterocolitica.

388. A preferred oral therapy for antibiotic-induced pseudomembranous enterocolitis is:


a. a 2nd or 3rd generation fluoroquinolone.
b. ampicillin.
c. chloramphenicol.
d. metronidazole.
e. Pepto-Bismol.

389. Lactulose is useful in patients with hepatic encephalopathy because it:


a. acidifies the fecal stream, resulting in the trapping of ammonia as ammonium in the stool.
b. causes ammonia to be transmitted to the intracellular region through the sodium-potassium
membrane pump; once there, it is safely used in the Krebs cycle.
c. forces renal excretion of sodium bicarbonate, leading to compensatory hyperpnea and respiratory
alkalosis, which keeps ammonia nontoxic.
d. provides an alternative calorie source to ammonia, preventing its digestion and the subsequent
metabolic ketoacidosis.
e. works in a manner as yet undescribed.

390. A 47-year-old man complains of right upper quadrant pain, vomiting after meals, and dark urine.
Laboratory studies show AST 420, ALT 198. This is highly suggestive for:
a. obstructive biliary disease.
b. pancreatitis.
c. hepatitis B.
d. hepatitis C.
e. alcohol related liver disease.

391. From an Emergency Practitioners perspective, the most acutely devastating complication of
ulcerative colitis is:
a. carcinoma.
b. fistula formation.
c. hepatorenal syndrome.
d. malabsorption and electrolyte abnormalities.
e. toxic megacolon.

392. A 72 year-old bedridden patient is sent for evaluation of vomiting. On physical exam you find
several well-healed scars, a distended tympanic abdomen, quiet bowel sounds, and an empty rectal
vault. The plain film radiograph of the abdomen shows a markedly dilated single loop of colon on
the left side suggestive of a sigmoid volvulus. You need to consult a:
a. surgeon for laparotomy, detorsion, and reattachment of the sigmoid in the left lower quadrant.
b. surgeon for sigmoidoscopy, decompression and detorsion.
c. surgeon for hemicolectomy.
d. surgeon for sigmoid colectomy.
e. surgeon for total colectomy.
144

387. e The initial clinical picture of Yersinia enterocolitis resembles that of infection by other
th
Rosen 5 , invasive intestinal organisms: fever; colicky abdominal pain; watery, greenish, and
Chapter 89, sometimes bloody diarrhea; and constitutional symptoms of anorexia, vomiting, and
p. 1305 malaise. A substantial number of patients with yersiniosis, in particular adolescents
and young adults, develop an ileocecitis. In these cases, lower abdominal pain with
little or no diarrhea predominates and may perfectly mimic acute appendicitis. Large
gastrointestinal outbreaks have been traced to contaminated milk, largely because
physicians noticed an extraordinary jump in the number of negative appendectomies.
388. d Many cases of antibiotic-associated colitis are self-limited, provided that the offending
Rosen 5th, agent is discontinued. When antibiotic treatment for C. difficle is warranted, oral
Chapter 89, metronidazole is typically used unless the patient is significantly ill and merits IV
p. 1312-1313 therapy.
389. a Lactulose is a poorly absorbed sugar metabolized by colonic bacteria yielding lactic
th
Rosen 5 , acid. The salutary effects of this agent are related both to the acidification of the fecal
Chapter 85, stream, resulting in the trapping of ammonia as ammonium in the stool, and to its
p. 1260 cathartic action. The usual dosage of lactulose is 15 to 30 ml orally three or four times
daily or in a quantity sufficient to result in several loose bowel movements daily. The
principal adverse effect is excessive diarrhea, with resultant fluid and electrolyte
imbalance.
390. e In alcoholic liver disease serum transaminases aspartate aminotransferase (AST) and
Rosen 5th, alanine aminotransferase (ALT) are moderately elevated. Values in excess of 10 times
Chapter 85, normal are unusual and a relative predominance of AST to ALT is expected. Bilirubin
p. 1258 is commonly elevated with a wide range of possible values.
391. e Toxic megacolon occurs in up to 5% of cases of ulcerative colitis , usually during the
Rosen 5th, initial acute episode. The patient will appear septic, apathetic, and lethargic, with high
Chapter 90, fever, chills, tachycardia, and progressive abdominal pain, tenderness, and distention.
p. 1338 The cause of toxic dilation is unknown, but precipitating factors may include use of
antidiarrheal agents, vigorous use of cathartics or enemas, or barium enema
examinations. Toxic dilation occurs predominantly in the transverse colon, probably
because in the supine position air collects in the transverse colon.
392. b Once the diagnosis of sigmoid volvulus is made, a surgeon should be consulted. The
Rosen 5th, therapy of choice for a nonstrangulationg sigmoid volvulus is decompression and
Chapter 87, detorsion, using a rectal tube via the sigmoidoscope. Success is achieved in 85% to
p. 1334 95% of patients.
145

393. The two most common causes of diarrhea in AIDS patients are:
a. Cryptospiridium and Cytomegalovirus (CMV).
b. Giardia and Rhodococcus equi.
c. Salmonella and Campylobacter.
d. Cyclospora and Strongyloides stercoralis.
e. Toxoplasma gondii and Shigella.

394. A 70-year-old female presents with increasing constipation for the last few months. You complete
her history and physical exam and become worried that she may have a serious disorder. You are
most concerned about her:
a. age greater than 60.
b. rectal bleeding.
c. abdominal distention.
d. flatulence.
e. depression.

395. A 28 year-old woman received a liver transplant 10 days ago. Tonight she complains of low-grade
fever and body aches. Which of the following is true concerning liver post-transplant rejection?
a. Rejection often begins 1 to 2 weeks post-op.
b. Leukocytosis may occur.
c. Fever may occur.
d. Right upper quadrant pain may occur.
e. All of the above are true

396. The most common cause of acute food poisoning in the United States is:
a. Bacillus cereus.
b. Clostridium perfringens.
c. Escherichia coli 0157:H7.
d. Staphylococcal organisms.
e. Yersinia.

397. A 51 year-old businessman ate sushi several hours before he started vomiting and having crampy,
explosive diarrhea. He requires intravenous rehydration and anti-emetics. He now feels better and
asks for ice chips. You notice he spits them out, explaining, They burn my mouth, and my teeth
feel loose. He is probably suffering from:
a. ciguatera poisoning.
b. scombroid poisoning.
c. tetrodotoxin psychosis.
d. E. coli 0157:H7 poisoning.
e. disulfiram (Antabuse) reaction.

398. You can tell the suffering gentleman with the condition described in question #397 that he:
a. will probably require psychiatric counseling.
b. must wait for the antidote to be delivered from Japan.
c. can expect rapid improvement.
d. must avoid alcohol, as it can worsen his symptoms.
e. will need high-dose steroid therapy.
146

393. a Cryptosporidium and cytomegalovirus (CMV) infections are the most causes of
th
Rosen 5 , diarrhea in AIDS patients. The incidence of each is 15% to 40%.Chronic persistent
Chapter 89, diarrhea is most often from one coccidia, Cryptosporidium or Isospora belli
p. 1319
394. b The most concerning symptoms associated with constipation are rectal bleeding and
th
Rosen 5 , change in the caliber of the stool. These are warning signals suggesting possible
Chapter 2, colorectal cancer.
p. 210
395. e Rejection often occurs 1 to 2 weeks after surgery, with fever, right upper quadrant
th
Rosen 5 , pain, and elevated bilirubin and transaminases. Leukocytosis may occur but is
Chapter 178, nonspecific.
p. 2508
396. b Clostridium perfringens is probably the most common cause of acute food poisoning
th
Rosen 5 , in the US, constituting almost one fourth of all bacteria-associated food-borne illnesses
Chapter 89, and usually occurring in large outbreaks caused by ingestion of meat or poultry dishes.
p. 1308
397. a Classically, patients with ciguatoxin develop both GI and neurological symptoms. The
th
Rosen 5 , GI symptoms (e.g., nausea, vomiting, profuse watery diarrhea, crampy abdominal pain,
Chapter 89, and diaphoresis) tend to appear earlier. The panoply of neurological symptoms
p. 1311 consists largely of dysesthesias and paresthesias around the throat and the perioral
area; burning feet, which may resemble alcoholic peripheral neuropathy; loose,
painful teeth; and sometimes CNS changes, such as ataxia, weakness, vertigo, visual
hallucinations, and even confusion and coma. One symptom highly suggestive of
ciguatera fish poisoning is sensory reversal dysesthesia, in which cold objects are
398. d perceived to be warm and vice versa. Patients describe such distortion of temperature
perception very vividly. Another classic feature is either a return or a worsening of the
Rosen 5th, symptoms after ingestion of alcohol. Ciguatera poisoning lasts an average of 1 to 2
Chapter 89, weeks, but at least half of its victims are still symptomatic at 8 weeks. The
p. 1311 neurological symptoms, particularly the paresthesias and dysesthesias, tend to persist
longer than the GI symptoms and have been reported up to years later.
147

399. When assessing a patient for acute appendicitis, which of the following describes the obturator
sign?
a. pain produced by internal rotation of the flexed right hip.
b. pain produced by external rotation of the flexed right hip.
c. pain produced by extension of the right hip with the patient in the left lateral decubitus
position.
d. sensation of pain in the right lower quadrant with palpation of the left lower quadrant.
e. pain produced by the patient coughing.

400. A type of hemorrhoid that requires emergent surgical consultation is:


a. thrombosed external hemorrhoid.
b. second degree internal hemorrhoid.
c. third-degree internal hemorrhoid.
d. nonthrombosed fourth degree internal hemorrhoid
e. thrombosed or gangrenous fourth degree internal hemorrhoid.
148

399. a The obturator sign is the elicitation of pain as the right hip is flexed and internally
th
Rosen 5 , rotated. Other clinical signs of acute appendicitis include the psoas sign (increase of
Chapter 88, pain when the psoas muscle is stretched as the patient is asked to extend his/her right
p. 1294 hip), and Rovsing's sign ( referred right lower quadrant pain with palpation on the left
lower quadrant).
400. e Patients with acute, gangrenous, fourth-degree internal hemorrhoids should be referred
Rosen 5th, for emergent hemorrhoidectomy. Nonthrombosed fourth-degree hemorrhoids require
Chapter 91, nonemergent hemorrhoidectomy. Acutely thrombosed external hemorrhoids can be
p. 1347 excised in the ED for pain relief. Second- and third- degree internal hemorrhoids
require referral for elective surgery.
149

4.0 Cutaneous Disorders


502. A 25-year-old woman is worried about some patches on her back that did not tan this
summer. She probably has:
a. pityriasis rosea.
b. seborrheic dermatitis.
c. tinea versicolor.
d. vitiligo.
e. xerosis.

503. A 27-year-old woman has several dozen small, scaly, slightly itchy lesions on her back.
She also points out a 3-cm lesion that she noticed a few days before the above rash. You
can tell her that:
a. she may get some lesions in her mouth.
b. the rash is likely to reoccur every year or two.
c. chronic low-dose griseofulvin will be curative.
d. its cause is unknown.
e. topical hydrocortisone will shorten the course of the disease.

504. A wheal is:


a. an elevated skin lesion >0.5 cm in diameter and depth.
b. a nodule filled with expressible material.
c. a blister >0.5 cm in diameter filled with clear fluid.
d. a blister filled with cloudy or purulent fluid.
e. a papule or plaque of dermal edema; often with central pallor and irregular borders.

505. A 54-year-old woman recently started trimethoprim-sulfamethoxazole for a sinus infection.


She complains of low-grade fever, lethargy, loss
of appetite, and this rash (Figure 114-13). She
has similar lesions in her mouth.
a. She requires intravenous broad-spectrum antibiotic
coverage to prevent progression to toxic shock
syndrome.
b. This is Kaposis sarcoma; she probably has AIDS.
c. The severe form of this disease is called a Stevens-
Johnson syndrome.
d. Systemic steroids will shorten the course of the
disease.
e. It is caused almost exclusively by viral infections Rosen 5th, Figure 114-13

506. A 3-year-old boy has had several days of yellow crusting lesions on his forehead and chin.
They are itchy but not painful. The child is afebrile and nontoxic.
a. Group A streptococcus is the most common causative organism.
b. Systemic antibiotics have a higher cure rate than topical agents.
c. These lesions are not contagious.
d. Systemic antibiotics will prevent later occurrence of glomerulonephritis.
e. Without treatment, the lesions heal within 3 to 6 weeks.
150

502. c Tinea versicolor is a superficial yeast infection caused by Pityrosporum ovale.


th
Rosen 5 , Superficial scaling patches occur mainly on the chest and trunk but may extend to the
Chapter 114, head and limbs. As the name implies, lesions can be a variety of colors including pink,
p. 1637 tan, or white. The disease may be associated with pruritus, but medical care is often
sought because the spots do not tan. On physical examination, a fine subtle scale is
noted that may appear hypopigmented. A KOH preparation reveals short hyphae
mixed with spores (chopped spaghetti and meatballs).
503. d Pityriasis rosea is a mild skin eruption predominantly found in children and young
Rosen 5th, adults. The lesions are multiple pink or pigmented oval papules or plaques 1 to 2 cm
Chapter 114, in diameter on the trunk and proximal extremities. Mild scaling may be present. The
p. 1638 lesions are parallel to the ribs, forming a Christmas treelike distribution on the trunk.
Oral lesions are rare. In children, papular or vesicular variants of the disease may
occur. In half the cases, the generalized eruption is preceded by a week by the
appearance of a herald patch. This is a larger lesion, 2 to 6 cm in diameter, that
resembles the smaller lesions in other respects. The eruption is usually asymptomatic,
although pruritus may be present. Pityriasis rosea is self-limited, resolving in 8 to 12
weeks. Its cause is unknown, although a virus is suspected. Treatment is usually
unnecessary, except for symptomatic alleviation of bothersome pruritus.
504. e Macule: Flat, color differs from surrounding skin
Rosen 5th, Patch: A macule with surface changes (i.e., scale or wrinkling)
Chapter 114, Papule: Elevated skin lesion <0.5 cm in diameter
p. 1636 Plaque: Elevated skin lesion >0.5 cm in diameter; without substantial depth
Nodule: Elevated skin lesion >0.5 cm in diameter and depth
Cyst: Nodule filled with expressible material
Vesicle: Blisters <0.5 cm in diameter filled with clear fluid
Bullae: Blisters >0.5 cm in diameter filled with clear fluid
Pustule: Vesicle filled with cloudy or purulent fluid
Crust: Liquid debris that has dried on the skin surface
Scale: Visibly thickened stratum corneum; usually white
Lichenification: Epidermal thickening characterized by visible and palpable skin
thickening and accentuated skin markings
Induration: Dermal thickening that feels thick and firm
Wheal: Papule of dermal edema; often with central pallor and irregular borders
Erythema: Red appearance of skin caused by vasodilatation of dermal blood vessels
505. c Erythema multiforme is an acute, usually self-limiting disease precipitated by a variety
Rosen 5th, of factors. It is characterized by the sudden appearance of skin lesions that are
Chapter 114, erythematous or violaceous macules, papules, vesicles, or bullae. Their distribution is
p. 1648 often symmetrical, most commonly involving the soles and palms, the backs of the
hands or feet, and the extensor surfaces of the extremities. Treatment should begin
with a search for the underlying cause. Mild forms resolve spontaneously in 2 to 3
weeks. Systemic steroids are commonly used and provide symptomatic relief, but are
of unproven benefit in influencing the duration and outcome.
506. e Impetigo is a slowly evolving pustular eruption, most common in preschool children.
Rosen 5th, Currently, Staphylococcus aureus is the most common pathogen, with Group A
Chapter 114, streptococcus a distant second. Both systemic and topical therapy is equally successful
p. 1639 in treating impetigo. Lesions are very contagious among infants and young children
and less so in older children and adults. There is no evidence, however, that systemic
antibiotics prevent the development of acute glomerulonephritis. Without treatment,
impetigo heals within 3 to 6 weeks.
151

507. A 21-year-old man has fever and a urethral discharge. He also complains of a swollen
painful left knee and this rash.
a. He can be treated as an outpatient with oral antibiotics.
b. This condition occurs more in men than in women.
c. If the lesions are cultured, they are usually positive for the
organism.
d. The tick serves as a vector for this disease.
e. You expect the rash to be pustular rather than petechial and
hemorrhagic.

508. The most common medication-related cause of


erythema nodosum is:
a. sulfa-based drugs.
b. non-steroidal anti-inflammatory agents.
c. oral contraceptives.
d. penicillin.
e. fluoroquinolones.

509. The most common skin eruption due to drugs is:


a. bullae.
b. macules.
c. papules.
d. striae.
e. urticaria.

510. A 25-year-old woman had an abortion five days ago. She now has fever and a flu-like
syndrome. Today she started vomiting and got confused. Oral temperature 103.4F,
heart rate 120 / minute, blood pressure 80/40 mmHg. You also notice diffuse eryhtema.
You know that:
a. mucous membrane involvement would indicate this is not toxic shock syndrome.
b. you can also see pharyngitis, a strawberry tongue, conjunctivitis, and vaginitis with this illness.
c. supportive care and high-dose penicillin is the appropriate treatment.
d. desquamation tends to spare the hands and feet.
e. the skin rash may take several weeks to dissipate.

511. Nikolskys sign is often found in patients with:


a. acanthosis nigrans.
b. bullous pemphigoid.
c. hidradenitis suppurativa.
d. Rocky Mountain spotted fever.
e. toxic epidermal necrolysis.

512. Erysipelas is usually caused by:


a. Neisseria gonorrhoeae.
b. Coxsackievirus.
c. Bartonella tularensis.
d. Group A streptococcus.
e. Staphylococcus epidermidis.
152

507. e About 2% of patients with mucosal gonococcal infection will develop disseminated
th
Rosen 5 , GC infection. Hospitalization is recommended for patients in whom the diagnosis is
Chapter 114, uncertain and for those who have septic arthritis, meningitis, or endocarditis. It affects
p. 1640 women primarily. Fever and migratory polyarthralgias commonly accompany the skin
lesions. The rash of disseminated gonococcemia is pustular rather than obviously
petechial and hemorrhagic, as are the lesions of Rocky Mountain spotted efevr and
meningococcemia.
508. c A number of underlying conditions produce erythema nodosum: tuberculosis,
Rosen 5th, sarcoidosis, coccidioidomycosis, histoplasmosis, ulcerative colitis, regional enteritis,
Chapter 114, pregnancy, infections with streptococci, Yersinia enterocolitica, and chlamydia. As
p. 1653 with erythema multiforme, many cases of erythema nodosum are idiopathic. Oral
contraceptive agents are the leading cause of drug-induced cases. The differential
diagnosis includes traumatic bruises and subcutaneous fat necrosis.
509. e A given drug can produce a skin eruption of a different appearance in different patients
Rosen 5th, or a different appearance in the same patient on different occasions. The most
Chapter 114, common eruptions are urticaria (hives) and morbilliform rashes. Drug reactions tend
p. 1640 to appear within a week after the drug is taken, with the exception of reactions to
semisynthetic penicillins, which commonly occur later. Skin lesions may appear after
a drug has been discontinued and may worsen if the drug or its metabolites persist in
the system. Special note should be made of penicillin because it is the most common
cause of drug reaction. Serum sickness and urticaria are the most common
manifestations of penicillin allergy.
510. b In toxic shock syndrome, the rash is typically diffuse, blanching, macular
Rosen 5th, erythroderma. Accompanying nonexudative mucous membrane inflammation is
Chapter 114, common. Pharyngitis, sometimes accompanied by a strawberry tongue,
p. 1644 conjunctivitis, or vaginitis may be seen. As a rule, the rash fades within 3 days of its
appearance. This is followed by a full-thickness desquamation, most commonly
involving the hands and feet. Initial treatment of TSS consists of IV fluid replacement,
ventilatory support, pressor agents, penicillinase-resistant antibiotics, and drainage of
infected sites. Corticosteroids reduce the severity of illness and duration of fever if
initiated within 2 to 3 days after the onset of illness.
511. e Nikolskys sign is epidermal shearing following lateral pressure on unblistered skin.
Rosen 5th, The extension of a blister following application of pressure to its roof has earned the
Chapter 114, eponym Asboe-Hansens sign. Nikolsky's sign is most commonly found in scalded
p. 1643 skin syndrome (also called Ritter disease), toxic epidermal necrolysis, and pemphigus
vulgaris.
512. d Erysipelas is an acute superficial cellulitis characterized by a sharply demarcated
Rosen 5th, border surrounding skin that is raised, deeply erythematous, indurated, and painful. It
Chapter 131, usually involves the dermis, lymphatics, and most of the superficial subcutaneous
p. 1946 tissue. Erysipelas most often occurs in the very young and those age 50 to 60 years; it
is associated with small breaks in the skin, nephrotic syndrome, and postoperative
wounds. Patients usually appear toxic with a prodrome of fever, chills, and malaise
preceding the eruption of a bright red cellulitis predominantly on the lower extremities
or on the face. Group A Streptococcus, other streptococcal species, and S. aureus are
the involved pathogens.
153

513. A positive Hutchinson sign is found in patients with:


a. herpes zoster ophthalmicus.
b. Lyme disease.
c. measles.
d. mumps.
e. Rocky Mountain spotted fever.

514. Physical findings of patients with scarlet fever and Kawasaki disease are similar. The
finding most likely to make you think of scarlet fever would be:
a. strawberry tongue.
b. cervical adenopathy.
c. fever.
d. generalized erythematous rash.
e. circumoral pallor.

515. There is an outbreak of head lice in the local school district, and the local school nurse
calls for advice. You tell her that:
a. household contacts require treatment.
b. involvement of the interdigital web spaces is typical.
c. nits are seen more frequently than the adult louse form.
d. pediculosis capitis is more common in adults.
e. the organism is a mite.

516. Oral acyclovir is recommended for the treatment of an initial episode of:
a. genital herpes simplex.
b. herpes zoster.
c. oral herpes simplex.
d. varicella.
e. vaccinia.

517. Roseola infantum:


a. causes a high fever followed in several days by a desquamating rash.
b. frequently is a precursor to encephalitis.
c. generally occurs in school-age children.
d. is also called fifth disease.
e. is caused by a herpes virus.

518. A pinkish maculopapular rash that first appears on the face and associated with
generalized lymphadenopathy including suboccipital and postauricular nodes is most
typical of:
a. measles
b. Rocky Mountain spotted fever.
c. roseola.
d. rubella.
e. rubeola.

519. The primary lesions of pemphigus vulgaris are:


a. bullae.
b. macules.
c. nodules.
d. papules.
154

e. wheals.
155

513. a Herpes zoster keratoconjunctivitis occurs as a result of activation of the virus along
th
Rosen 5 , ophthalmic division of the trigeminal nerve. The rash follows dermatomal patterns,
Chapter 66, involves the forehead and upper eyelid, and produces significant pain. Involvement of
p. 918 the nasociliary nerve, manifested by zoster lesions on the tip of the nose (Hutchinsons
sign), is associated with a 76% risk of ocular involvement vs. 34% risk if the nerve is
not involved.
514. e Kawasaki disease, or mucocutaneous lymph node syndrome (MLNS) is a disease of
Rosen 5th, unclear etiology found predominantly in children less than 9 years of age. The
Chapter 123, diagnosis of this disorder is based on a prolonged fever associated with at least four of
p. 1804-1805 the following: (1) conjunctivitis, (2) rash, (3) lymphadenopathy, (4) changes in the
Chapter 114, oropharynx consisting of injection of the pharynx and lips with prominent papillae of
p. 1646 the tongue (strawberry tongue), and (5) extremity erythema and edema. Patients with
scarlet fever can have the erythematous rash, strawberry tongue, lymphadenopathy,
and fever, but also have facial flushing with circumoral pallor.
515. c Only occasionally are the adult lice forms found, looking like blue or black grains.
Rosen 5th, The nits often attach to the base of the hair shafts, appearing as white dots. Any sexual
Chapter 114, contacts should be treated; household contacts should be examined, and if uninfected,
p. 1649 no treatment is necessary. Scabies involves the interdigital web spaces and is a mite
infestation.
516. a Acyclovir is not routinely recommended for oral herpes, herpes zoster, or varicella
Rosen 5th, infections unless the patient is immunocompromised. Acyclovir reduces the duration
Chapter 124, of viral shedding and accelerates healing. It does not prevent recurrent episodes.
p. 1819
517. e Roseola infantum is the most common exanthem of children younger than 2 years of
th
Rosen 5 , age and occurs most often at about 1 year of age. The illness begins abruptly with the
Chapter 114, acute onset of fever, often as high as 41o C, lasting three to five days. Despite the
p. 1822 fever, the child usually remains active and alert. A fine, evanescent, rose-colored
maculopapular rash then appears on the trunk after lysis of the fever, which may last
for 1 or 2 days. The rash may spread to the face and extremities.
518. d This is a typical picture of rubella (German measles), particularly the involvement of
Rosen 5th, the postauricular nodes. It is important to recognize rubella in order to avoid maternal
Chapter 114, exposure that can result in severe birth defects (p. 1825). Roseola infantum is
Chapter 128, described in question #517 (p. 1822). Rocky Mountain spotted fever is manifested by
pages listed the abrupt onset of fever, headaches, chills, and malaise; rash develops on the second
in text of to fourth day, with blanching erythematous macules around the wrists and ankles.
answer They may become petechial (p. 1891-1896). Rubeola (measles) is a febrile illness,
with rash that starts on the face and spreads to the trunk on the third to fifth day of
illness. Koplik's spots-bright red spots with bluish centers opposite the molars-occur
early in the course of the measles (p. 1827).
519. a Pemphigus vulgaris (PV) is an uncommon, but important, dermatologic disorder. The
Rosen 5th, mortality rate before the use of steroids was approximately 95%. PV is a bullous
Chapter 114, disease, affecting both sexes equally, and is most common in patient 40 to 60 years of
p. 1653-1654 age. Nikolskys sign is present and characteristic of the disease.
156

520. Hidradenitis suppurativa:


a. begins as a painful deep erythematous nodule usually in the axilla or groin.
b. begins in childhood.
c. is easily treated with 10-days of a long-acting tetracycline.
d. is more common in men.
e. should never be drained in the emergency department.
157

520. a Hidradenitis suppurativa affects the apocrine sweat glands. Recurrent abscess
th
Rosen 5 , formation in the axillae and groin resembles localized furunculosis. The condition
Chapter 114, tends to be recurrent and may be extremely resistant to therapy. It is treated with
p. 1639 drainage of abscesses, but antistaphylococcal antibiotics are useful if administered
early and for a prolonged period.
158

5.0 Endocrine, Metabolic, Nutritional Disorders


521. The most common cause of coma in a patient with diabetes is:
a. alcohol intoxication.
b. diabetic ketoacidosis.
c. hypoglycemia.
d. sepsis.
e. stroke.

522. In a hypoglycemic patient who has known adrenal insufficiency , you should consider
giving intravenous:
a. ACTH.
b. amiodarone.
c. hydrocortisone.
d. sodium bicarbonate.
e. thyroxin.

523. The only agents capable of stimulating hepatic glucose production (glycogenolysis) within
minutes are:
a. glucagon and epinephrine.
b. glucagon and insulin.
c. glucagon and norepinephrine.
d. insulin and epinephrine.
e. insulin and norepinephrine.

524. In patients with hypoglycemia, signs of CNS dysfunction are common. The next most
common physical sign would be:
a. seizures.
b. focal weakness.
c. hypotension
d. tachypnea.
e. sweating.

525. The only anabolic hormone is:


a. adrenaline.
b. cortisol.
c. glucagon.
d. insulin.
e. testosterone.

526. In a patient with diabetic ketoacidosis, the primary reason for mental status change is the:
a. elevated blood sugar.
b. metabolic acidosis.
c. respiratory alkalosis.
d. hypoxemia.
e. elevated osmolarity.
159

521. c The most common cause of coma associated with diabetes is hypoglycemia, due to an
th
Rosen 5 , excess of administered insulin with respect to glucose intake. Hypoglycemia may be
Chapter 120, associated with significant morbidity and mortality. Severe hypoglycemia is usually
p. 1748 associated with a blood sugar below 40 to 50 mg/dl and impaired cognitive function.
522. c Steroid administration should be considered for hypoglycemia that is either resistant to
th
Rosen 5 , aggressive glucose replacement therapy or associated with the signs of adrenal
Chapter 120, insufficiency. The dose is 100 to 200 mg hydrocortisone IV in adults.
p. 1781
523. a The first defense against the development of hypoglycemia is a decrease in insulin
th
Rosen 5 , secretion, but both glucagon and epinephrine are also important for the acute protection
Chapter 120, against hypoglycemia. Both of these counterregulatory hormones are the only agents
p. 1745 capable of stimulating hepatic glucose production within minutes of their release into
circulation, primarily via glycogenolysisthe release of glucose from its intracellular
storage-depot glycogen.
524. e A depressed sensorium is most common. Described less frequently are seizure activity
Rosen 5th, and focal neurologic findings. Sweating is common.
Chapter 120,
p. 1748
525. d Insulin is the only anabolic hormone and is responsible for the metabolism and storage
th
Rosen 5 , of carbohydrates, fat, and protein. Counterregulatory hormones include glucagon,
Chapter 120, catecholamines, cortisol, and growth hormone.
p. 1745
526. e Mental confusion and coma are much more likely with serum osmolarity 340 mOsm/L
Rosen 5th, or higher. If the serum osmolarity is <340 mOsm/L in a comatose patient, another
Chapter 120, cause of the coma should be sought.
p. 1750
160

527. Urine dipstick for ketones uses a nitroprusside reaction, which measures:
a. acetoacetate.
b. beta-hydroxybutyrate.
c. insulin levels.
d. ketones.
e. lactic acids.

528. The lab values most consistent with a diagnosis of DKA are:
a. glucose >350 mg/dL, bicarbonate >20 mEq/L, arterial pH <7.3, severe ketonuria
b. glucose >250 mg/dL, bicarbonate >15 mEq/L, arterial pH >7.3, moderate ketonemia
c. glucose >250 mg/dL, bicarbonate <20 mEq/L, arterial pH <7.0, moderate ketonuria
d. glucose >350 mg/dL, bicarbonate <15 mEq/L, arterial pH <7.3, moderate ketonemia
e. glucose >700 mg/dL, bicarbonate >15 mEq/L, arterial pH >7.3, moderate ketonuria

529. The single most important initial treatment for the patient in diabetic ketoacidosis is:
a. oxygen therapy.
b. rapid bicarbonate administration.
c. rapid fluid administration.
d. rapid insulin administration.
e. rapid potassium replacement.

530. In a patient with diabetic ketoacidosis, the preferred method of insulin administration is:
a. intramuscular.
b. intravenous bolus high dose.
c. intravenous bolus low dose.
d. intravenous constant drip weight-based.
e. subcutaneous.

531. During treatment of diabetic ketoacidosis, the most potentially life-threatening electrolyte
abnormality is:
a. hypercalcemia.
b. hyperkalemia.
c. hypokalemia.
d. hypernatremia.
e. hyponatremia.

532. Concerning the use of sodium bicarbonate in the treatment of diabetic ketoacidosis, it has
been shown to:
a. rapidly improve the hypokalemia
b. improve the central nervous system acidosis.
c. improve intracellular acidosis.
d. assist oxyhemoglobin dissociation.
e. do none of the above dont use it.

533. Common presenting symptoms in a patient with alcoholic ketoacidosis include:


a. blurred vision.
b. delirium tremens.
c. dehydration
d. bloody diarrhea.
e. fetid breath odor.
161

527. a Urine ketone dipsticks use the nitroprusside reaction, which is a good test for
th
Rosen 5 , acetoacetate but does not measure beta-hydroxybutyrate. Although the usual
Chapter 120, acetoacetate / beta-hydroxybutyrate ratio in diabetic ketoacidosis is 1:2.8, it may be as
p. 1748 high as 1:30, in which case the urine dipstick does not reflect the true level of ketosis.
When ketones are in the form of beta-hydroxybutyrate, the urine ketone dipsticks may
infrequently yield negative reactions in patients with significant ketosis.
528. d Although the exact definition of DKA is variable, most experts agree that a blood
Rosen 5th, glucose greater than 350 mg/dL, bicarbonate level less than 15 mEq/L, and an arterial
Chapter 120, pH of less than 7.3 with moderate ketonemia constitute the disease.
p. 1752
529. c Rapid fluid administration is the single most important initial step in the treatment of
th
Rosen 5 , DKA. Fluid helps restore intravascular volume and normal tonicity, perfuse vital
Chapter 120, organs, improve glomerular filtration rate, and lower serum glucose and ketones. The
p. 1752 average adult patient has a water deficit of 100 mL/kg (5 to 10 L) and a sodium deficit
of 7 to 10 mEq/kg. NS is the most frequently recommended fluid for initial
rehydration even though the extracellular fluid of the patient is initially hypertonic.
530. d High dosages of insulin have potentially harmful effects, including a greater incidence
Rosen 5th, of iatrogenic hypoglycemia and hypokalemia. Because the half-life of regular insulin
Chapter 120, is 3 to 10 minutes, IV insulin should be administered by constant infusion rather than
p. 1752-1753 by repeated bolus. The current therapy of choice is regular insulin infused at 0.1
U/kg/hr up to 5 to 10 U/hr, mixed with the IV fluids. Regular insulin, 10 to 20 U/hr,
administered IM accomplishes similar effect but subjects the patient to repeated painful
injections. In theory, IM insulin may accumulate at a poorly perfused administration
site, failing to enter the systemic circulation in a timely manner.
531. c The development of severe hypokalemia is potentially the most life-threatening
Rosen 5th, electrolyte derangement during the treatment of DKA. This complication is avoidable
Chapter 120, if the pathophysiology is understood and the effects of therapy frequently monitored.
p. 1754
532. e To date, not a single study clearly demonstrates improved clinical outcome using
th
Rosen 5 , bicarbonate in the treatment of DKA. Acidotic patients routinely recover from DKA
Chapter 120, without alkali therapy. Routine use of supplemental bicarbonate in the treatment of
p. 1754 DKA is not recommended.
533. b Presenting symptoms of alcoholic ketoacidosis
Rosen 5th, Nausea 76%
Chapter 179, Vomiting 73%
p. 2524 Abdominal pain 62%
Shortness of breath 20%
162

534. Initial treatment of the patient with alcoholic ketoacidosis should begin with:
a. intravenous bicarbonate.
b. intravenous D5NS.
c. intravenous insulin.
d. intravenous Ringers lactate.
e. subcutaneous insulin.

535. In comparing patients with diabetic ketoacidosis (DKA) to those with hyperglycemic
hyperosmolar nonketotic coma (HHNC), those with DKA have:
a. greater fluid and electrolyte deficits.
b. higher serum glucose levels.
c. lower anion gaps.
d. more profound acidosis.
e. slower onset.

536. Na+ 136 mEq/L; K+ 4.1 mEq/L; Cl- 108 mEq/L; NaHCO3- 12 mEq/L; glucose 600 mg/dL;
arterial pH 7.10
a. The anion gap is 26.
b. The corrected potassium is 2.3 mEq/L.
c. The corrected potassium is 5.9 mEq/L.
d. The corrected sodium is 126 mEq/L.
e. The corrected sodium is 152 mEq/L.

537. In rehydrating the patient with hyperosmolar hyperglycemic coma, you should use:
a.D5NS.
b. hypertonic saline.
c.hypotonic saline.
d. isotonic saline.
e.Ringers lactate.

538. When fixing a patients fluid and electrolyte imbalances, they should be corrected in this
order:
a. sodium and chloride > volume > pH > potassium, calcium, and magnesium
b. volume > pH > potassium, calcium, and magnesium > sodium and chloride
c. potassium, calcium, and magnesium > volume > pH > sodium and chloride
d. pH > potassium, calcium, and magnesium > sodium and chloride > volume
e. volume > potassium, calcium, and magnesium > sodium and chloride > pH

539. The most common cause of metabolic acidosis in children is:


a. cystic fibrosis.
b. diabetes.
c. febrile seizure.
d. nephrotic syndrome.
e. prolonged diarrhea.

540. Common precipitating factors in the development of hyperglycemic hyperosmolar


nonketotic syndrome include:
a.fever.
b. prolonged bedrest.
c.hypokalemia
d. myocardial ischemia.
163

e.use of a salt substitute.


164

534. b Treatment of alcoholic ketosis is volume replacement with normal saline, glucose,
th
Rosen 5 , thiamine, and correction of hypokalemia. This can be accomplished with 5% dextrose
Chapter 179, in normal saline and either 30 mEq of potassium chloride or 30 mEq of oral potassium.
p. 2524 Bicarbonate is seldom necessary for the uncomplicated case but may be considered in
the rare patient who has a pH of less than 7.1. If no serious complicating illness is
present, the ketosis will reverse in 12 to 24 hours with this treatment.
535. d The prodrome for HHNC is significantly longer than that of DKA. The patient
Rosen 5th, typically manifests more profound electrolyte imbalance and dehydration than those
Chapter 120, with DKA. A patient with HHNC does not have a ketoacidosis caused by diabetes but
p. 1752 may have a lesser degree of metabolic acidosis. The mortality rate is much higher for
HHNC as most patients are elderly and have underlying cardiac and renal disease.
536. b The reported serum sodium is often misleading in DKA. The true value of sodium
Rosen 5th, may be approximated by adding 1.6 mEq/L to the reported value for every 100 mg/dl
Chapter 120, of glucose over the norm. Acidosis and dehydration contribute to high measured
p. 1752 serum potassium despite total body deficits. Correction for acidosis can be made by
subtracting 0.6 mEq/L from the laboratory value for every 0.1 decrease in pH below
7.4.
537. d Most authors agree that the use of isotonic saline (0.9% NaCl) is the most appropriate
Rosen 5th, initial crystalloid for the replacement of intravascular volume. It is hypotonic to the
Chapter 120, patients serum osmolarity and will more rapidly restore plasma volume. Once
p. 1756 hypotension, tachycardia, and urinary output improve, half- normal saline (0.45%
NaCl) can be used to replace the remaining free water deficit.
538.b When fluids and electrolytes are altered, they should be corrected in the following
Rosen 5th, orderly fashion: volume > pH > potassium, calcium, and magnesium > sodium and
Chapter 119, chloride. Equilibrium of fluid, electrolytes, and pH depends on adequate tissue
p. 1753-1754 perfusion and often corrects spontaneously with resolution of the underperfused state.
539. e Metabolic acidosis can be caused by one of three mechanisms: (1) increased
th
Rosen 5 , production of acids, (2) decreased renal excretion of acids, or (3) loss of alkali. The
Chapter 118, etiologies of metabolic acidosis can be clinically divided into those that create an
p. 1718 elevation in AG and those that do not. In the pediatric age group, dehydration from
prolonged diarrhea is the most common cause of metabolic acidosis.
540. c Usually some precipitating event causes a patient to develop an insidious state of
Rosen 5th, progressive hyperglycemia and hyperosmolarity, which goes unchecked. By far, acute
Chapter 120, infection is the most common precipitating cause of HHNS. Urinary tract infection
p. 1756 and pneumonia are most common, though uremia, viral illness, and a host of metabolic
and iatrogenic causes have been identified. Similarly, several drugs may predispose or
contribute to hyperglycemia, volume depletion, or other effects leading to HHNS,
especially diuretics, glucocorticoids, lithium, phenytoin, neuroleptics, beta-blockers,
mannitol, didanosine, and calcium-channel blockers.
165

541. It is well known that vomiting leads to hypokalemia. The reason for this is:
a.direct loss of potassium from stomach contents.
b. hypovolemia from volume loss leads to increases in aldosterone secretion, causing the kidney to
preserve sodium and bicarbonate in exchange for potassium, resulting in alkalosis which causes
potassium to shift into cells in exchange for hydrogen ions.
c.vomiting causes hyperventilation, leading to respiratory alkalosis and compensatory extracellular to
intracellular potassium shifts.
d. with the loss of hydrogen ions after a first episode of vomiting, the gastric parietal cells secrete
potassium, which is lost in further vomiting.
e.unknown.

542. An 86-year-old woman from a local nursing home presents in hyperosmolar hyperglycemic
coma. She was recently hospitalized for pyelonephritis and bacteremia. When you look at
her medication list, you suspect that her current condition is more than likely caused by:
a. aspirin.
b. levofloxacin.
c. thyroid replacement.
d. omeprazole (Prilosec).
e. hydrochlorothiazide.

543. In treating the patient mentioned in Question #542, you know that insulin
a is contraindicated.
b. should be given in higher doses than when treating DKA.
c. can safely and effectively be used intramuscularly.
d. can precipitate vascular collapse if it is given prior to volume expansion.
e. can cause a potentially lethal hyperkalemia if given too rapidly.

544. The patient in Question #542 has received 2000 cc of normal saline solution and 12 units
of regular insulin. Her repeat bedside glucose is 245 mg/dL. Your third liter of fluid should
be:
a.0.9% saline.
b. 0.45% saline.
c.Ringers lactate.
d. dextrose / 0.9% saline.
e.5% dextrose / 0.45% saline.

545. The most common cause of hyperthyroidism in the United States is:
a.Graves disease.
b. multinodular goiter.
c.pituitary adenoma.
d. thyroid cancer.
e.thyroiditis.

546. A 40-year-old woman complains of palpitations. Temperature 100.8oF, heart rate 140 /
minute, respiratory rate 22 / minute, blood pressure
148/80 mmHg. Her eyes are shown in this picture. You
immediately suspect:
a.amphetamine toxicity.
b.lithium overdose.
c.myxedema coma.
d.pheochromocytoma.
166

e.thyroid storm.
167

541. e Several drugs may predispose or contribute to hyperglycemia, volume depletion, or


th
Rosen 5 , other effects leading to HHNS, especially diuretics, glucocorticoids, lithium,
Chapter 120, phenytoin, neuroleptics, beta-blockers, mannitol, didanosine, and calcium-channel
p. 1755 blockers.
542. b The hypokalemia associated with vomiting has very little to do with the actual K+ lost
Rosen 5th, in the vomitus and much more to do with the metabolic alkalosis that follows. The
Chapter 119, hypovolemia from volume loss leads to increases in aldosterone secretion, which acts
p. 1728 on the kidney to preserve Na+ and bicarbonate in exchange for K+. The resultant
alkalosis also causes K+ to shift into cells in exchange for H+.
543. d Initial volume replacement should always precede the initiation of an insulin drip in
Rosen 5th, HHNS. If insulin is utilized prior to rehydration, intravascular volume may be further
Chapter 120, depleted. Insulin will cause a shift of osmotically active glucose into the intracellular
p. 1756 space, bringing free water with it. This will further deplete the intravascular
compartment and may precipitate vascular collapse. Another relative contraindication
to the early use of insulin in HHNS is hypokalemia. Insulin promotes transport of
potassium into cells and it should be withheld until the potassium level is known and
hypokalemia has been treated.
544. e Once serum glucose decreases to less than 300 mg/dL, the intravenous solution should
Rosen 5th, be changed to D5W 0.45 percent NaCl and the insulin infusion should be reduced to
Chapter 120, half or 0.05 (units/kg)/h.
p. 1756
545. a Graves disease is by far the most common cause, accounting for more than 80% of
th
Rosen 5 , cases of hyperthyroidism in the United States. Toxic multinodular and toxic
Chapter 173, (adenoma) nodular goiters are the next most frequent causes. Less common causes of
p. 2441 hyperthyroidism are thyroiditis, pituitary tumors, metastatic thyroid cancer, and
dermoid tumors or teratomas of the ovary.
546. e Common symptoms of thyroid storm include palpitations, dyspnea, and chest pain.
Rosen 5th, Fever is often present, but the tachycardia may be elevated out of proportion to the
Chapter 122, height of fever. The manifestations of ophthalmopathy include upper-lid retraction,
p. 1771 staring, lid lag (Graefes sign), exophthalmos, and extraocular muscle palsies.
168

547. The most important treatment for the reduction of morbidity and mortality in thyroid storm
is:
a.inhibit hormone synthesis.
b.block hormone release.
c.prevent peripheral conversion of T4 to T3
d.block the peripheral effects of thyroid hormone.
e.provide general support.

548. A drug to avoid in a patient in thyroid storm is:


a. acetaminophen.
b. aspirin.
c. calcium chloride.
d. calcium gluconate.
e. insulin.

549. You are treating a patient in thyroid storm. You have given propranolol two mg
intravenously. An appropriate follow-up regimen would be:
a.intravenous methimazole followed by rapid intravenous sodium iodide.
b. intravenous propylthiouracil (PTU) followed by slow intravenous sodium iodide.
c.oral propylthiouracil followed by slow intravenous sodium iodide.
d. oral sodium iodide followed by intravenous methimazole.
e.oral sodium iodide followed by intravenous propylthiouracil.

550. Secondary hypothyroidism is usually caused by panhypopituitarism and initially leads to:
a. thyroglobulin deficiency.
b. thyroid releasing hormone deficiency.
c. thyrotropin (thyroid stimulating hormone) deficiency.
d. thyroxin deficiency.
e. triiodothyrinine deficiency.

551. A 70-year-old patient is sent by her family doctor because


Shes just not acting right. Rectal temperature 95.2oF, heart
rate 48 / minute, blood pressure 102/72 mmHg. She is slow
to respond. You notice periorbital edema and a thick tongue.
Her EKG shows sinus bradycardia and low voltage, but no
ischemia. Her general appearance is as shown here. In
addition to abnormal thyroid function testing, you would also
expect to find:
a. hyperchloremia.
b. hyperglycemia.
c. hypocarbia.
d. hypomagnesemia.
e. hyponatremia.

552. Concerning the patient in Question #551, you will need to begin therapy using intravenous:
a. high-dose steroids and a diuretic.
b. levothyroxine and hydrocortisone.
c. thyroglobulin and ACTH.
d. thyrotropin and insulin.
e. triiodothyrinine and a beta-blocker.
169

547. d Initial treatment of thyroid storm consists of stabilization, airway protection,


th
Rosen 5 , oxygenation, intravenous fluids, and monitoring. Blockade of the peripheral
Chapter 122, adrenergic hyperactivity of thyroid crisis may be the most important factor in reducing
p. 1774 morbidity and mortality. Beta-blockade is currently the method of choice for
staunching the peripheral manifestations of thyroid storm. Propranolol can reduce
dysrhythmias, hyperpyrexia, tremor, palpitations, restlessness, anxiety, and perhaps
myopathy.
548. b Hyperpyrexia of thyroid storm should be treated aggressively with acetaminophen.
Rosen 5th, Aspirin should not be used because it displaces thyroid hormone from thyroglobulin,
Chapter 122, thus theoretically increasing the pool of metabolically active hormone. Ice packs and
p. 1774 hypothermia blankets may also be used.
549. c Thioamides, including propylthiouracil (PTU) and methimazole, inhibit thyroidal
Rosen 5th, peroxidase, thereby preventing hormone synthesis. PTU is generally preferred over
Chapter 122, methimazole because it has the additional minor effect of inhibiting peripheral
p. 1774 conversion of T4 to T3. PTU is given in an initial dose of 600 to 1000 mg by mouth
(PO) or by nasogastric (NG) tube, followed by 200 to 250 mg every 4 to 6 hours.
Further organification of iodine will be blocked within 1 hour of PTU administration,
but the drug should be continued for several weeks while the hyperthyroidism is
brought under control. Because preformed T4 and T3 are stored in the thyroid colloid,
release of hormone can occur for weeks despite synthesis inhibition. Thus prevention
of colloid hormone release is the second goal of therapy. Both iodine and lithium can
inhibit thyroid hormone release. Lithium is not generally used because it can be
difficult to titrate the dose, and toxic effects are common. Thioamides should be given
at least 1 hour before iodine therapy to prevent organification of the iodine. Lugols
iodine solution, 30 drops per day in 3 to 4 divided doses PO or by NG tube; potassium
iodide (SSKI), 5 drops every 6 hours PO or by NG tube; or sodium iodide, 1 g slow
intravenous (IV) drip every 8 to 12 hours, is acceptable.
550. c Regulation of synthesis and release of thyroid hormone is under the control of the
Rosen 5th, anterior pituitary gland via thyroid-stimulating hormone (TSH), or thyrotropin.
Chapter 122, Regulation of TSH in turn is by hypothalamic thyrotropin-releasing hormone (TRH)
p. 1778 and also by means of a feedback loop to the pituitary gland by circulating thyroxine
(T4) and triiodothyrinine (T3) levels. Thyroid hormone production depends on
adequate iodine intake and synthesis of thyroglobulin.
551. e Laboratory evaluation of patients with suspected myxedema coma may reveal anemia;
Rosen 5th, hyponatremia; hypoglycemia; elevated transaminases, creatine phosphokinase, and
Chapter 122, lactate dehydrogenase levels; hypercholesterolemia; and arterial blood- gas
p. 1778 abnormalities (decreased PO2 and increased PCO2). The electrocardiogram may
demonstrate sinus bradycardia, prolongation of the QT interval, and low voltage with
flattening or inversion of T waves.
552. b Specific therapy includes intravenous levothyroxine; an initial intravenous bolus is
Rosen 5th, administered, followed by a reduced daily dose until the patient can take oral
Chapter 122, medication. This has the advantage of repleting the T4 pool and allowing the hormone
p. 1778-1779 to enter tissues slowly. Stress dosages of corticosteroids, such as 300 mg of
hydrocortisone IV followed by 100 mg IV every 6 to 8 hours, are also routinely given
because myxedema may be either a manifestation of panhypopituitarism or a
coexisting condition with primary adrenal failure.
170

553. The most common precipitating factor for myxedema coma is:
a. barbiturates.
b. depression.
c. diuretics.
d. infection.
e. trauma.

554. A 62-year-old man presents with classic symptoms of hypothyroidism mild hypothermia,
bradycardia, hoarse voice, and bilateral carpal tunnel syndrome. You find no signs of
infection. He has an extensive cardiac history, including an automatic internal defibrillator,
and is on a waiting list for heart transplant. He takes many medications, but the one you
suspect is probably causing his thyroid malfunction is:
a. amiodarone (Cordarone).
b. bumetinide (Bumex).
c. enalapril maleate / felodipine (Lexxel).
d. lovastatin (Mevacor).
e. sotalol (Betapace)

555. The most common cause of acute adrenal insufficiency and adrenal crisis is
a. hypothalamic infarct.
b. fuunctional, from exogenous glucocorticoid administration.
c. non-adrenal catecholamine-secreting tumors.
d. post-partum panhypopituitarism (Sheehans syndrome).
e. Waterhouse-Friderichsen syndrome.

556. Aldosterone is the primary mineralocorticoid. It is regulated by the renin-angiotensin system and
acts to:
a. change the osmotic set-point in the hypothalamus.
b. decrease potassium excretion.
c. decrease water retention.
d. increase calcium reabsorption.
e. increase sodium reabsorption.

557. Common laboratory findings in a patient with adrenal insufficiency include:


a. high sodium, high potassium, high glucose.
b. high sodium, low potassium, low glucose.
c. low sodium, high potassium, low glucose.
d. low sodium, low potassium, high glucose.
e. low sodium, low potassium, low glucose.

558. A 28-year-old woman has just returned from a 10-day cruise. She forgot to bring her prednisone,
which she uses to treat her rheumatoid arthritis. She started vomiting yesterday and now appears
quite dehydrated. Pulse 120 / minute, blood pressure 84/42 mmHg. The best way to treat her
hypotension is:
a. corticosteroids.
b. intravenous fluids and corticosteroids.
c. intravenous fluids and pressor agents.
d. intravenous fluids.
e. pressor agents.
171

553. d A patient suspected of presenting with myxedema coma commonly has a prior history
th
Rosen 5 , of primary hypothyroidism or previous thyroid surgery. Medication noncompliance or
Chapter 122, coexisting stressors such as cold exposure, severe infection, or the addition of new
p. 1778 medications may precipitate the onset of myxedema coma. By far, infection is the
most common cause.
554. a The many complex effects of iodine-rich amiodarone on thyroid physiology may lead
Rosen 5th, to asymptomatic abnormalities of thyroid hormone levels, including an elevated TSH,
Chapter 122, as well as clinically relevant hypofunction of the thyroid gland. Hypothyroidism has
p. 1775 been estimated to occur in 1% to 32% of patients taking amiodarone.
555. b The most common cause of tertiary adrenal insufficiency and adrenal crisis is
th
Rosen 5 , functional, from exogenous glucocorticoid administration. Rapid withdrawal of
Chapter 122, steroids from patients with adrenal atrophy secondary to chronic steroid use may result
p. 1779 in collapse and death, especially under circumstances of increased stress.
556. e The major mineralocorticoid is aldosterone. The renin-angiotensin system and plasma
Rosen 5th, potassium concentration regulate aldosterone through negative feedback loops.
Chapter 122, Aldosterone acts to increase sodium reabsorption and potassium excretion, primarily in
p. 1781 the distal tubules of the kidneys.
557. c The usual laboratory findings in patients with primary adrenal insufficiency include
Rosen 5th, hyponatremia, hyperkalemia, hypoglycemia, and azotemia. Hyponatremia is present in
Chapter 122, 88% of cases and is usually mild to moderate; severe hyponatremia (<120 mEq/L) is
p. 1781-1782 rare. Hyperkalemia is present in 64% of cases, usually mild; the potassium level rarely
exceeds 7 mEq/L. Two-thirds of patients with adrenal failure have hypoglycemia and
the glucose levels are less than 45 mg/dl; the pathophysiology is decreased
gluconeogenesis and increased peripheral glucose use secondary to lipolysis.
558. b Several mechanisms produce hypotension. Cortisol deficiency, even in the presence of
Rosen 5th, normovolemia, can lead to hypotension by directly depressing myocardial contractility.
Chapter 122, Responsiveness to catecholamines is also reduced. If aldosterone deficiency coexists,
p. 1781 sodium wasting can lead to hypovolemia. Volume deficits are greater in primary than
in secondary adrenal insufficiency. Elevations in renin-angiotensin function and ADH
secretion are seen and partially compensate for the relative or absolute hypovolemia
present. Understandably, response to pressors is poor, to volume replacement better,
and to volume plus corticosteroids best.
172

559. You are seeing a young man whom you believe to have adrenal insufficiency. You have started
intravenous fluids and arranged for admission. Because of his hypotension, you wish to start
appropriate replacement therapy, but you know that he will need an ACTH stimulation test
performed to confirm your diagnosis. Therefore you choose to give:
a. cortisone acetate.
b. dexamethasone phosphate.
c. fludrocortisone acetate (Florinef).
d. hydrocortisone sodium succinate.
e. hypertonic 3% saline solution.

560. Adrenal crisis is an acute, life-threatening emergency that must be suspected and treated based upon
clinical impression. The primary treatment of choice for a patient known to have adrenal failure is:
a. cortisone acetate
b. dexamethasone phosphate
c. fludrocortisone acetate (Florinef)
d. hydrocortisone hemisuccinate
e. hypertonic 3% saline solution

561. The most important blood protein buffer is:


a. albumin.
b. fibrinogen.
c. glucose-6 phosphatase.
d. hemoglobin.
e. myoglobin.

562. The most common acid-base disorder seen in the seizing patient is:
a. metabolic acidosis.
b. metabolic alkalosis.
c. mixed respiratory / metabolic acidosis.
d. respiratory acidosis.
e. respiratory alkalosis.

563. You are treating a patient with diabetes whom you suspect to be in ketoacidosis. She has
a fruity smell on her breath and she is taking deep, rapid respirations. Which statement is
True?
a. DKA occurs most often in type II diabetics
b. The occurrence of DKA in Type II patients is extremely rare
c. DKA can be caused by any condition that reduces insulin availability or activity or that decreases
glucagons
d. DKA can be induced by emotional or physical stressors
e. In DKA, decreased lipolysis resulting in the breakdown of free fatty acids, leads to production of
ketoacids.

564. Choose the correct match.


a. acute respiratory acidosis increased PaCO2, decreased pH, normal HCO3
b. chronic respiratory acidosis increased PaCO2, decreased pH, increased HCO3
c. acute respiratory alkalosis decreased PaCO2, increased pH, normal plasma HCO3
d. chronic respiratory alkalosis decreased PaCO2, increased pH, decreased plasma HCO3
e. all of the above are correct.
173

559. b If the diagnosis of adrenal failure is unconfirmed, dexamethasone phosphate, 4 mg IV


th
Rosen 5 , every 6 to 8 hours, is the corticosteroid replacement that should be used while an
Chapter 122 ACTH stimulation test is performed. Dexamethasone is approximately 100 times more
p.1782 potent than cortisol, and this amount of dexamethasone will not factitiously elevate
serum cortisol determinations. Replacement with hydrocortisone could confound
interpretation of serum cortisol determinations.
560. d For a patient known to have adrenal failure, 100 mg of hydrocortisone hemisuccinate
Rosen 5th, IV every six to eight hours should be used. If IV access cannot be obtained, cortisone
Chapter 122, acetate, 100 mg IM every six to eight hours, may be used, but its absorption is erratic
p. 1782 and not as reliable as the IV route.
561. d Many protein buffers in blood are effective in maintaining acid-base homeostasis. The
th
Rosen 5 , most important is hemoglobin, which can buffer large amounts of H+, preventing
Chapter 118, significant changes in the pH. If hemoglobin did not exist, venous blood would be 800
p. 1714-1715 times more acidic than arterial blood, circulating at a pH of 4.5 instead of the normal
venous pH of 7.37.
562. d Transient respiratory acidosis is the most common acid-base disorder seen acutely in
Rosen 5th, the seizure patient. Treatment centers on control of the seizure activity and assisted
Chapter 118, ventilation if required. Intravenous bicarbonate is not recommended because these
p. 1717 patients resolve their acid-base disorder with return of spontaneous ventilation.
563. d DKA presents clinically as a triad: hyperglycemia (usually >200 mg/dl), ketonemia
Rosen 5th, (>1:2 dilutions) and academia (pH <7.3). DKA can be caused by any condition that
Chapter 118, reduces insulin availability or activity or that increases glucagons. DKA occurs most
p. 1715 often in type I diabetic patients with little or no endogenous insulin; however, its
occurrence in type II patients, particularly obese African-Americans, is not as rare as
once thought. Increase lipolysis, resulting in the breakdown of free fatty acids, leads to
production of ketoacids. Precipitating events usually include infections, surgery, and
emotional and physical stressors.
564. e All are correct.
Rosen 5th,
Chapter 118,
p. 1716-1722
174

565. Treatment for patients with saline responsive metabolic alkalosis might include:
a. intravenous mineral acids (i.e. arginine monohydrochloride)
b. normal saline solution.
c. acetazolamide.
d. dialysis.
e. all of the above.

566. Choose the correct statement about lactic acidosis:


a. The body does not have a mechanism to break down lactic acid, accounting for its
toxicity.
b. Most lactic acidosis is associated with normal tissue perfusion.
c. The liver converts about 20% of lactate into glucose.
d. Pyruvate metabolism does not affect the fate of lactic acid.
e. Since most lactic acid is cleared by the kidneys, its excretion is enhanced by using
intravenous bicarbonate.

567. When considering electrolytes:


a. calcium is primarily intracellular.
b. magnesium is primarily extracellular.
c. phosphorus is primarily extracellular.
d. potassium is primarily intracellular.
e. sodium is primarily intracellular.

568. Daily maintenance fluid requirements for an average-sized patient with a fever of 101oF is
about:
a. 1000 1500 cc.
b. 1500 2500 cc.
c. 2000 3000 cc.
d. 3000 4000 cc.
e. 4000 5000 cc.

569. The most common cause of redistributive hyponatremia is:


a. diuretic use
b. hyperglycemia
c. laxative abuse
d. protein-wasting nephropathy
e. water intoxication

570. In patients with symptomatic chronic hyponatremia, correction must take place in a
controlled fashion. Overaggressive saline administration can lead to:
a. acute renal failure.
b. ARDS.
c. coma, seizures, and quadraparesis.
d. fulminant hepatic failure.
e. life-threatening cardiac dysrhythmias.

571. The only known mechanism for hypervolemic hypernatremia is:


a. overdose on mercurial diuretics.
b. licorice ingestion.
c. salicylate toxicity.
d. excess sodium intake.
175

e. laxative abuse.
176

565. a In a patient with saline responsive alkalosis, treatment is directed toward correcting the
th
Rosen 5 , urinary excretion of HCO3-. Administration of NaCl and KCl suppresses both renal
Chapter 118, acid excretion and HCO3- excretion. In patients who are severely volume depleted,
p. 1721 administration of IV mineral acids (e.g., arginine monohydrochloride) may be
necessary. In edematous states for which saline therapy may be contraindicated,
acetazolamide will increase the excretion of HCO3-, treating both the alkalosis and the
edema.
566. d Lactic acidosis is a marker of hypoperfusion and ongoing shock. A product of
Rosen 5th, anaerobic metabolism, lactic acidosis develops when an imbalance exists between
Chapter 118, lactic acid production and subsequent conversion by the liver and kidney. Pyruvic acid
p. 1119-1120 is metabolized aerobically into CO2 and H2O or anaerobically to lactic acid. Lactic
acid is then buffered by HCO3-, forming lactate. The liver converts 80% of lactate into
CO2 and H2O and 20% to glucose. This reaction regenerates HCO3- in the process and
balances the acid-base balance.
567. d Electrolytes are classified according to electronic charge, cations (positively charged
Rosen 5th, ions) and anions (negatively charged ions). The principal cations in the intracellular
Chapter 119 compartment are K+ and Mg2+, whereas the principal anions are PO4- and proteins. In
the plasma and interstitial fluids, which together make up the extracellular fluid space,
Na+ is the predominant cation, while Cl- and HCO3- are the major anions.
568. d To maintain balance, an average normal adult requires approximately 2000 to 3000 mL
Rosen 5th, intake of water per day. This accounts for the volume of water lost in a day due to
Chapter 119 insensible and urinary losses. The insensible loss can accelerate dramatically in the
setting of fever (500 mL per 1C fever), sweating (up to 1500 mL), and gastrointestinal
losses.
569. b Hyponatremia with an increase in osmotically active solutes occurs when there is an
Rosen 5th, accumulation of large quantities of solutes restricted primarily to the ECF space. In
Chapter 119, this setting, there is a net movement of water from the ICF to the ECF, thereby
p. 1724 effectively diluting the ECF [Na+]. The most common cause of this is hyperglycemia.
Each 100-mg/dL increase in plasma glucose decreases the serum [Na+] by 1.6 to 1.8
mEq/L.
570. c Acute hyponatremia may be corrected at rates of up to 1 to 2 mEq/L/hr, and chronic
Rosen 5th, hyponatremia should be corrected at a rate not greater than 0.5 mEq/L/hr. In general
Chapter 119, the serum sodium should not be corrected to above 120 mEq/L or increased by more
p. 1726 than 10 mEq/L in a 24-hour period. Hypertonic saline should be administered through
a controlled intravenous (IV) infusion, with careful attention to fluid input and output
and frequent assessment of serum electrolytes. The approximate required dose of
hypertonic saline can be calculated with the following formula:
(desired [Na+] measured [Na+]) x (0.6) (weight in kg) = mEq [Na+] administered
Overaggressive correction of the serum sodium may have serious consequences.
Central pontine myelinolysis (CPM), also known as cerebral demyelination, involves
the destruction of myelin in the pons and is thought to result from rapid elevation of
the serum sodium. Patients may develop cranial nerve palsies, quadriplegia, or coma.
CPM is more likely to occur in patients with chronic hyponatremia than in those with
acute hyponatremia.
571. d Excessive sodium intake, accidentally, intentionally, or iatrogenically, may cause
Rosen 5th, hypernatremia in the absence of corresponding intake of water. Because the kidney
Chapter 119, can usually excrete an increased sodium load effectively, most cases are seen in
p. 1726-1727 patients with renal insufficiency.
177

572. An elderly woman is found unconscious in her non-air conditioned apartment during a July
heat spell. Heart rate 125 / minute, blood pressure 70/palpable. Her serum sodium is 170
mEq/L, so you make sure that the intravenous which is running contains:
a. 0.9% saline.
b. 0.45% saline.
c. 0.225% saline.
d. 5% dextrose.
e. 5% dextrose / 0.9% saline.

573. In a patient with a potassium level of 7.6 mEq/L and this


rhythm strip, the medication which should be given first is:
a. calcium gluconate 5 10 mEq intravenously to stabilize
cardiac membranes.
b. furosemide 40 mg intravenously to promote renal K+
excretion.
c. glucose 50 gm intravenously to move K+ to the
intracellular space.
d. sodium bicarbonate 50 mEq intravenously to assist in K+ redistribution.
e. sodium polystyrene (Kayexalate) 15 to 50 gram orally or rectally to bind K+ for rectal
excretion.

574. In the absence of acute shifts caused by acid-base disturbances, a decrease of the serum
potassium by 1.0 mEq/L may reflect what deficit of potassium?
a. 80 mEq/L
b. 150 mEq/L
c. 225 mEq/L
d. 300 mEq/L
e. 370 mEq/L

575. If you were to inadvertently rapidly bolus a liter of 0.45% saline into a normovolemic
osmotically balanced patient, the first change you might notice is:
a. contraction of both extracellular fluid and intracellular fluid.
b. contraction of extracellular fluid, expansion of intracellular fluid.
c. expansion of both extracellular fluid and intracellular fluid.
d. expansion of extracellular fluid, contraction of intracellular fluid.
e. expansion of extracellular fluid, no change in intracellular fluid.

576. In a patient with diabetes insipidus:


a. fluid restriction is essential.
b. hypernatremia is the rule.
c. intranasal vasopressin may be helpful.
d. there is an osmolar gap.
e. urine specific gravity is high.

577. A patient is found to have a serum calcium level of 4.5 mEq. You expect most of her
symptoms to be related to the:
a. central nervous system.
b. pulmonary system.
c. cardiovascular system.
d. neuromuscular system.
e. gastrointestinal system.
178

572. a The primary goals in the emergency management of hypovolemic hypernatremic


th
Rosen 5 , patients are to restore volume deficits and to maintain organ perfusion. Treatment
Chapter 119, should begin with an infusion of isotonic solution (0.9% NaCl). Once the patient is
p. 1727p. hemodynamically stable, the remaining free water deficits can be replaced.
573. a Treatment of hyperkalemia involves three processes: membrane stabilization
th
Rosen 5 , (especially cardiac tissue), shifting potassium from extracellular to intracellular, and
Chapter 119, removing excess potassium from the body. Membrane stabilization is achieved with
p. 1731 calcium gluconate or calcium chloride. This is indicated in any patient with unstable
dysrhythmias or hypotension. Several ampules of calcium ( 10 ml or 10% solution)
may be required. Because of the brief duration of action (approximately 20-40
minutes), other measures should also be instituted promptly.
574. c In the absence of acute shifts caused by acid-base disturbances, a decrease
Rosen 5th, of the serum potassium by 1.0 mEq/L may reflect a 370 mEq deficit of total
Chapter 119, potassium.
p. 1729
575. b Hypotonic saline infusion will lower the extracellular fluid osmotic strength, which
th
Rosen 5 , will then cause a shift of extracellular fluid into the cells until the osmolalities are
Chapter 119, equalized.
p. 1727
576. c Diabetes insipidus (DI) results in loss of large amounts of dilute urine because of the
th
Rosen 5 , loss of concentrating ability in the distal nephrons. DI can be central (lack of ADH
Chapter 119, secretion from the pituitary) or nephrogenic (lack of responsiveness to circulating
p. 1726 ADH). Patients are usually able to maintain near-normal serum levels as long as
access to water is maintained. Patients will have a low urine specific gravity (<1.005)
and low urine osmolality. Those with central DI require the administration of
parenteral or intranasal vasopressin.
577. d The clinical manifestations of hypocalcemia depend not only on the serum level but
Rosen 5th, also on the rapidity with which it declines. Although the signs and symptoms of
Chapter 119, hypocalcemia are numerous, the effects on neuromuscular function predominate. A
p. 1733 declining serum calcium level is associated with progressive neuromuscular
hyperexcitability.
179

578. The most common cause of symptomatic hypokalemia is:


a. hyperemesis.
b. diuretic therapy.
c. beta-agonist therapy.
d. diabetes insipidus.
e. SIADH.

579. Patients who have undergone prolonged starvation and are suddenly fed can develop:
a. hypocalcemia.
b. hypoglycemia.
c. hypokalemia.
d. hyponatremia.
e. hyperphosphatemia.

580. In an adult patient with tetany and a positive Trousseau sign 5 days after thyroidectomy,
an appropriate treatment would be:
a. 10 ml of 10% calcium chloride IV bolus.
b. 100 ml of 10% calcium gluconate IV slow drip.
c. 25 ml of intravenous calcium biphosphonate.
d. 50 ml of sodium bicarbonate, then 200 mcg intravenous PTH.
e. fresh frozen plasma 4U.

581. A 61-year-old man complains of


weakness, numbness around his
mouth, and no appetite. When the
nurse checks his blood pressure she
notes this finding. You send serum
calcium, expecting it to be low. You
also order an EKG, expecting to find:
a. frequent PVCs.
b. prolonged QT interval.
c. second or third degree heart block.
d. shortened PR interval.
e. tachycardia.

582. More than 90% of cases of hypercalcemia are associated with:


a. adrenal insufficiency.
b. dietary indiscretion.
c. malignancy and primary hyperparathyroidism.
d. sarcoidosis and other granulomatous diseases.
e. vitamin A or D toxicity.

583. A 54-year-old woman who is being treated for breast cancer complains of fatigue, malaise,
and loss of appetite. Clinically she is mildly dehydrated. Her serum calcium is 14.3 mg/dl.
You begin treatment in the Emergency Department, realizing that:
a. glucocorticoids are contraindicated.
b. hypotonic saline administration will lead to normalization more than half the time.
c. the side effects of digoxin are accentuated in the presence of hypercalcemia.
d. simultaneously occurring hypermagnesemia must be recognized and treated.
e. thiazide diuretics are preferred over loop diuretics.
180

578. b Diuretic therapy, the most common cause of hypokalemia in clinical practice, increases
th
Rosen 5 , sodium delivery to the distal tubule, promoting potassium excretion. Associated
Chapter 119, volume depletion and high levels of aldosterone cause K+ and H+ excretion and may
p. 1728 worsen hypokalemia. In addition, alkalosis from H+ excretion promotes cellular
potassium uptake, further lowering the serum potassium. Other disorders can cause
significant renal potassium loss. These include osmotic diuresis, high
579. c mineralocorticoid states, magnesium depletion, and high urinary concentrations of
Rosen 5th, anions such as penicillin. Intrinsic renal causes of potassium loss include renal tubular
Chapter 119, acidosis (RTA), chronic interstitial disease, and drugs that affect tubular potassium
p. 1728 reabsorption. RTA type 1 is caused by a defect in H+ secretion in the distal tubule,
and RTA type 2 is associated with a similar defect in the proximal tubule.
Hypokalemia resulting from decreased dietary intake is rare. However, when poor
intake is combined with other factors (e.g., vomiting or diarrhea, high insulin or
aldosterone levels), severe hypokalemia may result. Patients suffering from prolonged
starvation may become hypokalemic when they are fed because insulin secretion and
increased cellular uptake cause potassium to move into cells.
580. a Two different calcium formulations are readily available in most Emergency
Rosen 5th, Departments: (1) 10-ml ampules of 10% calcium chloride, which contain 360 mg of
Chapter 119, elemental calcium, and (2) 10-ml ampules of 10% calcium gluconate, which contain 93
p. 1733 mg of elemental calcium. For the adult patient the recommended initial dose is 100 to
300 mg of elemental calcium given as calcium chloride or calcium gluconate. This
dose of calcium will increase the serum ionized calcium for only a short time (1 to 2
hours) and should be followed by repeated doses or an infusion at a rate of 0.5 to 2
mg/kg/hr.
581. b Severe hypocalcemia causes a decrease in myocardial contractility and rarely
Rosen 5th, bradycardia, hypotension and symptomatic CHF. Patients with preexisting cardiac
Chapter 119, dysfunction and those taking digoxin or diuretics are especially at risk. The ECG may
p. 1733 demonstrate QT prolongation and an inverse relationship exists between the serum
calcium and the QT interval. However, the ECG is a poor predictor of hypocalcemia
and should not be used to rule in or rule out this disorder.
582. c Hypercalcemia is a relatively common entity. It is defined as a total calcium level
Rosen 5th, exceeding 10.5 mg/dL or an ionized calcium level exceeding 2.7 mEq/L. Over 90% of
Chapter 119, occurrences are associated with hyperparathyroidism or malignancy.
p. 1734
583. c Calcium potentiates the action of digoxin, and the side effects of digoxin are
th
Rosen 5 , potentiated when hypercalcemia is present. The administration of isotonic saline is the
Chapter 119, first step in the management of severe hypercalcemia. Once the intravascular volume
p. 1735 has been restored to normal, the serum calcium will usually have decreased by 1.6 to
2.4 mg/dl, although hydration alone rarely leads to complete normalization. Loop
diuretics such as furosemide inhibit the resorption of calcium in the thick ascending
loop of Henle, increasing the calciuric effect of hydration. Volume expansion must
precede the administration of furosemide. Thiazide diuretics should not be used
because they enhance distal absorption of calcium and may worsen hypercalcemia.
Glucocorticoids are useful in patients with sarcoidosis, vitamin A or D intoxication,
multiple myeloma, leukemia, or breast cancer. They work by inhibiting bone
resorption and gastrointestinal absorption of calcium.
181

584. The most likely EKG finding in a patient with severe magnesium depletion is:
a. bradycardia.
b. PVCs and other ventricular dysrhythmias.
c. prolonged QT interval.
d. second or third degree heart block.
e. shortened PR interval.

585. Clinically significant hypermagnesemia is encountered almost exclusively in patients with:


a. adrenal insufficiency.
b. exogenous magnesium administration, like antacids.
c. impaired renal magnesium excretion.
d. rhabdomyolysis.
e. tumor lysis syndrome.

586. You are treating a pregnant woman for pre-eclampsia while awaiting emergent delivery.
She shows signs of iatrogenic hypermagnesmeia. After infusing isotonic saline and giving
a loop diuretic, you can reverse her respiratory depression and hypotension by giving:
a. 20 ml of 10% calcium chloride IV bolus.
b. 25 ml of intravenous potassium phosphate.
c. 40 mEq potassium chloride intravenously over two hours.
d. 50 100 mEq IV sodium bicarbonate.
e. methotrexate.

587. A 45-year-old man complains of myalgias and severe weakness. You have treated him in
the past for alcohol-related diseases. You suspect he is suffering from:
a. hypercalcemia.
b. hyperchloremia.
c. hyperkalemia.
d. hyponatremia.
e. hypophosphatemia.

588. A 68-year-old man complains of shortness of breath. He takes digoxin for chronic atrial
fibrillation. You listen to his heart and find an irregular rhythm. You know that digitalis-
induced dysrhythmias can be made worse by:
a. hypocalcemia.
b. hypoglycemia.
c. hyperkalemia.
d. hypomagnesemia.
e. hypophosphatemia.

589. The most common cause of hyperphosphatemia is:


a. adrenal insufficiency.
b. exogenous phosphorus administration.
c. renal failure.
d. rhabdomyolysis.
e. tumor lysis syndrome.
182

584. b Dysrhythmia is the most common cardiovascular manifestation of hypomagnesemia.


th
Rosen 5 , A number of studies demonstrate an increased incidence of supraventricular
Chapter 119, dysrhythmias (atrial fibrillation, multifocal atrial tachycardia, paroxysmal
p. 1738 supraventricular tachycardia) and ventricular dysrhythmias (PVCs, ventricular
tachycardia, torsade de pointes, ventricular fibrillation) in patients who are magnesium
deficient. Digitalis-induced dysrhythmias are also more likely in the presence of
hypomagnesemia. Because magnesium is an essential cofactor for the Na+-K+
ATPase pump that is inhibited by digitalis, hypomagnesemia typically worsens the
manifestations of digitalis toxicity.
585. c Clinically significant hypermagnesemia is encountered almost exclusively in the
Rosen 5th, setting of renal insufficiency. Serum magnesium levels rise as the creatinine clearance
Chapter 119, falls below 30 ml/min and typically reach approximately 2.5 mEq/L as renal function
p. 1738 nears zero.
586. a Patients with severe hypermagnesemia should receive IV calcium. Calcium directly
th
Rosen 5 , antagonizes the membrane effects of hypermagnesemia and reverses respiratory
Chapter 119, depression, hypotension, and cardiac dysrhythmias. For life-threatening
p. 1739 manifestations of hypermagnesemia, 100 to 200 mg of calcium, as either 10% calcium
gluconate (93 mg calcium per ampule) or 10% calcium chloride (360 mg calcium per
ampule), is a reasonable dose.
587. e Although diabetic ketoacidosis is an important cause of hypophosphatemia, decreased
Rosen 5th, phosphate intake and impaired intestinal phosphate absorption are other causes of
Chapter 119, hypophosphatemia. Up to 50% of alcoholics are hypophosphatemic. Increased renal
p. 1740 excretion and decreased intake are the proposed mechanisms. Hypophosphatemia may
be exacerbated when glucose-containing solutions are administered because these
cause phosphate to shift from the serum into cells.
588. d Hypomagnesemia, like hypokalemia and hypercalcemia, will worsen the
Rosen 5th, manifestations of digitalis toxicity. Digitalis-induced dysrhythmias are more likely in
Chapter 119, the presence of hypomagnesemia.
p. 1738
589. c Renal failure is the most common cause of hyperphosphatemia. The serum phosphate
th
Rosen 5 , typically remains normal until the creatinine clearance falls below 30 ml/min.
Chapter 119, Hyperphosphatemia is usually mild unless an exogenous phosphate load is given
p. 1741
183

6.0 Environmental Disorders


590. Which statement is true concerning the physiology of cold exposure?
a. Just as with heat exposure, humans develop physiologic adaptation to the cold.
b. During cold stress, peripheral vasodilation limits radiant heat loss.
c. Cutaneous blood flow plays a minimal role in maintaining heat homeostasis.
d. The unsheltered and homeless remain the group most likely to suffer cold-related injury.
e. Peripheral cooling of the blood activates the preoptic anterior hypothalamus, which then
orchestrates temperature regulation.

591. The ultimate determinant of progressive tissue damage from cold-exposure appears to be
injury to the:
a. clotting cascade.
b. complement cascade.
c. microvasculature.
d. platelets.
e. red blood cells.

592. Predisposing factors to the development of frostbite include:


a. anhidrosis.
b. hypertension.
c. loose clothing and boots.
d. overeating.
e. vasoconstrictors and vasodilators.

593. The most common presenting symptom of frostbite is:


a. burning pain.
b. gnawing pain.
c. numbness.
d. sharp pain.
e. tingling and other dysesthesias.

594. Some studies suggest that chilblains (pernio) can be both treated and prevented with:
a. beta-agonists (terbutaline, albuterol).
b. beta-blockade.
c. naloxone (Narcan).
d. nifedipine (Procardia).
e. steroid burst.

595. When treating an Emergency Department patient who has experienced frostbite:
a. a common error is overshoot rewarming, resulting in a heat damage to the tissues.
b. gentle, passive rewarming is less destructive to damaged tissues than aggressive
warming.
c. rapid rewarming can lead to hypokalemia and severe metabolic alkalosis.
d. rethawing is essentially a painless procedure.
e. rewarming should be continued until the part feels pliable and you see distal erythema.
184

590. e The preoptic anterior hypothalamus is the central thermostat. Cutaneous blood flow in
Rosen 5th the euthermic 70 kg human averages 200 to 250 ml/min. Heat stress causes
Chapter 133, vasodilatation that can increase this amount to 7000 ml/min. In contrast, extreme cold-
p. 1972 induced vasoconstriction reduces flow tenfold to less than 50 ml/min. In contrast to
heat exposure, humans do not appear to display significant physiologic adaptation to
the cold. The unsheltered and the homeless are no longer the most likely group at risk.
Most cases of civilian frostbite now result from routine exposure to cold by individuals
who have not given due consideration to risk factors for cold injury, such as
participants in outdoor recreational activities that produce exposure to unanticipated
drastic climatic changes.
591. c The ultimate determinant of progressive tissue damage appears to be injury to the
Rosen 5th microvasculature. Endothelial cells are the tissue most susceptible to freezing injury.
Chapter 133, After thawing, the vasculature is only patent temporarily. Platelet and erythrocyte
p. 1973 aggregates promptly clog and distort the vasculature. Intense vasoconstriction coupled
with arteriovenous shunting occurs at the interface between normal and damaged
tissue. The injured viable vasculature remains distorted. Local arteritis, medial
degeneration, and intimal proliferative thickening are seen.
592. e Predisposing factors to cold injury include:
Rosen 5th Physiologic genetic, core temperature, previous cold injury, acclimatization,
Chapter 133, dehydration, overexertion, trauma: multisystem, extremity, dermatologic diseases,
p. 1974 physical conditioning, diaphoresis, hyperhidrosis, hypoxia
Mechanical constricting or wet clothing, tight boots, vapor barrier, inadequate
insulation, immobility or cramped positioning
Psychological mental status, fear, panic, attitude, peer pressure, fatigue, intense
concentration on tasks, hunger, malnutrition, intoxicants
Environmental ambient temperature, humidity, duration of exposure, wind chill
factor, altitude, associated conditions quantity of exposed surface, area heat loss:
conductive, evaporative aerosol propellants
Cardiovascular hypotension, atherosclerosis, arteritis, Raynaud syndrome, anemia,
sickle cell disease, diabetes, hypovolemia, shock, vasoconstrictors, vasodilators
593. c The symptoms of frostbite usually reflect the severity of the exposure. The most
Rosen 5th common presenting symptom is numbness, present in more than 75% of patients. All
Chapter 133, patients will have some initial sensory deficiency in light touch, pain, or temperature.
p. 1973 Anesthesia is produced by intense vasoconstrictive ischemia and neurapraxia.
594. d Management of chilblains is supportive. The affected skin should be rewarmed, gently
Rosen 5th bandaged, and elevated. Some European studies support the use of nifedipine, at a
Chapter 133, dose of 20 mg tid, as both prophylactic and therapeutic.
p. 1974
595. e Rewarming should be continued until the part feels pliable and distal erythema is
Rosen 5th noted. Frozen or partially thawed tissue should be rapidly and actively rewarmed by
Chapter 133, immersion in gently circulating water that is carefully maintained at a temperature of
p. 1975 37 to 40 C by thermometer measurement. A common error is premature termination
of rewarming, which results in a partial thaw. The acute thawing of large amounts of
distal musculature extinguishes peripheral vasoconstriction, resulting in the sudden
return of cold, hyperkalemic, acidotic blood to the central circulation. This produces
core temperature after-drop, which is dysrhythmogenic.
185

596. Hypothermia can cause many physiologic changes. Which statement is true.
a. Osborn J waves are both diagnostic and prognostic. The size of the J wave is related to
the arterial pH.
b. Atrial fibrillation is rarely seen as a rhythm of hypothermia. Its presence should clue you
to seek a primary cardiac pathology.
c. Asystole and VF occur spontaneously when the core temperature falls below 25 C.
d. The core afterdrop phenomenon is an interesting finding in the laboratory, but of little
clinical significance.
e. Cerebral metabolism increases 6% to 7% for each 1C decline in temperature.

597. Blood gas analyzers warm the blood to 37C (99F), which increases the partial pressure
of dissolved gases. This results in an ABG report showing incorrect values, especially:
a. higher oxygen, higher carbon dioxide levels, higher pH.
b. higher oxygen, higher carbon dioxide levels, lower pH.
c. higher oxygen, lower carbon dioxide levels, lower pH.
d. lower oxygen, higher carbon dioxide levels, lower pH.
e. lower oxygen, lower carbon dioxide levels, lower pH.

598. Which statement is true concerning laboratory findings in a hypothermic patient?


a. A patients hematocrit may be deceptively low.
b. Acute hypothermia initially elevates blood sugars, but if hyperglycemia persists during
rewarming, you should suspect hemorrhagic pancreatitis or diabetic ketoacidosis.
c. BUN and hematocrit must be closely followed to determine volume status.
d. Extracellular plasma potassium level is highly temperature dependent.
e. White blood cell counts and platelet counts tend to remain normal.

599. In treating a hypothermic patient with a Bair Hugger, you are heating the patient by:
a.convection.
b.conduction.
c.evaporation.
d.transmission.
e.radiation.

600. Administering warm, humidified oxygen via mask or endotracheal tube to a hypothermic
patient:
a. helps prevent the inevitable rewarming dehydration.
b. is an excellent method for rapid rewarming.
c. serves as a heat sump.
d. works best by minimizing heat loss from the lungs.
e. works to promote oxygen use and prevent carbon dioxide retention.

601. The physiology of heat-related disease has been very well described. For instance, we
know that:
a. in a warm environment, convection is the most important mechanism of heat dissipation.
b. skin temperature changes correlate well with changes in rate of heat loss.
c. sweating occurs because of core temperature changes.
d. the major mechanisms of accelerating heat loss are sweating and peripheral vasodilation
e. wind at a velocity of 10 m/sec cools twice as much as wind at velocity 5 m/sec.
186

596. c Asystole and VF occur spontaneously when the core temperature falls below 25C. J
Rosen 5th waves are potentially diagnostic but not prognostic. They may appear at any
Chapter 134, temperature less than 32C. The size of the J wave is not related to arterial pH but
p. 1981 does increase with temperature depression. An analysis of ECGs of accidental
hypothermia patients commonly notes atrial fibrillation when the core temperature is
less than 32C. Core temperature afterdrop is a clinically relevant consideration when
treating patients with a large temperature gradient between the core and the periphery.
Cerebral metabolism decreases 6% to 7% for each 1C decline in temperature.
597. b Blood gas analyzers warm the blood to 37C (99F), which increases the partial
Rosen 5th pressure of dissolved gases. This results in an ABG report showing higher oxygen and
Chapter 134, carbon dioxide levels and a lower pH than the patients actual values. Optimal ABG
p. 1985 levels in the setting of hypothermia have not been determined, and current literature
concludes that correction of ABGs for temperature is unnecessary as a guide to
therapy.
598. b A patients hematocrit may be deceptively high as the result of a decreased plasma
Rosen 5th volume. Splenic, hepatic, and splanchnic sequestration in hypothermia decreases
Chapter 134, leukocyte and platelet counts. Acute hypothermia initially elevates blood sugar levels
p. 1985 through catecholamine-induced glycogenolysis, diminished insulin release, and
inhibition of cellular membrane glucose carrier systems. When hyperglycemia persists
during rewarming, one should suspect hemorrhagic pancreatitis or diabetic
ketoacidosis. The plasma potassium level is independent of the primary hypothermic
process. Because of hypothermic fluid shifts the hematocrit and BUN levels are poor
indicators of a patients actual fluid status.
599. a The Bair Hugger uses hot forced air circulated through a blanket. The air exits
Rosen 5th apertures on the patient side of the cover, which allows a convective transfer of heat.
Chapter 134,
p. 1991
600. d Inhalation rewarming provides a fairly small heat gain and is not effective for rapid
Rosen 5th rewarming. It does, however, minimize heat loss from the lungs, a potential loss of up
Chapter 134, to 30% of the total metabolic heat production.
p. 1991
601. d Temperature-sensitive structures are located both peripherally in the skin and centrally
th
Rosen 5 , in the body. Skin temperature changes correlate poorly with changes in rate of heat
Chapter 135, loss. The skin temperature effects heat loss since a person resting in a warm
p. 1998 environment initiates sweating, even though the core temperature remains constant.
Sweating and peripheral vasodilation are the major mechanisms by which heat loss
may be accelerated. In a warm environment, evaporation of sweat from the skin is the
most important mechanism of heat dissipation. The major effect of wind is attained at
a velocity of 0.5 to 5 m/sec; higher velocities do not appreciably increase cooling.
187

602. Acclimatization is defined as "a constellation of physiologic adaptations that appear in a


normal person as the result of repeated exposures to heat stress." Daily exposure to work
and heat for 100 min/day results in near-maximal acclimatization in 7 to 14 days. This is
characterized by:
a. a later onset of sweating at a higher core temperature.
b. diminished plasma volume.
c. diminished sweat volume to prevent dehydration.
d. increased sweat electrolyte concentration to preserve homeostasis.
e. lower heart rate with higher stroke volume.

603. The most critical variable to prevent heat stroke in young, healthy individuals is:
a. ambient temperature.
b. fluid intake.
c. relative humidity index.
d. sodium intake.
e. sweat sodium level.

604. Patients suffering from heat cramps invariably display:


a. hyperchloremia.
b. hyperkalemia.
c. hypernatremia.
d. hypokalemia.
e. hyponatremia.

605. The main difference between heat exhaustion and heat stroke is:
a. heart rate and blood pressure
b. height of temperature.
c. level of CNS dysfunction.
d. serum transaminases elevations.
e. sweating.

606. Concerning exertional vs. classic heat stroke:


a. classic heat stroke occurs in young people with increased heat production.
b. hypoglycemia is more frequently found in exertional heatstroke.
c. sweating is profuse in classic heatstroke.
d. the absolute height of core temperature is the most important prognostic factor.
e. the distinction between the two is important, as treatment varies depending on the cause.

607. Of methods listed, the preferred treatment to rapidly lower core temperature in heatstroke
is:
a. cooling blanket.
b. evaporative cooling using large circulating fans and skin wetting.
c. gastric lavage.
d. ice packs to axillae and groin.
e. peritoneal dialysis.

608. Lightning is:


a. direct current.
b. alternating current.
c. a combination of both direct and alternating current.
d. neither direct nor alternating current, but a unidirectional current impulse.
188

e. a voltage phenomenon.
189

602. e Acclimatization is characterized by an earlier onset of sweating (at a lower core


th
Rosen 5 , temperature), increased sweat volume, and lowered sweat electrolyte concentration. It
Chapter 135, is hastened by modest salt deprivation and delayed by high dietary salt intake. As
p. 1998 acclimatization proceeds, the sweat sodium concentration drops from approximately 65
mEq/L to as low as 5 mEq/L. The cardiovascular system plays a major role in both
acclimatization and endurance training, largely resulting from a 10% to 25% expansion
of plasma volume that occurs in both situations. Heart rate is lower and associated
with a higher stroke volume.
603. b Exertional heat stroke is most likely to occur in young, healthy people involved in
Rosen 5th, strenuous physical activity. Fluid intake is the most critical variable. Dehydration
Chapter 135, may be minimized by education on work-rest cycles and fluid consumption, and
p. 2004 through provision of cool, pleasantly flavored fluids.
604. e Heat cramps appear to be related to salt deficiency. Several steel mills virtually
th
Rosen 5 , eliminated heat cramps in their employees by encouraging salt consumption.
Chapter 135, Anecdotal reports from Germany and England indicate that laborers salt their beer to
p. 2001 successfully prevent heat cramps. Heat cramp victims exhibit hyponatremia,
hypochloremia, and low urinary sodium and chloride levels.
605. c Mild heat exhaustion and full-blown heat stroke represent extremes of the spectrum of
Rosen 5th, heat illness, and intermediate cases may prove difficult to differentiate. Heat
Chapter 135, exhaustion should not be diagnosed in the presence of major CNS dysfunction
p. 2004 (seizures, coma) or severe hyperthermia (greater than 40.5C).
606. b The distinction between exertional and nonexertional heatstroke is moot, because
Rosen 5th, signs, symptoms, and management are the same. The primary factor that contributes to
Chapter 135, the morbidity and mortality of heat illness is the severity of underlying disease and not
p. 2004 the absolute height of the core temperature. Sweating is totally absent in the majority
of classic heatstroke patients. Hypoglycemia in exertional heatstroke may occur as the
result of increased glucose use and hepatic damage resulting in impaired
gluconeogenesis.
607. b Evaporative cooling is the most widely used cooling method. Immersion in ice water
Rosen 5th, results in a rapid reduction of core temperature to less than 39C within 10 to 40
Chapter 135, minutes. Application of ice packs is commonly used; although technically easier to
p. 2006 perform than cold water immersion, this method sacrifices the conductive cooling
power of water immersion, resulting in slower cooling. Cooling blankets may be a
useful adjunct but will not produce rapid cooling if used exclusively. Peritoneal
dialysis with cold fluids, successful in dogs, remains untested in humans. Cold-irrigant
gastric or rectal lavage will not provide significant heat exchange if used as the
primary cooling modality.
608. d Lightning is neither a direct current nor an alternating current. At best, it is a
Rosen 5th, unidirectional massive current impulse. Therefore lightning is classed as a current
Chapter 136, phenomenon, rather than a voltage phenomenon.
p. 2012
190

609. A 79-year-old woman is brought from her non-air conditioned apartment when neighbors
had not seen her for more than two days. She is comatose with a rectal temperature of
41.6oC (106.8oF). Her heart rate is 46 and her blood pressure is 70 palpable. You begin
rapid cooling techniques and address the other vital sign abnormalities by giving:
a. atropine 1 mg by intravenous bolus.
b. isoproterenol by intravenous drip.
c. norepinephrine by intravenous drip.
d. normal saline solution 2000 cc wide open.
e. nothing initially the vital signs should all normalize as the patient cools down.

610. Which statement concerning the physiology of electrical injuries is true?


a. Direct current exposure to the same voltage tends to be three times more dangerous than
alternating current.
b. The thorax is the most common site of contact with an AC electric source.
c. The higher the resistance of a tissue to the flow of current, the less likely is transformation of
electrical energy to thermal energy.
d. Tissue resistance remains constant throughout length of exposure to current.
e. Tetany occurs when muscle fibers are stimulated at 40 to 110 cycles per second.

611. A kissing burn occurs:


a. at extensor surfaces in electrical exposure when the current causes flexion at a joint, protecting
that area but leaving the exposed extensor surfaces at greater risk for damage.
b. at flexor surfaces in electrical exposure when the current causes flexion at a joint, and the
moisture in the flexor crease leads to an arc burn and extensive deep tissue damage.
c. when a child sucks an electrical outlet.
d. when a child bites an electrical cord.
e. when lightning kisses the skin, leaving a typical fernlike pattern.

612. The paramedics are calling for medical command. They are on their way to a local golf
course where a tournament was interrupted by an unexpected thunderstorm. Their initial
report from the scene says there are 10 victims of a lightning strike, two of whom are
apparently dead, with no signs of breathing or heartbeat. Three victims are comatose but
breathing, and another was reported to be seizing but is now awake and confused. The
other four have flash burns and thermal burns where their clothing caught fire, but these
fires have been extinguished. You tell the medics that they should:
a. be careful when approaching these victims, as they may still be electrically charged.
b. treat the comatose victims first, just as in any mass casualty situation.
c. treat the apparently dead victims first, as there is a good chance they can be resuscitated.
d. bring the seizure patient first, as he will require rapid loading with phenytoin or fosphenytoin in
order to avoid going into status epilepticus.
e. preemptively countershock the comatose patients before they go into fibrillation.

613. Many victims of electrical injury have extensive burns. Their intravenous fluid needs
should be calculated by:
a. using the Parkland formula.
b. using the Brooke formula.
c. using the Edison formula.
d. serial BUN and creatinine.
e. maintaining urine output of 1 ml/kg/hr or greater.
191

609. e Hypotension is common in heat stroke and is usually caused by peripheral vasodilation
th
Rosen 5 , resulting in high-output cardiac failure, in addition to dehydration. Blood pressure
Chapter 135, usually rises with cooling. If this does not occur, or if the patient monitored invasively
p. 2007 has a low central venous or pulmonary capillary wedge pressure, a fluid challenge of
250 to 500 ml of 0.9% saline should be given rapidly. The use of alpha-adrenergic
agents such as norepinephrine is not recommended because they promote
vasoconstriction without improving cardiac output or perfusion, decrease cutaneous
heat exchange, and may enhance ischemic renal and hepatic damage. The role of
isoproterenol, dopamine, or dobutamine is unclear. Atropine and other anticholinergic
drugs that inhibit sweating should be avoided.
610. e Alternating current exposure to the same voltage tends to be three times more
Rosen 5th, dangerous than direct current. Continuous muscle contraction, or tetany, can occur
Chapter 136, when the muscle fibers are stimulated at between 40 and 110 times per second.
p. 2010 Electrical transmission frequency used in the United States is 60 cycles per second.
The hand is the most common site of contact via a tool that is in contact with an AC
electric source. The higher the resistance of a tissue to the flow of current, the greater
the potential for transformation of electrical energy to thermal energy at any given
current. As the tissue breaks down under the energy of the current flow, its resistance
may change markedly, making it difficult to predict the amperage for any given
electrical injury.
611. b A peculiar type of burn associated with electrical injury is the kissing burn, which
Rosen 5th, occurs at the flexor creases. As the current causes flexion of the extremity, the skin of
Chapter 136, the flexor surfaces at the joints touches. Combined with the moist environment that
p. 2014 often occurs at the flexor areas, the electric current may arc across the flexor crease,
causing arc burns on both flexor surfaces and extensive underlying tissue damage.
612. c Traditional rules of mass casualty triage do not apply to lightning victims. In the
Rosen 5th, absence of cardiopulmonary arrest, victims are highly unlikely to die of any other
Chapter 136, cause. Thus triage of lightning victims should concentrate on those who appear to be
p. 2016 in cardiorespiratory arrest. Cardiopulmonary resuscitation should be started on those
who have no pulse or respirations. When multiple victims are involved, the evaluation
of those who are breathing may be delayed because they are likely to survive the
incident.
613. e Fluids should be administered at a rate sufficient to maintain a urine output of at least
Rosen 5th, 0.5 to 1.0 ml/kg/hr in the absence of heme pigment in the urine and 1.0 to 1.5 ml/kg/hr
Chapter 136, in its presence.
p. 2017
192

614. The mother of this 2-year-old boy heard him crying and
found him with this lesion. You find no other sign of
injury, and the child is hemodynamically stable. You
have consulted the oral surgeon and the trauma
surgeon. The child easily takes liquids, and he stops
crying after an appropriate dose of acetaminophen with
codeine. Appropriate disposition would be to:
a. admit to a monitor bed.
b. admit to a burn center.
c. admit to a floor bed for observation.
d. discharge home with careful instructions to parents.
e. 12-hour overnight observation in Emergency Department or Short-Stay Unit.

615. Which statement is true concerning dysbarism?


a. A diver needs to descend only 33 feet in seawater to double the atmospheric pressure.
b. Barotrauma of ascent is also called squeeze.
c. If a diver ascends too rapidly to the surface, carbon dioxide comes out of solution, forming gas
bubbles in the tissues and blood.
d. The greatest risk for barotrauma is in deep water.
e. The solubility of gases is affected by temperature: the lower the temperature, the lower the
solubility.

616. The most common medical complaint of SCUBA divers is:


a. the bends.
b. inner ear squeeze.
c. middle ear squeeze.
d. nitrogen narcosis.
e. swimmers ear.

617. External ear barotrauma:


a. occurs during ascent if water is trapped in the external auditory canal, causing a relative positive
pressure.
b. is more common than middle ear barotrauma.
c. is diagnosed clinically, as there are no physical findings.
d. can be prevented by use of earplugs or a tight fitting wetsuit hood.
e. becomes painful as the tympanic membrane bulges outward.

618. The most severe form of pulmonary over-pressurization syndrome (POPS) is:
a. alveolar hemorrhage.
b. arterial gas embolism.
c. pneumomediastinum.
d. pneumothorax.
e. subcutaneous emphysema.

619. The most important physiologic change occurring during high-altitude acclimatization is:
a. compensatory tachycardia and hypertension.
b. diminished pulmonary compliance and resistance.
c. increased hemoglobin.
d. increased minute ventilation.
e. renal excretion of bicarbonate.
193

614. d Pediatric patients with oral burns may generally be safely discharged if close adult care
th
Rosen 5 , is assured. There is no evidence that an isolated oral burn correlates with cardiac
Chapter 136, injury or myoglobinuria. In general, these patients require surgical and dental
2018 consultation for oral splinting, eventual debridement, and occasionally reconstructive
surgery. After appropriate consultation, if hospitalization is not deemed necessary, the
childs parents should be warned about the possibility of delayed hemorrhage and
receive instructions to apply direct pressure by pinching the bleeding site and to
immediately return to the ED.
615. a Although a mountain climber would need to ascend to 18,000 feet to reduce
Rosen 5th, atmospheric pressure by 50%, a diver needs to descend only 33 feet in seawater to
Chapter 137, double atmospheric pressure. The solubility of gases is affected by temperature: the
p. 2021 lower the temperature, the higher the solubility. Barotrauma results when a diver is
unable to equalize the pressure within air-filled structures to the ambient pressure of
the environment during ascent or descent. Proportionally, changes in volume are
greater as a diver approaches the surface. Thus, the greatest risk for barotrauma is in
shallow water. Barotrauma of descent, or squeeze, as it is known in common diving
parlance, results from the compression of gas in enclosed spaces as ambient pressure
increases with underwater descent. If a diver ascends too rapidly to the surface,
nitrogen comes out of solution, forming gas bubbles in the tissues and blood.
616. c Middle ear barotrauma, (barotitis or ear squeeze,) is the most common complaint of
Rosen 5th, scuba divers, experienced by 30% of novice scuba divers and 10% of experienced
Chapter 137, divers. The middle ear is an air-filled space with solid bony walls except for the
p. 2022 tympanic membrane. The Eustachian tube is the only anatomic passage to the external
environment. As the diver descends, each foot of water exerts an additional 23 mmHg
pressure against the intact tympanic membrane. Ear squeeze occurs when a negative
differential pressure is created within the middle ear because the diver could not
equalize to ambient pressure, leading to pain. If equilibration of middle ear pressure
does not occur, the floppy medial third of the Eustachian tube collapses shut, making
any further attempts at equalization futile. Further pressure increases can cause the
TM to rupture.
617. e When a diver is submerged, the external ear canal usually fills with water. If the
Rosen 5th, external ear canal is occluded (e.g., by cerumen, foreign bodies, exostoses, or
Chapter 137, earplugs), water entry is prevented, and compression of the enclosed air with descent
p. 2022 will have to be compensated for by tissue collapse, outward bulging of the tympanic
membrane, or hemorrhage. This is typically manifested by pain or bloody otorrhea.
618. a The most severe form of POPS is arterial gas embolism. AGE is the second leading
Rosen 5th, cause of mortality of sport divers after drowning, accounting for approximately 30% of
Chapter 137, diving-related deaths. Typically it results when air bubbles are forced across the
p. 2025 alveolar-capillary membrane, escape into the pulmonary venous circulation, and
proceed through the left atrium and ventricle and into the arterial circulation. Clinical
symptoms and signs are in part the result of mechanical obstruction by gas bubbles.
AGE can also result from a right-to-left shunt of venous bubbles, such as in a diver
with a PFO.
619. d The most important physiologic change that occurs during acclimatization to high
Rosen 5th, altitudes is an increase in minute ventilation, causing a decrease in the partial pressure
Chapter 138, of carbon dioxide in arterial gas (PaCO2). The alveolar gas equation states that, as the
p. 2036 PaCO2 decreases, a corresponding increase in PaO2 occurs, thereby increasing arterial
oxygenation. Thus the level of ventilation determines alveolar oxygen for a given
inspired oxygen tension.
194

620. You are practicing in an emergency department near an international airport. A 23-year-
old woman has just flown back from the Caribbean, where she had gone on her
honeymoon. Despite the signs posted at the resort, she took one last dive an hour
before going to the airport. When she got off the airplane, she complained of limb
weakness, paresthesias, numbness, and low back and abdominal pain. The leg
symptoms started on the flight as a distal prickly sensation that advanced proximally,
followed by progressive sensory or motor loss. You suspect Type II decompression
sickness. You also know:
a. ground ambulance to a hyperbaric facility is the preferred method of transportation; air
transportation must be avoided.
b. hyperbaric treatment must start within 12 hours or it will not be beneficial.
c. if shes pregnant, decompression sickness should not affect the developing fetus.
d. more than half of all decompression sickness patients have delayed symptoms occurring 12 or
more hours after surfacing.
e. the goals of recompression therapy are to reduce the washout of nitrogen by decreasing the tissue-
blood nitrogen gradient, and to increase oxygen delivery to ischemic tissue.

621. Concerning the epidemiology of acute mountain sickness:


a. high-altitude illness is uncommon under 10,000 feet, even with rapid ascent.
b. most serious altitude illness occurs above 18,000 feet.
c. above 18,000 feet, complete acclimatization takes 50% longer than acclimatization at 12,000 feet.
d. the partial pressure of oxygen in the atmosphere decreases as altitude rises, but remains a constant
20.93% of the barometric pressure.
e. it has been well-demonstrated that children have a lower incidence of acute mountain sickness
than adults.

622. The most common fatal manifestation of severe high-altitude illness is:
a. high-altitude cerebral edema.
b. high-altitude pulmonary edema.
c. acute mountain sickness.
d. pulmonary embolism.
e. dehydration.

623. In high-altitude pulmonary edema (HAPE):


a. most cases occur within 24 hours of ascent.
b. marked dyspnea on exertion, fatigue with minimal to moderate effort, and dry cough are early
manifestations.
c. the symptoms of AMS preclude the development of HAPE.
d. paradoxically, the lungs remain clear to auscultation until the end stages of the condition.
e. hypothermia is a common finding.

624. The most effective therapy for high-altitude pulmonary edema is:
a. acetazolamide.
b. descent, rest, and oxygen.
c. dexamethasone.
d. furosemide.
e. morphine sulfate.
195

619. a Ground transport to a hyperbaric facility is preferred over air transportation, if at all
th
Rosen 5 , feasible, as an increase in altitude will lower the ambient pressure and allow
Chapter 137, microbubbles to expand. The goals of recompression therapy are to reduce the
p. 2028 mechanical obstruction of air bubbles, to facilitate the washout of nitrogen by
increasing the tissue-blood nitrogen gradient, and to increase oxygen delivery to
ischemic tissue. More than half of all DCS patients will become symptomatic in the
first hour after surfacing, with most of the rest experiencing symptoms within six
hours. DCS may be particularly dangerous to a developing fetus in the womb of a
scuba diving mother because the majority of fetal circulation bypasses the pulmonary
bed through the foramen ovale and the ductus arteriosis. Patients can obtain some
benefit from recompression even if treatment is initiated more than 24 hours after the
dive.
620. d High-altitude illness is common with rapid ascent above 8200 feet. Most serious
Rosen 5th, altitude illness occurs between 10,000 and 18,000 feet. Above 18,000 feet, complete
Chapter 138, acclimatization generally is not possible and long visits above this level result in
p. 2035 progressive deterioration. The partial pressure of oxygen (PO2) in the atmosphere also
decreases as altitude rises, but it remains a constant 20.93% of the barometric pressure.
Limited data suggest that children probably have the same incidence of acute mountain
sickness as adults.
622. b High-altitude pulmonary edema is the most common fatal manifestation of severe
Rosen 5th, high-altitude illness.
Chapter 138,
p. 2042
623. b The initial symptoms of HAPE usually begin insidiously 2 to 4 days after arrival at
th
Rosen 5 , high altitude. Most cases occur during the second night, but HAPE may develop
Chapter 138, rapidly, with early symptoms apparent after just a few hours at high altitude. Marked
p. 2042 dyspnea on exertion, fatigue with minimal to moderate effort, and dry cough are early
manifestations of the disease. These symptoms may be subtle but noticeable when
comparing the victim with others in the group. The symptoms of AMS usually occur
concurrently with the development of HAPE. The physical examination reveals a few
rales in mild HAPE usually found in the region of the right middle lobe, progressing to
unilateral or bilateral rales, then diffuse bilateral rales and audible rhonchi and gurgles
that can be heard without the stethoscope. Elevated temperatures are common.
624. b If HAPE is recognized early and treated properly, death usually can be avoided.
Rosen 5th, Descent to a lower altitude, bed rest, and supplemental oxygen are the most effective
Chapter 138, methods of therapy.
p. 2043
196

625. The most sensitive physical finding for the recognition of high-altitude cerebral edema is:
a. cerebellar ataxia.
b. inability to do reverse serial sevens.
c. loss of deep tendon reflexes.
d. nystagmus.
e. tinnitus.

626. When considering victims of drowning or near-drowning:


a. freshwater immersion leads to hypotonicity in a majority of victims.
b. salt water immersion leads to hypertonicity in a majority of victims.
c. most near-drowning victims require therapy for electrolyte abnormalities.
d. cerebral hypoxia is the final common pathway in all drowning victims.
e. All of the above statements are true.

627. It is now known that the most important factor(s) to correct in treating the near-drowning
victim is/are:
a. acidosis and volume overload.
b. blood volume and tonicity.
c. cardiovascular function.
d. pulmonary injury and hypoxia.
e. serum electrolyte concentrations.

628. A 25-year-old man was pulled comatose from the bottom of a private swimming pool.
Immersion time was unknown. His pupils were dilated, and his breathing was agonal.
CPR was initiated at the scene, and the paramedics intubated him at once. On arrival in
the Emergency Department his heart rate is 42 and his blood pressure palpable at 66
mmHg. You know that:
a. he will be hyperkalemic because of hypotonic cell lysis.
b. he will have dilutional hypokalemia.
c. he will have dilutional hyponatremia.
d. he will have dilutional anemia.
e. his odds of surviving neurologically intact are higher than his odds of dying.

629. Optimal treatment for patients with hypoxemic brain injury after near-drowning includes:
a. barbiturate coma.
b. continuous positive airway pressure
c. corticosteroids.
d. hypothermia.
e. neuromuscular blockade.

630. A three-year-old boy was pulled from an icy river. His estimated time underwater was 15
minutes. He is without spontaneous vital signs, but you know that there is a chance for
recovery. Aggressive measures are instituted and should be maintained until:
a. barbiturate coma is induced
b. core temperature reaches 30 to 35C.
c. emergent dialysis is arranged.
d. high-dose steroids are give intravenously.
e. naloxone has been administered.
197

625. a The finding of cerebellar ataxia is the most sensitive test for early recognition of
th
Rosen 5 , HACE. This early appearance of ataxia reflects the particular sensitivity of the
Chapter 138, cerebellum to hypoxia. Ataxia alone is an indication for immediate descent.
p. 2045
626. d Pathophysiologic differences between freshwater and seawater aspirations have
th
Rosen 5 , traditionally been emphasized. In freshwater aspiration, hypotonic fluid passes
Chapter 139, through the lungs into the pulmonary and systemic vasculature. Blood volume
p. 2051 increases within 3 to 4 minutes after aspiration, and the concentrations of most serum
electrolytes decrease. A rise in the serum potassium level may occur from the lysis of
red blood cells as a result of the effects of hypotonic freshwater or hypoxemia.
Conversely, aspirated saltwater, which is 3 to 4 times more concentrated than plasma,
leads to the development of an osmotic gradient across the alveolar membrane;
protein-rich fluid from the capillaries moves into the alveoli and interstitium, leading
to pulmonary edema. Blood volume decreases, and serum electrolyte concentrations
increase. Although these differences are demonstrated in experimental animal studies,
most victims of drowning or near-drowning do not aspirate enough fluid to cause life-
threatening changes in blood volume or serum electrolyte concentrations.
627. d The early literature on submersion incidents emphasized changes in serum electrolyte
Rosen 5th, concentrations, blood volume, and cardiovascular function. It is now clear that
Chapter 139, pulmonary injury and hypoxia are the factors of primary importance in the near-
p. 2052 drowning victim. The CNS injury seen in near-drowning victims is also a hypoxic-
ischemic event. Acidosis is often a prominent feature, developing from
hypoventilation, hypoxia, and poor perfusion.
628. e Of near-drowning victims who are comatose, 44% survive neurologically intact, 17%
Rosen 5th, to 24% survive with severe neurologic damage, and 27% to 39% die. Decreased serum
Chapter 139, bicarbonate is common and indicates tissue hypoxia (as well as elevated lactic acid
p. 2052 levels). The other electrolytes are usually normal on presentation. Similarly, the
complete blood count levels are often normal, with the exception of leukocytosis.
629. e The optimal treatment for patients with anoxic brain injury remains unclear. The
Rosen 5th, classic protocol of neuromuscular blockade, barbiturate coma, dehydration,
Chapter 139, hyperventilation, hypothermia, and corticosteroids (HYPER therapy), has been
p. 2054 abandoned because it did not demonstrate improved outcomes. Supportive care
without barbiturates, steroids, induced hypothermia, routine intracranial pressure
monitoring, or muscle paralysis is the standard in many intensive care centers.
630. b The safest guideline is to continue resuscitation efforts until the core temperature
Rosen 5th, reaches 30 to 35C. Cerebral death cannot be accurately diagnosed in the
Chapter 139, hypothermic patient with core temperatures below this level.
p. 2054
198

631. Africanized honeybees (killer bees) cause fatalities because of:


a. aggressiveness of the swarms, leading to multiple stings.
b. increased human sensitivity to the venom.
c. increased venom toxicity.
d. the relative insensitivity of its victims to epinephrine (adrenalin).
e. their propensity to sting near the face and throat, leading to rapid airway compromise.

632. The insect from the hymenoptera family with the most dangerous venom is thought to be
the:
a. fire ant (Solenopsis invicta).
b. honey bee (Apis mellifera).
c. hornet (Vespula maculata).
d. wasp (Chlorion ichneumerea).
e. yellow jacket (Vespula maculiforma).

633. Dapsone may be helpful in the management of the local effects of envenomation by the:
a. black widow spider.
b. brown recluse spider.
c. fire ant.
d. golden orb weaver spider.
e. pickled herring.

634. Care of a presumed black widow spider bite in a stable, asymptomatic ED patient
includes:
a. broad spectrum antibiotics including anaerobic coverage
b. immediate admission to an intensive care unit.
c. incision and drainage at the site.
d. intermittent tourniquet application proximal to the site.
e. tetanus immunization.

635. A 4-year-old child started screaming after tried to pet his mothers boyfriends pet
tarantula. He has probably been exposed to this arthropods:
a. alkali saliva in the eye, as the beast is an excellent spitter.
b. cardiotoxin through a sting.
c. irritating abdominal hairs, which the creature can throw several feet like a javelin.
d. neurotoxin through a bite.
e. pinchers.

636. The part of the United States with the highest reported incidence of venomous snake bites
is the:
a. Midwest.
b. Northeast.
c. Northwest.
d. South.
e. West Coast.

637. In using antivenin to treat the victim of snakebite, it is important to remember that:
a. dosing is weight-based.
b. if intravenous access cannot be obtained, the antivenin can be given intramuscularly.
c. pregnancy is a contraindication.
d. the first vial should be injected around the site of envenomation.
199

e. the need for doses beyond the first several vials is based on clinical response.
200

631. a Envenomations from these aggressive arthropods are most dangerous to very young or
th
Rosen 5 , elderly patients and those with concomitant medical conditions. Killer bees have
Chapter 55, colonized northern Mexico and now have moved into the southern United States,
p. 793 including California, Arizona, and Texas, where the mean high temperatures are at
least 60 F. This type of bee is not more toxic, only more aggressive.
632. b The signs and symptoms of bee and wasp stings vary, depending on the degree, type,
Rosen 5th, and location of envenomation as well as the characteristics of the victim. Bee and
Chapter 55, wasp venom can cause serious injuries other than allergic types of reactions, depending
p. 793 on the number of stings, the species of insect, the size and previous health of the
victim, and the anatomic area stung. For example, a sting in the tongue or throat may
quickly compromise the airway. Honeybee venom causes a much greater release of
histamine per gram than other hymenopteran venom and thus is more dangerous.
Certain species of honeybee release a pheromone, isoamylacetate, when the ovipositor
is pulled from the abdomen after stinging a victim. This pheromone attracts other bees
to the victim and thus incites multiple stings.
633. b Dapsone, 50 to 200 mg/day, has been shown to be helpful in preventing the local
Rosen 5th, effects of the venom. However, dapsone may cause methemoglobinemia and
Chapter 55, hemolysis in patients with glucose-6-phosphate dehydrogenase (G6PD) deficiency.
p. 796 Hyperbaric oxygen has been shown to decrease lesion size in animals. Analgesics and
antibiotics should be used as indicated during the course of the disease, although
infection is not common.
634. c The ED care consists of obtaining a history of the circumstances surrounding the bite, a
Rosen 5th, description of the appearance of the spider, any significant past medical history,
Chapter 55, medications, and allergies to insect bites, horses, or horse serum. The wound site
p.794 should be inspected for fang marks and cleansed with soap and water. Tetanus
immunization should be instituted. The patient should be observed for about 4 hours.
If the patient does not develop any symptoms and the spider was not positively
identified as a black widow, the patient may be discharged with instructions to return
to the ED if any symptoms develop.
635. c Tarantulas are popular pets in the United States, and most native species are relatively
Rosen 5th, nontoxic. Tarantulas are unusual in that the abdominal hairs can be thrown by the
Chapter 55, spider and embedded in human skin and the eye. These hairs can cause allergic
p. 796 reactions and severe conjunctivitis and must be removed. A recent import from
Thailand, the cobalt blue tarantula, Haplopelma lividum, is a very aggressive spider
with a toxic venom.
636. d The incidence of reported venomous snakebites is greatest in the southern United
Rosen 5th, States, which has the largest number of venomous snakes. States having the highest
Chapter 55, death rates were North Carolina, Arkansas, Texas, and Georgia.
p. 787
637. e The smaller the body of the patient, the larger is the relative initial dose required; a
th
Rosen 5 , bitten child usually receives more venom in proportion to body weight and thus
Chapter 55, requires more antivenin to neutralize it. All antivenin should be given intravenously.
p. 791 Pregnancy is not a contraindication to antivenin therapy. Administration of antivenin
at or around the site of the bite is not recommended. The need for subsequent doses is
based on the clinical response.
201

638. The greatest number of envenomations by marine animals are caused by the:
a. cephalopods, like the blue-ringed octopus.
b. coelenterates, like the Portuguese man-of-war.
c. gastropods, like coneshells and nudibranches.
d. echinoderms, like starfish and sea urchins.
e. scorpaenidae, like the zebrafish and stonefish.

639. 5% acetic acid (vinegar) is used to neutralize nematocysts from the marine animal:
a. box jellyfish
b. saltwater catfish.
c. sea urchin.
d. stingray.
e. zebrafish.

640. The symptom most consistently associated with pit viper bites is:
a. anaphylaxis.
b. compartment syndrome.
c. immediate burning pain
d. petechiae.
e. purpura.

641. A 35-year-old woman was intoxicated at a picnic and passed out in a mound of fire ants.
She has evidence of hundreds of bites, including in her ear canals. Treatment consists of:
a. a bicarbonate slurry to neutralize the ant acid.
b. high-dose steroids.
c. intubation and barbiturate coma for 24-hours until the pain has subsided.
d. symptomatic care.
e. warm soaks to neutralize the heat-labile toxin.

642. Narcotic analgesics should not be used when treating the sting from a:
a. Africanized killer bee.
b. bark scorpion.
c. Portuguese man-of-war.
d. puss caterpillar.
e. stingray.

643. When seeing a patient who has ingested poisonous mushrooms, you know that:
a. the earlier the symptoms start, the more likely an ingestion is lethal.
b. coprine-containing mushrooms like the inky cap can cause an Antabuse-type reaction if
consumed with alcohol.
c. the majority of mushroom fatalities in the USA occur from ingesting gyromitrin-containing
mushrooms.
d. amatoxins are used as an abortifacient.
e. psilocybin is structurally related to phencyclidine (PCP).
202

638. b Because of their large numbers, coelenterates account for the greatest number of
th
Rosen 5 , envenomations by marine animals. This includes the Portuguese man-o-war, true
Chapter 55, jellyfish, fire corals, stinging hydroids, sea wasps, sea nettle, and anemones. Most of
p. 797 these organisms are sessile, but some are free floating.
639. a Nematocyst injuries are treated by first removing the nematocysts without allowing
th
Rosen 5 , them to discharge. Tentacles should be removed with a gloved hand or forceps. The
Chapter 55, remaining nematocysts should be fixed by pouring vinegar over the wound area. Fresh
p. 799 water should not be used because it may stimulate continued nematocyst discharge.
640. c Immediate burning pain is characteristic of pit viper bites. This differentiates it from
th
Rosen 5 , other snakebites. Petechiae may occur, as well as anaphylaxis if there is an immune
Chapter 55, response. These do not occur as consistently. Compartment syndrome has not been
p. 789 reported despite severe edema.
641. d Treatment of fire ant stings consists of local wound care. If there is evidence of
th
Rosen 5 , systemic reaction, give the usual treatment for anaphylaxis. Desensitization should be
Chapter 55, directed to any person exhibiting a potentially life-threatening reaction to these
p. 793-794 arthropods.
642. b For Centruroides exilicauda (box scorpion) envenomations that occur in Arizona,
th
Rosen 5 , antivenin is available from the Antivenom Production Laboratory of Arizona State
Chapter 55, University. Narcotic analgesics and barbiturates have been reported to increase the
p. 797 toxic effects of the venom and should be avoided.
643. b The inky-cap or shaggy mane contains coprine and can cause a typical disulfiram
Rosen 5th, reaction if washed down with alcohol. In general, if toxicity begins within 2 hours of
Chapter 158, ingestion of a mushroom, the clinical course will most likely be benign. If symptoms
p. 2203 begin 6 hours or later after ingestion, however, in general the clinical course will be
more serious and potentially fatal. Nearly all fatalities in the United States occur from
the ingestion of the Amanita species (Amanita phalloides, Amanita virosa, and
Amanita verna). Amatoxin does not cross the placental barrier. Psilocybin is
structurally related to lysergic acid diethylamide (LSD).
203

644. Intractable seizures can occur after ingestion of:


a. aloe (Aloe barbadensis).
b. amanita phalloides.
c. Lily of the Valley (Convallaria majalis).
d. pyracantha (firethorn).
e. water hemlock (Cicuta maculata).

645. Physostigmine may be needed to treat the toxic effects of:


a. dieffenbachia (dumbcane).
b. jimsonweed (Datura stramonium).
c. poinsettia (Euphorbia pulcherrima).
d. pokeweed (Phytolacca americana).
e. fava beans and Chianti.

646. Patients who ingest large amounts of dieffenbachia experience symptoms of calcium
oxalate crystal ingestion, including:
a. burning and irritation of the mouth.
b. generalized urticaria.
c. hypertension and tachycardia.
d. hypotension and bradycardia.
e. vomiting and diarrhea.

647. The radiation that penetrates all layers of the body and is the primary cause of acute
radiation syndrome is:
a. alpha rays.
b. beta rays.
c. gamma rays.
d. neutrons.
e. photons.

648. You are evaluating a patient 48 hours after a major radiation injury. He had severe
nausea and vomiting, but is feeling better now. His absolute lymphocyte count is 1200,
about 50% of normal. He should be triaged to the:
a. "expectant" death group.
b. "impending" death group.
c. survival improbable group.
d. survival possible group.
e. survival probable group.
204

644. e The water hemlock contains the neurotoxin cicutoxin. Seizure activity in severe
th
Rosen 5 , exposure occurs in the first hour after ingestion and has resulted in high fatality rates.
Chapter 158,
p. 2199
645. b Jimsonweed's effects are most frequently seen when leaves are dried and smoked for
th
Rosen 5 , their hallucinogenic effects. Jimsonweed seeds contain potent anticholinergic agents,
Chapter 158, and ingestion may result in the anticholinergic syndrome typified by fever, tachycardia,
p. 2200 thirst, urinary retention, decreased secretions, and mydriasis. For severe cases
physostigmine may be useful.
646. a Dieffenbachia spp. has over 30 different species including the dumbcane, mother-in-
Rosen 5th, laws tongue, dumb plant, and tuft root. Some of these names refer to the inability to
Chapter 158, talk that can occur after biting into parts of this plant. Typically the mucous
p. 2200 membranes of the mouth are immediately affected with severe pain, swelling, and the
sensation of biting into glass. These effects are due to calcium oxalate crystals,
packaged into bundles known as raphides, which are found in cellular structures known
as idioblasts. These idioblasts also contain proteolytic enzymes that are ejected out of
the idioblasts with the oxalate crystals when plant parts are chewed. Treatment is
aimed at pain relief and local supportive measures, typified by eating ice cream.
647. c Gamma rays are the form of radiation that penetrate tissues deeply and interact with
Rosen 5th, every layer of tissue. They are emitted from radioisotopes after beta decay, and are the
Chapter 140, primary cause of the acute radiation syndrome.
p. 2057
648. d The survival possible group includes those in whom nausea and vomiting are relatively
th
Rosen 5 , brief, lasting 24 to 48 hours, followed by an asymptomatic period. After the initial
Chapter 140, symptoms, these patients exhibit thrombocytopenia, granulocytopenia, and
p. 2061 lymphopenia. The severity of these changes depends on individual susceptibility and
the level of the initial radiation dose. These patients should be admitted for fluid and
electrolyte therapy if vomiting is severe. Antiemetics may be given as needed for
nausea and vomiting, but they may be ineffective. Also, protective isolation
precautions are indicated, particularly if the absolute lymphocyte count at 48 hours is
lower than 1200 or 50% of the baseline value.
205

7.0 Head, Ear, Eye, Nose and Throat Disorders


649. You are examining a 5 year-old girl who knocked out her two upper central incisors in a fall
at a local amusement park. She is visiting her aunt, who does not know whether these are
her primary teeth. You know that:
a. normal primary dentition consists of 12 mandibular and 12 maxillary teeth.
b. the upper central incisors are the first teeth to erupt at approximately 10 months of age
c. all primary teeth should be present by 3 years of age.
d. permanent dentition does not begin to erupt until 7 to 8 years of age.
e. incisors are the first permanent teeth to erupt.

650. Trismus is caused by irritation of the:


a. buccinator muscle.
b. external pterygoid muscle.
c. masseter muscle.
d. platysma muscle.
e. temporalis muscle.

651. A 64-year-old woman complains of fever, neck pain and swelling, and difficulty swallowing.
She has a toxic appearance, and her neck is thicken, red, and tender. The bacteria most
likely responsible for this infection is:
a. Bacteroides fragilis.
b. hemolytic streptococcus.
c. Klebsiella.
d. Pseudomonas aeruginosa.
e. Staphylococcus aureus.

652. A 29-year-old woman complains of a toothache. She is afebrile and there is no facial
swelling, but she has very poor dentition and evidence of gingivitis. You drain her
periapical abscesses and then prescribe:
a. cephalexin.
b. ciprofloxacin.
c. an antifungal.
d. tetracycline.
e. trimethoprim-sulfamethoxazole.

653. A 22-year-old college student complains of severe mouth pain three days after removal of
two impacted mandibular molars. She looks very uncomfortable, but has normal vital
signs. Her exam shows probable dry sockets (post-extraction alveolar osteitis). You can
offer her best relief of her symptoms by:
a. having her hold an ice cube on the socket.
b. packing the socket with gauze impregnated with penicillin.
c. packing the socket with gauze soaked in 4% lidocaine.
d. packing the socket with gauze soaked in eugenol (oil of cloves).
e. suturing the gum over the exposed socket.
206

649. d The normal primary or deciduous dentition consists of 10 mandibular and 10 maxillary
Rosen 5th teeth. The primary dentition is important for mastication, cosmetics, growth, and
Chapter 65, development and functions as a physiologic space maintainer. Starting at the midline
p. 892 and moving posteriorly in any quadrant, the normal dentition consists of a central
incisor, lateral incisor, canine, and two primary molars. The lower central incisor is
the first tooth to erupt, at approximately 6 months of age; all primary teeth should be
present by 3 years of age. If not, further investigation for developmental or endocrine
abnormalities is warranted. The permanent dentition begins to erupt at approximately
5 to 6 years of age with the appearance of the first molar.
650. c Any irritation of the internal pterygoid or masseter muscles results in trismus. Trismus
Rosen 5th results in the ability to open the mouth only a few millimeters, limiting visualization of
Chapter 65, the pharynx and making diagnosis of lateral or retropharyngeal space involvement
p. 898 difficult.
651. b Ludwigs angina is a bilateral board-like swelling involving the submandibular,
Rosen 5th submental, and sublingual spaces with elevation of the tongue. The most serious
Chapter 65, immediate sequela is airway obstruction. A characteristic brawny induration is
p. 898 present; there is no fluctuance for incision and drainage. Hemolytic streptococcus is
most commonly responsible for the infection, although a mixed staphylococcal-
streptococcal flora is not uncommon, and both may lead to an overgrowth of anaerobic
gas-producing organisms, including Bacteroides fragilis.
652. e Pain from a periodontal abscess or swelling of the gingiva occurs when food or pus
Rosen 5th becomes trapped in a pocket. Treatment consists of a small conservative stab
Chapter 65, incision at the most fluctuant point to establish drainage, saline rinses, and antibiotic
p. 898 coverage. Using tetracycline for patients more than 8 years of age is preferable
because it provides better coverage for the gram-negative and anaerobic organisms
found in the gingival pocket.
653. d Acute alveolar osteitis or dry socket may occur 3 to 4 days after an extraction. The
Rosen 5th patient has a pain-free interval followed by sudden onset of excruciating pain
Chapter 65, associated with a foul odor. The pathophysiology involves premature loss of the
p. 899 healing blood clot from the socket with a localized infection of the bone. Treatment of
consists of an anesthetic nerve block, gentle irrigation of the socket, and packing the
socket with iodoform gauze saturated with a medicated dental paste such as Seda-
Dent, or barely dampened with eugenol (oil of cloves), giving almost immediate
relief.
207

654. A 27-year-old homeless man complains of fever, malaise, bad breath, and a feeling of
wooden and loose teeth. You examine him and find submandibular and anterior cervical
chain nodes. You also expect him to:
a. be culture-negative for anaerobic organisms.
b. have petechiae elsewhere on his body.
c. have the worst lesions in the location shown and near the posterior molars.
d. not be toxic in appearance.
e. respond poorly to antibiotics, as this is a surgical condition.

655. A 12-year-old boy was accidentally struck in the mouth with a baseball bat and has a
chipped tooth (see picture). The missing fragment is soaking in milk. You gently wipe
the tooth with some dry gauze and see a drop of blood. You should now:
a. arrange urgent follow-up with dentist or endodontist.
b. pack the pocket with gauze impregnated with penicillin.
c. pack the pocket with gauze soaked in 4% lidocaine.
d. pack the pocket with gauze soaked in eugenol (oil of cloves).
e. glue the fragment back into place using cyanoacrylate glue.

656. A 32-year-old camper avulsed his upper medial incisors three hours ago when his hatchet
ricocheted into his mouth while he was setting up his tent. He put his teeth into his shirt
pocket, hiked out of the woods, and drove to see you. You need to:
a. gently rinse the teeth and reimplant them, as there is an excellent chance they will
take.
b. gently rinse the teeth and reimplant them, but refer the patient to an endodontist and
warn him that the teeth will probably not survive.
c. scrub the teeth thoroughly to remove tissue and contaminants, then reimplant them.
d. soak the teeth in a povidone-iodine solution (Betadine) and call the dentist.
e. soak the teeth in normal saline and call the dentist.

657. Herpangina and hand-foot-and-mouth disease both:


a. are winter diseases.
b. improve rapidly with acyclovir.
c. are caused by Coxsackie virus group A.
d. can lead to overwhelming sepsis in the asplenic patient.
e. cause generalized lymphadenopathy.

658. A 26-year-old woman was yawning and now cannot close her mouth. You should:
a. consult oral surgery for surgical repair.
b. inject both temperomandibular joints with local anesthetic / steroid.
c. intubate to control the airway.
d. order Panorex or Panellipse radiograph.
e. reduce the dislocation, using conscious sedation if necessary.

659. A 47-year-old workman accidentally splashed an unknown chemical in his right eye. He
complains of a burning sensation and blurred vision. On arrival in the Emergency
Department your protocol calls for the triage nurse to:
a. check the pH; if acid, irrigate with a bicarbonate solution; if alkali, irrigate with boric acid
or another weak acid.
b. check visual acuity.
c. determine what chemical caused the injury and begin irrigation.
d. emergently consult ophthalmology.
208

e. immediately irrigate his eye with a liter of neutral solution.


209

654. c Acute necrotizing ulcerative gingivitis (ANUG) is a periodontal lesion in which


Rosen 5th bacteria actually invade nonnecrotic tissue. ANUG lesions are commonly
Chapter 65, accompanied by systemic manifestations of fever, malaise, and regional
p. 898 lymphadenopathy. The lesions can involve any part of the gingiva but are more
common in the anterior incisor and posterior molar regions. Anaerobic bacteria such
as Treponema, Selenomonas, Fusobacterium, and Prevotella are uniformly identified.
Antibiotics provide dramatic relief within 24 hours, as do dilute (3%) hydrogen
peroxide rinses.
655. a A drop of blood indicates a pulpal exposure, a true dental emergency (Ellis Class II or
Rosen 5th III). A general dentist, pedodontist, or endodontist should be notified immediately if
Chapter 65, possible, or follow-up instructed for the next working day. If no dentist is available, a
p. 902 piece of moist cotton can be placed over the exposed pulp and covered with a piece of
dry foil or sealed with a temporary root canal sealant (e.g., Cavet).
656. b It has been known since the mid-1960s that an avulsed tooth can be successfully
Rosen 5th replanted if it is returned to its socket within 30 minutes of the avulsion. A 1% chance
Chapter 65, of successful replantation is lost for every minute that the tooth is outside of its socket;
p. 903 the worst situation is to allow the tooth to be transported in a dry medium. Storage in
plain water is not much better. Teeth that have been avulsed for longer than 30
minutes will invariably require endodontic therapy.
657. c Herpangina is caused by coxsackievirus group A, types 16, 8, 10, and 22 most
Rosen 5th commonly. Most commonly occurring in the summer and autumn, herpangina
Chapter 65, presents with a sudden onset of high fever, sore throat, headache, and malaise followed
p. 1830 by eruption of oral vesicles 1 to 2 mm in size within 24 to 48 h. The disease lasts 7 to
10 days and is treated symptomatically. Coxsackievirus type A16, and occasionally
types A4, A5, A9, and A10, are associated with hand, foot, and mouth disease. Fever
is usually of short duration, and the disease lasts 5 to 8 days. Treatment is palliative.
658. e Reduction may be attempted in closed anterior dislocations without fracture. Most
Rosen 5th, attempts are made easier with analgesia.
Chapter 65, A short-acting intravenous muscle relaxant (e.g., midazolam) helps to decrease muscle
p. 906 spasm. Appropriate airway and hemodynamic monitoring is required. A systemic
analgesic (e.g., narcotic) may also be considered. Conscious sedation has also been
used successfully.
659. e Treatment should begin at the scene with immediate irrigation using copious amounts
Rosen 5th of water. Irrigation should continue for at least 30 minutes before any attempt to
Chapter 66, transport the patient to the hospital. Any particles should be removed from the
p. 910 fornices using a cotton swab. Upon hospital arrival, irrigation should continue.
Topical anesthetics and manual lid retraction may be needed for proper irrigation.
Irrigation is needed until the pH of the tear film is neutral as tested by Nitrazine paper
dipped into the inferior conjunctival fornix.
210

660. An apprentice welder complains of foreign body sensation and severe photophobia in both
eyes, eight hours after finishing a full day of welding. He gets good pain relief from
instillation of tetracaine drops, and fluorescein-enhanced slit lamp exam shows diffuse
punctate keratitis. Appropriate treatment is:
a. systemic antibiotics, topical anesthetic drops, systemic steroids.
b. topical and oral antimicrobials for infection prophylaxis, topical anesthetic drops for pain
relief.
c. topical antimicrobials for infection prophylaxis, topical anesthetic drops for pain relief.
d. topical antimicrobials for infection prophylaxis, topical short-acting cycloplegics for pain
relief, oral narcotics for breakthrough pain.
e. topical antimicrobials for infection prophylaxis, topical steroid drops, oral narcotics for
pain relief.

661. The major short-term complication from hyphema is:


a. cataract.
b. hypopyon.
c. glaucoma.
d. rebleeding.
e. vitreous hemorrhage and floaters.

662. A 23-year-old gentleman had a disagreement with an under-employed boxer at a local


entertainment establishment. During their discussion, he became punched in the eye.
After visiting his lawyer, he came to the Emergency Department complaining of pain. You
find him to be photophobic, with a midposition, poorly reactive pupil. You choose to treat
him with:
a. acetazolamide orally.
b. beta-blocker eye drops.
c. homatropine methylbromide 5%.
d. pilocarpine 4% drops.
e. topical antibiotic and an eye patch, with rapid follow-up.

663. A college basketball player complains of double vision just in my right eye after he was
elbowed in the face during a scrimmage. You suspect:
a. cataract.
b. hyphema.
c. malingering.
d. stroke
e. subluxed lens.

664. Scleral rupture:


a. can occur with normal visual acuity.
b. causes a bulging anterior chamber.
c. is best diagnosed by gentle palpation of the globe.
d. is confirmed by a negative Seidel test.
e. occurs most frequently at the limbus and insertion points of extraocular muscles.

665. You are examining a teen-age girl who was struck in the left eye by a batted softball. She
tells you she saw showers of sparks, which have now resolved. You suspect a possible
retinal tear, which:
a. always causes diminished visual acuity.
b. can be diagnosed with direct ophthalmoscopy and a slit lamp.
211

c. causes anterior chamber cell and flare and a mid-position poorly reactive pupil.
d. requires urgent ophthalmologist follow-up for best results.
e. rarely involves the macula.
212

660. d Ultraviolet light from sun lamps, tanning booths, high-altitude environments, or
Rosen 5th welders arc results in direct corneal epithelial damage. After a latent period of 6 to 10
Chapter 66, hours, patients develop a foreign body sensation, tearing, intense pain, photophobia,
p. 911 and blepharospasm. Examination reveals decreased visual acuity, injected conjunctiva,
and diffuse punctate corneal lesions, often with a discrete lower border defining the
cornea protected by the inferior lid. Treatment consists of a short-acting cycloplegic
and a topical broad-spectrum antibiotic ointment. Eye patching may be used for
patient comfort on the more affected eye. Oral narcotics are commonly needed.
Patients should never be prescribed topical anesthetics because frequent use retards
healing and can lead to corneal ulcer formation. Patients should have ophthalmologic
follow-up in 24 hours.
661. d The major complication of hyphema is rebleeding, which occurs after 2 to 5 days when
Rosen 5th the initial clot retracts and loosens. Rebleeding is more common in those with visual
Chapter 66, acuities of 20/200, initial hyphema covering more than one third of the anterior
p. 913 chamber, medical attention delayed more than 1 day after injury, and elevated
intraocular pressure at the initial examination. Other complications include corneal
bloodstaining, acute or chronic glaucoma, and anterior or posterior synechia formation.
662. c Blunt injury of the globe may contuse and inflame the iris and ciliary body, resulting in
Rosen 5th ciliary spasm. Patients complain of photophobia and deep aching eye pain.
Chapter 66, Examination reveals perilimbal conjunctival injection (ciliary flush), cells and flare in
p. 913 the anterior chamber, and a small, poorly dilating pupil. These symptoms indicate
white blood cells and protein as a result of the inflammation. Treatment consists of
paralyzing the iris and ciliary body with a long-acting cycloplegic agent, such as
homatropine methylbromide 5%, given four times daily for 7 to 10 days. Prednisolone
acetate 1% may be given to help relieve the inflammation if there is no improvement
after 5 to 7 days, but should be avoided in patients with a corneal epithelial defect.
Resolution occurs within 1 week.
663. e Patients complain of monocular diplopia or visual distortion with lens subluxation and
Rosen 5th marked visual blurring with lens dislocation.
Chapter 66,
p. 913
664. e Blunt trauma causes scleral rupture by suddenly elevating intraocular pressure.
Rosen 5th Ruptures are most common at the insertions of the intraocular muscles or at the limbus,
Chapter 66, where the sclera is the thinnest. Clues to a ruptured globe or intraocular foreign body
p. 913 include shallow anterior chamber, hyphema, irregular pupil, significant reduction in
preinjury visual acuity, and poor view of the optic nerve and posterior pole on direct
ophthalmoscopy. If aqueous humor is leaking from the corneal wound, it will appear
as streaming fluorescent dye surrounded by an orange pool of solution on slit-lamp
examination (Seidel test).
665. d Tears and detachments from blunt trauma are common. Symptoms include floaters
Rosen 5th from bleeding, flashing lights from stimulation of retinal neurons, and visual field
Chapter 66, cuts or decreased visual acuity. Retinal tears or detachments do not cause pain.
p. 914 Examination may reveal the hazy gray membrane of the retina billowing forward, but
many tears are peripherally located and not seen with direct ophthalmoscopy. Visual
acuity may be normal unless the macula is involved. Indirect ophthalmoscopy is
warranted if historic clues to the presence of retinal tears are present. Ophthalmologic
consultation is warranted in all cases of suspected or proven retinal detachment.
213

666. A good antibiotic eyedrop for a patient with bacterial conjunctivitis is:
a. gentamicin.
b. neomycin.
c. sulfacetamide.
d. trimethoprim + polymyxin b (Polytrim).
e. erythromycin.

667. There is frequently confusion about the difference between periorbital and orbital cellulitis.
In a patient with orbital cellulitis,
a. a frequent cause is spread from sinus infection, especially the ethmoid sinuses.
b. high-dose steroids may be necessary.
c. the globe itself is not at risk.
d. the most likely cause is a gram-negative rod.
e. The infection usually starts as an allergic reaction to an insect bite.

668. Corneal abrasions associated with contact lens wear should be treated with:
a. fluoroquinolone ointment + patch.
b. patch + fluoroquinolone drops+ steroid drops.
c. patch + topical anesthetics + systemic antibiotics.
d. tobramycin ointment.
e. topical anesthetics + tobramycin drops.

669. Central retinal artery occlusion causes vision loss that is:
a. gradual and painful.
b. gradual and painless.
c. intermittent.
d. sudden and painful.
e. sudden and painless.

670. The most common ocular motor palsy involves the:


a. 2nd cranial nerve.
b. 3rd cranial nerve.
c. 4th cranial nerve.
d. 6th cranial nerve.
e. 8th cranial nerve.

671. The ophthalmologic condition considered to be a surgical emergency is:


a. hyphema.
b. hypopyon.
c. retrobulbar hemorrhage.
d. rust ring from metallic foreign body.
e. vitreous hemorrhage.

672. An indication that an eyelid laceration has penetrated the orbital septum is:
a. Bells phenomenon.
b. fat in the wound.
c. negative Seidels test.
d. ptosis.
e. subconjunctival hemorrhage.
214

666. d Treatment of acute bacterial conjunctivitis includes warm compresses and topical
Rosen 5th ophthalmic antibiotics. In uncomplicated acute bacterial conjunctivitis, topical
Chapter 66, trimethoprim and polymyxin is a good initial selection. Neomycin ophthalmic
p. 917 solutions should be avoided because of the high incidence of hypersensitivity
reactions. Medications should be continued for 7 days. Corticosteroids and eye
patching should be avoided.
667. a Orbital cellulitis is an orbital infection; therefore it is deep to the orbital septum. This
Rosen 5th is a serious ocular infection that has the potential to be life threatening.
Chapter 66, Staphylococcus aureus is the most common pathogen; however, Haemophilus
p. 909 influenzae should be considered in young children and mucormycosis in diabetics and
immunocompromised patients. Polymicrobial infection is common. Orbital extension
of paranasal sinus infection (especially ethmoid sinusitis) is the most frequent source.
668. d Abrasions related to the wearing of soft contact lenses pose a risk of Pseudomonas
Rosen 5th infection and likewise should not be patched. These patients should be treated with
Chapter 66, tobramycin ointment QID, followed by a fluoroquinolone (Ciloxan, Ocuflox) drop
p. 920 or tobramycin drop QID once the epithelial defect starts to close.
669. e The first branch off the internal carotid artery is the ophthalmic artery, which supplies
Rosen 5th the central retinal artery, which, in turn, provides the blood supply to the inner retina.
Chapter 66, If the central retinal artery becomes occluded, the retina will infarct and become pale,
p. 925 less transparent, and edematous.
670. d The sixth cranial nerve has a long and tortuous course, and defects of this nerve are the
Rosen 5th most commonly reported ocular motor palsies. Patients with sixth cranial nerve palsies
Chapter 66, have an esotropia that is worsened by lateral gaze and will often turn their heads
p. 925 laterally toward the paretic side to compensate. Sixth-nerve palsy is caused by a
variety of diseases. Aneurysm, vascular disease (diabetes, hypertension,
atherosclerosis), trauma, neoplasm, multiple sclerosis, meningitis, thyroid eye disease,
and increased intracranial pressure may all cause dysfunction.
671. c An orbital hematoma or retrobulbar hemorrhage can occur after blunt trauma. It can
Rosen 5th result in a significant elevation in intraorbital pressure within the enclosed space,
Chapter 66, resulting in compression of the optic nerve and central retinal artery. Symptoms
p. 910 include proptosis, visual loss, and increased intraocular pressure. Any compromise in
the retinal blood supply is an indication for emergent surgical decompression.
672. b There is no subcutaneous fat in the eyelids themselves; protrusion of orbital fat into the
Rosen 5th wound indicates penetration of the orbital septum. Ptosis indicates injury to the levator
Chapter 66, muscle or aponeurosis of the upper lid. Subconjunctival hemorrhage results from
p. 915 rupture of small subconjunctival vessels, and by itself is benign. Bell's phenomenon is
the normal upward rotation of the globe with reflex blinking. Seidels test to use to
check for globe rupture.
215

673. Middle ear aspirates from patients with bullous myringitis usually grow:
a. Chlamydia trachomatis.
b. Escherichia coli.
c. Mycoplasma pneumoniae.
d. nothing.
e. Streptococcus pneumoniae.

674. The most common complication of acute otitis media is:


a. bacteremia.
b. hearing loss.
c. mastoiditis.
d. meninigitis.
e. sinusitis.

675. The most common intracranial complication of acute otitis media is:
a. lateral venous sinus thrombosis.
b. encephalitis.
c. epidural abscess.
d. meningitis.
e. subdural abscess.

676. Otitis externa:


a. can be life-threatening in an immunocompromised patient.
b. can easily be confused with a Ramsay-Hunt syndrome.
c. can initially be treated using any mildly acid solution (boric acid, acetic acid, etc.).
d. is a disease of fall and winter.
e. is usually caused by streptococcus group A.
216

673. e Although we are taught to think about atypical bacteria in a patient with bullous
Rosen 5th myringitis, middle ear aspirates in this condition usually grow S. pneumoniae and H.
Chapter 67, influenzae. Mycoplasma pneumoniae is uncommon.
p. 929
674. b Hearing impairment is the most common complication in otitis media. Almost all
Rosen 5th children with OM will have a temporary conductive hearing loss; sensorineural deficit
Chapter 67, occurs less commonly, probably as a spread of infection through the round window.
p. 930 This may contribute to the association of OM with decreased or delayed speech,
language, or cognitive development.
675. d Meningitis is the most common intracranial complication, more from hematogenous
Rosen 5th spread than direct invasion. The symptoms include headache, meningismus, fever,
Chapter 67, nausea, emesis, irritability, lethargy, and altered mental status.
p. 930
676. a Necrotizing external otitis is an extremely aggressive form of otitis externa, which can
Rosen 5th lead to invasive disease and death. The most common bacterial causes of external
Chapter 67, otitis are Pseudomonas aeruginosa and S. aureus. It occurs most often in the summer
p. 931 and is common in the tropics. Herpes zoster oticus, also known as the Ramsay Hunt
syndrome, is a viral manifestation of disease affecting the auricle, with resulting facial
paralysis that may involve multiple cranial nerves. Acetic acid possesses antibacterial
and antifungal properties that are more important than its acidifying properties.
217

8.0 Hematologic Disorders


677. A patient with anemia, thrombocytopenia, renal failure, normal coagulation tests and a
clear sensorium probably has:
a. (ITP) idiopathic thrombocytopenic anemia.
b. (TTP) thrombotic thrombocytopenic purpura.
c. (HUS) hemolytic-uremic syndrome.
d. (DIC) disseminated intravascular coagulation.
e. autoimmune hemolytic anemia.

678. The viral agent implicated in an aplastic crisis of patients with sickle cell disease is:
a. adenovirus (atypical).
b. herpes simplex.
c. parvovirus.
d. coxsackie virus.
e. HTLV-IV.

679. The etiologic agent implicated in hemolytic uremic syndrome is:


a. giardia.
b. Escherichia coli O157:H7.
c. scombroid toxin.
d. atypical enterovirus / poliovirus.
e. Enteromoeba histolytica.

680. The most helpful lab study in diagnosing disseminated intravascular coagulation is the:
a. D-dimer, which is elevated.
b. partial thromboplastin time (PTT), which is decreased.
c. fibrinogen level, which is elevated.
d. prothrombin time, which is prolonged.
e. fibrin degradation products (FDP), which are diminished.

681. Hemolytic-uremic syndrome is most commonly seen in:


a. neonates.
b. infants and children 6 months to 4 years of age .
c. adolescents.
d. women age 30 to 50.
e. both sexes, over age 75.

682. The initial dose of factor VIII required for a 60-kg male with severe hemophilia A in whom
you suspect a ruptured spleen is:
a. 1,500 units
b. 2,850 units
c. 3,000 units
d. 6,000 units
e. 5,700 units
218

677. c ITP generally presents with isolated thrombocytopenia. TTP causes neurologic
Rosen 5th symptoms in addition to the other symptoms. DIC will have abnormal coagulation
Chapter 116, studies. Autoimmune hemolytic anemia may cause severe rapid anemia, which may
p. 1692-1693 present with angina or congestive heart failure.
678. c Aplastic crises can be precipitated by viral infections (particularly parvovirus B19),
Rosen 5th folic acid deficiency, or the ingestion of bone marrow toxins such as phenylbutazone.
Chapter 161, Bone marrow erythropoiesis is slowed or stopped. The hematocrit falls to as low as
p. 1823 10%, and the reticulocyte count falls to as low as 0.5%. The white blood cell count
and platelet counts usually remain stable.
679. b In children, the development of HUS often follows a prodromal infectious disease,
Rosen 5th usually diarrhea or an upper respiratory infection. Diarrhea, particularly that
Chapter 167, associated with Escherichia coli serotype 0157:H7, as well as with Shigella, Yersinia,
p. 2341-2342 Campylobacter, and Salmonella, may be antecedent.
680. d MOST USEFUL: HELPFUL:
Rosen 5th Prothrombin time prolonged Activated partial thromboplastin time usually
Chapter 116, Platelet count usually low prolonged
p. 1697-1698 Fibrinogen level low Thrombin clot time prolonged
Table 116-3 Fragmented red blood cells should be present
Fibrin degradation products and D-dimers elevated
681. b Hemolytic-uremic syndrome (HUS) is a disease mainly of infancy and early childhood,
Rosen 5th with a peak incidence between 6 months and 4 years of age. Mean age at presentation
Chapter 167, is 3 years, and it is rare after age 5 years.
p. 2341
682. c A 60-kg patient with a life threatening hemorrhage who requires 100% correction will
Rosen 5th need 50 mL/kg = 3000 units of factor VIII. Formula: FactorVIII (units) = Kg./2 X
Chapter 116, Activity Desired
p. 1695
219

683. A 44-year-old woman with a history of TTP, in remission for 30 days, presents to the ED
complaining of lethargy. Laboratory results would likely show:
a. elevated LDH, elevated reticulocyte count, elevated red blood cell count.
b. elevated LDH, elevated reticulocyte count, elevated creatinine.
c. decreased platelet count, decreased red blood cell count, decreased reticulocyte count.
d. decreased platelet count, decreased reticulocyte count, decreased LDH.
e. none of the above.

684. A 54-year-old woman complains of back pain for several months. She now has two days
of urine and stool incontinence and numbness when she wipes with toilet paper. You
suspect spinal cord compression, which:
a. commonly occurs as a presenting symptom in a patient with a previously unsuspected
malignancy.
b. is always associated with back pain, but not tenderness.
c. usually responds to aggressive chemotherapy.
d. usually causes asymmetric motor or sensory defects.
e. can usually be diagnosed by plain radiographs, with CT or MRI adding little.

685. In a patient with hemolytic anemia from any cause, you would expect to find:
a. decreased haptoglobin, increased LDH, increased indirect bilirubin, decreased reticulocytes.
b. decreased haptoglobin, increased LDH, increased indirect bilirubin, increased reticulocytes.
c. increased haptoglobin, decreased LDH, decreased indirect bilirubin, increased reticulocytes.
d. increased haptoglobin, increased LDH, increased direct bilirubin, increased reticulocytes.
e. increased haptoglobin, increased LDH, increased indirect bilirubin, increased reticulocyte.

686. You are evaluating a patient whom you suspect to be severely anemic. You know that:
a. pallor is present only with chronic anemia.
b. the RBC distribution width (RDW) is useful in differentiating iron deficiency anemia from
thalassemia.
c. purpura, petechiae, and jaundice usually indicate an acute anemia.
d. orthostasis represents a chronic anemia.
e. retinal hemorrhages indicate an acute anemia.

687. The most helpful laboratory study to differentiate poor red blood cell production from
increased red cell destruction is the:
a. sedimentation rate.
b. sideroblast level.
c. serum iron level.
d. total to direct bilirubin ratio.
e. reticulocyte count.

688. Choose the correct pairing.


a. hypochromic microcytic anemia hypothyroidism
b. chronic disease hyperchromic anemia
c. macrocytic anemia lead poisoning
d. myelodysplastic anemia cells may be large or normal size, seen in toxins and renal failure
e. megaloblastic anemia impaired RNA synthesis
220

683. b Thrombotic thrombocytopenic purpura is classically seen as the constellation of


Rosen 5th thrombocytopenic purpura, microangiopathic hemolytic anemia, fluctuating neurologic
Chapter 116, symptoms, renal disease, and fever. However, only 40% of cases present with the
p. 1693 classic pentad. The platelet count ranges from 10,000/mm3 to 50,000/mm3, and
generalized purpura and bleeding complaints are common. Anemia is universal, with
hematocrit commonly less than 20%. The hemolysis may cause jaundice or pallor, and
the blood smear characteristically contains numerous schistocytes and fragmented red
blood cells.
684. a Spinal cord tumors produce neurologic symptoms by compression, invasion, or
Rosen 5th destruction of myelinated tracts. The resulting neurologic symptoms are directly
Chapter 100, related to both the growth rate and location of the tumor. Most tumors of the spinal
p. 1504 cord are metastatic in origin. Approximately 5% to 10% of patients ultimately
diagnosed with cancer first present with a spinal metastasis. Most metastases occur in
the thoracic spine, and nearly 20% will have disease at multiple levels. In 95% of
patients with spinal neoplasm, the initial complaint is pain, either in the back at the
level of the tumor or in a radicular distribution. Nighttime pain that is severe is
characteristic of spinal neoplasm.
685. d Haptoglobin binds hemoglobin on a molecule-for-molecule basis. Its absence implies
Rosen 5th saturation and degradation after binding with hemoglobin and is an early finding in
Chapter 115, hemolysis. It has a normal range of 40 to 180 mg/ml, is decreased in hepatic failure,
p. 1676 and increases as an acute-phase reactant. After haptoglobin is bound, hemoglobin
binds with hemopexin, transferrin, and albumin before circulating in its free form.
LDH is released when the RBC is broken down peripherally or in the marrow. It is
elevated in hemolytic, thalassemic, sideroblastic, and megaloblastic anemias. It may
also be seen in cases of uremia, polycythemia vera, and erythroleukemia. Bilirubin is
often delivered to the liver faster than the conjugating mechanism can handle it.
Conjugated or indirect bilirubin may rise as high as 4 to 5 mg/dl even with normal liver
function. Higher levels connote some degree of underlying hepatic insufficiency.
686. b The RBC distribution width (RDW) is a measure of the homogenicity of the RBCs
Rosen 5th measured. It is automatically calculated as the standard deviation of the MCV divided
Chapter 115, by the MCV multiplied by 100. A normal RDW is 13.5% + 1.5%. It is useful in
p. 1667 differentiating iron deficiency from thalassemia.
687. e The reticulocyte retains its ribosomal network for about 4 days, of which 3 are spent in
Rosen 5th the bone marrow and 1 in the peripheral circulation. The red blood cell matures as the
Chapter 115, reticulocyte loses its ribosomal network and circulates for 110 to 120 days. Under
p. 1665 steady state conditions, the rate of red blood cell production equals the rate of
destruction. Red blood cell mass remains constant as an equal number of reticulocytes
replace the destroyed, senescent erythrocytes during the same period.
688. d RBC indices are useful in classifying production deficit anemias. The mean
Rosen 5th corpuscular volume (MCV) is a measure of RBC size. Decreases and increases reflect
Chapter 115, microcytosis and macrocytosis, respectively. The mean corpuscular hemoglobin
p. 1667 (MCH) value incorporates both the RBC size and hemoglobin concentration.
It is influenced by both and is the least helpful of the indices. The MCH concentration
is a measure of hemoglobin concentration (MCHC). Low values represent
hypochromia. High values are noted only when decreased cell membrane relative to
cell volume exists, such as in spherocytosis.
221

689. The most common inherited bleeding disorder is:


a. hemophilia A.
b. hemophilia B.
c. von Willebrands disease
d. hereditary DIC.
e. Christmas disease.

690. Your patient is profoundly anemic and you believe she would benefit from a blood
transfusion. In weighing the benefits against the risks, you tell her that the most common
adverse effect is:
a. hepatitis C transmission.
b. hepatitis B transmission.
c. human immunodeficiency virus (HIV) transmission.
d. febrile nonhemolytic reaction.
e. graft versus host reaction.

691. The anemia in adult sickle-cell disease is due to:


a. poor marrow production.
b. abnormal protein-binding capacity.
c. increased red-cell destruction.
d. overactive antibody-antigen response.
e. sequestration.

692. Long-term effects of sickle-cell disease can include:


a. strokes.
b. liver failure.
c. renal failure.
d. bone infarcts.
e. all of the above.

693. In a patient with sickle-cell disease, infectious crises are primarily due to:
a. leukopenia.
b. marrow shutdown.
c. poor antibody production.
d. functional asplenia.
e. antibiotic resistance.

694. A 12-year-old girl with sickle cell disease is brought by her mother after she passed out
twice. She was kept home from school the last few days for a cold. When you ask the
child to stand, you must catch her to prevent her from falling to the ground. This is
suspicious for:
a. salmonella sepsis.
b. sequestration crisis.
c. acute chest syndrome.
d. aplastic crisis.
e. hemolytic crisis.

695. In a patient with sickle cell disease who is having a typical vaso-occlusive crisis:
a. oral hydration is seldom sufficient.
b. jaundice is a sign of serious disease.
c. oxygen is always helpful.
222

d. exchange transfusions are becoming the treatment of choice.


e. morphine is preferred over meperidine as the pain medication of choice.
223

689. c Von Willebrands disease is the most common hereditary bleeding disorder, with an
Rosen 5th estimated prevalence of 1%. The disease occurs in 5 to 10 persons per million
Chapter 116, population as an autosomal dominant trait with a variable penetrance pattern. A rare
p. 1697 X-linked inheritance has been described.
690. d This most common and least serious transfusion reaction is characterized by fever,
Rosen 5th chills, and malaise. Reactions are frequently related to antileukocyte and antiplatelet
Chapter 5, antibodies and seen in multiply transfused patients. Treatment is symptomatic with an
p. 50 analgesic/antipyretic and an antihistamine. If recurrent febrile reactions occur in a
patient, leukocyte-poor RBCs (washed, frozen-thawed-deglycerolized, filtered) should
be considered. If a febrile reaction occurs in a first-time transfusion, it should be
treated the same as an extravascular hemolytic reaction until proved otherwise.
691. c Although most of the diagnostic and therapeutic problems of sickle cell disease are
Rosen 5th related to vasoocclusive crises, other serious complications must be anticipated. Sickle
Chapter 115, cell disease is a chronic hemolytic state with reasonably compensated hematocrit
p. 1678 values in the 20% to 30% range and elevated reticulocyte counts. This compensated
balance may be disrupted by a rare iron deficiency or more commonly a folate
deficiency. A potentially life-threatening aplastic crisis may be seen with an acute
postinfectious or folate deficiency suppression of erythropoiesis. This aplastic
condition is suspected when the hemoglobin level falls 2 g/dl or more from previous
stable levels, and the reticulocyte count remains low (< 2%).
692. e Long-term effects of sickle cell disease are protean and devastating. Space prohibits
Rosen 5th listing them here.
Chapter 115,
p. 1679,
Table 115-7
693. d Adults with fever require a careful evaluation and laboratory assessment, including
Rosen 5th appropriate cultures. Early institution of appropriate antibiotics is necessary in those
Chapter 115, patients with a discernible source of infection. In children and adults, Staphylococcus
p. 1678-1679 and Pneumococcus species and Haemophilus influenzae are particularly common. An
increased incidence of Salmonella osteomyelitis also occurs. The origin of this related
immunologic deficiency is believed to be multifactorial, with functional asplenia,
poorly migrating neutrophils, and decreased opsonin production as contributors.
694. b Children with sickle cell disease may have an acute splenic sequestration syndrome.
Rosen 5th This syndrome involves acute splenic enlargement from increased intrasplenic sickling
Chapter 115, and obstruction. The child may have lassitude and be in shock.
p. 1678
695. e Present therapies are directed toward symptomatic relief and attempts to stop the cycle
Rosen 5th of deoxygenated sickling and intravascular sludging. These include rest, adequate
Chapter 115, nutrition, hydration, oxygenation, analgesia, transfusion, and therapy for infection.
p. 1679 Most patients with sickle cell anemia are mildly dehydrated because of urine-
concentrating difficulties. Fluid replacement can be oral or IV. Oxygen through a
nasal cannula at 2 to 4 L/min may help hypoxic patients and may be given to any
patient with HbSS as a low-risk treatment modality with potential benefit. No standard
pain management exists for sickle cell disease. The most important aspect of pain
management in these patients is a consistent, thorough, and attentive approach that
offers true pain relief. Rarely, transfusions are given for control of bony or visceral
crises. This is not an ED procedure and is considered only after hematologic
consultation. Prophylactic transfusions to dilute HbS levels are also recommended in
pregnancy and before major surgery.
224

696. The laboratory tech calls to tell you that she sees Howell-Jolly bodies on the peripheral
smear of one of your patients. You tell her that you suspected as much, since the patient:
a. has a G6PD deficiency.
b. has a history of malaria.
c. is asplenic.
d. is HIV positive.
e. is named Howell and is very jolly.

697. Obstetric causes of disseminated intravascular coagulation (DIC) include:


a. placental abruption
b. amniotic fluid embolism
c. acute fatty liver of pregnancy
d. eclampsia
e. all of the above.

698. The cornerstone of Emergency Department management of DIC is:


a. hemodynamic stabilization and treatment of the underlying disorder.
b. rapid correction of thrombocytopenia.
c. aggressive resuscitation with colloid.
d. pan-culture and broad-spectrum antibiotic coverage.
e. rapid intubation and hyperventilation.

699. Heparin has selective use in the treatment of DIC when fibrin deposition and thrombosis
dominate the pathologic picture. An example of such a condition is:
a. meningococcemia.
b. purpura fulminans.
c. abruptio placentae.
d. severe liver disease.
e. trauma.

700. Heparin-induced thrombocytopenia:


a. does not occur with low molecular weight heparins.
b. requires a minimum number of units, so a heparin flush is always safe.
c. can paradoxically cause thrombosis, ischemia, and amputation.
d. never occurs during the first 24 hours of infusion.
e. is easily treated with warfarin and fresh-frozen plasma.
225

695. c The presence of Howell-Jolly bodies on a peripheral blood smear indicates functional
Rosen 5th or true absence of a spleen.
Chapter 177,
p. 2500
697. e Events during pregnancy which can lead to disseminated intravascular coagulation
Rosen 5th include placental abruption, amniotic fluid embolus, septic abortion, intrauterine fetal
Chapter 175, death (chronic DIC); women can also get DIC in HELLP syndrome.
p. 2481
698. a The goals of emergency care in cases of DIC include initial suspicion, aggressive
Rosen 5th diagnostic pursuit, understanding of potential life-threatening complications, and only
Chapter 116, rarely, initiation of therapy.
p. 1698
699. b Heparin has selective use in the treatment of DIC when fibrin deposition and
Rosen 5th thrombosis dominate the pathologic picture. Certain disease states are more associated
Chapter 116, with fibrin deposition, in which case heparin therapy should be considered. Examples
p. 1698 include purpura fulminans, retained dead fetus before delivery, giant hemangioma, and
acute promyelocytic leukemia. Heparin therapy is of little benefit in cases of
meningococcemia, abruptio placentae, severe liver disease, and trauma. Low doses of
heparin (300 to 500 U/hour) as a continuous infusion are currently recommended.
Low-molecular-weight heparin may also be used instead of unfractionated heparin.
700. c A number of drugs have been associated with thrombocytopenia of immunologic
Rosen 5th origin. Because of its relatively high frequency, heparin is an important cause of drug-
Chapter 116, induced thrombocytopenia in hospitalized patients. Platelets are activated by the
p. 1692 formation of an IgG-heparin complex. Low-molecular-weight heparin may be
associated with less thrombocytopenia than standard, unfractionated heparin; however,
both forms of heparin have cross-reactivity. Heparin-induced thrombocytopenia can
occasionally lead to the white clot syndrome, causing impaired peripheral
circulation, gangrene, and amputation.
226

9.0 Immune System Disorders


701. An important step in the work-up of arthritis is determining whether it is monoarticular or
polyarticular. Classification by number of joints involved can help narrow the diagnosis. Which
pairing below is correct?
a. gout polyarticular arthritis
b. Lyme disease monoarticular arthritis
c. pseudogout polyarticular arthritis
d. Reiters syndrome monoarticular arthritis
e. rheumatoid disease polyarticular arthritis

702. A 22-year-old female with no prior medical history presents with joint pain and swelling. When
evaluating a patient with a complaint of nontraumatic joint pain, you know that:
a. arthritis in the first carpometacarpal joint implies rheumatoid disease.
b. bursitis causes a diffuse joint pain.
c. fever is unusual in a patient with arthritis which is not a septic joint.
d. gonococcal arthritis tends to cause migratory oligoarticular symptoms.
e. true arthritis causes pain on active motion, but not on passive motion.

703. In evaluating a patient with acute arthritis, which laboratory study may be of any value in
determining the underlying etiology?
a. CPK.
b. platelet count.
c. sedimentation rate.
d. serum protein level.
e. uric acid level.

704. A 38-year-old man complains of an isolated swollen knee joint. He is afebrile and nontoxic. After
appropriate explanation and preparation, you perform arthrocentesis and obtain 55 ml of somewhat
cloudy, straw-colored joint fluid. You know that:
a. absolute cell counts can be used to rule out a septic etiology.
b. arthrocentesis is contraindicated if infection of any kind covers the joint.
c. most of the WBCs in septic arthritis are PMNs, whereas eosinophils predominate in inflammatory
arthritis.
d. synovial fluid glucose, lactic acid, viscosity, mucin clot, and total protein can all help differentiate
the etiology of the arthritis.
e. two red-topped tubes should be sufficient for any studies which need to be done.

705. You suspect a patient may have rheumatoid arthritis, so you order an EKG, thinking that you might
find:
a. bigeminy.
b. left anterior hemiblock.
c. prolonged PR interval.
d. right ventricular hypertrophy.
e. shortened QT interval.
227

701. e Of the listed arthritides, only pseudogout typically presents as monoarticular arthritis.
th
Rosen 5 , Other monoarticular arthritides are osteoarthritis, septic arthritis, and gout. In addition,
Chapter 110, trauma and hemarthrosis usually produce monoarticular symptoms. Besides those
p. 1585 listed other diseases that produce polyarticular symptoms include rubella, rheumatic
fever, Reiter's syndrome, and serum sickness. See Table 110-1, page 1585.
702. d Gonococcal arthritis tends to occur in two or three joints (oligoarticular). True arthritis
Rosen 5th, produces generalized joint pain, warmth, swelling, and tenderness. Discomfort
Chapter 110, increases with both passive and active motion of the joint because the inflamed
p. 1585 synovium is exquisitely sensitive to stretching, and because all parts of the joint are
involved in the inflammatory process. Periarticular inflammation (bursitis, tendinitis,
or localized cellulitis) tends to be focal. Tenderness and swelling do not occur
uniformly across the joint, and pain is produced only with certain movements. Patients
with any inflammatory arthritis may have low-grade fever. The first carpometacarpal
joint is frequently involved in osteoarthritis
703. c Laboratory tests other than synovial fluid analysis are of limited diagnostic value for
Rosen 5th, evaluating acute arthritis in the ED. The two most general screening tests are a
Chapter 110, complete blood count and an erythrocyte sedimentation rate, which is elevated in
p. 1586 almost all cases of inflammatory arthritides. The serum uric acid level is not helpful in
diagnosing acute gouty arthritis; in the acute phase of the disease, the serum uric level
may be normal.
704. b Emergent arthrocentesis is contraindicated if infection of any kind covers the area to be
Rosen 5th, punctured. Most of the WBCs in both septic and severe inflammatory arthritis are
Chapter 110, PMNs. Prediction rules regarding cell count and the likelihood of septic arthritis are
p. 1587 common, but are in no way absolute, and cell counts should not be used to rule out a
septic etiology. Rather, bacterial cultures should be obtained if there is any suspicion
of infection. Other tests (i.e., synovial fluid glucose, lactic acid, viscosity, mucin clot,
and total protein) have limited utility in ruling out infection and are no longer routinely
recommended. Specimens for cellular analysis should be submitted in tubes with
ethylenediaminetetraacetic acid (EDTA) anticoagulant (lavender top), whereas
specimens for crystal analysis should be transported in tubes with liquid heparin (green
top). Chemical analysis, serology, and viscosity should be analyzed on fluid submitted
in a red top tube.
705. c ECG is indicated for patients with arthritis who have a history of chest pain or
Rosen 5th, complaints that might be related to the heart, or physical examination findings of a new
Chapter 110, or changing heart murmur, evidence of congestive heart failure, or cardiomegaly. In
p. 1589 carditis, prolongation of the P-R interval is the most common finding, and if
pericarditis is present, acute diffuse ST segment elevations may be noted.
228

706. The most common bacterial cause of septic arthritis is:


a. Gram negatives.
b. group A streptococcus.
c. Haemophilus influenzae.
d. Neisseria gonorrhoeae.
e. Staphylococcus aureus.

707. The joint which most frequently becomes septic is the:


a. ankle.
b. hip.
c. knee.
d. shoulder.
e. wrist.

708. Concerning the treatment of gout:


a. allopurinol increases uric acid elimination and is useful during an acute attack.
b. colchicine should not be used prophylactically.
c. if NSAIDs are contraindicated, intramuscular injections of ACTH are useful.
d. probenecid lowers uric acid by diminishing production.
e. response to colchicine is diagnostic for the disease.

709. Pseudogout differs from gout in that:


a. indomethacin is contraindicated.
b. the attack is more severe.
c. the crystals cannot be identified using microscopy.
d. the knee is the most commonly involved joint.
e. the typical patient is younger.

710. The two viruses that most commonly cause arthritis are:
a. coxsackievirus and hepatitis A.
b. Epstein Barr virus and enteroviruses.
c. hepatitis C and parvovirus.
d. mumps and adenoviruses.
e. rubella and hepatitis B.

711. The most common stool pathogen implicated in Reiters syndrome is:
a. Campylobacter spp.
b. Chlamydia trachomatis.
c. Salmonella typhi.
d. Shigella flexneri.
e. Yersinia enterocolitica.
229

706. e The microbiology of septic arthritis has remained fairly constant over time. Overall,
th
Rosen 5 , Staphylococcus aureus is still the most common cause of septic arthritis.
Chapter 110, Staphylococci, streptococci, gram-negative organisms, and anaerobes in relatively
p. 1590 constant proportions cause the remaining cases of monoarthritis. N. gonorrhoeae
accounts for ~20% of cases of monoarticular septic arthritis, but it is a more common
pathogen in patients with polyarthritis, which is the usual presentation.
707. c Typically, infectious arthritis affects a single joint, with the most common joints
Rosen 5th, infected being the knee (40% to 50%), hip (13% to 20%), shoulder (10% to 15%),
Chapter 110, wrist (5% to 8%), ankle (6% to 8%), elbow (3% to 7%), and the small joints of the
p. 1591 hand or foot (5%). In approximately 20% of cases, several joints may be involved at
the same time. Even in septic polyarthritis, however, the knees are the most common
sites of infection.
708. c Colchicine is effective for gout, pseudogout and other crystal arthritides, so it cannot
Rosen 5th, be used to make the specific diagnosis of gout. Long-term therapy of gout is designed
Chapter 110, to decrease serum uric acid levels either by decreasing production (allopurinol) or
p. 1592 increasing excretion (probenecid). ACTH is also recommended for those patients with
contraindications to NSAIDs. The dose of ACTH is 40 IU to 80 IU given IM. Uric
acid lowering agents should not be started during an acute attack. Colchicine may be
given prophylactically for 6 to 12 months to suppress flare-ups.
709. d In a patient with pseudogout, the knee is the joint most commonly involved, followed
Rosen 5th, by the wrist, ankle, and elbow. The average attack is not as severe as acute gout. In
Chapter 110, general, these patients are between the sixth and eighth decades that have a previous
p. 1592 history of arthritic attacks. Joint fluid examination shows the weakly positive
birefringent crystals of calcium pyrophosphate dihydrate. The crystals appear
rhomboidal on regular light microscopy. Treatment for an acute attack is similar to the
therapy for acute gout: NSAIDs or oral colchicine, although the latter is not as
effective as with gout.
710. e The two viruses that most commonly cause arthritis are rubella and hepatitis B, but
Rosen 5th, arthritis can also occur with mumps, adenoviruses, Epstein-Barr virus, and
Chapter 110, enteroviruses. The pathophysiology of arthritis in the viral diseases appears to be
p. 1593 deposition of soluble immune complexes in the synovium with resultant inflammation.
Patients with rubella arthritis often are young women.
711. d Reiters syndrome represents the clinical manifestation of a reactive arthritis that
Rosen 5th, occurs in genetically susceptible hosts after infection with Chlamydia trachomatis in
Chapter 110, the genitourinary tract, or Salmonella, Shigella, Yersinia, or Campylobacter organisms
p. 1595 in the GI tract. Salmonella enteritis leads to reactive arthritis in up to 4% of cases;
Shigella flexneri is the most common stool isolate causing Reiters syndrome.
230

712. The triad of fever, joint pain, and rash in a woman of childbearing age should suggest the diagnosis
of systemic lupus. You also know that:
a. bundle branch block is the most common cardiac manifestation, reported in 30% of patients.
b. exudative pleural effusions are relatively common.
c. persistent hematuria is seen in approximately 50% of patients.
d. neurologic presentations, such as seizures, stroke, psychosis, migraines, and peripheral
neuropathies, are frequently the first signs of disease.
e. unlike rheumatoid arthritis, the inflammation of the hands is asymmetric.

713. A 77-year-old woman complains of headache, low-grade fever, malaise, myalgias, intermittent
blurred vision, and other nonspecific symptoms. You suspect giant cell arteritis, knowing that:
a. absence of temporal artery tenderness would eliminate the possibility of this condition.
b. elevated C-reactive protein level may be a helpful diagnostic clue.
c. polycythemia is a common finding.
d. sedimentation rate will be two or three times the normal rate.
e. steroids must be withheld until arterial biopsy can be done.

714. The hallmark physical finding (as opposed to most common finding) of Behets syndrome is:
a. green sclera.
b. hypopyon uveitis.
c. intracranial hypertension with a multiple sclerosislike syndrome.
d. iritis and optic neuritis.
e. recurrent, painful genital aphthous ulcers.

715. Henoch-Schnlein purpura primarily effects:


a. arterioles and capillaries in children.
b. veins and venules in adults.
c. arteries and venules in children.
d. arterioles and veins in adults.
e. capillaries and venules in children.

716. You are evaluating a 35-year-old with a history of systemic lupus. She complains of chest pain.
You know that:
a. Libman-Sachs vegetations are infectious excrescences on the aortic valve, representing bacterial
endocarditis.
b. lupus pericarditis requires high-dose steroid therapy.
c. pericardial effusions are found in more than half of lupus patients.
d. pericarditis is the most common cardiac manifestation of SLE.
e. SLE patients have no increased risk of coronary artery disease.

717. Acute HIV syndrome is characterized by nonspecific symptoms such as fatigue, weight loss,
diarrhea, pharyngitis, and adenopathy, seen:
a. within 24 hours of exposure.
b. 1 week after exposure.
c. 2 to 6 weeks after exposure.
d. 12 to 24 weeks after exposure.
e. 4 to 6 months after exposure.
231

712. b Pleural effusions, seen in 12% of SLE patients, are usually exudative in nature. Like
th
Rosen 5 , rheumatoid arthritis, the inflammation of the hands, specifically the proximal
Chapter 112, interphalangeal and the metacarpophalangeal joints, is symmetric. Clinical nephritis,
p. 1609 defined as persistent proteinuria, is seen in approximately 50% of patients. Nervous
system manifestations are varied and include seizures, stroke, psychosis, migraines,
and peripheral neuropathies, but are rarely the initial sign. Pericarditis is the most
common cardiac manifestation of SLE, reported in 30% of patients.
713. b Temporal or giant cell arteritis is most common in branches of the carotid artery but
Rosen 5th, may involve any large or medium artery. The disease is most commonly seen in
Chapter 112, women in the sixth and seventh decades of life. The classic symptoms of TA are
p. 1613 consistent with ischemia to the organs fed by branches of the internal and external
carotid artery: visual loss in one eye, temporal artery tenderness, and jaw claudication.
Patients may complain of nonspecific, vague symptoms such as malaise, weight loss,
and fever. Headache may be the initial complaint. Although the diagnosis is made
clinically, helpful laboratory findings include elevated ESR (usually greater than 100
mm/hr on a Westergren blot), elevated C-reactive proteins, and anemia. The definitive
diagnosis is made by temporal artery biopsy. Most patients are extremely sensitive to
glucocorticoids, and treatment should be started for any patient with a high clinical
suspicion of TA. The steroids do not significantly change the results of the biopsy and
may prevent progression to visual loss.
714. b Recurrent, painful aphthous ulcers that involve the oral mucosa and genitals are
Rosen 5th, clinically predominant, but the hallmark of Behets, a hypopyon uveitis, is seen
Chapter 112, rarely. Other eye involvement includes iritis, uveitis, and optic neuritis, all of which
p. 1615 can lead to blindness. CNS vasculitis, resulting in meningoencephalitis, intracranial
hypertension, or a multiple sclerosislike syndrome, can also occur
715. a Henoch-Schnlein purpura (HSP) affects mainly the arterioles and capillaries, with
Rosen 5th, peak incidence between 4 and 11 years of age, although adults may also be affected.
Chapter 112, The syndrome occurs most often in the spring following a viral upper respiratory
p. 1616 infection. Other inciting agents associated with HSP include insect stings and drugs.
716. d Pericarditis is the most common cardiac manifestation of SLE, reported in 30% of
th
Rosen 5 , patients. Signs and symptoms include fever, tachycardia, chest pain, and transient
Chapter 112, rubs. Pericardial effusions, however, are found in only 20% of patients. Purulent
p. 1609 pericarditis associated with Staphylococcus aureus and tuberculosis has occurred in
patients taking steroids. Pericarditis in SLE is usually self-limited. Libman-Sachs
vegetations on the mitral valve are noninfectious and related to autoimmune
deposition. SLE patients with hypertension and hypercholesterolemia are at a
markedly increased risk of coronary artery disease. Myocarditis is clinically apparent
in only 10% of patients; however, it is present in 40% at autopsy.
717 .c Acute HIV syndrome usually occurs 2 to 6 weeks after initial exposure and is
Rosen 5th, characterized by multiple nonspecific symptoms such as fever, fatigue, adenopathy,
Chapter 126 diarrhea, pharyngitis, weight loss, and rash. These symptoms may last up to 3 weeks
p. 1845 and usually resolve without the patient seeking medical advice.
232

718. A 24-year-old man with AIDS complains of headache, fever, stiff neck, and confusion. You
suspect cryptococcal meningitis, knowing that the most accurate diagnostic study will be:
a. CSF cryptococcal antigen.
b. CSF fungal culture.
c. CSF India ink stain.
d. Serum cryptococcal antigen.
e. Serum cryptococcal antibody.

719. In an AIDS patient who complains of diarrhea, the pathogen most likely to lead to a bacteremia is:
a. Campylobacter jejuni.
b. enteroadherent E. coli.
c. Isospora spp.
d. Salmonella spp.
e. Shigella spp.

720. Kaposis sarcoma is:


a. commonly found on the palms and the soles.
b. highly lethal.
c. more common in HIV-positive women, due to the estrogen effect.
d. painful, blanching raised brown-black or purple papules and nodules.
e. the second most common manifestation of AIDS.

721. The treatment of Kawasaki Syndrome is intended to prevent:


a. coronary artery aneurysms.
b. fistula formation from the draining lymph nodes.
c. fulminant liver failure.
d. meningoencephalitis.
e. paraplegia or quadriplegia.

722. A middle-aged woman is brought from a local shopping mall by ambulance. She is confused and
profusely diaphoretic, and can answer no questions. The medics only report that she was
hypotensive and bradycardic, but she did not respond to 1 mg of intravenous atropine. As you
examine the patient you note a large well-healed midchest scar and a Medic Alert bracelet stating
Heart Transplant Patient. You know you should treat her bradycardia with:
a. dobutamine.
b. dopamine.
c. epinephrine.
d. higher doses of atropine.
e. isoproterenol.
233

718. a Of the diagnostic tests for cryptococcus, identifying cryptococcal antigen in the CSF is
th
Rosen 5 , 100% sensitive and specific. India ink has a 60% to 80% sensitivity; fungal culture
Chapter 126, and serum cryptococcal antigen have a 95% sensitivity for identifying cryptococcus.
p. 1849
719. d Salmonella is a particular problem in patients with HIV, often producing recurrent
th
Rosen 5 , bacteremia and other significant clinical disease. Campylobacter infection usually
Chapter 126, causes a proctocolitis. Isospora and Cryptosporidium are protozoal infections that
p. 1852 produce a chronic watery diarrhea. Cytomegalovirus is a viral opportunistic infection
that is associated with diarrhea and other gastrointestinal disease (such as
hepatobiliary) in HIV patients.
720. e Kaposis sarcoma is the second most common manifestation of AIDS and appears
Rosen 5th, more often in homosexual men than in other risk groups. Clinically, it consists of
Chapter 126, painless, raised brown-black or purple papules and nodules that do not blanch.
p. 1852 Common sites are the face, chest, genitals, and oral cavity; however, widespread
dissemination involving internal organs may occur. Since cutaneous Kaposis sarcoma
is not generally associated with significant rates of morbidity or mortality, therapy is
indicated only for extensive, painful, or cosmetically disfiguring lesions.
721. a The treatment of Kawasaki syndrome is directed toward the amelioration of symptoms
Rosen 5th, and the prevention of coronary aneurysms. Gamma-globulin 2 gm/kg intravenously
Chapter 161, should be administered over 12 hours, followed by high-dose aspirin therapy (100
mg/kg/24 hr PO given in divided doses every 6 hours for 14 days). Low-dose aspirin
therapy (3 to 5 mg/kg/24 hr PO) should continue until acute phase reactants return to
normal over 3 to 5 months. A pediatric cardiologist should follow children to monitor
cardiac status.
722. e Exogenous pressor drugs work well in the transplanted heart. Upregulation of Beta-
Rosen 5th, adrenergic receptors appear to occur in the graft, with a slightly increased response to
Chapter 178, norepinephrine and isoproterenol.
p. 2508
234

10.0 Systemic Infectious Disorders


723.A 22-year-old sexually active female complains of dull bilateral lower abdominal pain and dysuria.
She denies fever, chills, nausea or vomiting. She also denies vaginal discharge. Urinalysis
confirms a urinary tract infection. Urine pregnancy test is negative. Her last normal menstrual
period was 2 days ago. Pelvic examination demonstrates no cervical motion tenderness or adnexal
pain on either side. You should still have a high index of suspicion for:
a. appendicitis.
b. an ovarian cyst.
c. ectopic pregnancy.
d. mittelschmerz.
e. a sexually transmitted disease.

724.You are evaluating a 20-year-old man who returns 10 days after being treated for STD. He was
given ceftriaxone 125 mg IM and azithromycin 1 gm orally. He now complains of dysuria and
mild testicular pain. He is afebrile, and has a normal cremasteric reflex and a positive Prehn's sign.
You suspect resistant chlamydia, and confirm your suspicion with:
a. blood culture and CBC.
b. ELISA or DNA probe of urethral secretions
c. testicular ultrasound
d. urethral culture.
e. urine dipstick for nitrates/leukocytes.

725.You are treating a 19-year-old woman for salpingitis. You also tell her that she should:
a. advise her partner to be treated.
b. undergo HIV counseling and testing through her family doctor or a public health clinic.
c. avoid douching, as this has been implicated in salpingitis and cervicitis.
d. not resume normal sexual activity for at least one week.
e. all of the above are true

726.The STD-causing organism which usually produces a characteristic skin lesion is:
a. Chlamydia trachomatis.
b. Gardnerella vaginalis.
c. Neisseria gonorrhoeae.
d. Treponema pallidum.
e. Trichomonas vaginalis.

727.A 23-year-old woman complains of "vaginal pimples. She has not had sex for 3 months. You
examine the patient and suspect genital warts. The best treatment is:
a. acyclovir 400 mg five times daily for two weeks.
b. famciclovir 500 mg three times daily for two weeks.
c. metronidazole 2 grams oral once.
d. oseltamivir 250 mg twice daily for five days.
e. podofilox 0.5% solution or gel.
235

723. e Many patients who present to the Emergency Department complaining of vague
Rosen 5 ,th abdominal pain and/or dysuria are diagnosed with a urinary tract infection, treated and
Chapter 93, discharged. However, more than half of these patients also have positive cultures for a
p. 1394 sexually transmitted disease. Therefore, the clinician must have a high index of
suspicion for diagnosing an STD, especially in a young sexually active patient. Many
of these diagnoses are delayed or missed completely. The patient just had her last
menstrual period 2 days prior, has a negative urine HCG and clinical presentation
inconsistent with an ectopic pregnancy or ovarian cyst.
724. b The clinical examination of the patient appears to indicate a urethritis and possible
Rosen 5th, mild epididymitis. The best lab test would be polymerase chain reaction (PCR) or
Chapter 93, enzyme immunoassay techniques (ELISA). The urinalysis may be helpful, but
p. 1413 oftentimes lacks the sensitivity/specificity to verify this diagnosis. A CBC and blood
culture would not be considered useful in this patient.
725. e Patients who have contracted an STD are at higher risk for acquiring HIV, therefore, it
Rosen 5th, is advisable that they seek counseling and testing for HIV. They should also refrain
Chapter 93, from sexual activity until they are no longer capable of spreading or potentially "ping-
p. 1393 ponging" the disease between themselves and their partner. Their partner should also
be evaluated and treated.
726. d Syphilis, caused by the spirochete, Treponema pallidum usually causes a painless
Rosen 5th, chancre with indurated borders on the shaft of the penis during the primary stage of the
Chapter 93, disease. Chlamydia trachomatis is an intracellular organism and has not been reported
p. 1393 to cause a characteristic lesion. Neisseria gonorrhoeae and Trichomonas vaginalis
also do not cause an obvious, characteristic lesion.
727. e The current recommended treatment for HPV is podofilox 0.5% solution or gel applied
Rosen 5th, to the affected area twice daily for three days, followed by four days of no therapy,
Chapter 93, with the cycle repeated up to four times. Podofilox is similar to podophyllum but
p. 1394 because it is less concentrated, it can be self-applied. Another potential treatment is
imiquimod 5% cream applied three times a week at bedtime for up to 16 weeks. Some
physicians may do cryotherapy in their offices, but not in the ED.
236

728. Dark field microscopy may be helpful in laboratory diagnosis of:


a. crab lice (Pediculosis pubis).
b. genital warts..
c. HSV-II infections.
d. Treponema pallidum.
e. Trichomonas vaginalis.

729. An 18-year-old male complains of severe dysuria and difficulty urinating to the point of modest
urinary retention. He has a several, painful, vesiculopustular lesions on his glans penis. You make
a smear of fluid from one of the lesions and successfully demonstrate large intranuclear inclusions.
Your treatment regimen should include:
a. famcyclovir.
b. metronidazole.
c. nystatin.
d. penicillin G.
e. podofilox.

730. An antibiotic category considered safe for use at any time in pregnancy is the:
a. aminoglycosides.
b. fluoroquinolones.
c. penicillins.
d. sulfonamides.
e. tetracyclines.

731. A 27-year-old woman returns 4 days after having nasal packing placed for a nosebleed. Her oral
temperature is 102.9oF, and blood pressure is 86/48 mmHg. She is still oozing blood from around
the packing, and has a diffuse red rash, especially on her palms and soles. You should treat this
patient with:
a. a glucose-containing intravenous fluid, with 2 ampoules of bicarbonate added.
b. antibiotics, being certain to cover for both streptococcal and staphylococcal infection.
c. high-dose glucocorticoids.
d. platelet transfusions and fresh-frozen plasma.
e. subcutaneous epinephrine and intramuscular diphenhydramine.

732. A 37-year-old man complains of fever and blurred vision. He is HIV positive with a CD4 count of
<50. He probably has:
a. cataracts.
b. central retinal vein occlusion.
c. CMV retinitis.
d. glaucoma.
e. iritis.

733. A 2-week-old baby is visiting with his mother; they are from an underdeveloped section of
Africa. Mother says that the baby cant suckle. The child has a low-grade fever, increased heart
rate, and irritability. The umbilical cord stump looks red and infected. The baby probably has:
a. botulism.
b. meningitis.
c. polio.
d. rabies.
e. tetanus.
237

728. d Dark field (or darkfield) microscopy is used to assist in the laboratory diagnosis of
Rosen 5 ,th syphilis, caused by the spirochete, Treponema pallidum. Trichomonas is diagnosed by
Chapter 93, plain microscopy. The other conditions listed can usually be diagnosed clinically.
p. 1393
729. a This patient appears to have herpes simplex urethritis. The Tzanck cytology test
Rosen 5 ,th demonstrated evidence of a viral infection, most likely HSV-II. The appropriate
Chapter 93, treatment is an antiviral agent, such as acyclovir, famcyclovir, or valicyclovir.
p. 1392
730. c Penicillins as a group are generally believed to be the antibiotics that are safest to use
th during pregnancy.
Rosen 5 ,
Chapter 174, Aminoglycosides gentamicin is category C, streptomycin is category D.
p. 2449 Fluoroquinolones category C, should be avoided during pregnancy, possible
detrimental effects on cartilage and joint development.
Sulfonamides as a class are category B, but trimethoprim and Bactrim are
category C
Tetracyclines category D, should not be used during pregnancy unless no other
alternatives are available.
731. b This patient had a tampon placed in her nares. She now has fever, hypotension and
Rosen 5th, erythroderma, hailing the onset of toxic shock syndrome. She will most likely need
Chapter 131, hemodynamic support in the form of pressors and fluids, and antibiotics to treat both
p. 1947 streptococcal and staphylococcal infections.
732. c CMV retinitis occurs in 10% to 30% of HIV-infected patients and is the most common
Rosen 5th, cause of blindness in patients with AIDS. With advances in HAART, reduced
Chapter 126, incidences of CMV retinitis have been observed, but discontinuation of HAART may
1853 result in intraocular inflammation. CMV retinitis typically produces severe necrotic
vasculitis and retinitis. When present, it may be asymptomatic or cause diminished
visual acuity, photophobia, scotoma, redness, or pain. It is diagnosed by its
characteristic appearance on indirect ophthalmoscopy of fluffy white retinal lesions,
often perivascular. Differential diagnosis includes toxoplasmosis, syphilis, HSV
infection, HZV infection, and TB.
733. e Neonatal tetanus is a form of generalized tetanus that occurs almost exclusively in
Rosen 5th, underdeveloped countries, where maternal immunization is inadequate and
Chapter 123, contaminated material is used to cut and dress the umbilical cords. The incubation
p. 1792 period is short, with symptoms beginning during the first week of life. Early clinical
manifestations include irritability and poor sucking and swallowing.
238

734. Although Clostridium tetani is sensitive to many antibiotics, the antibiotic of choice to treat a
patient suffering from tetanus is:
a. penicillin.
b. linezolid.
c. a third-generation cephalosporin.
d. tetracycline.
e. a third- or fourth-generation fluoroquinolone.

735. In a patient with cephalic tetanus, the cranial nerve most likely to be involved is the:
a. ocular nerve.
b. olfactory nerve.
c. facial nerve.
d. trigeminal nerve.
e. vagus nerve.

736. In the United States, the animal most likely to infect a human with rabies is the:
a. raccoon.
b. squirrel.
c. rat.
d. guinea pig.
e. chipmunk.

737. A 36-year-old camper was the victim of an unprovoked attack by a fox, which bit her several times
and then ran away. She has not received any pre-exposure prophylaxis to rabies, so she now needs:
a. administration of HDVC, scrubbing the wound with virucidal agent, and waiting for the
authorities to determine if the animal in question is rabid before administering HRIG because of
the risk of anaphylaxis.
b. irrigation and scrubbing of the wound only as this is a low-risk exposure.
c. irrigation of the wound with saline and administration of a mixture of HRIG and HDVC into the
area surrounding the wound.
d. irrigation with sterile water and administration of human diploid cell vaccine (HDCV).
e. scrubbing and irrigation with a virucidal agent (e.g., povidone-iodine), followed by
administration of HDVC and human rabies immune globulin (HRIG) at separate anatomic sites.

738. A stray dog bites your hospitals CEO. Her family doctor calls for your advice on rabies treatment,
so you tell him that:
a. bites on the head and neck have a shorter incubation period than those on the extremities.
b. the incubation time in humans cannot exceed 5 years.
c. the usual incubation time from bite to disease is 10 to 14 days.
d. the virus replicates locally and spreads via lymphatic channels.
e. mortality following an inoculation is equal regardless of the bite location.

739. Your CEO wants rabies post-exposure prophylaxis, so you begin the series after counseling her
that:
a. if pregnant, she will probably abort.
b. if she is taking steroids, she should stop during the time she is receiving the shots.
c. rabies antibody levels should be checked at 30 and 90 days after receiving PEP.
d. HRIG is contraindicated if she has an allergy to eggs or horse serum.
e. tetanus booster must be delayed for at least two weeks following HRIG administration.
239

734. b Toxin production is eliminated by treatment of C. tetani infection. Because surgical


th care and antibiotic use can cause a transient release of tetanospasmin, the EP should
Rosen 5 ,
Chapter 123, consider delaying these measures until after the antitoxin has been administered. The
p. 1793 wound should be debrided and cleansed, and foreign bodies should be removed.
Penicillin, tetracycline, erythromycin, and metronidazole are all effective against C.
tetani. Most references recommend penicillin G, 10 to 24 million U per day IV in
divided doses in adults (for pediatrics, use 100,000 U/kg/day in divided doses) for 10
to 14 days. Doxycycline 100 mg IV every 12 hours is an alternative. Metronidazole
(500 mg orally every 6 hours) may have greater efficacy than penicillin.
735. c Cephalic tetanus is manifested by trismus plus cranial nerve palsies. Cephalic tetanus
Rosen 5th, is rare, accounting for 1% to 3% of all tetanus cases. Most of these cases occur after
Chapter 123, facial trauma or otitis media. Patients develop trismus and palsies of cranial nerve III,
p. 1792 IV, VII, IX, X, or XII ipsilateral to the site of local infection. The most commonly
involved cranial nerve is the facial nerve (VII). The clinical course is variable. In one
third of cases, resolution of symptoms is complete. Two thirds of these cases progress
to generalized tetanus.
736. a In the 1960s and 1970s, the majority of wildlife rabies was found in skunks in the
Rosen 5th, United States. However, in the late 1970s, an epizootic among raccoons began in the
Chapter 125, mid-Atlantic states and spread north and south to cover the entire Eastern seaboard;
p. 1834-1836 now the majority of wildlife rabies in the United States is attributable to raccoons. The
source of this epizootic appears to be the inadvertent translocation of rabid raccoons
from the southeastern states to the mid-Atlantic region to stock the area for hunting.
This determination was based on the fact that the raccoons first seen in the mid-
Atlantic states, and now along the entire east coast, carry the same antigenic variant as
those in the southeastern states. Although the total number of rabid raccoons appears
to have peaked, the population continues to spread geographically north and westward.
737. e Prophylaxis consists of three steps: wound care, passive immunization, and active
Rosen 5th, immunization. No step in this treatment should be omitted. Rabies is easily killed by
Chapter 125 sunlight, soap, or drying, and experimental studies have shown that scrubbing and
p.1839 flushing the wound with benzalkonium chloride, 20% soap solution, or Ivory soap was
nearly 100% protective when performed within 3 hours of inoculation of virus. Rabies
immunoprophylaxis requires both passive immunization with antibody (immune
globulin) and active immunization with vaccine.
738. a Although the incubation time for rabies is generally from 30 to 90 days, it has been
Rosen 5th, documented to take as long as 7 years. The virus replicates locally and travels along
Chapter 125 peripheral nerves at a rate between 8 and 20 mm/day. For this reason, head and neck
p.1837 bites have a shorter incubation period and higher mortality (with treatment) compared
to lower extremity bites. The risk of developing rabies is between 5% and 80%,
depending on the animal, the size of the bite/severity of exposure, and location on the
body.
739. b Prophylaxis, including both passive and active immunization, given during pregnancy
Rosen 5th, does not result in an increase in fetal wastage, congenital defects, or side effects and
Chapter 125, should not be withheld when indicated. Corticosteroids, antimalarials, and other
p. 1839 immunosuppressives can interfere with the development of active immunity and
should be withheld during the course of treatment if possible. Patients with
immunosuppressive illnesses should be monitored for antibody response.
240

740. Treatment of choice for a patient with falciparum malarium is:


a. chloroquine.
b. pyrimethamine-sulfadoxine.
c. quinine plus doxycycline.
d. rapid-infusion intravenous quinine.
e. primaquine.

741. Acute Plasmodium vivax can cause:


a. disseminated intravascular coagulation.
b. headache.
c. hypoglycemia.
d. pulmonary edema.
e. renal failure.

742. The best diagnostic test to support the diagnosis of malaria is:
a. abnormally low sedimentation rate.
b. complete blood count revealing microcytosis, schistocytes, and rouleaux.
c. microscopic urinalysis showing trophozoites.
d. peripheral blood smear showing parasites.
e. polymerase-chain reaction (PCR) DNA testing.

743. A 24-year-old entomologist was camping in New Hampshire a week ago and pulled off some deer
ticks (Ixodes dammini), so he took 200mg of oral doxycycline to prevent Lyme disease. Now he
complains of malaise, high fevers, headache, and dark urine. On his physical exam you see no rash,
but find hepatosplenomegaly. His CBC shows pancytopenia, and his urine looks quite dark. You
should now treat him with:
a. 3rd or 4th generation fluoroquinolone.
b. clindamycin and quinine.
c. one month of intravenous ceftriaxone.
d. two weeks of amoxicillin / clavulanate.
e. two weeks of doxycycline.

744. A pathognomonic finding for Chagas disease (Trypanosoma cruzi) is:


a. Kerley C lines on chest x-ray.
b. Romaas sign, painless unilateral periorbital edema.
c. spontaneous rhabdomyolysis and hematuria.
d. trifascicular heart block without cardiomegaly.
e. urine which turns green when left in a cool dark room.

745. The best way to prevent hookworm infection is to:


a. avoid raw fish.
b. avoid swimming in brackish water.
c. cook wild game thoroughly before eating.
d. wash hands frequently.
e. wear shoes.
241

740. c In the past, chloroquine phosphate was the treatment of choice for acute,
Rosen 5 ,th uncomplicated attacks of malaria. Resistance to chloroquine has been steadily
Chapter 127, increasing, and now the drug should be used only in regions of known chloroquine
p. 1869 sensitivity. Quinine and doxycycline given together are the drugs currently
recommended for the treatment of falciparum malaria. Quinine can be given orally or
parenterally for more serious and life-threatening infections. When given
intravenously, too rapid infusion of quinine can cause profound hypoglycemia.
741. b Irregular fevers are the hallmark of malaria. Other symptoms may include anemia,
Rosen 5th, headache, nausea, chills, lethargy, abdominal pain, and upper respiratory complaints.
Chapter 127 Acute falciparum infection can have the following complications: cerebral malaria,
p.1869 hypoglycemia (especially in children), metabolic acidosis, severe anemia, renal failure,
pulmonary edema, disseminated intravascular coagulation (DIC), and death.
742. d Thick and thin blood films are the gold standard for the diagnosis of malaria. Viewing
Rosen 5th, several slides may be necessary if the parasite burden is not overwhelming. Giemsa or
Chapter 127 Wrights stains are both adequate for this purpose when used with ordinary light
p.1869 microscopy. The diagnosis often can be made in a simply equipped laboratory. Even
if the parasite is not visualized, the physician should still treat for malaria if clinically
suspected.
743. b Babesiosis is a malaria-like illness that is becoming increasingly prevalent in the
Rosen 5th, northeastern United States (Babesia microti), the northwestern United States (B.
Chapter 127 gibsoni), and Europe (B. divergens). The organism is a protozoan similar in structure
p.1869 and life cycle to the plasmodia. It is transmitted by the deer tick, Ixodes dammini, the
vector of Lyme disease. Patients develop fatigue, anorexia, malaise, myalgia, chills,
high spiking fevers, sweats, headache, emotional lability, and dark urine. They have
hepatosplenomegaly, anemia, thrombocytopenia, leukopenia, elevated liver enzyme
levels, and signs of hemolysis with hyperbilirubinemia and decreased haptoglobin.
The treatment of choice is quinine and clindamycin.
744. b The vector for Chagas disease is the reduviid, or kissing bug, which inhabits the
Rosen 5th, walls and roofs of thatched dwellings built adjacent to forest in South America. The
Chapter 127 reduviid bites the patient, often in the periorbital region, and excretes feces containing
p.1874 the trypomastigote of T. cruzi. The trypanosome enters the inflamed bite wound or
other mucosal or conjunctival surfaces, causing local swelling, called a chagoma.
Romaas sign, painless unilateral periorbital edema, is pathognomonic but rarely seen.
745. e Hookworm infection has been recognized as a major cause of anemia worldwide. The
Rosen 5th, larvae penetrate human skin, usually through the feet. Adult worms penetrate into
Chapter 127 intestinal mucosa and feed, causing significant ongoing luminal blood loss. Eggs
p.1871 defecated in the soil mature through a rhabditiform larval form to the infective
filariform larva.
242

746.A 39 year-old woman complains of a severe headache, high fever, stiff neck, and photophobia. She
appears toxic and cannot touch her chin to her chest. Her left arm also drifts downward when held
at full extension. You must now:
a. start antibiotic, do spinal tap, do head CT.
b. do spinal tap, start antibiotic if positive, do head CT.
c. start antibiotic, do head CT, spinal tap if CT negative.
d. start antibiotic, do head CT; dont bother with spinal tap since already treating.
e. do head CT, do spinal tap, start antibiotic if indicated.

747.Babesiosis is a:
a. bacterium spread by the bite of a flea.
b. fungus spread by the bite of a flea.
c. protozoa spread by the bite of a tick.
d. spirochete spread by the bite of a rat.
e. virus spread by the bite of a tick.

748.In a patient with Babesiosis, the pathognomonic finding is:


a. abnormal hemoglobin-hematocrit ratio (>>1:3).
b. green urine which fluoresces with Woods lamp.
c. intraerythrocytic parasite on blood smear ('Maltese cross' formation).
d. oval liver-abscess on ultrasound.
e. spirochetal invasion of platelets.

749.The hallmark of a patient with Cat Scratch Disease is:


a. anorexia, emesis, weight loss, thinning of hair.
b. chronic tender lymphadenopathy.
c. headache, lace-like rash on chest and abdomen.
d. malaise or fatigue with splenomegaly.
e. severe pharyngitis with normal-looking throat and negative cultures.

750.An immunocompetent patient with Cat Scratch Disease can be treated with:
a. outpatient intravenous antivirals.
b. inpatient intramuscular antibiotics.
c. oral antibiotics.
d. oral antivirals.
e. watchful waiting.

751.A 43-year-old woman has uncontrollable watery diarrhea. You ask her about food intake and recent
travel in an effort to determine the correct etiology, such as:
a. giardia from home-canned vegetables.
b. salmonella from backpacking and stream water.
c. scombroid poisoning from red snapper.
d. staphylococcus from potato salad.
e. Vibrio parahemolyticus from fried rice.
243

746. c Patients with a markedly depressed sensorium that precludes careful neurologic
th
Rosen 5 , examination, or those with a focal neurologic deficit, papilledema, seizures, or
Chapter 103, evidence of head trauma must be considered to be at risk for a herniation syndrome
p. 1532 that may be exacerbated by an LP. If the presentation is an acute, fulminating, febrile
illness and bacterial meningitis is the concerning diagnosis, early initiation of
antimicrobial therapy is mandatory because of the association of prognosis and time to
treatment. The algorithmic alternatives are therefore (1) immediate LP, followed by
initiation of antibiotic treatment before obtaining the results or (2) initiation of
antibiotic treatment, followed by a cranial CT scan and then an LP. This latter choice
of empiric treatment with antibiotics is now the routine in many institutions. This
reflects the efficacy of current methodologies of means other than bacteriologic
cultures.
747. c Babesiosis is a malaria-like disease with the etiologic agents being protozoan parasites:
Rosen 5th, Babesia microti and Babesia equi. The major zoonotic reservoirs are domesticated
Chapter 127, mammals, rodents, and deer. Ixodes ticks functions as the principal vector. Blood
p. 1869 transfusions have been implicated in the transmission of babesiosis. Laboratory tests
often show evidence of hemolysis, liver dysfunction, anemia, thrombocytopenia, and
748.c renal failure. Approximately 20% of the patients with babesiosis have a concurrent
Rosen 5th, infection with Lyme disease. Diagnosis is made by finding intraerythrocytic ring
Chapter 127, forms on a Giemsa-stained peripheral blood smear, though false-negative results can
p. 1869 occur when the level of parasitism is low.
749. b Cat scratch disease most often occurs in young (80% less than 21 years of age)
immunocompetent hosts. The initial site develops a transient erythematous papule or
750. e pustule for which the patients do not often seek medical attention. They usually come
Rosen 5th, after the development of persistent regional lymphadenopathy in an area of the body
Chapter 54, draining lymph from the scratch or bite, usually 7 to 12 days after the injury.
p.777 Symptomatic lymphadenopathy usually resolves spontaneously in 2 to 4 months. The
bacterium is sensitive to many antibiotics.
751. d Staphylococcal-related food poisoning is caused by the multiplication of an
Rosen 5th, enterotoxin-forming strain of Staphylococcus organisms in the food before ingestion.
Chapter 89 Contamination of food with Staphylococcus organisms is extremely common because
p.1308 the organism can be grown from the hands of approximately 50% of persons. Most
protein-rich foods will support the growth of staphylococci, especially ham, eggs (even
hard boiled), custard-filled pastries, mayonnaise, and potato salad.
244

752. Two weeks ago, a 65 year-old woman cut her hand while gardening. She vigorously washed the
wound and observed for infection. Today she complains of a painful arm and jaw. She has no
fever, but practically bites the thermometer in half when the hospital fire alarm rings. You know
that:
a. her condition is caused by an anaerobic, gram-positive bacillus, which is nonencapsulated and
forms spores.
b. the mortality rate from this disease is more than 50% in the United States.
c. the spatula test will be negative.
d. patients presenting early after exposure to the bacterium have a better prognosis.
e. broad-spectrum antibiotics are the hallmark of treatment.

753. Choose the true statement about cellulitis.


a. Blood cultures are usually positive.
b. Dicloxacillin is a good antibiotic choice for treatment.
c. Temperature is usually elevated and there is a left shift in the white blood cell count.
d. The most common sites are face, then upper extremities, then lower extremities.
e. The patient usually remembers a break in the skin.

754. Impetigo is the most common skin infection seen in the emergency department.
a. The prevalence is greatest in homeless, alcoholics, and others with poor immune systems.
b. Fever and malaise are usually present.
c. Regional lymphadenitis is present in virtually every case.
d. Treatment is combined oral and topical antibiotic therapy.
e. In bullous impetigo, Nikolskys sign is negative.

755. A six-year-old non-immunized boy has fever, sore throat, difficulty swallowing, and malaise. His
throat exam shows an adherent gray pseudo-membrane. His treatment must include antitoxin
therapy plus:
a. active immunization.
b. antiviral therapy.
c. active immunization and antibiotics.
d. passive immunization and an antiviral.
e. active immunization and steroids.

756. A 42-year-old immigrant laborer stepped on a nail protruding from a 2 x 4 while he was helping
tear down an old barn. He is not certain whether he has ever had a tetanus shot. He should get:
a. tetanus toxoid only, as this is not a tetanus prone wound.
b. tetanus toxoid and tetanus immunoglobulin initially, with referral for rest of immunization.
c. tetanus toxoid and tetanus immunoglobulin, even if he had been immunized in last 5 years, since
this is a tetanus-prone wound.
d. tetanus toxoid now, withhold tetanus immune globulin until he can check his immunization
status; otherwise, TIG given too soon after prior immunization greatly increases the probability of
adverse reactions.
e. tetanus immunoglobulin, with second dose in 24 hours because of short half-life.

757. The most common CNS complication of pertussis is:


a. encephalitis.
b. encephalopathy.
c. intracerebral hemorrhage.
d. seizure.
e. vasculitis and stroke.
245

752. a Trismus is the presenting symptom in most patients with tetanus and is caused by
th
Rosen 5 , increased masseter muscle tone. Patients may complain of lockjaw and present to a
Chapter 123, dentist or oral surgeon. As the other facial muscles become involved, the characteristic
p. 1791 sardonic smile (risus sardonicus) appears. Other early symptoms include irritability,
weakness, myalgia, muscle cramps, dysphagia, hydrophobia, and drooling. The muscle
rigidity increases as the disease progresses. The time from an initial symptom to the
first muscle spasm is called the onset period. A shorter onset period is predictive of a
poorer prognosis. In the most severe form of tetanus, muscle rigidity becomes
generalized and reflex muscle spasms may be precipitated by external stimuli (noise,
light, touch) or occur spontaneously.
753. b In addition to being caused by trauma or breaks in the skin layer, cellulitis may also be
Rosen 5th, secondary to hematogenous and lymphatic dissemination. Fever is not commonly seen
Chapter 131, in cellulitis. Without systemic involvement WBC counts are normal or only slightly
p. 1944-1945 elevated without left shift. Blood cultures are rarely positive. Staphylococcus and
streptococcus cause cellulitis most commonly on the lower extremities, then the upper
extremities, and 3rd the face.
754. e Impetigo is the most common skin infection seen in the emergency department. The
Rosen 5th, prevalence is greatest in young children, particularly those under the age of 6 years.
Chapter 131, Impetigo may occur sporadically or, occasionally, in epidemics. There are no
p. 1949 associated systemic manifestations such as fever or malaise. Regional lymph nodes
may be minimally enlarged. The treatment of impetigo is oral antibiotic therapy or an
appropriate topical antibiotic for limited eruptions. A first generation oral
cephalosporin or erythromycin provides effective oral therapy. Mupirocin is the only
topical agent with proven efficacy. Combination topical and systemic therapy is
unnecessary.
755. c Equine serum diphtheria antitoxin should be administered promptly after the clinical
Rosen 5th, diagnosis of respiratory diphtheria. Antitoxin can be obtained by contacting the CDC.
Chapter 123, The size and location of the membrane, the duration of illness, and the patients overall
p. 1787 degree of toxicity determine the dosage of antitoxin. The Committee on Infectious
Diseases of the American Academy of Pediatrics (AAP) recommends 20,000 to
40,000 U for pharyngeal or laryngeal involvement of 48 hours duration, 40,000 to
60,000 U for nasopharyngeal lesions, and 80,000 to 100,000 U for extensive disease of
3 or more days duration or for diffuse swelling of the neck. In addition to antitoxin
therapy, antibiotics should be administered for 14 days. Erythromycin, 40 to 50
mg/kg/day (up to 2 g) IV or orally in divided doses; intramuscular (IM) aqueous
crystalline penicillin, 100,000 to 150,000 U/kg/day in four divided doses; or procaine
penicillin, 25,000 to 50,000 U/kg/day in two divided doses for 14 days given IM every
12 hours is acceptable.
756. b Patient with no known history of tetanus immunization should be given tetanus
Rosen 5th, immunoglobulin and tetanus toxoid in two different locations in body in ED and
Chapter 123, referred to receive the rest of the immunization. Any patient with known history of
p. 1793 adequate immunization within last 5 years does not need to have tetanus toxoid or
immunoglobulin even if the wound is contaminated. Tetanus immunoglobulin (TIG)
does not need to be repeated since the half-life of TIG is 25 days.
757. d CNS complications of pertussis include seizures (3%), encephalopathy (0.9%), and
Rosen 5th, intracerebral hemorrhage. The mechanisms may include hypoxia, a toxin, or
Chapter 123 secondary infection by neurotropic viruses. CNS hemorrhages may occur as a
p.1789 consequence of the increased cerebral vascular pressures generated during the
paroxysm of coughing and whooping.
246

758. In a patient with pertussis, the severity and duration of cough can be lessened by using:
a. a macrolide antibiotic.
b. non-narcotic cough suppressants.
c. narcotic-based cough suppressants.
d. pertussis immune globulin.
e. antihistamines.

759. A 27-year-old woman complains of weakness, lightheadedness, dry mouth, difficulty speaking, and
double vision. She was at a family gathering yesterday where her aunt had served some home-
canned foods. You suspect botulism, knowing that:
a. distal muscles are weaker than proximal muscles.
b. lower extremity muscles more affected than those of the upper extremity.
c. ocular signs include ptosis, extraocular palsies, and pinpoint pupils.
d. the gag reflex should be normal.
e. the patient with botulism is usually alert and afebrile.

760. Treatment of the patient in Question #759 should include:


a. antibiotics to prevent further toxin release.
b. antitoxin to neutralize both circulating and bound toxin.
c. intubation if forced vital capacity goes to less than 50% of predicted.
d. magnesium-containing cathartics.
e. saline enemas to cleanse the GI tract of residual toxin.

761. Clinical criteria for sepsis include:


a. heart rate >110 beats/minute.
b. more than 20% immature (band) forms of WBCs.
c. respiratory rate >24 breaths/minute.
d. temperature >38C (100.4oF) or <36C (96.8oF)
e. WBC count greater than 15,000/mm3.

762. Patients with asplenia (post-surgical) or inadequate splenic function (as occurs in sickle cell
disease) may develop a fulminant type of pneumococcemia termed overwhelming post-
splenectomy infection (OPSI), which is characterized by
a. meningitis, DIC, and hemorrhagic pancreatitis.
b. pituitary infarct, hemorrhagic thyroiditis, and asterixis.
c. septic shock, adrenal hemorrhage, and DIC.
d. septic shock, coronary artery occlusion, and fulminant hepatitis.
e. shock, rotatory nystagmus, and hemorrhagic hepatitis.

763. You are seeing a 16-year-old girl with flu-like symptoms. She is about 14 hours into the illness.
Your rapid screening test shows she has influenza type B. Her parents are insistent that you have to
give her something. The only medicine which might help this illness and has been approved for
this age group is:
a. amantadine.
b. famcyclovir.
c. oseltamivir.
d. rimantadine.
e. zanamivir.
247

758. d Antibiotics are indicated but may not alter the severity or duration of illness when
th
Rosen 5 , initiated after the catarrhal phase of illness. Corticosteroids, especially in infants, may
Chapter 123 reduce the severity and course of illness, and beta2-adrenergic agonists may reduce the
p.1789-1790 frequency and severity of the paroxysmal coughing episodes. Early clinical trials with
pertussis immune globulin have shown significant improvement in lymphocytosis and
paroxysmal coughing in infants. This may be a useful treatment for very ill patients in
the future. Standard cough suppressants and antihistamines are ineffective.
759. e The patient with botulism is usually alert and is afebrile unless secondary infection is
Rosen 5th, present. Postural hypotension may be present. Ocular signs are prominent and include
Chapter 123 ptosis, extraocular palsies, and markedly dilated and fixed pupils. The absence of
p.1796 ocular abnormalities does not exclude the diagnosis. Dryness of the mouth, tongue,
and pharynx leads to a red, dry appearance of the mucous membranes. The gag reflex
is depressed or absent. Muscle weakness is usually present but varies from mild to
severe. Neck muscles are often weak. Upper extremity muscles are more affected
than those of the lower extremity. Proximal muscles are weaker than distal muscles.
Deep tendon reflexes may be normal, symmetrically decreased, or absent. Sensory
examination is normal.
760. e A decrease in vital capacity to less than 30% of predicted is a standard criterion
Rosen 5th, mandating mechanical ventilation. Saline enemas and cathartics have been
Chapter 123 recommended to cleanse the GI tract of residual toxin. Magnesium-containing
p.1797 cathartics should be avoided because magnesium can exacerbate muscle weakness.
Antitoxin neutralizes only circulating toxin and has no effect on bound toxin.
Antibiotics are not currently recommended for food-borne botulism, as they may
increase cell lysis and promote toxin release.
761. d Sepsis is the systemic response to infection, manifested by two or more of the
Rosen 5th, following: (1) temperature greater than 38C or less than 36C; (2) heart rate >90
Chapter 123 beats/min; (3) respiratory rate >20 breaths/min or PACO2 less than 32 mm Hg; and (4)
p.1798 WBC count greater than 12,000/mm3, less than 4000/mm3, or greater than 10%
immature (band) forms.
762. c Patients with asplenia (post-surgical) or inadequate splenic function (as occurs in sickle
Rosen 5th, cell disease) may develop a fulminant type of pneumococcemia termed overwhelming
Chapter 123 post-splenectomy infection (OPSI), which is characterized by septic shock, adrenal
p.1799 hemorrhage, and DIC.
763. e Amantadine and rimantadine are effective in preventing and treating influenza A, but
th
Rosen 5 , have no activity against influenza B. Zanamivir and oseltamivir act by inhibiting the
Chapter 124 activity of neuraminidase, an enzyme involved in the release of viral progeny from
p.1813, 1817 infected cells. Influenza strains are identified by their hemagglutin and neurominidase
composition (e.g., H1N3, H2N4, etc.) Zanamivir is approved for the treatment of
influenza A and B, administered by inhalation, and is approved for use in patients older
than 12 years of age. Oseltamivir is approved for the treatment of influenza A and B,
an oral medication approved for patients older than 18.
248

764. Virtually all patients with meningococcemia have:


a. fever.
b. meningitis.
c. petechiae.
d. seizures.
e. Waterhouse-Friderichsen syndrome.

765. The most common exanthem of immunized children less than 2 years of age is:
a. erythema infectiosum.
b. roseola.
c. rubella.
d. rubeola.
e. varicella.

766. Parvovirus:
a. has been implicated in aplastic crisis in children with sickle cell disease.
b. has been implicated in hemolytic crisis in children with sickle cell disease.
c. has been implicated in sequestration crisis in children with sickle cell disease.
d. has been implicated in vaso-occlusive crisis in children with sickle cell disease.
e. has been implicated in equestrian crisis in children with sickle cell disease.

767. The correct match is:


a. varicella = first disease
b. rubella = third disease
c. scarlet fever = fourth disease
d. exanthem subitum = fifth disease.
e. erythema infectiosum = sixth disease

768. Flaviviruses, such as the West Nile virus, are spread to humans through the vector of:
a. bats.
b. fleas.
c. mites.
d. mosquitoes.
e. ticks.

769. Mycobacterium tuberculosis:


a. affects almost one-fifth of the worlds population.
b. can be found in greatest abundance in immigrants from Africa and Eastern Asia.
c. has been on the decline since emergence of better antibiotics in the 1980s.
d. is especially problematic in nursing home patients, as these patients are a primary reservoir for
disease.
e. is the second leading infectious cause of death worldwide.

770. Choose the true statement:


a. Hemoptysis in tuberculosis occurs early and is usually copious.
b. In children with pulmonary tuberculosis, the most common radiographic findings are hilar
adenopathy, mediastinal lymphadenopathy, and consolidated pneumonia.
c. Fewer than 5% of patients with tuberculosis have extrapulmonary manifestations.
d. Night sweats and weight loss are the most common symptoms of pulmonary tuberculosis.
e. Shortness of breath is common and is not predictive of severity.
249

764. a An elevated temperature is present in 71% to 89% of cases of meningococcemia, and


th
Rosen 5 , hypothermia is present in 4% of cases. Although skin lesions are present in 71% of
Chapter 123, cases, petechiae, or purpura, are present in only 49% of cases. Shock occurs in 42%
p. 1801-1802 of patients, arthritis in 8%, and seizures in 8%. The Waterhouse-Friderichsen
syndrome occurs in 10% to 20% of patients with meningococcal disease and is
characterized by extreme severity and rapid clinical deterioration, including
vasomotor collapse and shock.
765. b Roseola infantum is the most common exanthem of children younger than 2 years of
Rosen 5th, age and occurs most often at about 1 year of age. The illness begins abruptly with the
Chapter 124, acute onset of fever, often as high as 41C, lasting 3 to 5 days. Despite the fever, the
p. 1822 child usually remains active and alert.
766. a Parvovirus B19 has a propensity for attacking and causing a marked reduction in
Rosen 5th, erythroid cell precursors and has been established as a cause of transient aplastic crisis
Chapter 124, in patients with chronic hemolytic anemia. Recovery is associated with reappearance
p. 1823 of reticulocytes in the peripheral smear 7 to 10 days after their disappearance. Rare
cases of encephalitis and pneumonia have been reported.
767. b The rash associated with rubella (third disease) is one of the classic common
Rosen 5th, exanthems of childhood. It may be similar to the rash of measles (rubeola, or first
Chapter 124, disease), scarlet fever (second disease), a variant of scarlet fever or toxin-producing
p. 1825 staphylococcal disease (fourth disease), erythema infectiosum (fifth disease), and
roseola (exanthem subitum, or sixth disease).
768. d More than 60 flaviviruses have been identified, with more than 20 causing human
Rosen 5th, disease. Three of the most common, all transmitted to humans via a mosquito vector,
Chapter 124, are the agents of yellow fever, dengue, and St. Louis encephalitis. The West Nile virus
p. 1825 has recently been reported in numerous areas of the United States, including Louisiana
and the East Coast.
769. d Tuberculosis is the world's leading cause of infectious death. Over one third of the
Rosen 5th, world's population is infected, and each year over 3 million people die from the
Chapter 129, disease. Currently in the United States, elderly people with dormant infections are the
p. 1903 main reservoir of TB reactivation. The largest numbers of people with TB originate
from Mexico, the Philippines, and Vietnam.
770. b Cough is the most common symptom of pulmonary tuberculosis. Hemoptysis is
Rosen 5th, usually minor but often indicates extensive involvement. Shortness of breath from
Chapter 129, parenchymal lung involvement is unusual, and if present, indicates extensive
p. 1906 parenchymal disease or tracheobronchial obstruction. While the majority of cases of
Table 129-1 tuberculosis are pulmonary, up to 15% of cases will have extrapulmonary
manifestations. In children with TB, The most common radiographic findings include
hilar adenopathy, mediastinal lymphadenopathy, or consolidated pneumonia.
250

771. Multi-Drug Resistant Tuberculosis (MDRTB) is defined as resistance to:


a. ethambutol.
b. rifampin.
c. streptomycin.
d. isoniazid.
e. two or more first-line anti-tuberculosis drugs.

772. The most common complication of cavitary pulmonary tuberculosis is:


a. bacteremia.
b. empyema.
c. endobronchial spread.
d. pericardial effusion.
e. pneumothorax.

773. Risks for acquiring TB may also be stratified by age. We know that:
a. infants and toddlers have a much lower incidence of TB than adults.
b. children 5 to 10 years of age are relatively resistant to TB.
c. infants and toddlers only rarely have extrapulmonary disease
d. young adults rarely develop cavitary lesions
e. the elderly are most likely to manifest classic disease of cough, hemoptysis, and cavitary
lesions.

774. Concerning the use of anti-tuberculosis drugs, you know that:


a. ethambutol is the preferred treatment in small children and infants,
b. fluoroquinolones are becoming first-line agents.
c. oral streptomycin is a second-line agent.
d. pyrazinamide works in the acid environment of the macrophage.
e. rifampin causes blue-green discoloration of blood, sweat and tears.

775. Hantavirus infections are usually spread by:


a. being bitten by an infected flea.
b. being bitten by an infected mosquito.
c. being bitten by an infected tick.
d. drinking contaminated water.
e. inhaling aerosolized contaminated mouse droppings.

776. Patients with the hantavirus pulmonary syndrome:


a. are treated with chloramphenicol and ampicillin.
b. can develop a paradoxical metabolic alkalosis.
c. develop bradycardia as an early sign of systemic illness.
d. have a 50 to 70% mortality rate.
e. have an annoying brassy paroxysmal cough.

777. The most common source of infection in the septic patient is the:
a. central nervous system.
b. gastrointestinal system.
c. integumentary system.
d. respiratory system.
e. urogenital system.
251

771. b Multi-drug resistance tuberculosis is defined as resistance to two or more first-line


th
Rosen 5 , antituberculous agents. Multidrug resistance may occur when a single drug is added to
Chapter 129, a failing regimen, an intervention equivalent to giving monotherapy.
p. 1919
772. c Endobronchial spread is the most common complication of cavitary disease. It is seen
th
Rosen 5 , on x-ray examination as 5- to 10-mm poorly defined nodules clustered in dependent
Chapter 129, portions of the lungs. Empyema is rare (1% to 4%) and is more common in patients
p. 1907 with extensive, progressive parenchymal disease. Pericarditis may result from the
anatomic relationship of the mediastinal lymph nodes to the pericardial sac. Rupture
of lymph nodes into the pericardium may lead to pericardial tamponade.
Pneumothorax is seen in less than 5% of patients.
773. b Risks for acquiring TB may also be stratified by age. Infants and toddlers have poorly
Rosen 5th, developed cell-mediated immunity, so they have a much higher incidence of TB than
Chapter 129, adults. Children 5 to 10 years of age are relatively resistant to TB. Infants and
p. 1906 toddlers commonly have extrapulmonary disease and acute lower and midlung
bronchopneumonia that rarely progresses to cavitary disease. Young adults show the
adult pattern of apical pulmonary disease, including cavity formation, suggesting
reactivation. Because of decreased immunocompetence, the elderly tend to manifest
the disease similar to young children.
774. d Rifampin causes orange discoloration of urine, tears, and sweat. Pyrazinamide works
Rosen 5th, in the acid environment of the macrophage. Visual acuity and red-green color
Chapter 129 perception should be measured when using ethambutol. Because of the difficulty in
p.1917 visual testing in small children and infants, ethambutol should be avoided in these
populations. Streptomycin must be given parenterally. Fluoroquinolones have a
limited role; they are less effective than first-line agents and are used mainly in the
treatment of drug-resistant disease. If used singly, resistance may quickly develop.
775. e Hantaviruses cause a hemorrhagic fever with renal syndrome. Rodents carry the
agents, and the virus is transmitted via aerosols infected from rodent urine. More than
776. d 100,000 cases occur in Asia and Europe, and the mortality rate is approximately 6%.
Rosen 5th, In 1994, a previously unknown hantavirus was found to cause a pulmonary syndrome
Chapter 124, associated with tachypnea, hemoconcentration, thrombocytopenia, and leukocytosis.
p. 1829 Cases occurred predominantly in the southwestern United States, were associated with
a mortality rate higher than 50%, and were believed to have been transmitted from the
deer mouse, Peromyscus maniculatus. The virus has been named the Muerto Canyon
or Sin Nombre virus.
777. d The respiratory system is the most common focus of infection in the septic patient.
Rosen 5th, History of a productive cough, fevers, chills, upper respiratory symptoms, and throat
Chapter 132, and ear pain should be elicited. Examination should include detailed evaluation,
p. 1961 looking for focal infection such as exudative tonsillitis, sinus tenderness, tympanic
membrane injection, and crackles or dullness on lung auscultation. Also, pharyngeal
thrush should be noticed as a marker of an immunocompromised state.
252

778. You have just intubated a 70-year-old man who presented with fever, cough, confusion, and
respiratory failure. Even with a non-rebreather mask you were unable to get his oxygen saturation
above 85%. Your next priority should be to:
a. keep his hemoglobin above 10 g/dl.
b. keep his oxygen saturation above 96%.
c. keep his positive end-expiratory pressure to less than 8 cm of water.
d. keep his systolic blood pressure below 170 mm hg.
e. keep his transalveolar pressures (plateau pressures) below 35 cm of water.

779. Pseudomonas bacteria are responsible for bone and joint infections in three main settings. The first
is in puncture wounds to the foot. The other two are:
a. children with cystic fibrosis and alcoholics with peri-articular puncture wounds.
b. diabetics with eye prostheses and patients with Gram-negative toxic shock syndrome.
c. patients with implanted orthopedic devices and IV drug users, who may develop hematogenous
osteomyelitis, often in the spine.
d. pediatric patients with glycogen storage deficiency disease and under-treated patients with
Pseudomonas endocarditis.
e. sickle cell patients with avascular necrosis and AIDS patients with long-time steroid use for PCP
treatment.

780. Bone scanning is a very useful test in diagnosing osteomyelitis. The radionuclide with the highest
sensitivity, approaching 90%, is:
a. 111In.
b. 127Ne.
c. 23SkDu.
d. 67Ga.
e. 99mTc MDP.
253

778. e There are no specific recommendations in the acute setting of trying to lower blood
th
Rosen 5 , pressure below 170 mm Hg. Positive end-expiratory pressure is an effective way to
Chapter 132, increase arterial oxygen delivery. This pressure may need to be increased well above 8
p. 1962-1963 cm H2O in the acute setting. Most experts believe that transfusion is only warranted
once the hemoglobin drops below 8 g/dL. Keeping plateau pressures below 35 cm
H2O has been shown to reduce the incidence of ventilator-induced lung injury.
779. c Pseudomonas bacteria are responsible for bone and joint infections in three main
Rosen 5th, settings. The first is in puncture wounds to the foot. Pseudomonas does not appear to
Chapter 130, grow on puncture objects, but rather is intimately associated with shoe gear that may
p. 1928 be inoculated into the wound and produce soft-tissue infection and osteomyelitis.
Patients in whom prosthetic devices are implanted during orthopedic surgery are at risk
for Pseudomonas bone and joint infection. IV drug users may develop hematogenous
osteomyelitis, often in the spine, from Pseudomonas bacteria.
780. e The technetium methylene diphosphonate (99mTc MDP) scan is a sensitive test for
Rosen 5th, osteomyelitis in patients who have no existing bone abnormalities. Most series report
Chapter 130, a sensitivity of greater than 90% with the three-phase scan. Additional testing with
p. 1930 gallium citrate 67Ga and indium oxine. 111In is often used to compensate for the limited
specificity of the technetium-99m scans.
254

11.0 Musculoskeletal Disorders (Nontraumatic)


781. Normal compartment pressures are:
a. near 0 mmHg
b. 10-20 mmHg
c. 20-40 mmHg
d. 40-60 mmHg
e. 60-80 mmHg

782. Carpal tunnel syndrome is characterized by:


a. improvement with repetitive motion.
b. negative Tinel sign.
c. positive Finkelsteins test.
d. positive Phalens test.
e. radial nerve palsies.

783. Which cervical nerve root innervates the thumb?


a. C2
b. C4
c. C6
d. C8
e. T2

784. The small finger is innervated by the:


a. palmar cutaneous branch of median nerve.
b. dorsal cutaneous branch of ulnar nerve.
c. median nerve.
d. radial nerve.
e. ulnar nerve.

785. To test the motor branch of the ulnar nerve, ask the patient to
a. abduct the fingers.
b. extend the fingers.
c. extend the wrist.
d. flex the fingers.
e. flex the wrist.

786. A boutonnire or swan neck deformity


a. always involves a fracture.
b. present with DIP flexion.
c. is the same as a mallet finger.
d. presents with PIP flexion and DIP extension.
e. requires emergent surgical repair.
255

781. a Normal compartment pressure is 0 mm Hg. Blood flow to the microcirculation is


th
Rosen 5 , generally impaired when tissue pressures reach 30 mm Hg or more; however, some
Chapter 42, patients can tolerate much higher compartment pressures without compartment
p. 479 syndrome developing. Controversy exists over attempts to define compartment
syndromes on the basis of specific tissue pressure. The tolerance to tissue ischemia
varies among individuals because of shock, compensatory hypertension, altered tone in
resistance vessels, and other unknown factors. Inadequate perfusion and relative
ischemia begin when the tissue pressure within a closed compartment rises to within
10 to 30 mm Hg of a patients diastolic pressure.
782. d The most sensitive provocative test to diagnose carpal tunnel syndrome is the wrist
Rosen 5th, flexion test, or Phalens test (sensitivity 76%, specificity 80%). The patient fully
Chapter 44, flexes the wrists for 60 seconds while the forearms are held in a vertical position. The
p. 548-549 test is positive if paresthesias or numbness develops in the median nerve distribution.
Another test, Tinels sign, is positive if light tapping or percussion over the median
nerve at the wrist produces pain or paresthesias in the median nerve distribution.
Finkelstein's test is positive in de Quervains tendonitis.
783. c C2 Occiput C7 Index finger L2L3 Medial thigh
Rosen 5th, C3 Thyroid cartilage C8 Small finger L4 Knee
Chapter 36, C4 Suprasternal notch T4 Nipple line L5 Lateral calf
p. 347, C5 Below clavicle T10 Umbilicus S1 Lateral foot
Figure 36-22 C6 Thumb L1 Femoral pulse S2S4 Perianal region
784. e FIGURE 4325. Cutaneous
th
Rosen 5 , nerve supply of the hand.
Chapter 43, M, median
p. 505, R, radial
Figure 43-25 U, ulnar
PCM, palmar cutaneous
branch of median nerve
DCU, dorsal cutaneous
branch of ulnar nerve.

785. a The dorsal interosseous muscles abduct the fingers away from the midline. The ulnar
Rosen 5th, nerve innervates them. Wrist and finger extension is under control of the radial nerve,
Chapter 43, while the median nerve controls flexion.
p. 505
786. d A boutonnire deformity is a disruption of the extensor hood near the PIP that presents
th
Rosen 5 , with PIP flexion and DIP extension. Initial treatment is by splinting the PIP in
Chapter 43, extension. A mallet finger is a disruption of the extensor tendon at the DIP.
p. 523, Boutonnire deformities are also seen in patients with rheumatoid arthritis
Figure 43-52
256

787. A felon:
a. can be treated by the emergency physician using incision and drainage followed by antibiotics.
b. is most commonly caused by group A streptococcus.
c. is the same as a paronychia.
d. requires hospital admission.
e. requires nail bed removal.

788. Patients with flexor tenosynovitis:


a. almost never recall a precipitating event.
b. have a negative Kanavels sign.
c. have exquisite pain with finger flexion.
d. have no pain when the proximal tendon sheath is palpated.
e. present with diffuse fusiform swelling.

789. Volkmans contracture


a. is associated with child abuse.
b. is associated with supracondylar fractures
c. is caused by a brachial nerve injury.
d. takes weeks to develop.
e. usually presents with a pulseless extremity.

790. Legg-Calv-Perthes disease is caused by:


a. a hip joint infection, most commonly due to Staphylococcus aureus.
b. a genetic bone matrix defect.
c. degeneration of the acetabulum.
d. avascular necrosis of the femoral head.
e. repetitive microtrauma to the articular cartilage of the knee.

791. A 3-year-old child is diagnosed with a septic arthritis of the hip. The organism which is most likely
causing his infection is:
a. Enterobacter.
b. Neisseria gonorrhoeae.
c. Pseudomonas aeruginosa.
d. Staphylococcus aureus.
e. Streptococcus pneumoniae.

792. A 14-year-old boy complains of pain in his groin, which sometimes goes to his thigh and knee. The
pain is dull, vague, intermittent or continuous, and is exacerbated by physical activity. He walks
with his leg in external rotation, and has an antalgic gait. You strongly suspect the child has:
a. Legg-Calv-Perthes disease.
b. Osgood-Schlatter disease.
c. septic arthritis.
d. slipped capital femoral epiphysis.
e. transient toxic synovitis.
257

787. a A felon is a subcutaneous pyogenic infection of the pulp space of the distal finger or
th
Rosen 5 , thumb. The patient presents with marked throbbing pain and a red, tense distal pulp
Chapter 43, space. Staphylococcus aureus is the most common organism, but Streptococcus species,
p. 529-530 anaerobes, and gram-negative organisms are also encountered. Most felons can be
drained adequately with a limited incision and drainage procedure. Most felons have
significant associated cellulitis that should be treated with oral antibiotics. A first-
generation cephalosporin or antistaphylococcal penicillin should be prescribed for 7 to
10 days or until the infection has abated.
788. e Flexor tenosynovitis is a surgical emergency that must be diagnosed promptly by the
Rosen 5th, examining physician and managed aggressively by both the emergency physician and
Chapter 43, the hand surgeon. Recognizing the classic clinical findings described by Kanavel makes
p. 531 the diagnosis. The four cardinal signs are tenderness over the flexor tendon sheath,
symmetric swelling of the finger, pain with passive extension, and a flexed posture of
the involved digit at rest.
789. b Ten percent of children with supracondylar fracture lose the radial pulse temporarily,
Rosen 5th, most often as a result of swelling and not direct brachial artery injury. Reducing the
Chapter 45, fracture, avoiding flexing the elbow more than 90 degrees, and elevating the arm help
p. 565-566 prevent secondary obstruction to arterial flow. Compartment syndrome, or Volkmanns
ischemic contracture as a result of prolonged ischemia of the forearm, is a dreaded
complication but fortunately is rare in this era, with a reported incidence of less than
0.5%.
790. d Perthesdisease is the name given to avascular necrosis of the pediatric femoral head. It
Rosen 5th, has also been called Legg-Calv-Perthes and Calv-Perthes disease. It occurs at a
Chapter 49, younger age than SCFE doesbetween the ages of 2 and 10. Its peak incidence is at 6
p. 643 years and it occurs 5 times more often in boys than in girls. The disease affects both
hips in 15% of patients.
791. d The most common organism implicated in acute septic arthritis is Staphylococcus
Rosen 5th, aureus. Acute septic arthritis occurs in all age groups but is more common in children:
Chapter 169, 70% of cases occur in children less than 4 years old, and the peak incidence is between
p. 2384, 6 and 24 months. Boys are affected twice as frequently as girls. Predisposing factors
Table 169-6 include preceding viral infection, trauma, immunodeficiency, hemoglobinopathy,
hemophilia with recurrent hemarthroses, diabetes, intravenous drug abuse, rheumatoid
arthritis, and intra-articular injections or operations. A total of 75% of septic arthritis
cases involve the joints of the lower extremity, with the knee being most commonly and
the hip second most commonly involved.
792. d Children with a stable slipped capital femoral epiphysis have symptoms of intermittent
Rosen 5th, limp and pain of several weeks to months duration. Stable slips make up
Chapter 169, approximately 90% of all cases. The pain of SCFE may be localized to the hip but more
p. 2387 commonly is poorly localized to the thigh, groin, or knee. With continued slippage,
internal rotation, flexion, and abduction are lost; and parents and children may note
progressive external rotation and shortening of the involved lower extremity with
subsequent difficulty in daily activities such as tying shoes. On examination, children
initially experience a slight loss of internal rotation and experience pain only at the
extremes of motion. The gait is antalgic and muscle atrophy is minimal. As the slip
becomes more severe, the gait becomes more antalgic, internal rotation is lost, abduction
and flexion of the hip increase, thigh and gluteal muscle atrophy is more pronounced,
and leg length discrepancy develops.
258

793. Carpal tunnel syndrome


a. is associated with ulnar nerve compression.
b. is ruled out with a negative Tinels sign.
c. presents with ulnar and dorsal hand numbness.
d. is reliably diagnosed with nerve conduction velocity studies.
e. Produces symptoms that worsen when the hands are placed in a dependent position.

794. In a patient with suspected acute cervical disc herniation:


a. axial load frequently relieves the symptoms.
b. the disease is more common in women than in men.
c. it tends to occur more in the upper vertebra.
d. the disc usually bulge in a direct posterior direction.
e. Valsalvas maneuver frequently replicates the symptoms.

795. A complaint of bilateral shoulder pain without history of trauma should raise the suspicion of:
a. C4 radiculopathy.
b. C5 radiculopathy.
c. C5 myelopathy.
d. C6 radiculopathy.
e. C6 myelopathy.

796. Chance fractures:


a. are usually at the thoracic spine level.
b. are associated with airbag deployment.
c. involve only the anterior and posterior columns.
d. involve only the middle and posterior columns.
e. are easily misdiagnosed as compression fractures.

797. The key sign of spinal stenosis is:


a. genital numbness and stool incontinence.
b. hyperreflexia and a negative anal wink.
c. hyporeflexia and a negative cremasteric reflex.
d. lower extremity pain exacerbated with walking and relieved with sitting.
e. urinary incontinence and stocking anesthesia.

798. Concerning the rotator cuff of the shoulder:


a. All four rotator cuff muscles originate on the scapula, traverse the glenohumeral joint, and insert
on the proximal humerus.
b. It consists of four muscles: the supraspinatus, the infraspinatus, the teres major, and the deltoid.
c. The infraspinatus muscle initiates arm elevation and abducts the shoulder.
d. The supraspinatus is an abductor of the humerus.
e. The rotator cuff muscles contribute little to the power of the upper extremity, providing less than
10% of the power in abduction and 20% percent in external rotation.

799. The muscle of the rotator cuff which is injured most frequently is the:
a. infraspinatus.
b. subscapularis.
c. supraspinatus.
d. teres minor.
e. teres major
259

793. a Carpal tunnel syndrome is a neuropathy of the median nerve at the wrist that occurs as a
th
Rosen 5 , result of compression of the median nerve within the carpal tunnel. The classic symptoms
Chapter 47, include a gradual onset of numbness, paresthesia, and pain in the thumb, index finger, and
p. 547-549 long finger. These symptoms are often bilateral and are worse during the night and after
strenuous activities. Typically, symptoms improve when the hands are shaken or held in a
dependent position. On physical examination the most sensitive provocative test is the
wrist flexion test, or Phalens test (sensitivity 76%, specificity 80%), performed by asking
the patient to fully flex the wrists for 60 seconds while the forearms are held in a vertical
position. The test is positive if paresthesias or numbness develops in the median nerve
distribution. Another test, Tinels sign, is positive if light tapping or percussion over the
median nerve at the wrist produces pain or paresthesias in the median nerve distribution.
Tinels sign is not as helpful as Phalen's test (sensitivity 64%, specificity 55%). Nerve
conduction studies have recently been used to confirm the diagnosis, with reports of a
sensitivity as high as 90%.
794. e Cervical disk disease occurs most commonly in those aged 35-55 years, and frequently
Rosen 5th, with a prior history of trauma. Lateral to posterolateral is the usual direction of herniation
Chapter 47, with directly posterior being rare. It occurs most commonly in the lower cervical
p. 606-608 vertebrae, C4-C7. Axial load and Valsalvas maneuver both aggravate the symptoms.
795. d C6 lesions are frequent and may cause pain along the upper neck, down the biceps, along
Rosen 5th, the lateral forearm, and into the dorsal radial aspect of the hand. Motor weakness usually
Chapter 47, involves the biceps or wrist extensors, making elbow flexion and wrist extension difficult.
p. 609
796. e Chances fracture is a vertebral fracture, usually lumbar, involving the posterior spinous
th
Rosen 5 , process, pedicles, and vertebral body. Caused by simultaneous flexion and distraction
Chapter 42, forces on the spinal column, usually associated with use of lap seatbelts. The anterior
Table 42-1 column fails in tension along with the middle and posterior columns. It may be
misdiagnosed as a compression fracture.
797. d Spinal stenosis complaints usually come from older patients. Pain is diffuse, usually
Rosen 5th, involves the back, and often radiates down one or both legs, associated with paresthesias.
Chapter 47, It is usually resolved by rest, although it takes longer to resolve than true vascular
p. 616 claudication. Patients also note pain relief with back flexion. A typical story involves the
patient who is able to walk uphill without pain but develops pain walking downhill when
the back is extended. Impotence and incontinence may be present with spinal stenosis.
798. a The supraspinatus, infraspinatus, and teres minor insert into facets of the greater
Rosen 5th, tuberosity, whereas the subscapularis inserts into the lesser tuberosity. All originate on the
Chapter 46, scapula and traverse the glenohumeral joint, and serve as a dynamic stabilizer of the
p. 600-601 glenohumeral joint. The acromioclavicular joint itself has only 5 to 8 degrees of
movement, but allows for 40 to 50 degrees of clavicular rotation. Due to its unique bony
structure, it is the most highly mobile yet least stable joint in the body. The supraspinatus
provides most of the first 30o of extension, while the infraspinatus is an external rotator.
799. c Most rotator cuff tears involve the dominant arm and occur in men over age 40 years. The
Rosen 5th, occupational history is significant for strenuous work requiring overhead activity. Most
Chapter 46, tears occur near the attachment of the supraspinatus and can extend anteriorly into the
p.601 subscapularis or posteriorly into the infraspinatus. Tears can be classified according to
their size, completeness, pattern location, or duration. A clinically useful system is to
divide tears into acute or chronic types.
260

800. Slipped capital femoral epiphysis:


f. affects girls more than boys.
g. frequently presents with gradual onset of knee pain.
h. is most common under age 10 years.
i. is treated conservatively.
j. usually has normal x-rays.

801. Failure to mobilize the shoulder after a stroke is a major risk factor for:
a. adhesive capsulitis.
b. biceps tendonitis.
c. carpal tunnel syndrome.
d. deltoid dawn syndrome.
e. rotator cuff disease.

802. Patients with bicipital tendonitis have intense pain, especially against resistance, when they attempt
to:
a. extend the elbow.
b. extend the wrist.
c. flex the elbow.
d. pronate the forearm.
e. supinate the forearm.

803. Transient toxic synovitis is


a. a diagnosis of exclusion
b. an uncommon cause of hip pain
c. associated with high fever and laboratory abnormalities.
d. caused by trauma
e. treated with antibiotics

804. A 27-year-old athletic woman complains of right knee pain. She has no history of significant
trauma. There is no effusion on exam, but there is swelling and tenderness in the popliteal fossa
and in the calf. A lower extremity ultrasound is interpreted as normal. The most likely diagnosis
is:
a. anterior cruciate ligament injury
b. Bakers cyst
c. deep venous thrombosis.
d. Osgood-Schlatter disease
e. patellar dislocation

805. Osgood- Schlatter Disease:


a. classically has no reproducible tenderness
b. is another name for patellar tendon apophysitis
c. is far more common in boys than in girls.
d. presents with acute pain.
e. requires surgery.
261

800. b Slipped capital femoral epiphysis is most common in obese adolescent males ages 10-
th
Rosen 5 , 17. It is rare before age 10. The etiology is unclear. It often presents with gradual
Chapter 169, onset of pain referred to the knee. The x-rays have a characteristic melting ice cream
p. 2387 cone appearance. It is treated surgically with open reduction internal fixation.
801. a Although the cause of adhesive capsulitis remains unknown, any condition associated
th
Rosen 5 , with prolonged dependency of the arm can result in capsular contraction. This
Chapter 46, includes voluntary immobilization after calcific tendinitis, rotator cuff injury,
p. 603 mastectomy, or a distal upper extremity injury (Colles fracture).
802. e On examination of a patient with bicipital tendinitis, there is tenderness over the biceps
th
Rosen 5 , tendon as it passes through the bicipital groove. This is best demonstrated with the
Chapter 46, arm in abduction and external rotation. Active range of motion is limited by pain, but
p. 602 the passive range remains intact. Supination against resistance (Yergasons test) with
the arm adducted and the elbow flexed to 90 degrees reproduces the pain.
803. a Transient synovitis is the most common cause of hip pain in childhood. It is a self-
Rosen 5th, limited inflammatory condition caused by a nonpyogenic inflammatory response of the
Chapter 169, synovium. Although it has been reported in children as young as 3 months and it
p. 2382 occasionally occurs in adults, its peak incidence is between 3 and 6 years of age.
Transient synovitis of the hip affects boys more commonly than girls and has a slight
predilection for the right side. The cause is unknown, and the diagnosis is one of
exclusion. Treatment is conservative rest and antiinflammatories.
804. b A Bakers, or popliteal, cyst is a herniation of the synovial membrane through the
Rosen 5th, posterior aspect of the capsule of the knee, or it may occur through escape of fluid
Chapter 50, through the normal communication of an anatomic bursa adjoining the
p. 697 semimembranosus or gastrocnemius. It is usually the result of an internal knee
derangement with recurrent synovitis caused by a torn meniscus, loose body,
instability, degenerative change, or other factors. It generally causes a mass of varying
size occupying the posteromedial corner of the knee and often produces pressure, pain,
and limitation of range of motion. Rupture of the bursa with resultant escape of fluid
into the calf may produce a clinical picture similar to thrombophlebitis. Differentiation
from other clinical entities may require aspiration, ultrasound, or an MRI scan..
805. c Tibial tubercle apophysitis was first described by Osgood and Schlatter in 1903. It is
Rosen 5th, believed to be caused by repeated traction to the anterior portion of the developing
Chapter 50, ossification center of the tibial tuberosity. Past reports have suggested a large male
p. 699-700 predominance for the development of Osgood-Schlatter disease, but as more girls and
young women have joined competitive sports, no significant sex differences have been
found. The disease is characterized by painful swelling over the tibial tubercle that is
exacerbated by activity, relieved by rest, and usually of several months duration.
Surgery may be required if the conservative treatment fails, but it cannot be
implemented until the epiphysis is closed, at which time the problem is usually
resolved.
262

806. Concerning pelvic fractures:


a. when present, they usually have associated bladder or urethral disruption.
b. they are usually seen in association with blunt lower urinary tract injury.
c. urethral disruption is more common in women than men.
d. they have no relation to thoracic aortic injury.
e. the first treatment is application of MAST garments.

807. Coccygeal fractures:


a. usually require an x-ray to diagnose.
b. they can be well evaluated by a rectal exam.
c. when displaced, reduction should be attempted.
d. occur more often in men than women.
e. usually heal rapidly and with minimal discomfort.

808. Plantar fasciitis:


a. causes pain and tenderness localized to the medial aspect of the heel.
b. causes pain that is worse in the evening and better after exercising.
c. eventually will require surgery.
d. is always associated with heel spurs.
e. is improved by avoiding Achilles tendon stretching and by walking barefoot on hard surfaces.

809. The tarsal tunnel syndrome:


a. causes pain on the dorsal foot near the first metatarsophalangeal joint.
b. causes tenderness over the tip of the lateral malleolus.
c. involves the anterior tibial nerve.
d. usually requires surgery.
e. can be diagnosed by the aid of selective nerve block.

810. Mortons neuroma is found most frequently in women age 25 to 50, and causes a severe,
lancinating, debilitating pain in the:
a. ankle.
b. foot.
c. knee.
d. thumb.
e. wrist.
263

806. b The overall incidence of bladder or urethral disruption associated with pelvic fracture
th
Rosen 5 , ranges from 7% to 25%; however, more than 80% of patients with lower urinary tract
Chapter 48, injury after blunt trauma have a pelvic fracture. Urethral disruption is less common in
p. 627-628 women, but it does occur. There is a well-documented association between pelvic
fracture and injury to the thoracic aorta. This is presumably the result of the enormous
forces required to produce either injury. In particular, anteroposterior (AP)
compression fractures are associated with an 8 times greater incidence of aortic rupture
than is seen in the overall blunt trauma population. Initial treatment is fluid
resuscitation followed by considerations for stabilization and possibly embolization.
807. b Radiographic confirmation of a coccygeal fracture is not always necessary. Displaced
Rosen 5th, fractures are often seen on the lateral view, but the diagnosis is evident on rectal
Chapter 48, examination. Attempts at reduction are not recommended. Undisplaced fractures may
p. 632 be difficult to demonstrate radiographically. The physician must decide whether the
knowledge gleaned from x-ray studies will alter the therapy to a degree that warrants
radiation exposure to the pelvis, especially considering that most of these fractures
occur in women. Because of muscle action on the fragment, healing is slow and
patients must be cautioned that discomfort may be prolonged.
808. a The plantar fascia is a tough layer of the sole that is functionally significant during foot
Rosen 5th, strike and the early stance phase of walking. Plantar fasciitis, an overuse injury of
Chapter 51, insidious onset, usually begins with pain on first weight bearing in the morning or after
p. 733 prolonged sitting. This progresses to persistent pain during gait. Pain and tenderness
are localized to the medial aspect of the heel. Plantar fasciitis accounts for 9% of
running injuries and is particularly common in cavus feet. It is associated with heal
spurs about 50% of the time. Treatment is usually conservative with surgery rarely
required.
809. e Compression of either the abductor digiti quinti nerve or the posterior tibial nerve (the
Rosen 5th, tarsal tunnel syndrome) can cause subcalcaneal heel pain. Diagnosis of these
Chapter 51, conditions is difficult and is sometimes facilitated by assessing the impact of selective
p. 733 nerve block with local anesthesia. Initial treatment of these conditions is similar to that
for plantar fasciitis, although local steroid injections or surgical release may be
required.
810. b An important cause of unilateral metatarsalgia is a perineural fibrosis of the
Rosen 5th, intermetatarsal plantar digital nerve, more commonly known as a Mortons neuroma.
Chapter 51, This neuropathy of unknown cause was first described in 1876 and usually involves
p. 733 the second-third or third-fourth interspace, causing lancinating pain with weight
bearing. The pain may be associated with paresthesias and can radiate into the toes. In
addition, after burn pain may persist during rest. The pain of a Mortons neuroma is
reproduced when the affected interspace is pinched or when the metatarsal heads are
compressed together. Hence, pain may occur intermittently with tight-fitting footwear
(e.g., rock climbing shoes or ski boots). Crepitus or a nodule may be palpable.
Treatment usually involves surgical excision or neurolysis.
264

12.0 Nervous System Disorders


811. The 30-day mortality rate for patients experiencing a stroke is:
a. less than 5%.
b. 10%.
c. 20 to 25%.
d. about 35%.
e. more than 50%.

812. A 74-year-old man has a history of hypertension and high cholesterol. He complains of sudden
painless blindness in his left eye. You suspect embolic stroke, with the origin of the clot being the:
a. anterior cerebral artery.
b. carotid artery.
c. circle of Willis.
d. heart.
e. middle cerebral artery.

813. An 82-year-old man is brought by his family after sudden onset of one-sided weakness, falls, and
an inability to speak. He can follow simple commands using the non-weak side. This localizes his
stroke to the:
a. anterior cerebral artery, dominant cortex.
b. anterior cerebral artery, nondominant cortex.
c. brainstem.
d. middle cerebral artery, dominant cortex.
e. middle cerebral artery, nondominant cortex.

814. Lacunar infarcts are:


a. commonly associated with chronic hypertension.
b. uncommon in African-Americans.
c. primarily located in the cerebellum.
d. frequent causes of cognitive defects, aphasia, and memory impairment.
e. rarely subcortical.

815. A 54-year-old male with a long history of smoking and hypertension says his right hand felt dead
for about 20 minutes this morning, and he was dragging his right foot. He dropped his coffee mug
and had a hard time lacing his shoes. He states he now feels 100% normal. You examine him
and find no focal deficits. You discuss your findings with him, explaining:
a. 20% of patients with TIA will experience a stroke within 5 years.
b. a CT will show no evidence of an infarction.
c. carotid artery territory TIAs uniformly demonstrate arterial disease on angiography.
d. most TIAs last less than 30 minutes.
e. patients with resolved new-onset TIAs can safely be discharged from the Emergency Department
if they have a normal exam and negative CT scan.
265

811. c Stroke is the third leading cause of death in the United States and the leading cause of
th
Rosen 5 , adult disability. It afflicts over 700,000 patients per year, with an in-hospital mortality
Chapter 95, of almost 15% and a 30day mortality of 20% to 25%. Even among survivors, over
p. 1433 half are left with a permanent disability and one third need assistance in the activities
of daily living.
812. b Blood is supplied to the brain by the anterior and posterior circulation. The anterior
Rosen 5th, circulation originates from the carotid system and perfuses four fifths of the brain,
Chapter 95, including the optic nerve, retina, and frontoparietal and anterotemporal lobes. The first
p. 1434 branch off the internal carotid artery is the ophthalmic artery, which supplies the optic
nerve and retina. As a result, the sudden onset of painless monocular blindness
(amaurosis fugax) identifies the stroke as involving the anterior circulation
(specifically the carotid artery) at or below the level of the ophthalmic artery.
813. d Aphasia is important to recognize because it usually localizes a lesion to the dominant
Rosen 5th, (usually left) cerebral cortex in the middle cerebral artery distribution. Aphasia may be
Chapter 95, expressive, receptive, or a combination of both. Wernickes aphasia occurs when the
p. 1436 patient cannot process sensory input such as speech and thus fails to understand verbal
communication (receptive aphasia). Brocas aphasia refers to the inability to
communicate verbally in an effective way, even though understanding may be intact
(expressive aphasia). Aphasia should be distinguished from dysarthria, which is a
motor deficit of the mouth and speech muscles; the dysarthric patient articulates poorly
but understands words and word choices.
814. e Lacunae or small-vessel strokes involve small terminal sections of the vasculature and
Rosen 5th, more commonly occur in African Americans and patients with diabetes and
Chapter 95, hypertension. A history of hypertension is present in 80% to 90% of patients who have
p. 1434 lacunae strokes. The subcortical areas of the cerebrum and brainstem are often
involved. The infarctions range in size from a few millimeters to 2 cm and occur most
commonly in the basal ganglia, thalamus, pons, and internal capsule. They may be
caused by small emboli or by a process termed lipohyalinosis, which occurs in
patients with hypertensive cerebral vasculopathy. Although nearly 20 lacunae
syndromes have been described, the most common of these are pure motor strokes,
pure sensory strokes, or ataxic hemiparesis. Because they are subcortical and well
localized, lacunae strokes do not cause cognitive defects, aphasia, simultaneous
sensorimotor findings, and loss of consciousness or memory impairment.
815. d The majority of TIAs last less than 30 minutes. Up to 50% of patients who experience
Rosen 5th, a TIA will go on to develop a stroke within 5 years. At least 50% of patients with TIA
Chapter 95, in the carotid distribution do not have angiographically demonstrated arterial disease.
p. 1435 Although clinical deficits of TIAs resolve within 24 hours, studies have shown that up
to 64% of patients have CT evidence of infarction. Current standard is that a patient
with a new TIA requires hospital admission. The exception is a patient with only
minimal anterior circulation symptoms who has had a complete ED evaluation
including CT scan, carotid Doppler or MRA, and an echocardiogram if indicated.
Close neurologic follow up must be arranged.
266

816. A 76-year-old man had sudden onset of nausea and vomiting, and then became quite lethargic. His
exam shows poor pain and temperature perception on the right face and on the left arm and leg.
These symptoms are characteristic of a stroke in the distribution of the:
a. anterior cerebral artery.
b. middle cerebral artery.
c. posterior communicating artery.
d. anterior communicating artery.
e. vertebral basilar system.

817. The standard brain imaging study for a patient with presumed acute stroke is:
a. contrast-enhanced CT scan.
b. gadolinium-enhanced MRI.
c. magnetic resonance angiography.
d. noncontrast MRI.
e. noncontrast CT scan.

818. An 84-year-old woman has an acute ischemic stroke. She presents within 30 minutes of symptom
onset and her head CT shows only atrophy. She has no contraindication to fibrinolytic therapy, as
described in the National Institute of Neurologic Disorders and Stroke criteria. She weighs 50 kg,
so you choose to treat her using:
a. 10 mg bolus, 50 mg infusion over 30 minutes.
b. 10 mg bolus, 90 mg infusion over 60 minutes.
c. 100 mg bolus.
d. 5 mg bolus, 45 mg infusion over 60 minutes.
e. 50 mg bolus.

819. A 70-year-old man has a large hemorrhagic stroke. His blood pressure is 240/160 mm Hg, even
after he has received supplemental oxygen and comfort measures in the two hours you have been
caring for him. You should manage his blood pressure by giving him:
a. intravenous labetalol.
b. intravenous nitroprusside.
c. more time.
d. sublingual nifedipine.
e. transdermal clonidine patch.

820. The metabolic condition most frequently mistaken for stroke is:
a. Addisonian crisis.
b. hyperglycemia.
c. hypocalcemia.
d. hypoglycemia.
e. thyroid storm.

821. The patient who has had a classic tonic-clonic (grand mal) seizure:
a. has good recall for events leading up to and during the seizure.
b. invariably has post-ictal confusion and lethargy.
c. is usually able to avoid self-injury.
d. may experience an aura lasting a minute or two before the actual seizure.
e. may have side-to-side head thrashing, rhythmic pelvic thrusting, and clonic alternating extremity
motions
267

816. e Unlike anterior circulation strokes, patients with posterior circulation stroke can
th
Rosen 5 , present with loss of consciousness, and they frequently have nausea and vomiting.
Chapter 95, Visual agnosia, the inability to recognize seen objects, may occur, as well as alexia, the
p. 1436 inability to understand the written word. A third nerve palsy may occur, and the
patient may experience homonymous hemianopsia. One of the more curious facets of
this syndrome is that the patient may be unaware of any visual problem (visual
neglect). Vertigo, diplopia, visual field defects, weakness, paralysis, dysarthria,
dysphagia, syncope, spasticity, ataxia, or nystagmus may occur with vertebrobasilar
artery insufficiency. One of the key features is that the sensory findings such as
deficits in pain and temperature perception occur on one side of the face and on the
opposite side of the body.
817. e An emergency noncontrast cranial CT is the standard imaging technique for evaluating
Rosen 5th, a patient with a potential stroke in the ED. It can quickly differentiate an ischemic
Chapter 95, stroke from intracerebral hemorrhage and other mass lesions. It can identify almost all
p. 1438 parenchymal hemorrhages greater than 1 cm and up to 95% of all subarachnoid bleeds.
The majority of ischemic strokes do not have gross signs of infarction on routine CT
scan for at least 6 to 12 hours, depending on the infarcts size. However, subtle, ultra-
early changes have been noted in up to one third of stroke patients evaluated by CT
within 3 hours of symptom onset.
818. d The total dose of rtPA for treating acute ischemic stroke is 0.9 mg/kg, with a maximum
Rosen 5th, dose of 90 mg; 10% of the dose is administered as a bolus, with the remaining amount
Chapter 95, infused over 60 minutes. Blood pressure and neurologic checks should be assessed
p. 1440-1441 every 15 minutes for 2 hours after starting the infusion.
819. b Nitroprusside is the agent most commonly recommended because one can obtain a
th
Rosen 5 , rapid and consistent lowering of the blood pressure to the desired level and
Chapter 95, adjustments can be made rapidly. It has rapid onset, it can be titrated, and has no effect
p. 1442 on mental status.
820. d Hypoglycemia is often responsible for an altered mental status and is a well-known
th
Rosen 5 , cause of sustained focal neurologic findings that persist for several days.
Chapter 95,
p. 1438
821. b By definition, patients lose consciousness in a generalized seizure, and no aura is
th
Rosen 5 , present. Some patients may experience a brief, vague prodrome or dysphoric state just
Chapter 96, before the ictal event. Convulsive generalized seizures are typified by the grand mal
p. 1446-1447 seizure in which the patient loses consciousness; stiffens with generalized muscular
hypertonus; and then rhythmically, violently contracts multiple, bilateral, and usually
symmetric muscle groups. A generalized convulsive seizure is followed by a postictal
state, headache, and drowsiness that may persist for hours.
268

822. Seizure is a common presenting symptom in an acute overdose of the anti-seizure medication:
a. carbamazepine (Tegretol)
b. gabapentin (Neurontin).
c. phenobarbital.
d. topiramate (Topamax).
e. valproic acid (Depakene)

823. Posttraumatic seizures (PTS):


a. are caused by epidural and intracerebral hematomas, but not subdural and traumatic subarachnoid
hemorrhages.
b. are more common in adults than children, especially in the early post-trauma stage.
c. are much more common if the dura is violated.
d. extinguish quickly, so that by one year after significant head trauma, the incidence of seizures is
the same as that of the general population.
e. occur in blunt head trauma as the result of increased intracranial pressure; thus, penetrating head
trauma does not usually cause seizures.

824. The most common cause of new onset seizures in geriatric patients is:
a. dehydration.
b. drug-drug interaction.
c. metastatic brain tumors.
d. primary brain tumors.
e. strokes.

825. A basic transport unit brings you a disheveled middle-aged man who has been seizing for about 10
minutes. As you start an intravenous line you ask the nurse to bring you a parenteral
benzodiazepine so you can stop the seizure. She asks you which one you prefer. You answer:
a. any of the three will do, as comparison studies have not demonstrated one drug to be clearly
superior.
b. diazepam, as it can be given intramuscularly if the intravenous line fails.
c. lorazepam, as it has clearly been demonstrated as the most efficacious.
d. lorazepam, diazepam, and midazolam, as the combination of all three drugs has been shown
especially efficacious in stopping status epilepticus.
e. midazolam, as it can be given endotracheally if the intravenous line fails.

826. A 24-year-old college student with a known seizure disorder failed to take any phenytoin for the
last week because of final exams. He had three seizures today and was brought by his roommate
for treatment. The patient weighs about 220 pounds, and his serum phenytoin level is 0. You need
to give him a loading dose of about:
a. 800 mg.
b. 1000 mg.
c. 1200 mg.
d. 1500 mg.
e. 1800 mg.

827. The most common recurrent head pain syndrome is:


a. cluster headache.
b. migraine headache.
c. temporal arteritis.
d. tension headache.
e. trigeminal neuralgia.
269

822. a The most common drug- and toxin-associated seizures occur in conjunction with illicit
th
Rosen 5 , drugs such as cocaine, amphetamines, and phencyclidine; with overdoses of
Chapter 96, anticholinergic agents such as cyclic antidepressants and antihistamines; as a
p. 1448-1449 manifestation of withdrawal from ethyl alcohol and sedative-hypnotics; and with toxic
levels and deliberate overdoses of diverse medications such as aspirin, theophylline,
isoniazid, lithium, and the anticonvulsants phenytoin and carbamazepine.
823. c Posttraumatic seizures (PTS) can occur as an acute result of blunt or penetrating head
Rosen 5th, trauma and as posttraumatic sequelae. Epidural, subdural, and intracerebral
Chapter 96, hematomas and traumatic subarachnoid hemorrhages can all be acutely ictogenic,
p. 1449 particularly as the intracranial pressure rises. Immediate PTS are more common in
children than in adults. Within the first year after significant head trauma, the
incidence of seizures is at least 12 times that of the general population. The incidence
of seizures after injury with neurologic deficit without dural violation is 7% to 39%;
when the dura is disrupted, the incidence is 20% to 57%.
824. e Ischemic or hemorrhagic stroke is the cause of new-onset seizures in 54% of elderly
Rosen 5th, patients. The overall incidence of seizures with stroke ranges from 4% to 15%; more
Chapter 96, than half occur within the first week after stroke. The incidence of epilepsy after
p. 1449 stroke is 4% to 9%. Seizures that occur acutely with stroke are thought to result from
local metabolic alterations in the CNS; these events are transient and the seizures are
often focal and self-limited. Seizures that develop later are more likely to be
generalized.
825. a Pertinent differences among benzodiazepines are the efficacy of diazepam when
Rosen 5th, administered rectally, endotracheally, or intraosseously; a relatively longer duration of
Chapter 96, seizure suppression with lorazepam; and efficacy of the rectal and intramuscular routes
p. 1452 of administration for midazolam. In comparison studies, no one drug is clearly
superior. All three may be used in patients of all ages, all have relatively short
duration of anticonvulsant action, and all can cause sedation, hypotension and
respiratory depression.
826. e Phenytoin is the primary drug used in the management of status epilepticus after initial
Rosen 5th, benzodiazepine therapy. It may also be used as primary therapy for patients whose
Chapter 96, seizures are less frequent. The recommended loading dose is 18 to 20 mg/kg IV, in
p. 1452, this 100 kg patient a dose of 1800 to 2000 mg is appropriate. Infusion should be
Table 96-1 limited to 50 mg/min due to the cardiotoxicity of the diluent. Patient should be on a
cardiac monitor.
827. d Tension headache is the most common recurrent pain syndrome, affecting more than
Rosen 5th, 75% of the population. Women are affected more frequently than men, and most
Chapter 97, patients are middle aged. The headaches do not cause significant disability, and
p. 1460 patients are able to continue with their normal daily activities. The median frequency
of headaches is six per month, and stress and lack of sleep are implicated as triggering
factors. The average duration of the headache is 4 to 13 hours, with a maximum of 72
hours.
270

828. You are seeing a young man in whom you have suspicion of a possible subarachnoid bleed. His CT
was negative, so you did a spinal tap. The cerebrospinal fluid looks a bit pink, but the tap was
difficult and you are worried about a possible traumatic tap. You ask the lab to centrifuge a tube
of fluid so you can look for xanthochromia, as the patient has had the headache for long enough you
might see this finding. The amount of time it takes for xanthochromia to develop is about:
a. 4 hours.
b. 8 hours.
c. 12 hours.
d. 16-18 hours.
e. 20-24 hours.

829. Chromatographic studies on the supernatant spinal fluid of the patient in Question #828 show that it
was, indeed, a traumatic tap. You ask the patient to lie flat for two hours before sending him home
with appropriate analgesia. Twenty-seven hours later he returns, saying he has an even worse
headache than before. You suspect postdural puncture headache (PDPH).
a. An autologous blood patch will relieve the headache in a majority of patients with this condition.
b. The headache is from a hyperstimulatory overproduction of cerebrospinal fluid and can be
relieved by a second tap with removal of more fluid.
c. This is an uncommon complication, occurring in less than 10% of patients.
d. You had correctly used a large needle to minimize the amount of time needed to remove fluid,
and this should have prevented the headache.
e. You realize you should have had him stay flat for at least two more hours to avoid this
complication.

830. The treatment of choice for the pain of trigeminal neuralgia is:
a. dental referral for oral orthotic.
b. local steroid injections.
c. oral carbamazepine.
d. oral nimodipine.
e. transdermal fentanyl.

831. The majority of cases of delirium are caused by


a. electrolyte disturbances.
b. hypoxemia.
c. liver failure.
d. recreational and prescribed drugs.
e. sepsis.

832. The most common metabolic disorder causing acute organic brain syndrome is:
a. hyperglycemia.
b. hypernatremia.
c. hyperthyroidism.
d. hypoglycemia.
e. hyponatremia.

833. Brain tumor can present acutely as delirium, especially if the growth is located in the:
a. brainstem.
b. frontal lobe.
c. occipital lobe.
d. parietal lobe.
e. temporal lobe.
271

828. c To differentiate a traumatic lumbar puncture from SAH, the patients cerebrospinal
th
Rosen 5 , fluid should be spun and the supernatant observed for xanthochromia. The yellowish
Chapter 97, pigmentation is secondary to the metabolism of hemoglobin to pigmented molecules of
p. 1461 oxyhemoglobin and bilirubin, a process that takes approximately 12 hours to occur.
829. a Post Dural Puncture Headache (PDPH) is the most common complication of lumbar
th
Rosen 5 , puncture, occurring in up to 40% of patients. The amount of time a patient remains
Chapter 97, recumbent after lumbar puncture does not appear to affect the incidence of headache.
p. 1464 Certain factors have been implicated as causes of PDPH, including the size or diameter
of the spinal needle, the orientation of the bevel during the procedure, and the amount
of fluid withdrawn. Smaller-diameter needles will cause less leakage, and it is
postulated that inserting the needle with the bevel up (i.e., bevel pointing up when the
patient is in the lateral position) will minimize damage to the dural fibers. Using
atraumatic needles or pencil-point needles (e.g., Whitaker or Sprotte) has also been
shown to significantly reduce the incidence of PDPH. Most PDPH headaches resolve
spontaneously within a few days with bed rest and mild analgesics. For severe
headaches lasting more than 24 hours, an epidural blood patch (autologous blood clot)
will relieve the headache in the majority of patients.
830. c Trigeminal neuralgia is characterized by paroxysms of severe unilateral pain in the
Rosen 5th, trigeminal nerve distribution lasting only seconds, with normal findings on neurologic
Chapter 97, examination. There is no pain between paroxysms. Treatment can be medical or
p. 1465 surgical. Carbamazepine is a very effective treatment. If it fails, the patient is unlikely
to have trigeminal neuralgia.
831. d Delirium can be defined as acute cognitive dysfunction secondary to some underlying
Rosen 5th, medical condition. Terms that have been used interchangeably with delirium include
Chapter 98, acute organic brain syndrome, acute confusional state, reversible cerebral dysfunction,
p. 1468-1469 metabolic encephalopathy, toxic encephalopathy, and febrile delirium. The word
delirium is derived from the Latin delirare, which literally means to go out of the
furrow (lira, Latin furrow), but is used figuratively to mean crazy or deranged.
Recreational and prescribed drugs are responsible for most cases of acute delirium.
Among the elderly, medications are the most common cause of delirium, accounting
for 22% to 39% of cases.
832. d The most common metabolic disorder causing acute organic brain syndrome is
Rosen 5th, diabetes mellitus. Hypoglycemia is the most common and readily reversible cause of
Chapter 98, acute confusion in the diabetic patient. Other causes of acute cognitive impairment in
p. 1470 the diabetic patient are hyperglycemia, hyperosmolarity, and acid-base abnormalities.
833. b Acute delirium occasionally can be the first manifestation of an intracranial space-
Rosen 5th, occupying lesion such as a subdural hematoma, tumor (especially frontal lobe), or
Chapter 98, hydrocephalus. The size and location of the lesion determine whether focal neurologic
p. 1471 findings are present.
272

834. One of the hallmarks of acute delirium is:


a. auditory, but not visual, hallucinations.
b. inability to catnap during the day.
c. paucity of emotions.
d. remote memory impairment.
e. short-term memory impairment.

835. In a patient with delirium, the EEG:


a. is helpful in less than 50% of cases.
b. is almost never normal, showing diffuse bilateral slowing.
c. is useless.
d. may show rapid focal spikes.
e. shows sleep abnormalities, but is normal during wakefulness.

836. In a patient with Alzheimers dementia, the cortical atrophy is most prominent in the:
a. cerebellum.
b. frontal lobes.
c. occipital lobes.
d. parietal lobes.
e. temporal lobes.

837. The drug of choice to treat agitated dementia is:


a. chlorpromazine (Thorazine).
b. diazepam (Valium).
c. haloperidol (Haldol).
d. hydroxyzine (Atarax).
e. lorazepam (Ativan).

838. A 57-year-old has progressive dementia and loss of balance. He smells of urine and is incontinent
as you examine him. He most likely has:
a. bovine spongioform encephalopathy (Mad Cow Disease).
b. Creutzfeldt-Jakob disease.
c. normal pressure hydrocephalus.
d. senile dementia, Alzheimer's type.
e. Wernicke-Korsakoff syndrome.

839. Current recommended treatment for Bells palsy is:


a. acyclovir 400 mg five times per day for 10 days.
b. cold packs.
c. prednisone 1 mg/kg/day for 5 days and then tapered over the next 5 days and acyclovir 400 mg
five times per day for 10 days.
d. prednisone 1 mg/kg/day for 5 days, with or without taper.
e. surgical ablation.

840. Concerning the cranial nerves:


a. cranial nerve III injury causes a dilated pupil due to loss of parasympathetic nerve fiber supply.
b. cranial nerve III injury will result in diplopia on attempted lateral gaze.
c. cranial nerve IV supplies the lateral rectus muscle.
d. cranial nerve V injury causes deafness.
e. cranial nerve VI damage will often lead to head tilt.
273

834. e One of the hallmarks of delirium is impairment in short-term memory, with inability to
th
Rosen 5 , learn and assimilate new information. Remote memory, or memory of past events, is
Chapter 98, usually preserved. Thought processes and speech may be disorganized. The delirious
p. 1471 patient has a reduced capacity to modulate fine emotional expression and may
demonstrate emotional lability. Characteristically, the sleep-wake cycle is reversed
because patients sleep during the day and are awake throughout the night. The
delirious patient may experience visual, auditory, tactile, gustatory, or olfactory
hallucinations, in contrast to patients with acute functional psychosis, who typically
experience only auditory hallucinations.
835. b Although rarely practical in the emergency setting, the EEG can be a valuable
Rosen 5th, diagnostic tool in determining the presence of delirium. A normal EEG result is
Chapter 98, incompatible with severe delirium; bilateral diffuse symmetric abnormalities are a
p. 1475 relatively consistent feature. In most cases the EEG abnormality consists of relative
generalized slowing with or without superimposed fast activity.
836. e On a broad scale there is cortical atrophy most prominent in the temporal and
Rosen 5th, hippocampal regions, caused by progressive synaptic and neuronal loss in the cerebral
Chapter 98, gray matter. This is generally followed by loss of white matter (subcortical atrophy).
p. 1477 Atrophy secondary to neuronal death is an important manifestation of Alzheimers
disease.
837. c Occasionally medications are needed for symptomatic treatment of agitation, sleep
Rosen 5th, disturbance, and depression. These patients typically do not improve with anxiolytics.
Chapter 98, Agitation can be controlled with a small dose of the butyrophenone haloperidol
p. 1483 (Haldol). The cardiovascular toxicity of this drug is minimal, and it is well tolerated
in elderly patients.
838. c The classic triad of progressive dementia, ataxia, and urinary incontinence (Wacky,
Rosen 5th, Wobbly, and Wet) occurs in patients with NPH, which affects younger patients than
Chapter 98, does primary degenerative dementia. More than half of the reported cases involve
p. 1481-1482 individuals less than 60 years of age. Hydrocephalus secondary to prior head trauma
or infection has a more favorable prognosis than primary hydrocephalus.
839. c Treatment of Bells palsy is controversial but most practitioners use prednisone for
Rosen 5th, anti-inflammatory effect at 60 mg PO daily or 1 mg/kg/day for 5 days and then tapered
Chapter 99, over the next 5 days, along with acyclovir 400 mg five times per day for 10 days and
p. 1489 follow-up with either ENT or neurologic consultation. The prognosis is generally
good for total recovery, but patients with total paralysis are at increased risk of long-
term or permanent paralysis and should be seen in follow-up within 2 to 3 days, and
patients with incomplete paralysis instructed to return if the weakness becomes total
paralysis.
840. a Cranial nerve III (oculomotor nerve) controls all function of extraocular muscles
Rosen 5th, except the superior oblique (CN IV, trochlear nerve) and the lateral rectus (CN VI,
Chapter 99, abducens nerve). CN III also controls pupillary constriction via parasympathetic
p. 1486- innervation and upper lid elevation by the levator palpebrae. Patients will often
1487, Table compensate for CN IV compression by head tilt. Memory device for ocular nerves
99-1 LR6SO4 all others 3.
274

841. You are attempting to clinically differentiate an acoustic neuroma from Mnires disease. The
finding more suggestive for an acoustic neuroma is:
a. asymmetric hearing loss.
b. balance problems.
c. intermittent ringing in the ears.
d. vertigo.
e. vomiting.

842. The primary pathology in a patient who refluxes fluid through the nose when he tries to swallow is:
a. CN V.
b. CN VII.
c. CN X.
d. CN XI.
e. CN XII.

843. You are evaluating a 36-year-old woman for a complaint of visual disturbance and colors looking
funny. Her eye examination is completely normal until you show her the bottle-top on a
cycloplegic agent. With her right eye, she says it is bright red. With her left eye, she says it
looks like its covered with wax. You are certain she has optic neuritis, so arrange for:
a. emergent double-contrast brain CT.
b. high-dose intravenous immunoglobulin therapy.
c. continuous mannitol infusion.
d. hyperbaric oxygen therapy.
e. urgent neurologic and ophthalmologic follow-up.

844. The most consistent finding in patients with a cauda equina syndrome is:
a. absent anal wink.
b. absent bulbocavernosus reflex.
c. low back pain.
d. urinary incontinence.
e. urinary retention.

845. Spinal subarachnoid hemorrhage:


a. can present like an intracerebral subarachnoid bleed.
b. cannot be diagnosed with lumbar puncture, as the bleed is extradural.
c. causes a painless paraplegia.
d. is best diagnosed using contract CT scan.
e. is usually the result of a patient being placed on anticoagulants.

846. Guillain-Barr syndrome is the most common cause of demyelinating polyneuropathies. It is


usually recognized through the findings of progressive symmetric:
a. distal motor weakness and hyperreflexia.
b. lower extremity weakness more than upper extremity weakness and hyperreflexia.
c. proximal and distal musculature weakness worse in the lower extremities, and hyporeflexia.
d. proximal motor weakness and hyperreflexia.
e. upper extremity weakness more than lower extremity weakness and hyporeflexia.
275

841. b Asymmetric sensorineural hearing loss has few causes other than acoustic neuroma.
th
Rosen 5 , Mnires disease may present a diagnostic dilemma because it can be asymmetrical.
Chapter 99, Mnires disease may be differentiated from acoustic neuroma in that the tinnitus of
p. 1491 Mnires disease is usually intermittent, while the tinnitus of acoustic neuroma is
typically continuous. In addition, patients with Mnires disease typically describe
true vertigo, whereas patients with an acoustic neuroma are more likely to describe
imbalance or dysequilibrium.
842. c Injury to the vagus nerve causes unilateral loss of palatal elevation: Patients complain
Rosen 5th, that on drinking liquids the fluid refluxes through the nose.
Chapter 99,
p. 1486,
Table 99-1
843. e Approximately 30% of patients presenting with acute optic neuritis will develop
th
Rosen 5 , multiple sclerosis within 5 years. In an initial study of patients with acute optic
Chapter 181, neuritis, treatment with a 3-day course of IV methylprednisolone reduced the rate of
p. 2553 development of multiple sclerosis over a 2-year period. However, 5-year follow-up of
this same patient cohort revealed no significant differences among treatment groups in
the development of multiple sclerosis. Use of oral steroids for hastening optic neuritis
is controversial. The Optic Neuritis Study Group had shown an increased risk of optic
neuritis recurrences in those patients treated with oral prednisone. However, a recent
randomized and controlled study of high-dose oral methylprednisolone in acute optic
neuritis showed improved recovery from optic neuritis at 1 and 3 weeks, but no effect
at 8 weeks or on subsequent attack frequency. Long-term visual outcome is no
different than observation alone.
844. e The cauda equina (horses tail) is the name given to the lumbar and sacral nerve
Rosen 5th, roots that continue on within the dural sac caudal to the conus medullaris. The
Chapter 100, etiology of the cauda equina syndrome is usually a ruptured, midline intervertebral
p. 1499 disk, most commonly occurring at the L4-L5 level. Tumors and other compressive
masses may cause the syndrome as well. Patients generally present with progressive
symptoms of fecal or urinary incontinence, impotence, distal motor weakness, and
sensory loss in a saddle distribution. Muscle stretch reflexes may also be reduced.
The presence of urinary retention is the single most consistent finding, with a
sensitivity of 90%. Low back pain may or may not be present.
845. a Patients with spinal subarachnoid bleed present with the paroxysmal onset of
Rosen 5th, excruciating back pain at the level of the hemorrhage. This pain may also be in a
Chapter 100, radicular distribution or into the flank. Patients may complain of headache and exhibit
p. 1502 cervical rigidity if the blood migrates into the intracranial subarachnoid space
simulating an intracranial subarachnoid hemorrhage. The diagnostic study of choice is
the MRI. Lumbar puncture also confirms the diagnosis of blood in the CSF.
846. c The majority of patients seek treatment days to weeks after resolution of a respiratory
Rosen 5th, or gastrointestinal illness and present in the ED with progressive, symmetric weakness
Chapter 101, of proximal and distal musculature. Signs and symptoms are worse in the lower
p. 1508 extremities and are associated with diminution or loss of deep tendon reflexes, variable
sensory findings, and sparing of the anal sphincter. Urinary retention from autonomic
dysfunction may occur, contributing to a clinical picture easily mistaken for a spinal
cord lesion or conus medullaris syndrome.
276

847. You are seeing a 67-year-old man in whom you suspect Guillain-Barr syndrome. His vital
capacity is adequate at 3 liters. Another way to monitor impending respiratory failure is to follow
his:
a. anal wink.
b. biceps strength.
c. bulbocavernosus reflex.
d. neck muscle extensors.
e. triceps strength.

848. Patients with distal symmetric polyneuropathies:


a. get numbness in their fingertips before lower extremity numbness reaches their knees.
b. get numbness on the tops of their feet before numbness reaches the plantar surface.
c. have weak foot dorsiflexion as their first motor sign.
d. improve transiently with non-steroidal anti-inflammatory treatment.
e. lose their ankle-jerks before they develop footdrop.

849. Select the correct pairing for the cause of upper extremity mononeuropathies:
a. Bridegrooms palsy = ulnar nerve.
b. Guyons canal = median nerve.
c. LOAF weakness = median nerve.
d. Saturday night palsy = ulnar nerve.
e. Tardy palsy = radial nerve.

850. The most striking feature of a complete common peroneal mononeuropathy is:
a. dorsal foot fasciculations.
b. footdrop.
c. great toe gangrene.
d. loss of ankle jerk.
e. total foot paresthesias.

851. The combination of weakness and sensory changes is found in patients with:
a. amyotrophic lateral sclerosis (ALS).
b. Guillain-Barr syndrome.
c. Lambert-Eaton myasthenic syndrome.
d. myasthenia gravis.
e. poliomyelitis.

852. Which statement is true concerning the edrophonium test, used in helping to differentiate
myasthenic crisis from cholinergic crisis.
a. A maximum dose of 5 mg can be given intravenously.
b. An intravenous test dose of 1 to 2 mg followed by muscle fasciculations and respiratory
depression demonstrates an allergic reaction and further edrophonium administration is
contraindicated.
c. Dramatic improvement of symptoms within 90 seconds is proof of cholinergic crisis.
d. The test is graded by measuring the distance between the upper eyelid and lower eyelid in the
most severely affected eye before and after intravenous edrophonium.
e. Edrophonium is a short-acting acetylcholinesterase stimulator.
277

847. d In patients with possible Guillain-Barr Syndrome who have normal pulmonary
th
Rosen 5 , function, extensor neck strength can be monitored to predict impending ventilatory
Chapter 101, failure.
p. 1509
848. a Distal symmetric polyneuropathies are the most common type of peripheral neuropathy
th
Rosen 5 , seen in emergency practice, with the preponderance of cases occurring in diabetics. As
Chapter 101, the process advances, the plantar surfaces of both feet will become dysesthetic before
p. 1510 the dorsum of either foot is involved. Weakness of dorsiflexion of the big toe is
usually the first motor sign, followed by weakness of foot dorsiflexion, foot drop, loss
of ankle jerks, and later a steppage gait. Sensory loss continues to move proximally,
and before it reaches the knees, the fingertips are usually involved.
849. c Most radial mononeuropathies are due to so-called Saturday night palsies, derived
Rosen 5th, from the association of improper positioning of the arm during deep, commonly
Chapter 101, inebriated sleep. Consequently, the radial nerve is trapped for a prolonged period
p. 1513 between the humeral shaft and some firm surface, causing an external compression
mononeuropathy. Bridegrooms palsy is another eponym for radial
mononeuropathy, so named because the brides head resting on the bridegrooms arm
during sleep may compress the radial nerve. A relatively unique feature of the ulnar
nerve is its propensity to develop tardy ulnar palsy occurring years after a traumatic
event. Compression of the ulnar nerve within Guyons canal is rare, but when it
occurs, it will affect all of the ulnar intrinsics and all the interossei. Median nerve
motor involvement in carpal tunnel syndrome is confined to the median intrinsics,
which innervate the Lumbricals (flexion of the MCP joints), and power thumb
Opposition, Abduction, and Flexion, therefore known as the LOAF muscles.
850. b The most striking feature of a complete common peroneal mononeuropathy is foot
Rosen 5th, drop caused by weakness of foot dorsiflexion. At testing, the everters of the foot are
Chapter 101, also weak, but the inverters, which are innervated by the tibial nerve, remain strong.
p. 1517 This may be the single most reliable feature distinguishing sciatic from common
peroneal mononeuropathy.
851. b Guillain-Barr syndrome is a sensorimotor polyneuropathy. Amyotrophic lateral
Rosen 5th, sclerosis results from a degeneration of the motor neuron without sensory involvement.
Chapter 101, Poliomyelitis affects the anterior horn cells and results in lower motor neuron disease
p. 1508- without sensory involvement. Muscular weakness and fatigability are the hallmarks of
1509, 1518 myasthenia gravis, but there are no sensory losses. The Lambert-Eaton myasthenic
Chapter 102, syndrome affects both nicotinic and muscarinic receptors. With repeated stimulation
p. 1522, 1523 the amount of ACh in the synaptic cleft increases, leading to an increase in strength,
the opposite of that seen with MG.
852. d The edrophonium (Tensilon) test is performed by measuring the distance from the
Rosen 5th, upper to the lower eyelid in the most severely affected eye before and after intravenous
Chapter 102, administration of the short-acting AChE blocking agent edrophonium. An IV test dose
p. 1523 of 1 to 2 mg is given first. Muscle fasciculations and respiratory depression within a
few minutes suggests that the muscle weakness is related to a cholinergic crisis, and
further edrophonium administration is contraindicated. If no adverse reaction is found,
and the patient does not dramatically improve in 30 to 90 seconds, a second dose of 3
mg is given. If there is still no response, a final dose of 5 mg is given for a total
maximum dosage of 10 mg in order to demonstrate benefit in the face of a presumed
myasthenic crisis.
278

853. Classic features of botulism would include:


a. ascending paralysis.
b. bowel and bladder incontinence
c. difficulty swallowing.
d. pinpoint pupils.
e. sore throat.

854. You are performing lumbar puncture on a 29-year-old woman with the worst headache of her
life. She is in the left lateral decubitus position. You recognize that:
a. a normal CSF protein is up to 45 mg/dl.
b. a normal opening pressure is 200 to 300 mmH2O.
c. an abnormal closing pressure must be accounted for.
d. normal CSF contains as many as 10 WBCs / mm3.
e. the CSF to serum glucose ratio is normally 0.4 to 1.

855. When peripheral cell counts are normal, the cerebrospinal fluid from a traumatic lumbar puncture
should contain around 1 WBC per:
a. 400 RBCs.
b. 500 RBCs.
c. 700 RBCs.
d. 1000 RBCs.
e. 1200 RBCs.

856. The vast majority of aseptic meningitis in the United States is caused by:
a. adenovirus.
b. arbovirus.
c. cytomegalovirus.
d. enterovirus.
e. herpes virus.

857. Your hospitals infection control nurse notifies you that the infant you admitted with meningitis is
growing meningococcus from both blood and cerebrospinal fluid cultures. You know that
prophylaxis with rifampin should be mandatory for:
a. daycare personnel where the child spent 6 hours, 2 days prior to presentation.
b. emergency department nursing personnel,
c. paramedics who performed mouth-to-mouth prior to intubating the child.
d. the resident who performed the spinal tap.
e. all of the above.

858. Surgery for intracerebral hemorrhage is generally not indicated, although it has been reported to be
life saving in selective cases of bleeding in the:
a. brainstem.
b. cerebellum.
c. pons.
d. putamen,
e. thalamus.
279

853. c The classic feature of botulism is a descending symmetric paralysis. Dysphagia,


th
Rosen 5 , diplopia, and dysarthria occur early. The toxin inhibits cholinergic output, leading to
Chapter 102, constipation and urinary retention. There is no pain. Pupils are often dilated and
p. 1525 nonreactive.
854. a The normal CSF protein in adults ranges from 15 to 45 mg/dL. An elevated CSF
th
Rosen 5 , protein, usually higher than 150 mg/dL commonly occurs with acute bacterial
Chapter 103, meningitis. CSF glucose is normally in a ratio of 0.6 to 1 to the serum glucose, except
p. 1534 with marked systemic hyperglycemia, when the ratio is closer to 0.4 to 1. The normal
CSF pressure in an adult varies from 50 to 200 mm H2O. This may increase several-
fold when in the sitting position. Normal adult CSF contains no more than 5
leukocytes/mm3 with at most one granulocyte (PMN). Closing pressure is not
necessary.
855. c When peripheral cell counts are normal, the CSF from a traumatic lumbar puncture
Rosen 5th, should contain around 1 WBC per 700 RBCs.
Chapter 103,
p. 1533
856. d Viral meningitis may likewise be caused by a variety of etiologic agents.
th
Rosen 5 , Enteroviruses are statistically encountered most commonly. Unfortunately, precise
Chapter 103, definition of the etiologic agent is often impossible.
p. 1528
857. c Health care workers are not at any increased risk for the disease and do not require
th
Rosen 5 , prophylaxis unless they have had direct mucosal contact with the patients secretions,
Chapter 103, as might occur during mouth-to-mouth resuscitation, endotracheal intubation, or
p. 1538 nasotracheal suctioning. The general rule is 2 hours within 2 feet of a clinically ill
patient to require prophylaxis.
858. b Surgery is not beneficial in most cases of intracerebral hemorrhage. Surgery is more
Rosen 5th, efficacious in patients with cerebellar hemorrhage. The clinical course in cerebellar
Chapter 95, hemorrhage is notoriously unpredictable. Patients with minimal findings may
p. 1443 deteriorate suddenly to coma and death with little warning. For this reason, most
neurosurgeons will consider emergency surgery for patients with cerebellar
hemorrhage within 48 hours of onset.
280

859. The ascending reticular activating system (ARAS) is located in the:


a. cerebellum.
b. hippocampus.
c. hypothalamus.
d. pons.
e. thalamus.

860. Coma preceded by delirium suggests:


a. alcohol intoxication.
b. endogenous toxins, such as ammonia.
c. hypoglycemia.
d. hypoxemia.
e. stroke.

861. The paramedics have brought you a 43-year-old man who is completely unresponsive and smells
heavily of fermented beverage. The recommended method of noxious stimulus to arouse him is:
a. inserting a cotton swab into his posterior nasopharynx.
b. pinching his nipples.
c. squeezing his fingernail beds.
d. twisting his eyebrows.
e. yelling in his ears

862. Dysconjugate gaze in the vertical plane generally results from:


a. alcohol intoxication.
b. barbiturate intoxication.
c. hypoglycemic coma.
d. hypoxemia.
e. pontine or cerebellar lesions.

863. The cardinal sign of inner ear disease is:


a. abnormal nystagmus.
b. ataxia.
c. dizziness.
d. dysdiadochokinesis.
e. hearing loss.

864. A feature that is characteristic of central vertigo is:


a. brief duration.
b. gradual onset.
c. hearing loss.
d. severe intensity, especially with head movement.
e. tinnitus.

865. An 84-year-old woman with a history of coronary artery disease complains of new onset of brief
episodes of dizziness, dysarthria, ataxia, and double vision. She has a very ataxic gait, but an
otherwise nonfocal exam. Her most likely diagnosis is:
a. benign positional vertigo.
b. cerebellar hemorrhage.
c. Mnires disease.
d. purulent labyrinthitis.
e. vertebrobasilar insufficiency.
281

859. d The ascending reticular activating system (ARAS) is the neuroanatomic structure
th
Rosen 5 , responsible for arousal. It is located in the pons. The ARAS acts as a gateway for
Chapter 15, sensory and somatic stimuli to the cerebral cortex and as a trigger for arousal from
p. 137 sleep. When the ARAS is dysfunctional, coma occurs because the cerebral cortex
cannot be aroused.
860. a Coma preceded by delirium suggests alcohol intoxication, sedative withdrawal, toxic
Rosen 5th, psychosis, or encephalitis. Coma preceded by confusion without delirium suggests
Chapter 15, hypoglycemia, endogenous toxins, or drug overdose.
p. 140
861. a A variety of noxious stimuli have been used, but perhaps the best stimulus is to tickle
th
Rosen 5 , the posterior nasopharynx with a cotton swab. This is not harmful or painful, does not
Chapter 15, leave a mark, and can be reproduced accurately by subsequent examiners.
p. 141
862. e Dysconjugate gaze in the vertical plane, called skew deviation, generally results from
th
Rosen 5 , pontine or cerebellar lesions. Dysconjugate gaze in the horizontal plane is normally
Chapter 15, observed in drowsiness and in various sedated states including alcohol intoxication,
p. 141 with parallel ocular axes reemerging when the patient awakens or slips deeper into
coma.
863. a Abnormal nystagmus is the cardinal sign of inner ear disease and the principal
Rosen 5th, objective evidence of abnormal vestibular function. The abnormal jerk nystagmus of
Chapter 13, inner ear disease consists of slow and quick components. The eyes slowly move in the
p. 125 direction of the diseased, hypoactive ear and then quickly jerk back to the midline.
Nystagmus caused by vestibular disease tends to be unidirectional and
horizontorotatory. If the nystagmus is vertical, a central lesion (either brainstem or
cerebral) is usually the cause.
864. b The main features of central vertigo are gradual onset; mild intensity; duration of
Rosen 5th, weeks to months; horizontal, rotatory, or vertical nystagmus (different directions in
Chapter 13, different positions); little change with head position; associated with more than one
p. 125 position; associated neurologic findings; and absence of auditory findings.
865. e Vertebrobasilar insufficiency (VBI) should be considered in any patient of advanced
Rosen 5th, age who has isolated new-onset vertigo without an obvious cause. The diagnosis is
Chapter 13, more likely in patients with a history of atherosclerosis. Initial episodes usually last
p. 127 seconds to minutes. Headache may be present. Neurologic symptoms may include
dysarthria, ataxia, weakness, numbness, and double vision. Tinnitus and deafness are
uncommon. Neurologic deficits are usually present but initial neurologic examination
may be normal. Because of the possibility of progression of new-onset VBI in the first
2472 hours, hospital admission and consideration of early MRA are probably
warranted even in the stable patient.
282

13.0 Obstetrics and Gynecology


866. There is an increased risk of candidal vaginitis in a woman:
a. with diabetes.
b. with frequent bladder infections.
c. who is menstruating.
d. who has gone through menopause.
e. who has multiple sex partners.

867. You have just diagnosed a 23-year-old woman with trichomonas vaginitis and wish to treat her with
oral metronidazole. You know also know that:
a. concomitant ingestion of ethanol inactivates the medication.
b. her partner requires treatment.
c. oral metronidazole is contraindicated if she has a near-term pregnancy.
d. there are thus far no reports of trichomonas being resistant to metronidazole.
e. trichomonas can become very aggressive during pregnancy.

868. Chlamydial infections:


a. are best treated with ceftriaxone (Rocephin).
b. are easily diagnosed because of copious discharge.
c. are frequent causes of infertility.
d. present exclusively as cervicitis or pelvic inflammatory disease.
e. should not be treated unless infection has been proven by culture or assay.

869. Which statement about pelvic inflammatory disease (PID) is true?


a. Symptoms usually begin just before the patients menstrual period.
b. PID is common in pregnancy.
c. Chronic PID is nearly always due to Neisseria gonorrhoeae.
d. Successful outpatient treatment of PID involves ceftriaxone (Rocephin) 125mg IM and twice-
daily doxycycline for 7 days.
e. Irregular bleeding is reported in about 40% of women diagnosed with PID.

870. Uterine fibroids are associated with:


a. Caucasian race.
b. frequent episodes of cystitis.
c. IUD use.
d. menorrhagia.
e. unexplained fevers.

871. A 37-year-old woman complains of a single genital ulcer for three days.
a. Absence of vaginal discharge reliably rules out granuloma inguinale.
b. Herpes can easily be ruled out with a negative Tzanck prep.
c. Lyme disease must be in your differential.
d. Non-treponemal antibody tests (VDRL and RPR) are often negative at this stage, so you cannot
reliably rule out syphilis.
e. The borders of the ulcer are ragged and the inguinal nodes are tender and swollen, so you can rule
out chancroid.
283

866. a Candida albicans, a yeast-like organism, is probably the most common cause of
th
Rosen 5 , vaginitis. Predisposing factors include antibiotic administration, pregnancy, oral
Chapter 93, contraceptives, steroid administration, restrictive clothing, and diabetes.
p. 1396
867. b Metronidazole (Flagyl) is a Category B drug in pregnancy, and contraindicated in
th
Rosen 5 , first trimester, but can be used in later trimesters if the benefits outweigh the risk.
Chapter 174, Trichomonas is rarely aggressive during pregnancy.
p. 2449
868. c Chlamydia trachomatis is an obligate, intracellular parasite that has the features of both
th
Rosen 5 , a virus and a bacterium. It is now the most common of all STDs and is one of the
Chapter 93, principal causes of infertility in women. An estimated 1 in 10 adolescent girls and 1 in
p. 1395 20 women of reproductive age are infected. In addition to causing LGV, acute urethral
syndrome, and PID, the organism causes mucopurulent cervicitis that is difficult to
clinically distinguish from that caused by N. gonorrhoeae. Chlamydia causes
nongonococcal urethritis and epididymitis in men. Men may also be asymptomatic
carriers of the infection. Azithromycin, 1 g orally in a single dose, is the preferred
treatment. Chlamydial infections also respond to doxycycline, 100 mg orally twice a
day for 7 days; or an erythromycin base 500 mg orally four times daily for 7 days.
869. e Typically, acute PID is manifested by an increased vaginal discharge, pelvic pain, or
Rosen 5th, symptoms of urethritis, beginning 3 to 5 days after menstruation. A history of
Chapter 93, antecedent irregular bleeding is elicited in 40% of cases. N. gonorrhoeae and
p. 1397-1398 Chlamydia organisms together account for approximately 80% of all cases of PID.
Table 93-4 Treatment: ceftriaxone, 250 mg IM OR cefoxitin, 2 g IM, + probenecid 1 g po AND
doxycycline, 100 mg po bid for 1014 days OR ofloxacin 400 mg po bid for 14 days
AND EITHER clindamycin 400 mg po qid for 14 days OR metronidazole 500 mg po
bid for 14 days.
870. d Leiomyomas (fibroids) are benign tumors of muscle cell origin and are the most
Rosen 5th, frequently occurring pelvic tumor. They are found in one of four white women and in
Chapter 28, one of two black women. They decrease in size during menopause, and enlargement is
p. seen early in pregnancy and, in some cases, oral contraceptive use. Up to 30% of
patients with leiomyomas experience pelvic pain and abnormal bleeding.
871. d Pain and the puttylike consistency of the ulcers distinguish chancroid from other
Rosen 5th, causes of genital ulcers. A painful unilateral inguinal bubo develops in 50% of
Chapter 93, patients. In primary syphilis, a small papule develops at the site of inoculation 10 to 90
p. 1394 days after exposure, which then becomes a painless, indurated ulcer, the classic
chancre, which heals spontaneously in 4 to 5 days. Serologic tests are often negative at
this stage. Tzanck slide preparation reveals the characteristic multinucleated giant
cells of herpes only about 50% of the time.
284

872. A 19 year-old college student presents with a first episode of Herpes genitalis.
a. Counseling should involve warnings about increased risk of PID.
b. Early treatment with acyclovir will prevent future outbreaks.
c. Famcyclovir intravenously will successfully prevent recurrences.
d. Systemic analgesics may be needed.
e. Systemic antibiotics are mandatory to prevent generalized dissemination.

873. A woman with pyelonephritis can safely be treated at home unless she:
a. has a prior history of hospitalization for pyelonephritis.
b. has an allergy to sulfa.
c. has ureteral obstruction due to stone.
d. is over age 45.
e. lives alone.

874. A previously healthy 56 year-old perimenopausal woman presents with dysfunctional uterine
bleeding. Her pregnancy test is negative, and pelvic examination reveals no pathology. Ultrasound
shows no structural abnormality.
a. Anovulatory bleeding is usually heavy, with long intervals between menstrual periods.
b. Coagulopathy screen (PT, aPTT, bleeding time) is mandatory.
c. Fallopian carcinoma still needs to be ruled out.
d. NSAIDs should be avoided, as they tend to worsen bleeding.
e. Pap smear should reveal the cause of bleeding.

875. Chlamydial organisms cause:


a. chancroid.
b. condyloma accuminata
c. granuloma inguinale.
d. lichen sclerosis et atrophicus.
e. lymphogranuloma venereum.

876. A 17 year-old female complains of vaginal itching. She has never been sexually active. She has
some mild redness around the introitus. Pregnancy test, wet mount, and KOH prep are negative.
a. Environmental exposures, such as douches, soaps, and body creams, should be eliminated.
b. Gynecologic consultation is mandatory.
c. She requires broad-spectrum antibiotic coverage for presumed occult infection.
d. She should stop drinking grapefruit juice, as she is developing an allergic dermatitis.
e. You can reassure the patient there is no pathology and let her go home.

877. For Morning after pills to be successful in preventing pregnancy, they must be taken within what
period of time after intercourse?
a. 12 hours
b. 24 hours
c. 36 hours
d. 72 hours
e. 7 days
285

872. d Treatment is not curative. Multiple antiviral agents and regimens are available.
th
Rosen 5 , Systemic antiviral agents provide partial control of the signs and symptoms and
Chapter 93, accelerate healing of the lesions, but do not affect the frequency or severity of
p. 1392 recurrences. Topical therapy is not effective. Patients with severe disease may need
hospitalization and intravenous therapy.
873. c Outpatient therapy for selected patients with pyelonephritis (young, otherwise healthy,
Rosen 5th, able to keep down fluids and antibiotics) is as safe and effective, and considerably less
Chapter 94, expensive, than comparable patients treated on an inpatient basis. Hospitalization is
p. 1410 required in the presence of (1) clinical toxicity, (2) inability to take oral medications,
(3) an immunocompromised state, (4) pregnancy, or (5) urologic abnormalities.
874. a In nonpregnant patients, most vaginal bleeding is related to dysfunctional
Rosen 5th, (anovulatory) uterine bleeding. This is usually managed with estrogen
Chapter 28, supplementation in consultation with a gynecologist. Other causes, such as neoplasm,
p. 229 suspected endometriosis, or ovarian cysts, are referred to a gynecologist. Before
discharge, it is important to assess the patients tolerance of the hemorrhage by
measuring vital signs, including orthostatics. A baseline hemoglobin/hematocrit is
recommended. Finally, other medical causes, such as hypothyroidism, hemostasis
disorders, or anticoagulant therapy, must be considered.
875. e Lymphogranuloma venereum (LGV) is an uncommon STD caused by C. trachomatis.
Rosen 5th, The initial lesion is a small, shallow, painless, evanescent vesicle, pustule, or ulcer that
Chapter 93, occurs on the labia, cervix, or penis 7 to 21 days after exposure. The patient usually
p. 1393 does not notice the lesion. Two to 12 weeks after exposure, localized inguinal lymph
nodes develop that can enlarge, coalesce, and ulcerate impressively. The groove
sign may appear to be created by the proliferation of inguinal lymphadenopathy
superior and inferior to the inguinal ligament.
876. a Chemical irritant vaginitis commonly occurs after the use of feminine hygiene
Rosen 5th, deodorants, douches, or medications not intended for the vagina. Diagnosis is by
Chapter 93, history. Stopping the use of the offending agent usually is sufficient treatment.
p. 1397
877. d Pregnancy prophylaxis must be initiated within 72 hours of the sexual encounter in
Rosen 5th, order to be most effective. Currently accepted therapy is the birth control pill Ovral
Chapter 61, (norgestrel plus ethinyl estradiol) 2 tablets orally initially and 2 tablets 12 hours later.
p. 860 Commercial preparations, called Preven and Plan B, are also available.
Box 61-3
286

878. A 25 year-old woman presents with pelvic pain and mucoid vaginal discharge. Her most recent
period ended 7 days ago. Examination shows right upper quadrant tenderness. She probably has:
a. non-infectious hepatitis.
b. cholecystitis.
c. concurrent right lower lobe pneumonia.
d. Fitz-Hugh-Curtis syndrome.
e. tubo-ovarian abscess.

879. An appropriate treatment for primary syphilis is:


a. ceftriaxone, 250 mg IM, daily for 5 days
b. doxycycline, 100 mg PO BID for 7 days
c. erythromycin, 500 mg PO QID for 7 days
d. levofloxacin 250 mg PO BID for 3 days.
e. penicillin G benzathine, 2.4 million units IM, as a single dose

880. Choose the correct statement:


a. Untreated bacteriuria in pregnancy is associated with a higher incidence of fetal morbidity.
b. Pregnancy causes increased peristalsis throughout the collecting system.
c. The prevalence of bacteriuria in women increases with pregnancy.
d. Asymptomatic bacteriuria in a pregnant woman can be observed.
e. Treatment with trimethoprim-sulfamethoxazole is contraindicated.

881. A sexually transmitted disease whose lesions have been reported to turn malignant, especially in
HIV-positive men, is:
a. chancroid.
b. condyloma accuminata.
c. gonorrhea.
d. granuloma inguinale.
e. syphilis.

882. Absolute contraindication to Emergency Department speculum and manual pelvic examination is:
a. active labor.
b. delivery less than one week prior to exam.
c. possible septic abortion.
d. third trimester bleeding.
e. vaginal discharge.

883. Choose the correct statement concerning urinary tract infection:


a. Bacteria most often enter the urinary tract via hematologic seeding.
b. UTIs in men generally begin to appear with the onset of prostatic hypertrophy.
c. A single catheterization in an outpatient setting carries a risk of infection of 5% to 10%.
d. About 10% of women with sterile bladder urine will grow 1000 to 100,000 bacterial colony
forming units (CFU) per milliliter from a midstream clean-catch specimen.
e. In men the sterility of the specimen is significantly affected by lack of cleansing.

884. Mammary thrombophlebitis (Mondors syndrome) is commonly associated with:


a. abscess and/or fistula.
b. cystic breast disease.
c. multiparity.
d. recent breast surgery.
e. menarche.
287

878. d Fitz-Hugh-Curtis syndrome (gonococcal perihepatitis) is an inflammation of the liver


th
Rosen 5 , capsule that results in band-like adhesions between the liver and the anterior abdominal
Chapter 93, wall. Typically the patient has a sudden onset of severe, pleuritic, sharp, right upper
p. 1398 quadrant pain that occurs days to weeks after symptoms of acute PID. Examination
usually reveals profound tenderness over the liver, some lower abdominal tenderness,
cervicitis, and culture evidence of PID. Laparoscopy may be required in diagnosis and
lysis of adhesions.
879. e Benzathine penicillin G 2.4 (Rosen says 24, an obvious typographical error) million
Rosen 5th, units IM in a single dose has remained the standard of care for primary syphilis.
Chapter 93, Doxycycline 100 mg PO bid for 2 weeks may be used for allergic individuals.
p. 1393
880. a Untreated bacteriuria in pregnancy is associated with a higher incidence of prematurity
th
Rosen 5 , and fetal morbidity. Maternal complications include a 20% to 40% incidence of acute
Chapter 94, pyelonephritis and an increased incidence of postpartum chronic pyelonephritis. The
p. 1406 physiologic changes that occur within the urinary tract of pregnant women include
ureteral and renal pelvis dilatation and reduced peristalsis throughout the collecting
system. The prevalence of bacteriuria in women does not change with pregnancy.
However, in contrast to bacteriuria in nonpregnant females, bacteriuria in pregnant
women, even if they are asymptomatic, must be treated. Reasonable antibiotic choices
include amoxicillin, cephalexin, and nitrofurantoin. Some authors recommend
trimethoprim-sulfamethoxazole if used before the third trimester.
881. b Condylomata acuminata are most often found in homosexual men but can be seen in
Rosen 5th, heterosexual men, women, and children. Because one half of HIV-positive patients
Chapter 91, have anal warts, HIV testing is recommended. The pink-to-gray warts are a result of
p. 1354-1355 hyperplastic epithelial growth and appear as vegetative papilliform growths. They may
coalesce to form a massive patch that obscures the anal verge. Squamous cell
carcinoma should be considered if the lesions are indurated. Progression to
intraepithelial neoplasia has been reported to be related to the level of
immunosuppression.
882. d In a patient with third trimester placenta previa, instrumentation, digital examination of
Rosen 5th, the cervix, and speculum examination can provoke severe, exsanguinating hemorrhage,
Chapter 175, and are therefore contraindicated.
p. 2741
883. b Bacteria most often enter the urinary tract via ascent through the urethra and into the
th
Rosen 5 , collecting system. UTIs in men generally begin to appear at age 50 years, concomitant
Chapter 94, with the onset of prostatic hypertrophy. A single catheterization in an outpatient
p. 1401-1403 setting carries a risk of infection of 1% to 3%. Up to 50% of women with sterile
bladder urine grew 1000 to 100,000 bacterial colony forming units per milliliter from a
midstream clean-catch specimen. In men, the specimen is not affected significantly by
lack of cleansing or by the timing of specimen collection.
884. d Mondors disease of the breast is a superficial phlebitis of the veins in the
Rosen 5th, subcutaneous tissue of the breast. This condition may occur postoperatively or after
Chapter 83, minor trauma. The patient presents complaining of a painful induration across the
p. 1217 costal margin. A cord may be present. The phlebitis may be tender for several weeks.
No treatment other than analgesia is required, but spontaneous fibrosis of the vessel
often leads to retraction of subcutaneous tissues, with puckering and hardening of one
quadrant of the breast. Follow-up evaluation with a surgeon is indicated. Mondors
disease of the penis has also been described.
288

885. A 15 year-old woman tells you that she is 7 weeks pregnant and her boyfriend recently tested
positive for Chlamydia and she wants to be treated. She plans on carrying the pregnancy to term.
She cant afford single-dose azithromycin, so you:
a. give her a prescription for 1 week of doxycycline 100 mg bid.
b. give her a prescription for 1 week of erythromycin 500 mg qid.
c. give her a prescription for 1 week of ofloxacin 300 mg bid.
d. refer her to a public health clinic.
e. refuse to treat her because of her age.

886. A 21 year-old woman is brought by ambulance from her dormitory room at a nearby college. She
appears ill, has a fever of 102.3oF, and a blood pressure of 74/38 mmHg. The palms of her hands
are peeling skin. She refuses a pelvic exam because My period is on and I have a tampon. The
organism which is MOST LIKELY causing her illness is:
a. Chlamydia trachomatis.
b. Staphylococcus aureus.
c. Streptococcus pyogenes.
d. Streptococcus viridans.
e. Yersinia enterocolitica.

887. A 28 year-old woman G5P1112 whose last period was more than 4 months ago presents with
cough, flank pain, and low-grade fever. You would expect to find which of the following as a
result of her presumed pregnancy?
a. increased blood pressure
b. decreased heart rate
c. increased temperature
d. protein, glucose, and ketones in the urine
e. mild anemia

888. A 21-year-old woman G1P0 whose last period was 36 weeks ago presents with dysuria. There is
no sign of pyelonephritis. Your most appropriate antibiotic for treatment is:
a. ampicillin.
b. ciprofloxacin.
c. doxycycline.
d. erythromycin estolate.
e. trimethoprim-sulfamethoxazole.

889. Placenta previa is characterized by:


a. high fever.
b. painful vaginal bleeding.
c. right upper quadrant fullness.
d. tender, tense uterus.
e. third trimester bleeding.

890. You begin magnesium administration in a woman with third trimester pre-eclampsia. You must
monitor her closely for signs of toxicity, which include:
a. fever.
b. hyperreflexia.
c. hypotension.
d. respiratory depression.
e. tachycardia.
289

884. b Clinical diagnosis is difficult during pregnancy because cervical mucus is usually
th
Rosen 5 , cloudy and contains WBCs. Routine Chlamydia screening during pregnancy is
Chapter 172, important to prevent complications of preterm labor and postpartum endometritis, both
p. 2430 of which are more common in patients who have chlamydial cervical infections.
Chlamydial conjunctivitis has become epidemic among infants born to mothers with
chlamydial cervical infections. In addition, neonatal pneumonia caused by Chlamydia
is common. Recommended treatment during pregnancy or breastfeeding is a 7-day
course of erythromycin or amoxicillin.
885. b Toxic shock syndrome often occurs in menstruating women who use vaginal tampons.
Rosen 5th, Staphylococcus aureus is isolated more than 90% of the time.
Chapter 131,
p. 1399
887. e The hemoglobin decreases slightly in pregnancy. The Centers for Disease Control and
th
Rosen 5 , Prevention defined anemia in pregnancy as less than 11 g/dl during the first and third
Chapter 171, trimesters and less than 10.5 g/dl during the second trimester.
p. 2400-2401
888. a Pregnant women with UTI are at increased risk for pyelonephritis, and pyelonephritis
th
Rosen 5 , is a risk factor for preterm labor and low birth weight, as a result of hydroureter,
Chapter 171, dilatation of the renal pelvis, and consequent urinary stasis. Pregnant patients with
p. 2430 asymptomatic bacteriuria identified on microscopic analysis should have a urine
culture performed and be treated if the culture is positive. Untreated, 25% will develop
symptomatic infection, while treatment will decrease the incidence of symptomatic
UTI by 80 to 90%. Symptomatic UTI requires a urine culture and antibiotic therapy.
Only 7-day regimens are recommended during pregnancy, including nitrofurantoin
(one tablet bid), ampicillin; or cephalexin. Trimethoprim-sulfamethoxazole is
relatively contraindicated in the first trimester because of anti-folate properties of
trimethoprim, and sulfonamides are contraindicated in the third trimester due to risk of
fetal hyperbilirubinemia.
889. e The classic symptom of placenta previa is painless bright red vaginal bleeding in the
Rosen 5th, third trimester. The uterus remains soft and fetal lie is often abnormal. The initial
Chapter 175, bleeding is often self-limited and not lethal. This bleeding usually occurs as cervical
p. 2471 effacement exposes the placenta. The earlier in the effacement process the bleeding
begins, the lower the placenta lies.
890. d Magnesium administration should always be accompanied by clinical observation for
Rosen 5th, loss of reflexes (which occurs at about 10 mg/dl) or respiratory depression (which
Chapter 172, occurs at levels over 12 mg/dl, although actual serum magnesium levels are rarely
p. 2430 monitored). The infusion should be stopped if signs of hypermagnesemia are seen;
such patients may require assisted ventilation. Calcium gluconate, 1 g given slowly
into a secure vein, will reverse the adverse effects of hypermagnesemia.
290

891. A 16-year-old woman G2P0020 complains of intermittent crampy right lower abdominal pain. She
thinks her last period was about 3 weeks ago, but is not certain. Her vital signs are normal. Urine
pregnancy test is positive, and quantitative beta-HCG is 1123 mIU. Transvaginal ultrasound shows
an intrauterine gestational sac, a fluid-filled cyst in the right adnexa, and a small amount of fluid in
the cul-de-sac. Your next step should be to:
a. administer RhoGAM and consult gynecology for emergent laparotomy.
b. check Rh status and discharge patient on methotrexate therapy, since she is stable.
c. refer for prenatal care; history and findings are consistent with intrauterine pregnancy and a
corpus luteum cyst.
d. obtain type and screen, CBC, and start a large-bore intravenous line in preparation for emergent
laparotomy.
e. obtain type and screen, CBC, and consult gynecology; you cannot rule out an ectopic pregnancy
based on available information.

892. A 24 year-old G2P1 female in her 39th week of pregnancy says she had a a gush of water from my
vagina about one hour ago. Her cervix is closed to your fingertip, but there is fluid in her vaginal
vault. Which test can verify the presence of amniotic fluid?
a. Look under a microscope at a spun specimen white blood cell presence confirms amniotic fluid.
b. Place a drop of the fluid on filter paper or a paper towel if a halo appears around the drop, this
is amniotic fluid.
c. Put a drop of the fluid on a Dextrose measurement stick if less than 60 mg/dL, this is amniotic
fluid.
d. Use a strip of nitrazine paper pH >7.0 indicates presence of amniotic fluid.
e. Use a urine dipstick if positive for protein, amniotic fluid is present.

893. A 35 year-old woman with a history of pregnancy-induced hypertension presents after rupture of
membranes and regular uterine contractions. She is 32-weeks pregnant by dates and size. You can
help suppress premature labor by using:
a. oxytocin.
b. prostaglandin synthase inhibitors.
c. beta-blocking agents.
d. steroids.
e. terbutaline.

894. The HELLP syndrome includes:


a. HE = hepatosplenomegaly.
b. H = hypotension.
c. EL = elevated liver enzymes
d. LL = low liver enzymes
e. LP = low protein.

895. Ectopic pregnancy:


a. can occur in normal fallopian tubes.
b. does not occur after tubal ligation.
c. is the leading cause of second trimester maternal death.
d. usually causes pain before vaginal bleeding.
e. usually causes symptomatic presentation during the first 4 weeks of gestation.
291

891. e No single diagnostic test or combination of laboratory tests is currently considered to


th
Rosen 5 , have sufficient negative predictive value to completely rule out ectopic pregnancy or
Chapter 172, abnormal intrauterine pregnancy or positive predictive value to definitively diagnose
p. 2418 them. Ruptured ectopic pregnancies requiring surgery have been reported with very
low or absent levels of HCG. As a result, it is risky to assume a benign course on the
basis of a low HCG level.
892. d Nitrazine paper testing will reveal a pH of 7.1 to 7.3 typical of amniotic fluid (normal
Rosen 5th, vaginal pH in pregnancy is 3.5 to 6.0).
Chapter 175,
p. 2470
893. e Tocolytic agents
th
Rosen 5 , Magnesium sulfate: 46 g bolus over 30 min, 24 g/hr infusion
Chapter 175, Terbutaline: 2.55.0 mg PO q24h, 0.250.50 mg SC, 0.0100.080 mg/min IV
p. 2469, Ritodrine: 1020 mg PO q4h, 0.0500.350 mg/min IV
Table 175-4 Isoxsuprine: 20 mg PO q4h
894. c The HELLP syndrome (an acronym for hemolysis, elevated liver enzymes, and low
th
Rosen 5 , platelets) is an important clinical variant of preeclampsia that has a predilection for the
Chapter 172, multigravid patient, in contrast to the primigravida, in whom preeclampsia is more
p. 2422 common.
895. a Ectopic pregnancy can occur in normal fallopian tubes and after tubal ligation. It
Rosen 5th, remains the leading cause of maternal death in the first trimester of pregnancy and is
Chapter 172, the second leading cause of maternal mortality overall. Amenorrhea of 4 to 12 weeks
p. 2416 is reported in approximately 70% of cases.
292

896. A 36 year-old woman in her 28th week of pregnancy complains of swollen hands and face. Her
blood pressure is 146/90 mmHg, her BUN is 12 mg/dL, and urine protein is >1 g/L. She has:
a. acute renal failure.
b. chronic hypertension of pregnancy.
c. eclampsia.
d. gestational hypertension.
e. preeclampsia.

897. Perimortem cesarean section is recommended only when:


a. an obstetrician is available to perform the procedure.
b. fetal heart sounds are fading.
c. resuscitation of the mother has stopped.
d. the fetus is at 26 weeks of gestation or greater.
e. volume resuscitation on the mother is completed.

898. The most common cause of death in a pregnant patient with toxemia is:
a. cardiogenic shock
b. cerebral hemorrhage.
c. disseminated intravascular coagulation.
d. renal failure.
e. sepsis.

899. A woman in third trimester of pregnancy with painful, dark vaginal bleeding probably has:
a. abruptio placenta.
b. molar pregnancy.
c. placenta previa.
d. pre-eclampsia.
e. uterine rupture.

900. The most common cause of blunt abdominal trauma in pregnant women is:
a. unknown.
b. domestic violence.
c. falls.
d. motor vehicle crashes.
e. self-inflicted.

901. Expected cardiovascular changes in pregnancy include:


a. heart rate, central venous pressure, blood pressure until it normalizes at term.
b. heart rate, central venous pressure, blood pressure which normalizes near term.
c. heart rate, central venous pressure, blood pressure.
d. heart rate, central venous pressure, blood pressure which normalizes near term.
e. heart rate, central venous pressure, blood pressure which normalizes near term.

902. Initial management of the hypotensive third-trimester trauma patient should include:
a. inflation of MAST garment.
b. left lateral decubitus position.
c. right lateral decubitus position.
d. semi-Fowlers positions when cervical spine cleared.
e. Trendelenberg position.
293

896. e The patient with mild preeclampsia has no evidence of organ damage aside from
th
Rosen 5 , proteinuria and mild systolic or diastolic blood pressure elevation. Mental status
Chapter 172, assessment, testing of reflexes, abdominal examination, liver function studies, and
p. 2422 coagulation studies should all yield normal results. Edema is often difficult to assess,
because pregnancy is normally associated with excess extracellular fluid and
dependent edema. The edema of preeclampsia is usually generalized, involves the face
and hands, and persists throughout the day.
897. d Perimortem cesarean section in the ED should be performed if uterine size exceeds the
Rosen 5th, umbilicus and fetal heart tones are present. Bedside US may also be used to quickly
Chapter 31, assess the presence of a fetal heart rate. No time should be wasted obtaining consent
p. 264 for delivery. Time since maternal circulation ceased is the critical factor in fetal
outcome. Reports show that 70% of children who survive perimortem cesarean
sections are delivered in less than 5 minutes of ED arrival. Accordingly, 4 minutes are
allowed in starting the procedure. The extra minute is used for actual delivery time.
898. b Cerebral hemorrhage is the most common cause of death in toxemia. It occurs in 60%
Rosen 5th, of patients who die after becoming eclamptic.
Chapter 171,
p. 2407
899. a Vaginal bleeding occurs in 80% of patients with abruptio placentae. Blood is
th
Rosen 5 , characteristically dark and the amount is often insignificant, although the mother may
Chapter 172, have hemodynamic evidence of significant blood loss. Uterine tenderness or pain is
p. 2420 seen in about two thirds of women; uterine irritability or contractions are seen in one
third.
900. d The most common cause of blunt abdominal trauma is motor vehicle crash, accounting
Rosen 5th, for up to 70% of acute injuries. This is followed by falls and direct assault in
Chapter 31, decreasing order of frequency.
p. 257
901. d Arterial blood pressure and vascular resistance decrease overall during normal
th
Rosen 5 , pregnancy. Arterial blood pressure decreases during the second trimester and slowly
Chapter 171, rises during the third trimester. Cardiac output at rest in the lateral recumbent position
p. 2401 steadily increases during pregnancy. The changes in blood pressure and cardiac output
during pregnancy reflect the physiologic balance of the decreasing blood pressure and
vascular resistance concurrent with the increase in blood volume, maternal weight, and
basal metabolic rate.
902. b Arterial blood pressure is lowest in the lateral recumbent position and highest when
Rosen 5th, sitting. Usually the blood pressure in the supine position is somewhere between these
Chapter 171, two levels. Some pregnant women become hypotensive in the supine position, referred
p. 2401 to as the supine hypotensive syndrome. Thus a supine pregnant woman with a normal
or hypotensive brachial pressure may have an even lower uterine blood pressure.
Accurate vital signs are best obtained with the patient in the left lateral decubitus
position.
294

17.0 Toxicologic Disorders


1001. Profound ketonuria without acidosis is caused by poisoning with:
a. ethylene glycol.
b. isoniazid.
c. isopropanol.
d. methanol.
e. salicylates.

1002. A 2 year-old male presents two hours after ingesting an unknown quantity of his mothers
prenatal vitamins. He is vomiting and has diarrhea. You should:
a. administer a dose of activated charcoal.
b. immediately administer fluid challenges and IV deferoxamine.
c. obtain an x-ray of the abdomen.
d. obtain serum iron level.
e. obtain WBC and serum glucose studies.

1003. A 21-year-old female with a history of depression was found unconscious and brought to
the emergency department. She is comatose without focal findings, has been intubated,
and is receiving mechanical ventilation. Temperature 99.0oF; heart rate 140/min;
respiratory rate 14 (assisted); BP 80/40 mmHg. Pupils are midposition and reactive and
axillae are dry. An ECG shows sinus tachycardia with a QRS interval of 0.15 seconds.
Arterial blood gas: pH 7.42; pCO2 37, pO2 106. Your next step should be to administer:
a. glucagon.
b. physostigmine.
c. propranolol.
d. sodium bicarbonate.
e. verapamil.

1004. Police bring you a 19-year-old man who is agitated, combative; and seems to be
hallucinating. Temperature 101.4oF, heart rate 120/min, respiratory rate 24/min, BP
130/85 mmHg. His pupils are dilated and he is drenched in sweat. He has most likely
ingested:
a. diphenhydramine.
b. jimson weed.
c. methamphetamine.
d. propoxyphene.
e. Thorazine.

1005. A 22-year-old intravenous drug abuser complains of heroin withdrawal. You would expect
to find:
a. lacrimation, miosis, yawning.
b. restlessness, hallucinations, tachycardia .
c. restlessness, rhinorrhea, vomiting.
d. seizures, miosis, piloerection.
e. seizures, mydriasis, piloerection.
295

1001. c Isopropanol, when metabolized, forms acetone. Acetone is a ketone but not an acid.
th
Rosen 5 , Isopropanol poisoning produces an elevated osmolar gap and ketonuria. Isopropanol
Chapter 149, poisoning does not produce metabolic acidosis and does not produce an elevated anion
p. 2134 gap. Poisoning by ethylene glycol, isoniazid, methanol, and salicylates all produce
metabolic acidosis and an elevated anion gap.
1002. b This patient is experiencing signs and symptoms of moderate iron poisoning and
Rosen 5th, should be treated with intravenous fluid challenges and the antidote deferoxamine.
Chapter 151, Activated charcoal does not bind to iron and should not be administered. Abdominal
p. 2152 x-rays are occasionally helpful, especially in cases of an unknown ingestion. Patients
who experience clinical signs and symptoms of moderate to severe signs iron
poisoning need to be treated aggressively. Treatment is not based on serum iron levels,
WBC counts, or serum glucose studies.
1003. d This patient is displaying signs and symptoms of tricyclic antidepressant poisoning
Rosen 5th, (anticholinergic toxicity, sodium channel blockade, hypotension). The antidote for
Chapter 145, tricyclic antidepressant poisoning is sodium bicarbonate. Glucagon is an antidote used
p. 2089-2093 for beta-blocker and calcium channel blocker toxicity. Physostigmine is an antidote
for anticholinergic poisoning but should not be given to patients suffering from sodium
channel blockade as asystole may result.
1004. c This patient is displaying signs and symptoms of sympathomimetic poisoning.
Rosen 5th, Methamphetamine is the only sympathomimetic agent among the five choices.
Chapter 148, Diphenhydramine and Jimson weed are anticholinergic agents. Anticholinergic
p. 2124 poisoning is very similar to sympathomimetic poisoning. The major difference
between these two poisonings is that anticholinergic poisoning produces dry skin while
sympathomimetic poisoning produces diaphoresis.
1005. c Heroin is an opioid. The typical signs and symptoms of opioid withdrawal are
Rosen 5th, restlessness, yawning, mydriasis, rhinorrhea, vomiting, abdominal pain, and
Chapter 156, piloerection.
p. 2183
296

1006. A comatose 32 year-old woman is brought from home. Her Glasgow Coma Score is 5,
and the paramedics have successfully intubated her. They tell you that she has bipolar
disease, and show you an empty prescription bottle labeled lithium carbonate 300mg.
The prescription was filled three days ago for 120 tablets. Your most appropriate
management is:
a. activated charcoal.
b. hemodialysis.
c. physostigmine.
d. sodium bicarbonate.
e. syrup of ipecac.

1007. The clinical presentation of strychnine poisoning most closely resembles:


a. botulism poisoning.
b. brown recluse spider bite.
c. organophosphate poisoning.
d. scombroid poisoning.
e. tetanus infection.

1008. The treatment most likely to be successful in the management of beta-adrenergic blocker
toxicity is:
a. dialysis.
b. emesis with syrup of ipecac.
c. esmolol.
d. glucagon.
e. theophylline.

1009. Digoxin:
a. decreases cardiac automaticity.
b. increases intracellular potassium concentration.
c. decreases vagal tone.
d. decreases cardiac contractility.
e. inhibits the Na+-K+ ATPase pump.

1010. A 64-year-old woman inadvertently overdosed on her quinidine sulfate. You expect her
EKG to show:
a. QRS prolongation.
b. QT shortening.
c. ST segment depression.
d. ST segment elevation.
e. T wave inversion.

1011. Bradycardia can be seen in patients with toxic ingestion of:


a. captopril.
b. enalapril.
c. hydralazine.
d. nitroprusside.
e. verapamil.
297

1006. b The indications for hemodialysis for lithium patients poisoned are: clinical signs of
th
Rosen 5 , severe poisoning, deteriorating clinical condition (e.g., seizures, coma, ventricular
Chapter 154, dysrhythmias), decreasing urine output, renal failure, and lack of expected drop in
p. 2172-2173 serum lithium level (20% in 6 hours). Lithium does not bind to activated charcoal.
1007. e Strychnine poisoning and tetanus infection are similar clinically because both illnesses
th
Rosen 5 , have a negative effect on glycine neurotransmission. Strychnine produces its toxicity
Chapter 123, by blocking glycine receptors. Tetanus, an infection caused by Clostridium tetani,
p. 1792-1793 causes its toxicity by preventing the release of glycine from pre-synaptic neurons.
1008. d Glucagon is widely accepted as first-line therapy for beta-blocker toxicity. It enhances
th
Rosen 5 , myocardial performance by increasing cAMP concentrations in a manner identical to
Chapter 146, that of catecholamines but is thought to act via its own receptor. Thus, glucagon may
p. 2110 bypass the blocked beta-receptor. Clinical experience with this antidote has generally
produced favorable results, often after other treatments have failed. A limited number
of animal models directly comparing glucagon to other therapies support the use of
glucagon.
1009. e Digoxin and the other cardiac glycosides produce their therapeutic effects by inhibiting
Rosen 5th, the Na+ - K+ ATPase pump. Other pharmacologic effects associated with digoxin
Chapter 146, include increased cardiac automaticity, decreased intracellular potassium
p. 2104 concentration, increased vagal tone, and increased cardiac contractility.
1010. a Toxicity with Class 1A antidysrhythmics (quinidine, procainamide, disopyramide)
Rosen 5th, typically produces a prolonged QRS interval by blocking myocardial sodium channels.
Chapter 74, Class 1A antidysrhythmics also have the potential to produce a prolonged QT interval.
p. 1059
1011. e Calcium channel antagonists block the slow calcium channels in the myocardium and
th
Rosen 5 , vascular smooth muscle, leading to coronary and peripheral vasodilation. They also
Chapter 146, reduce cardiac contractility, depress SA nodal activity, and slow AV conduction. In
p. 2111 overdose, verapamil has the deadliest profile, combining severe myocardial depression
and peripheral vasodilation. Hypotension and bradycardia occur early, and other
rhythm disturbances include AV block of all degrees, sinus arrest, AV dissociation,
junctional rhythm, and asystole. Nifedipine overdose more commonly causes reflex
sinus tachycardia from peripheral vasodilation. Calcium channel blockade has little
effect on ventricular conduction, so QRS widening is not seen early on. Hydralazine
poisoning typically causes hypotension and reflex tachycardia.
298

1012. The syndrome of anticholinergic poisoning includes:


a. hyperactive bowel sounds.
b. hypothermia.
c. miosis.
d. salivation.
e. seizures.

1013. A 21-year-old man is brought to the emergency department by his family. He is restless
combative, and vomiting. His mother hands you a suicide note she found at his bedside.
Temperature 102.1oF, heart rate 120, respiratory rate 30, blood pressure 120/76 mmHg.
HEENT exam is unremarkable. Lungs are clear, but he has Kussmaul respirations. Heart
exam shows only tachycardia. His abdomen is soft and non-tender. His neurologic exam
shows an agitated, combative patient, but is non-focal. He has most likely ingested:
a. digoxin.
b. diphenhydramine.
c. organophosphate pesticides.
d. salicylate.
e. tricyclic antidepressants.

1014. A 75 year-old man took a handful of digoxin tablets in a suicide attempt. An indication for
using the digoxin-specific Fab fragments (Digibind, DigiFab) would be:
a. a serum digoxin level of 5.5 ng/ml.
b. a serum potassium of 5.8 mEq/l.
c. altered mental status.
d. hepatic insufficiency.
e. renal insufficiency.

1015. Carbon monoxide poisoning causes:


a. decreased affinity of hemoglobin for oxygen.
b. decreased oxygen carrying capacity of hemoglobin.
c. oxidative phosphorylation uncoupling.
d. methemoglobinemia.
e. stimulation of the tissue cytochrome oxidase system.

1016. Acetaminophen:
a. does not bind to activated charcoal.
b. is metabolized by the cytochrome P-450 system.
c. serum levels should be plotted on the Done nomogram.
d. overdose causes acidosis and tinnitus.
e. should not be given in combination with aspirin.

1017. A patient who has ingested a large amount of methanol will probably present with:
a. confusion, stupor, coma, and seizures.
b. crystalluria and renal failure.
c. an elevated osmolar gap but a normal anion gap.
d. hypochloremia and hypotension.
e. severe hypoglycemia and severe hyponatremia.
299

1012. e The diagnosis of acute anticholinergic poisoning is suggested by characteristic physical


th
Rosen 5 , signs that constitute the anticholinergic syndrome or toxidrome. Mydriasis, dry
Chapter 144, mucous membranes, the absence of axillary sweat, flushed skin, fever, tachycardia,
p.2083 decreased or absent bowel sounds, and bladder distention constitute peripheral
evidence of muscarinic blockade. The presence of only one or two of these findings
does not exclude the diagnosis of an anticholinergic poisoning. The patient is often
alert and may be silly, agitated, violent, or incoherent. Visual hallucinations are
common. Central motor effects may manifest as myoclonus or choreoathetoid
movements in adults and children. Children seem more sensitive to the CNS stimulant
effects than adults and are more likely to have seizures, typically preceded by signs of
CNS irritability or depression.
1013. d This clinical scenario is most consistent with a salicylate overdose. Salicylate
Rosen 5th, poisoning causes nausea, vomiting, dehydration, hyperthermia, respiratory alkalosis,
Chapter 143, and metabolic acidosis. The metabolic acidosis subsequently causes the Kussmaul
p. 2077 respirations.
1014. b Recommendations for Administration of Digitalis Antibody Fragments in Adults
Rosen 5th, Severe ventricular dysrhythmias
Chapter 146,
Progressive and hemodynamically significant bradydysrhythmias unresponsive to
p. 2107, Box
atropine
146-4
Serum potassium greater than 5 mEq/L
Rapidly progressive rhythm disturbances or rising serum potassium
Coingestion of cardiotoxic drug as beta-blockers, calcium channel blockers, or
tricyclic antidepressants
Ingestion of plant known to contain cardiac glycosides plus severe dysrhythmias
(rare)
Acute ingestion greater than 10 mg plus any one of factors 1 through 6 above
Steady-state serum digoxin greater than 6 ng/ml plus any one of factors 1 through 6
above
1015. b Carbon monoxide binds to hemoglobin with an affinity 250 times that of oxygen.
Rosen 5th, Carbon monoxide poisoning causes reduced oxyhemoglobin saturation, a decreased
Chapter 153, hemoglobin-to-oxygen carrying capacity, and inhibition of the cytochrome oxidase
p. 2169 system.
1016. b Acetaminophen is primarily metabolized in the liver. About 90% of a therapeutic dose
Rosen 5th, is converted to the inactive glucuronide and sulfate conjugates. Less than 4% is
Chapter 142, excreted as unchanged Acetaminophen, and a similar fraction is conjugated with
p. 2069 glutathione by hepatic cytochrome P450 mixed function oxidases. Activated charcoal
does bind acetaminophen. Serum levels are plotted on the Rumack-Matthews
nomogram; the Done nomogram is used to plot serum salicylate levels. Salicylate
poisoning typically produces tinnitus and acidosis.
1017. a The methanol-toxic victim may be confused or, in severe cases, comatose. There may
Rosen 5th, be complaints of headache or vertigo, and seizures may occur. Visual disturbances are
Chapter 149, seen in approximately 50% of patients. These include diplopia, blurred vision,
p. 2128 decreased visual acuity, photophobia, descriptions of looking into a snow field,
constricted visual fields, and blindness. The clinician may find nystagmus, fixed and
dilated pupils, retinal edema, and optic atrophy or hyperemia of the optic disk.
300

1018. A 30-year-old man is found slumped over his workbench in the back of his jewelry store.
Minutes earlier he had complained of headache, dizziness, and weakness. Now he is
comatose and apneic with a thready pulse. There is no cyanosis. Basic Metabolic Panel
shows a high anion gap acidosis. Blood gas determinations on arterial and venous blood
samples show nearly identical pO2 values. The carboxyhemoglobin is 5%. Although the
patient is intubated and being hyperventilated with 100% oxygen, the acidosis persists.
Your next step is to:
a. administer intravenous atropine and pralidoxime (2-PAM).
b. administer intravenous methylene blue.
c. administer intravenous physostigmine and hydroxocobalamin (vitamin B12a).
d. administer intravenous sodium nitrite, then infuse sodium thiosulfate.
e. transfer the patient to a facility with a hyperbaric chamber.

1019. Drugs which can be removed by dialysis:


a. have a high molecular weight.
b. are highly protein bound.
c. have a large volume of distribution
d. are lipophilic
e. are water soluble

1020. A 35-year-old HIV positive man with a history of Pneumocystis carinii pneumonia presents
to the awake and alert, but markedly cyanotic. Paramedics report finding the patient in a
bed in a local hospice with a suicide note and pills on the bedroom floor. The patient's
cyanosis persists despite 100% oxygen via mask. The patient is wearing a sulfur allergy
alert bracelet. The most likely etiology for his cyanosis is:
a. ethylene glycol poisoning.
b. hemolysis secondary to a trimethoprim/sulfamethoxazole (Bactrim) overdose.
c. methemoglobinemia secondary to medication overdose.
d. pulmonic stenosis from endocarditis associated with atrial septal defect.
e. severe hypoxemia due to recurrence of Pneumocystis carinii pneumonia.

1021. Pralidoxime (2-PAM) reverses the toxicity of organophosphate insecticides by:


a. a direct beta-agonist effect which offsets bradycardia.
b. enhancing acetylcholine release from nerve endings.
c. enhancing norepinephrine release from nerve endings.
d. inducing metabolism of organophosphate insecticides.
e. reversing the inhibition of acetylcholinesterase.

1022. Hypocalcemia is a common finding in poisoning by:


a. acetaminophen.
b. diphenhydramine.
c. ethylene glycol.
d. iron.
e. methamphetamine.

1023. Which of the following uncouples oxidative phosphorylation?


a. acetaminophen
b. carbon monoxide
c. ethylene glycol
d. isoniazid
e. salicylates
301

1018. d This patient is most likely suffering from cyanide poisoning. Cyanide poisoning
th
Rosen 5 , typically causes headache, dizziness, and weakness followed by convulsions and coma.
Chapter 153, Metabolic acidosis occurs without evidence of cyanosis or hypoxia, and an abnormally
p. 2168 elevated venous oxygen pressure also is seen. Cyanide is used in many industries,
including ore extraction, metal polishing, and electroplating. Sodium nitrite and
sodium thiosulfate are found in the cyanide antidote kit. Atropine and pralidoxime use
antidotes used for organophosphate poisoning. Methylene blue is an antidote that
reverses methemoglobinemia. Physostigmine is an antidote occasionally used for
anticholinergic poisoning. Hyperbaric oxygen is used to treat carbon monoxide
poisoning.
1019. e Hemodialysis is generally reserved for specific toxins that must be both potentially life
Rosen 5th, threatening and amenable to removal by this method. The benefits include the ability
Chapter 154, to remove toxins that are already absorbed from the gut lumen, removal of substances
p. 2173 that do not adhere to activated charcoal, and the ability to remove both the parent
compound and the active toxic metabolites. Hemodialysis is much less effective where
the toxin ingested has a large volume of distribution, has a large molecular weight, or
is highly protein bound.
1020. c This patient most likely ingested dapsone, a medication used to treat Pneumocystis
pneumonia for patients allergic to sulfur. Dapsone toxicity can produce significant
methemoglobinemia and a resulting cyanosis that does not reverse with oxygen. Other
agents that can cause methemoglobinemia include lidocaine, aniline dyes,
sulfonamides, phenacetin, and the fluoroquinolones.
1021. e The second part of acetylcholinesterase inhibition treatment is the use of pralidoxime
Rosen 5th, (Protopam, 2-PAM) to break up the organophosphate acetylcholinesterase complex
Chapter 157, and restore cholinesterase activity at both muscarinic and nicotinic sites. Pralidoxime
p. 2190 can also combine with unbound organophosphates and prevent their subsequent
binding to nerve terminals.
1022. c Poisoning by ethylene glycol can produce hypocalcemia secondary to the precipitation
Rosen 5th, of calcium oxalate crystals. Hypocalcemia may be severe enough to cause tetany and
Chapter 149, prolongation of the QT interval. Two forms of urinary calcium oxalate crystals may be
p. 2131 identified on microscopic examination of the urine: dihydrate (octahedral) form and
monohydrate (monoclinic) form).
1023. e Salicylates enhance lipolysis, inhibit various enzymes involved in energy production
Rosen 5th, and amino acid metabolism, and uncouple oxidation phosphorylation. Because
Chapter 143, oxidative phosphorylation is a major buffer of hydrogen ions, impairment of oxidative
p. 2076 phosphorylation by salicylate results in metabolic acidosis. Carbon monoxide inhibits
electron transport.
302

1024. Which of the following inhibits electron transport?


a. chlorine gas
b. cyanide
c. dapsone
d. hydrofluoric acid
e. isopropanol

1025. A 3-year-old boy is brought to the ED because of lethargy. He is difficult to awaken and
slightly diaphoretic. Temperature 97.6oF; heart rate 22; respiratory rate 12; blood pressure
60/30 mmHg. ECG shows sinus bradycardia with a QRS interval of 0.1 s and peaked T
waves. The serum potassium level is 8.2. This child has ingested:
a. captopril.
b. clonidine.
c. digoxin.
d. propranolol.
e. verapamil.

1026. Acute inorganic mercury and arsenic poisoning share many clinical features. Which of the
following suggests the presence of acute inorganic mercury salt poisoning over acute
inorganic arsenic poisoning?
a. Delayed onset of peripheral neuropathy
b. Hemorrhagic gastroenteritis
c. Radio-opaque substance noted on KUB
d. Rapid onset of renal failure
e. Shock

1027. A 5-year-old boy presents with lethargy. His heart rate is 30 bpm, and blood pressure is
50/30 mmHg. On ECG, the QRS complex is narrow. He has probably ingested:
a. clonidine.
b. doxepin (Sinequan).
c. jimson weed.
d. phenylpropanolamine.
e. thioridazine.

1028. A 32-year-old woman complains of vomiting, hematemesis, and abdominal pain. Vital
signs: temperature 99.oF; heart rate 130/min; respiratory rate 44/min, blood pressure
111/80 mm Hg. She is diaphoretic. Laboratory studies reveal: Na+ 150, K+ 4.8, Cl- 110,
CO2 12, pH 7.35, pCO2 18, pO2 104, urine ketones 3+. After fluid challenge, your next
step should be:
a. gastric lavage and deferoxamine.
b. fomepizole (4-MP) and hemodialysis.
c. intravenous ethanol and hemodialysis.
d. intravenous sodium bicarbonate and hemodialysis.
e. n-acetylcysteine and activated charcoal.

1029. In a patient suffering from serotonin syndrome, it is reasonable to give:


a. benztropine (Cogentin).
b. bromocriptine (Parlodel).
c. cyproheptadine (Periactin).
d. L-dopa.
e. ondansetron (Zofran).
303

1024. b Cyanide is a potent inhibitor of the electron transport chain. Inhibition of the electron
th
Rosen 5 , transport chain leads to anaerobic metabolism and metabolic acidosis, as the body
Chapter 153, tissues cannot utilize oxygen. Carbon monoxide and hydrogen sulfide also inhibit the
p. 2167 electron transport chain.
1025. c This scenario is most consistent with acute digoxin (cardiac glycoside) poisoning.
th
Rosen 5 , Acute digoxin (cardiac glycoside) poisoning typically causes lethargy, confusion,
Chapter 146, bradyarrhythmias or supraventricular tachydysrhythmias, hyperkalemia, and nausea
p. 2107 and vomiting. Unless ventricular escape rhythms occur, the QRS complex typically
stays narrow (less than 0.12 s). Poisoning by clonidine, propranolol, and verapamil
can produce a similar presentation; however, hyperkalemia is NOT typical. Captopril
poisoning does not typically produce cardiovascular compromise.
1026. d Acute inorganic mercury and arsenic poisoning can produce hemorrhagic
Rosen 5th, gastroenteritis, shock, and delayed onset of peripheral neuropathy. In addition, both
Chapter 151, inorganic arsenic and mercury are radio-opaque on an x-ray. Acute inorganic mercury
p. 2155, 2157 poisoning typically causes rapid onset of renal failure (within 24 hours) secondary to
acute tubular necrosis.
1027. a Clonidine (an imidazoline) poisoning typically produces bradycardia, hypotension, and
CNS depression. Unlike the tricyclic antidepressants (e.g., doxepin) and the
phenothiazines (e.g., thioridazine), clonidine and other imidazolines do NOT block
myocardial sodium channels and do NOT prolong the QRS complex. Jimson weed is a
plant with anticholinergic properties; jimson weed poisoning typically produces
tachycardia. Phenylpropanolamine was available as an over-the-counter diet
preparation and poisoning typically produced hypertension and cerebrovascular
complications. For safety reasons, the FDA recalled all products containing
phenylpropanolamine in the year 2000.
1028. d This scenario is most consistent with acute salicylate poisoning. Acute salicylate
Rosen 5th, poisoning causes nausea, vomiting, dehydration, hyperthermia, respiratory alkalosis,
Chapter 143, and metabolic acidosis. The metabolic acidosis subsequently causes the Kussmaul
p. 2077, 2078 respirations. Abdominal pain and hematemesis can occur. The treatment for severe
salicylate poisoning includes fluid resuscitation, alkalinization of the serum and urine,
and hemodialysis.
1029. c Serotonin syndrome occurs when there is an excess of the neurotransmitter serotonin at
Rosen 5th, the receptor. This syndrome typically occurs with the concomitant use of two
Chapter 145, serotonergic drugs. Overdoses of a single serotonergic agent may also produce
p. 2096 serotonin syndrome. Serotonin syndrome typically produces neuromuscular
symptoms, cognitive-behavior symptoms, and dysfunction of the autonomic nervous
system. Several investigators have reported successful treatment of serotonin
syndrome with the serotonin receptor antagonist cyproheptadine.
304

1030. A 34-year-old man is brought by ambulance with refractory seizures and a heart rate of
104/min. The rhythm strip shows a QRS complex of 0.08 seconds. He has probably
ingested:
a. gamma-butyrolactone (GBL).
b. gamma-hydroxybutyrate (GHB).
c. isoniazid (INH).
d. nortriptyline.
e. quinidine.

1031. A 62-year-old man is lethargic two hours after undergoing outpatient bronchoscopy. Heart
rate 120/min, respiratory rate 26/min, blood pressure 160/90 mmHg. He is cyanotic and
diaphoretic. Laboratory studies show: pH 7.28; pCO2 28, pO2 88; Hgb 12 g/dL. The on-
call intensivist inserts a Swan-Ganz catheter, and the reading show a cardiac output of 5
L/min, oxygen consumption 330 ml/min (normal), and mixed venous pCO2 20 mmHg.
Your most appropriate action is:
a. check for osmolar gap and consider dialysis.
b. check pulse oximetry.
c. exchange transfusion.
d. intravenous methylene blue.
e. intravenous sodium thiosulfate and sodium nitrite.

1032. A 22 year-old woman ingested an unknown substance two hours ago. She is vomiting
and has diarrhea. Her vital signs are normal. Lab studies show pH 7.4; pCO2 40; pO2 95;
Na+ 136, K+ 4.0, Cl- 110, HCO3 24, acetaminophen level 0, salicylate level 0. Over the
next 8 hours she develops confusion, rigidity, and convulsions. ECG shows first-degree
heart block. She probably ingested:
a. acetaminophen.
b. ciprofloxacin.
c. lithium carbonate.
d. oil of wintergreen.
e. salicylate.

1033. The oxyhemoglobin dissociation curve shifts to the right in:


a. acidosis.
b. alkalosis.
c. carboxyhemoglobin.
d. methemoglobin.
e. hypothermia.

1034. Alcohol dehydrogenase metabolizes:


a. disulfiram.
b. gamma-hydroxybutyrate (GHB).
c. gyrometra mushrooms.
d. isopropanol.
e. Rohypnol.

1035. A 22-year-old woman is brought from her dormitory room, where her roommate found her
comatose. Heart rate 95/min; respiratory rate 18/min; BP 180/95 mmHg. Physical
examination shows muscular rigidity and rotary nystagmus. She has probably ingested:
a. amphetamine.
b. ecstasy.
305

c. phencyclidine.
d. phenobarbital.
e. diazepam.
306

1030. c Poisoning by all these agents may produce seizures. Gamma-butyrolactone (GBL) and
th
Rosen 5 , gamma-hydroxybutyrate (GHB) do NOT typically cause refractory seizures.
Chapter 118, Nortriptyline and quinidine block myocardial sodium channels. Severe poisoning by
p. 1719 nortriptyline and quinidine may produce seizures but usually do so when the QRS
complex is prolonged (greater then 0.10 s). Isoniazid poisoning blocks the production
of the inhibitory neurotransmitter gamma-aminobutyric acid (GABA) and causes
seizures that are refractory to conventional therapy (e.g., benzodiazepines,
barbiturates). The antidote for isoniazid poisoning is pyridoxine (vitamin B6).
1031. d This clinical scenario is typical for acquired methemoglobinemia, with the source
Rosen 5th, being the local anesthetic (e.g., benzocaine) used during the bronchoscopy. The
Chapter 182, treatment for symptomatic methemoglobinemia is methylene blue.
p. 2574
1032. c Acute lithium poisoning typically produces nausea, vomiting, and diarrhea. Lithium
th
Rosen 5 , poisoning also produces central nervous system toxicity (lethargy, confusion,
Chapter 154, spasticity, rigidity, convulsions, etc.) and cardiovascular toxicity (ECG conduction
p. 2172 defects, bradycardia, ST-T wave changes, ventricular arrhythmias). Because of
lithiums prolonged absorption and distribution period, presentation of the CNS and
cardiovascular toxicity may be delayed. Oil of wintergreen (methyl salicylate) is an
extremely toxic form of salicylate.
1033. a The following factors shift the oxyhemoglobin dissociation curve to the left:
methemoglobinemia, carboxyhemoglobinemia, alkalosis, hypothermia. The following
factors shift the oxyhemoglobin dissociation curve to the right, inhibiting the release of
oxygen at the tissue level: acidosis, hyperthermia, and elevations in 2,3-
diphosphoglycerate concentration.
1034. d Isopropanol, the main ingredient found in rubbing alcohol, is metabolized by alcohol
Rosen 5th, dehydrogenase to acetone. Disulfiram (Antabuse) inhibits the enzyme aldehyde
Chapter 149, dehydrogenase.
p. 2134
1035. c Phencyclidine, a dissociative anesthetic agent, is a common agent of abuse. Mild to
th
Rosen 5 , moderate intoxication with phencyclidine typically causes agitation, confusion,
Chapter 180, hallucinations, delusions, ataxia, rigidity, rotary nystagmus, and coma. Severe
p. 2541 phencyclidine toxicity causes hypertension and tachycardia. Amphetamine and ecstasy
(sympathomimetics) poisoning typically cause tachycardia. Phenobarbital and
diazepam (sedative hypnotics) poisoning typically causes muscular relaxation.
307

1036. A patient poisoned with cyanide will have:


a. convulsions.
b. cyanosis.
c. garlic odor to breath.
d. metabolic alkalosis.
e. rotten egg odor to skin.

1037. A 44-year-old male ingests an unknown substance. In less than 10 minutes he collapses
in convulsions. He probably ingested:
a. Amanita phylloides mushrooms.
b. insulin.
c. phenobarbital.
d. salicylates.
e. strychnine.

1038. An elevated methemoglobin fraction may be seen in:


a. aldehyde dehydrogenase deficiency.
b. gastroenteritis in a newborn.
c. glucose-6-phosphate deficiency (G6PD).
d. isoniazid overdose.
e. thiamine deficiency.

1039. Centrilobular hepatic necrosis is seen in poisoning with:


a. Percocet.
b. jimson weed.
c. ecstasy.
d. Nerium oleander.
e. theophylline.

1040. Regarding gastrointestinal decontamination after overdose:


a. Activated charcoal is the recommended gastrointestinal decontamination procedure of choice.
b. Activated charcoal prevents absorption of substances such as lithium and iron supplements.
c. In an overdose, the stomach should be emptied by both ipecac and gastric lavage.
d. Lavage is most effective if performed within 6 hours of ingestion of large quantity of pills.
e. Multidose charcoal is most useful to enhance elimination of a life-threatening dose of medication.

1041. A 37-year-old man says he took a handful of over the counter pain medicine yesterday
morning. In addition to an acetaminophen concentration, the lab test most sensitive in
detecting possible liver injury is:
a. acid phosphatase.
b. alkaline phosphatase.
c. aspartate aminotransferase (AST or SGOT).
d. bilirubin.
e. lactated dehydrogenase (LDH).

1042. The cut-off time for using N-acetylcysteine in a patient presenting after acetaminophen
overdose is:
a. 8 hours.
b. 24 hours.
c. 36 hours.
d. 2 days.
308

e. none.
309

1036. a Cyanide poisoning typically causes headache, dizziness, and weakness followed by
th
Rosen 5 , convulsions and coma. In addition, metabolic acidosis occurs without evidence of
Chapter 153, cyanosis or hypoxia. Garlic odor is characteristic of arsenic and organophosphate
p. 2167-2168 poisoning. Rotten egg odor is characteristic of hydrogen sulfide poisoning.
1037. e Strychnine competitively blocks the glycine receptor. Glycine is an inhibitory
th
Rosen 5 , neurotransmitter released by the postsynaptic inhibitory neurons in the spinal cord. By
Chapter 123, blocking the glycine receptor, strychnine causes increased neuronal excitability, which
p. 1792-1793 results in generalized seizure-like contraction of the skeletal muscles. Strychnine is
absorbed rapidly following ingestion. Salicylate poisoning can produce seizures but
not in such a rapid time frame. Insulin, when ingested, is metabolized before
absorption and typically does not cause hypoglycemia.
1038. b Neonates have reduced levels of the endogenous enzyme used to reduce
methemoglobin back to its normal state. When neonates are exposed to oxidant stress
(e.g., viral induced gastroenteritis) they are at risk for developing elevated levels of
methemoglobin. Isoniazid poisoning causes seizures refractory to standard therapy.
Thiamine deficiency can lead to Wernickes encephalopathy, Korsakoffs syndrome, or
Beriberi. Patients with glucose-6-phosphate deficiency (G6PD) are not predisposed to
methemoglobinemia.
1039. a Percocet (oxycodone/acetaminophen) may produce opioid toxicity but also liver
Rosen 5th, toxicity secondary to the acetaminophen. Jimson weed typically causes anticholinergic
Chapter 142, poisoning. Ecstasy causes sympathomimetic toxicity. Nerium oleander is a plant with
p. 2069 cardiac glycoside properties, and poisoning with this plant resembles poisoning seen
with digoxin. Theophylline is a phosphodiesterase-inhibitor that does not typically
produce liver damage in an overdose setting.
1040. a Activated charcoal is the procedure of choice for GI decontamination. Complications
Rosen 5th, of gastric emptying procedures, such as aspiration, are avoided with activated charcoal.
Chapter 141, In such an overdose, whole bowel irrigation may be considered. Gastric emptying
p. 2066 does not require both ipecac and gastric lavage. If performed within one hour of
ingestion, gastric lavage may be useful. Multidose charcoal may be useful in removing
drugs already absorbed, not to enhance elimination.
1041. c Once potential risk has been established, determine acetaminophen concentration
Rosen 5th, ([APAP]) 4 hours after the ingestion, or as soon as possible after 4 hours. In many
Chapter 142, cases [APAP] alone will be adequate to establish risk. When the physical findings are
p. 2071 consistent with hepatic injury, or the timing of the ingestion cannot be established,
aspartate transaminase (AST) should also be measured to establish the presence or
absence of hepatic injury. Those at risk for APAP-induced hepatotoxicity on the basis
of [APAP], or with evidence of hepatic injury, should be treated with NAC.
1042. e All durations of NAC administration are effective when started early, indicating that
Rosen 5th, shorter courses of treatment are appropriate when NAC is used for its preventive
Chapter 142, actions. In contrast, protocol comparisons and series involving late NAC
p. 2074 administration suggest that a longer duration of treatment may be needed when treating
established liver injury. Rather than a single duration of therapy for all patients, it may
be appropriate to use different treatment protocols on the basis of clinical course or
other markers. No definitive cut-off point for beginning therapy has been described.
310

1043. Choose the true statement concerning non-steroidal anti-inflammatory overdose:


a. Because of a high volume of distribution and slow metabolism, urinary alkalinization,
hemodialysis, or hemoperfusion can enhance NSAID elimination.
b. Children with ingestions of less than 300 mg/kg of ibuprofen do not require medical evaluation.
c. Because of zero-order kinetics, elimination half-lives are substantially prolonged in overdose.
d. NSAIDs can cause lethal dysrhythmias.
e. Overdose with mefenamic acid (Ponstel) is associated with seizures, which can occur several
hours after ingestion.

1044. You are seeing a 23-year-old graduate student who overdosed on a bottle of over-the-
counter sleeping pills. He is poorly responsive, delirious, and irritable, with frequent
myoclonic jerks. His mucous membranes are dry and pupils dilated. In an attempt to
prevent seizures, you decide to give the appropriate antidote:
a. 1 to 2 mg rapid IV push.
b. 1 to 2 mg slow IV push over 2 to 4 minutes.
c. 5 mg IM.
d. 5 mg rapid IV push.
e. 5 mg slow IV push over 2 to 4 minutes.

1045. The most significant toxicity which can be seen with bupropion (Wellbutrin, Zyban) is:
a. neuroleptic malignant syndrome.
b. orthostasis.
c. seizures.
d. serotonin syndrome.
e. torsade de pointes.

1046. In a patient who has overdosed on tranylcypromine (Parnate), you should know that:
a. asymptomatic patients need 24 hours of observation.
b. bradycardia and hypotension are early findings.
c. bretylium is the drug of choice for ventricular dysrhythmias.
d. hemodialysis should be considered for severe overdoses.
e. symptoms usually start within 1 to 2 hours of ingestion.

1047. Caustic gastrointestinal burns are possible after ingestion of:


a. cigarettes.
b. furniture polish.
c. hair relaxer.
d. rodenticides.
e. rubbing alcohol.

1048. In a patient with massive topical exposure to white phosphorus, you might expect to see
a. severe metabolic alkalosis.
b. fulminant pulmonary edema.
c. intractable seizures.
d. hypocalcemia and prolonged QT on ECG.
e. severe disseminated intravascular coagulopathy.
311

1043. e NSAIDs are not known to be primary causes of dysrhythmias, but fluid and electrolyte
th
Rosen 5 , abnormalities may place patients at risk for cardiac dysrhythmias. Children with
Chapter 142, ingestions of less than 100 mg/kg of ibuprofen do not require medical evaluation.
p. 2079 Because of high protein binding and rapid metabolism, urinary alkalinization,
hemodialysis, or hemoperfusion does not enhance NSAID elimination. Elimination
half-lives are not substantially prolonged in overdose.
1044. b Physostigmine antagonizes the effects of anticholinergic medications by inhibiting
Rosen 5th, acetylcholinesterase and thereby increasing the concentration of acetylcholine in the
Chapter 143, synaptic space. In the absence of a significant anticholinergic overdose, however,
p. 2085 physostigmine may exacerbate asthma and may produce seizures, bradycardia,
salivation, diarrhea, and bronchorrhea. It should therefore be used with caution and
never as a first-line drug. The proper dose is 1 to 2 mg slow IV push in adults and 0.5
mg slow IV push in children. Doses may be repeated as needed. Rapid infusion, even
with significant anticholinergic toxicity, may produce seizures.
1045. c Seizures may occur not only with bupropion overdose but also when the maximum
Rosen 5th, daily dose is exceeded. Bupropion-induced seizures should be treated with
Chapter 145, benzodiazepines. Phenobarbital should be used to treat patients with recurrent seizures
p. 2097 or status epilepticus. Hypotension, neuroleptic malignant syndrome, and serotonin
syndrome do not occur unless other medications are coingested.
1046. a Signs and symptoms of MAO-I overdose are usually not apparent during the first 6 to
Rosen 5th, 12 hours following ingestion. For this reason, all patients with MAO-I overdose
Chapter 145, should be admitted for 24 hours. This is in contrast to patients with suspected or
p. 2100 known MAO-I interactions with food, beverages, and drugs. These patients will have
onset of symptoms within minutes to hours of ingestion and may be discharged if they
remain asymptomatic for 6 hours. Bradycardia and hypotension are late findings with
MAO-I toxicity. Hemodialysis, hemoperfusion, and forced diuresis do not enhance
elimination. Lidocaine is the drug of choice for ventricular dysrhythmias. Bretylium
is contraindicated because it releases biogenic amines.
1047. c Ingestion of base-containing substances produces significant injury because bases
Rosen 5th, cause liquefactive necrosis, which allows deep penetration of the caustic material.
Chapter 147, Such substances include lye (NaOH and KOH), ammonia (NH4OH4), hair relaxers, hair
p. 2115 straighteners, and electric dishwasher soaps.
1048. d Metabolic derangements have been identified after white phosphorus burns. Serum
Rosen 5th, electrolyte changes consist of decreased serum calcium and increased serum
Chapter 57, phosphorus. ECG abnormalities include prolonged QT interval, bradycardia, and ST-
p. 818 segment-T-wave changes, which may explain the sudden death occasionally seen in
patients with apparently inconsequential white phosphorus burns.
312

1049. The most appropriate initial treatment for a chemical burn is:
a. bleach soaks.
b. copious irrigation with water.
c. ice packs.
d. pain management.
e. silver sulfadiazine dressings.

1050. There are reports of perforated aorta, left atrium, and bronchial artery in patients who did
not completely wash down the commonly-prescribed medication:
a. potassium chloride
b. cephalexin.
c. ibuprofen.
d. codeine.
e. nitroglycerin.
313

1049. b Injury will continue as long as there is contact between the offending agent and the
th
Rosen 5 , skin. Contaminated clothes should be removed, but irrigation should be initiated
Chapter 57, immediately and continued while the clothing is removed.
p. 814
1050. c More than 70 different pills may cause damage when in contact with esophageal mucus
th
Rosen 5 , for prolonged periods. Patients who take medications in the supine position, or who
Chapter 147, take pills without water, are at higher risk. Pills most likely to adhere are doxycycline,
p. 2115-2116 tetracycline, potassium chloride, and aspirin. Potassium chloride is particularly
dangerous and has caused penetration into the aorta, left atrium, and bronchial artery.
314

18.0 Traumatic Disorders


1051. In treating a patient with traumatic brain injury, the ideal cerebral perfusion pressure is:
a. 40 mmHg or greater.
b. 50 mmHg or greater.
c. 60 mmHg or greater.
d. 70 mmHg or greater.
e. 80 mmHg or greater.

1052. Uncal transtentorial herniation would typically result in:


a. ipsilateral fixed and dilated pupil, ipsilateral hemiparesis
b. ipsilateral fixed and dilated pupil, contralateral hemiparesis
c. contralateral fixed and dilated pupil, ipsilateral hemiparesis
d. contralateral fixed and dilated pupil, contralateral hemiparesis
e. bilateral fixed / dilated pupils with quadriparesis

1053. Epidural hematoma is most often associated with a skull fracture that tears the:
a. external carotid artery.
b. internal carotid artery
c. middle meningeal artery
d. middle meningeal vein
e. subarachnoid bridging veins

1054. A 14-year-old boy was knocked out when struck on the head with a golf ball. He now
opens his eyes when stimulated, moans in pain, and pulls away from noxious stimuli. His
Glasgow Coma Score is:
a. 14
b. 12
c. 10
d. 8
e. 6

1055. Mannitol reduces intracranial pressure by:


a. diuresis
b. pH elevation
c. pH reduction
d. vasoconstriction
e. vasodilation

1056. A 94-year-old woman is sent from a local nursing home after falling from her wheelchair.
Her transfer note asks you to rule-out subdural. You know that:
a. blood collects in the subdural space more quickly than in an epidural hematoma.
b. infants and toddlers rarely develop subdural hematomas.
c. most subdural hematomas are due to penetrating head injury.
d. the risk of developing a subdural hematoma decreases with age.
e. the usual mechanism is a sudden acceleration-deceleration of brain parenchyma and tearing of
bridging veins.
315

1051. d Cerebral perfusion pressure (CPP) is estimated as MAP minus ICP. Cerebral blood
th
Rosen 5 , flow (CBF) is dependent on CPP. As CPP drops below 60mm Hg, the autoregulation
Chapter 34, of CBF is lost, CBF declines, and the resultant tissue ischemia critically affects
p. 288 cerebral metabolism.
1052. b The most common traumatic herniation syndrome is uncal herniation, The classic signs
th
Rosen 5 , and symptoms are caused by compression of the ipsilateral uncus of the temporal lobe
Chapter 34, on the edge of the tentorium cerebelli as the brain is forced through the tentorial hiatus.
p. 290 As compression of the uncus begins, the third cranial nerve is compressed; anisocoria,
ptosis, impaired extraocular movements, and a sluggish pupillary light reflex develop
ipsilateral to the expanding mass lesion. As herniation progresses, compression of the
ipsilateral oculomotor nerve eventually causes ipsilateral pupillary dilation and
nonreactivity. Contralateral hemiparesis develops as the ipsilateral peduncle is
compressed against the tentorium. With continued progression of the herniation,
bilateral decerebrate posturing eventually occurs. In up to 25% of patients the
contralateral cerebral peduncle is forced against the opposite edge of the tentorial
hiatus. Hemiparesis is then detected ipsilateral to the dilated pupil and the mass lesion.
This is termed Kernohans notch syndrome and causes false-localizing motor findings.
1053. c Epidural hematomas (EDHs) are blood clots that form between the inner table of the
Rosen 5th, skull and the dura. Eighty percent are associated with skull fractures across the middle
Chapter 34, meningeal artery and are therefore located in the temporoparietal region. The
p. 307 incidence of skull fractures in children with EDH is lower than in adults because the
elasticity of the skull during childhood permits it to spring back to its original position
instead of breaking after a significant impact.
1054. d The GCS assesses a patient's best eye, verbal, and motor responsiveness. The score
Rosen 5th, ranges from 3 (unresponsive) to 15. This patient receives a 2 for eye response: opens
Chapter 34, eyes in response to pain; a 2 for verbal response: moaning, no recognizable words; and
p. 292 a 4 for motor response: pulls away from pain in flexion.
1055. a The timely administration of mannitol can be lifesaving. Mannitol (0.25 to 1 g/kg)
th
Rosen 5 , effectively reduces cerebral edema by osmotic diuresis, producing an osmotic gradient
Chapter 34, that prevents further cellular edema and draws tissue water into the vascular space.
p. 295 This reduces brain volume and provides increased space for an expanding hematoma
or brain swelling. The osmotic effects of mannitol occur within minutes and peak
about 60 minutes after bolus administration. The ICP-lowering effects of a single
bolus may last for 6 to 8 hours.
1056. e Subdural hematomas (SDHs) are blood clots that form between the dura and the
Rosen 5th, arachnoid. SDHs are caused by movement of the brain relative to the skull, as is seen
Chapter 32, in acceleration-deceleration injuries. They are common in patients with brain atrophy,
p. 273 such as alcoholic or elderly patients where superficial bridging vessels traverse greater
Chapter 34, distances, and are thus more prone to rupture with rapid movement of the head. The
p. 308 slow bleeding of venous structures in SDHs delays the development of clinical signs
and symptoms. SDHs are common in patients less than 2 years of age.
316

1057. The oculocephalic response (dolls eyes maneuver) tests the integrity of the:
a. cerebellum.
b. hippocampus.
c. hypothalamus.
d. occipital visual fields.
e. pons.

1058. You have just received by ambulance a comatose 19-year-old college student with severe
midface fractures following a collision with a lamppost. Medics were unsuccessful in field
intubation, so you prepare to do rapid sequence intubation, knowing that:
a. thiopental can raise both systemic and intracerebral blood pressure.
b. etomidate is contraindicated.
c. ketamine reduces intracerebral pressure, but may cause severe laryngospasm.
d. pretreatment with lidocaine is not indicated.
e. succinylcholine should be avoided unless a defasciculating dose of a nondepolarizing agent has
first been given.

1059. You are looking at skull x-rays on a child who was dropped on his head onto a concrete
surface. You are trying to differentiate possible skull fracture from normal suture lines,
knowing that:
a. fractures are less lucent than vascular grooves and sutures.
b. fractures are most commonly seen in the temporal region.
c. fractures are narrowest at the ends and widest at the middle.
d. fractures are rarely more than 3 mm wide.
e. most fractures are visible only on one view.

1060. Rapid rises in the ICP can lead to the Cushing reflex of
a. disconjugate gaze, Cheyne-Stokes breathing, and tachycardia.
b. hypertension, bradycardia, and respiratory irregularities.
c. hypotension, frequent PVCs, and decerebrate posturing.
d. upgoing toes, diminished deep tendon reflexes, and Kussmaul respirations.
e. wandering nystagmus, pulsus alternans, and tardive dyskinesia.

1061. You have thoroughly examined a patient who fell off scaffolding. You found only a basilar
skull fracture and clear CSF rhinorrhea. This patient needs:
a. a lumbar puncture.
b. admission for prophylactic intravenous antibiotics.
c. outpatient observation only.
d. outpatient therapy with oral prophylactic antibiotics.
e. serial head CTs.

1062. Most head injuries seen in the Emergency Department are classified as mild and
moderate. Concerning this group:
a. patients with an initial GCS of 9 or greater who later deteriorate to a GCS of 8 generally have a
good prognosis.
b. less than 10% of moderately head-injured patients have an abnormal CT scan.
c. most moderate head trauma patients become asymptomatic within two weeks of injury.
d. signs and symptoms of concussion usually are completely resolved by 6 hours.
e. the most common complaint after concussion is headache.
317

1057. e In the acute setting, brainstem activity is assessed by the patient's respiratory pattern,
th
Rosen 5 , pupillary size, and eye movements. The oculocephalic response (doll's eyes maneuver)
Chapter 34, tests the integrity of the pontine gaze centers. This response cannot be tested until
p. 293 cervical spine fractures have been ruled out. The oculovestibular response (cold water
calorics) assesses the brainstem.
1058. e In general the agents used for RSI in the head-injured patient are the same as those for
Rosen 5th, other patients, although attention must be given to the increased ICP that can occur
Chapter 1, with any physical stimulation of the respiratory tract. Lidocaine (1.5 to 2 mg/kg IV
p. 13 push) effectively attenuates the cough reflex, hypertensive response, and increased ICP
Chapter 34, associated with intubation. Thiopental may also be effective but should not be used in
p. 295 hypotensive patients. If succinylcholine is used, premedication with a subparalytic
dose of a nondepolarizing agent should be considered if time permits, since
fasciculations produced by succinylcholine may increase ICP. The degree of ICP
elevation and its clinical significance are unclear, however, and must be balanced
against the need for rapidly establishing an airway. Etomidate (0.3 mg/kg IV), a short-
acting sedative-hypnotic agent, has beneficial effects on ICP by reducing cerebral
blood flow and metabolism. While ketamine is exceptionally hemodynamically stable,
it increases ICP and should not be used in head-injured patients.
1059. c In general on skull x-rays, fractures are more lucent than vascular grooves and sutures.
Rosen 5th, Sutures are usually less than 2 mm wide in adults; fractures are often 3 mm or greater
Chapter 34, in overall width and tend to be widest in the midportion and narrow at each end.
p. 305 Linear fractures are most common in the temporoparietal, frontal, and occipital regions
of the skull and can usually be visualized on more than one radiographic view.
1060. b Progressive hypertension associated with bradycardia and diminished respiratory effort
Rosen 5th, is a specific response to acute, potentially lethal rises in ICP. This response is called
Chapter 34, the Cushing reflex, or Cushing's phenomenon, and its occurrence indicates that the ICP
p. 290 has reached life-threatening levels. The full triad of hypertension, bradycardia, and
respiratory irregularity is seen in only one third of cases of life-threatening increased
ICP.
1061. c Most CSF leaks spontaneously resolve without complications in 1 week. Therefore, in
Rosen 5th, general, antibiotics are not given prophylactically during the first week of CSF
Chapter 34, rhinorrhea. If a patient with a previously diagnosed CSF leak returns to the ED later
p. 305 with fever, the diagnosis of meningitis should be strongly suspected and appropriate
workup and antibiotic treatment initiated immediately.
1062. d Patients with an initial GCS of 9 or greater who later deteriorate to a GCS of 8 or less
Rosen 5th, have a poorer outcome than those who originally presented with a GCS less than 8.
Chapter 34, Approximately 40% of moderately head-injured patients have an abnormal CT scan,
p. 297, 300 and 10% lapse into coma. Most moderate head trauma patients remain symptomatic
for extended periods after head injury. At 3 months after trauma, up to 70% are unable
to return to work, 90% have memory difficulties, and more than 90% complain of
persistent headaches. Almost 50% are left with a permanent disability that interferes
with their previous daily activities. A wide spectrum of transient neurologic symptoms
may occur with concussion; the most common complaints are confusion and amnesia
for the traumatic event. Adults may complain of seeing stars or feeling nauseated,
dizzy, or disoriented for a brief period. A brief LOC may last for seconds up to several
minutes, but many patients report no LOC. In children, acute symptoms of concussion
differ from adults and may include restlessness, lethargy, confusion, or irritability. On
presentation, they may be vomiting, be tachycardic, or appear pale. These signs and
symptoms usually are completely resolved by 6 hours.
318

1063. The clinical picture most suggestive for a central cord syndrome is:
a. paralysis below the level of C4.
b. sensory loss below C4.
c. complete motor paralysis with loss of pain and temperature sensation distal to the lesion.
d. ipsilateral paralysis and loss of proprioception and vibratory sensation plus loss of pain and
contralateral loss of pain and temperature sensation.
e. weakness more pronounced in arms than legs and worse in hands than in the proximal upper
extremity.

1064. The maximum neurologic deficit following spinal cord injury is seen:
a. at the time of injury.
b. within hours of initial injury.
c. in the first days after injury.
d. in the first two weeks after injury.
e. six months after injury.

1065. The cervical spine injury most likely to be stable is:


a. bilateral facet dislocation.
b. burst fracture of C1 (Jeffersons).
c. clay shoveler fracture.
d. hangmans fracture.
e. rotary dislocation C1 on C2.

1066. Penetrating neck injury is defined as any wound that violates the:
a. digastric muscle
b. trachea
c. platysma
d. sternomastoid muscle
e. omohyoid muscle

1067. The gold standard for evaluating penetrating trauma to the neck is:
a. angiography.
b. color Doppler ultrasound.
c. contrast CT scan.
d. magnetic resonance imaging.
e. trans-esophageal echocardiography.

1068. A 22-year-old man walks into your Emergency Department. He is covered in blood and
pointing to his throat, where you see a large laceration and a hematoma that is displacing
his trachea. His voice is hoarse and stridorous. You must immediately:
a. consult ENT for emergent tracheostomy.
b. do a cricothyrotomy.
c. insert a laryngeal mask airway (LMA).
d. perform nasotracheal intubation.
e. perform rapid sequence intubation.
319

1063. e Central cord syndrome is a hyperextension injury often seen in patients with
th
Rosen 5 , degenerative arthritis of the cervical vertebrae. The ligamentum flavum buckles into
Chapter 36, the cord, resulting in a concussion or contusion of the central portion of the cord. This
p. 349 injury affects the most central portions of the pyramidal and spinothalamic tracts.
Since nerve fibers that innervate distal structures are located in the periphery of the
spinal cord, these patients have a greater neurologic deficit in the upper extremities
than in the lower extremities.
1064. b The maximum neurologic deficit after blunt spinal cord trauma often is not seen
Rosen 5th, immediately but rather seems to progress and extend over many hours. The ultimate
Chapter 36, extent of spinal cord injury can also be affected by reversible and preventable factors,
p. 345 such as hypoxia, hypotension, hyperthermia, hypoglycemia, and mishandling by
medical personnel.
1065. c Fractures involving C1 and C2 are generally considered anatomically unstable because
Rosen 5th, of their location and the relative paucity of ligamentous and muscle support. In
Chapter 36, assessing the stability of spinal injuries below C2, it is helpful to think of the spine as
p. 331 consisting of two columns. The anterior column is formed by alternating vertebral
bodies and intervertebral disks held in alignment by the anterior and posterior
longitudinal ligaments. The posterior column, which contains the spinal canal, is
formed by the pedicles, transverse processes, articulating facets, laminae, and spinous
processes. If both columns are traumatically disrupted, the spine moves as two
separate pieces. Such a lesion is considered mechanically unstable, and the likelihood
of a spinal cord injury resulting from even a slight motion is great. On the other hand,
if only one column is disrupted, the other column resists further movement. Bilateral
facet dislocation is an extremely unstable condition that is often associated with spinal
cord injury. The clay shoveler's fracture is an oblique fracture of the base of the
spinous process of one of the lower cervical segments. Because this injury involves
only the spinous process, it is stable and not associated with neurologic involvement.
The hangman's fracture occurs when the cervicocranium is thrown into extreme
hyperextension as a result of abrupt deceleration. Bilateral fractures of the pedicles of
the axis occur with or without dislocation. This lesion is unstable.
1066. c The platysma muscle, sandwiched between the superficial and deep cervical fascia,
Rosen 5th, covers the anterolateral neck. It has clinical significance because of its superficial
Chapter 37, location and proximity to vital structures. If the platysma muscle is violated, injury to
p. 372 these structures should be suspected.
1067. a To date, arteriography remains the gold standard for diagnosing vascular injuries.
Rosen 5th, Despite a sensitivity and specificity of nearly 100% and a complication rate of less
Chapter 37, than 2% for arteriography, other, less invasive diagnostic tests have been evaluated for
p. 377 accuracy, speed, cost, and efficacy. Duplex ultrasonography has been used to exclude
cervical vascular injury in patients with both penetrating and blunt trauma. Despite
clinical success, limitations of ultrasonography include the risk of missing zone I and
III injuries and the lack of 24-hour availability at many centers.
1068. e Recent literature supports orotracheal RSI by in trauma patients with blunt or
Rosen 5th, penetrating neck injuries, and it should be considered the first-line airway technique
Chapter 37, unless contraindications exist. Concerns over sedative-induced muscle relaxation
p. 373 leading to airway distortion, although possible, have not been validated. Orotracheal
RSI is often successful even after neck trauma with airway distortion. RSI also has
been shown to be superior to intubation without paralytics. If the cervical spine must
remain immobilized, an assistant should maintain in-line stabilization of the head and
neck..
320

1069. Clinical findings of infraorbital anesthesia and inability to gaze vertically suggest:
a. zygomatic arch fracture
b. orbital wall fracture
c. orbital floor fracture
d. LeFort I fracture
e. LeFort II fracture

1070. Typical components of a tripod fracture are:


a. central maxillary fracture, orbital floor fracture, zygomatic arch fracture.
b. infraorbital rim fracture, zygomatico-frontal suture diastasis, zygomatico-temporal disruption.
c. orbital wall, pterygoid plate, nasal septum.
d. supraorbital rim fracture, zygomatico-frontal suture diastasis, zygomatico-temporal disruption.
e. zygomatico-frontal suture diastasis, zygomatico-temporal disruption, zygomatic arch fracture.

1071. In a dyspneic chest trauma patient with distended neck veins, hypotension, tracheal
deviation to the right and absent breath sounds on the left, the correct approach is:
a. confirm hemothorax with chest radiograph, place chest tube.
b. confirm simple pneumothorax with chest radiograph, place chest tube.
c. confirm tension pneumothorax with chest radiograph, place chest tube.
d. convert tension pneumothorax to simple pneumothorax with insertion of 14 gauge cannula in left
chest.
e. treat suspected cardiac tamponade with emergency pericardiocentesis.

1072. In a dyspneic chest trauma patient with distended neck veins, hypotension, midline
tracheal and equal breath sounds, the correct approach is:
a. confirm cardiac tamponade with chest radiograph, administer normal saline, perform
pericardiocentesis.
b. confirm cardiac tamponade with echocardiogram, administer normal saline, perform
pericardiocentesis.
c. suspect cardiac tamponade, administer normal saline, perform pericardiocentesis.
d. suspect bilateral tension pneumothoraces, perform bilateral needle decompression.
e. suspect bilateral simple pneumothoraces, place bilateral chest tubes.

1073. In a hypotensive chest trauma patient with unilateral absent breath sounds and a chest
radiograph showing a massive fluid collection, the ED approach involves:
a. immediate tube thoracostomy.
b. immediate thoracotomy.
c. fluid resuscitation to a systolic BP greater than 90, followed by operative repair.
d. early administration of O negative blood followed by tube thoracostomy.
e. early administration of fresh frozen plasma followed by tube thoracostomy.
321

1069. c Patients with orbital floor fractures may have impaired


th
Rosen 5 , upward gaze, diplopia due to entrapment of the inferior
Chapter 35, rectus muscle within the fracture fragments, and
p. 324, 325 infraorbital hypoesthesia. Radiographic evaluation may
show the "hanging drop" sign of herniated contents into
the maxillary sinus depression of bony fragments into the
maxillary sinus, or emphysema of the orbit. Clouding of
a maxillary sinus on the side of the trauma must be
assumed to be an orbital fracture until proved otherwise.

Figure 35-9 Appearance of eyes and


limitation of motion typically present
in blowout fracture. Fracture of right
orbit is shown here.
1070. b The zygoma has two major components: the body or malar eminence and the
Rosen 5th, zygomatic arch. Two common fractures of this bone are (1) depression of the malar
Chapter 35, eminence, with fracture lines usually at the zygomaticotemporal suture, at the
p. 324 zygomaticofrontal suture, and through the infraorbital foramen (the so-called trimalar
or tripod fracture), and (2) depression of the zygomatic arch. Clinical signs of a tripod
fracture include flatness of the cheek, anesthesia in the distribution of the infraorbital
nerve, diplopia or change in consensual gaze, or a palpable step defect.
1071. d This description is classic for tension pneumothorax. It occurs when the injury acts
Rosen 5th, like a one-way valve and leads to a progressive increase of intrapleural pressure. Air
Chapter 38, enters on inspiration but cannot exit with expiration, which compresses the vena cava
p. 388, 391 and distorts the cavoatrial junction, leading to decreased diastolic filling of the heart
and subsequent decreased cardiac output. These changes result in the rapid onset of
hypoxia, acidosis, and shock. When the diagnosis of tension pneumothorax is
suspected clinically, the pressure should be relieved immediately with needle
thoracostomy, which is performed by inserting a large-bore (16-or 18-gauge) through
the second or third interspace anteriorly or the fourth or fifth interspace laterally on the
involved side.
1072. c This patient has all the clinical signs of cardiac tamponade. Although an
Rosen 5th, echocardiogram would confirm the diagnosis, there may not be time do so in a
Chapter 38, hemodynamically unstable patient. When the patient arrives in the ED, volume
p. 400 expansion with crystalloid solution via two or three large-bore (14-or 16-gauge)
catheters should be established immediately. Aspiration of as little as 5 to 10 ml of
blood by pericardiocentesis may result in dramatic clinical improvement. Reducing
the total intrapericardial volume to just below the critical level allows compensatory
mechanisms to maintain adequate hemodynamics. Whenever possible,
pericardiocentesis should be performed under sonographic guidance because this
approach will increase success rate and decrease the incidence of complications.
1073. a This patient has a massive hemothorax. Treatment consists of restoring the circulating
Rosen 5th, blood volume, controlling the airway as necessary, and evacuating the accumulated
Chapter 38, blood. Tube thoracostomy allows constant monitoring of the blood loss as well as
p. 391 reexpansion of the lung. A large-bore tube (36 to 40 French) should be inserted in the
fifth interspace at the anterior axillary line and connected to underwater-seal drainage
and suction. Although small hemothoraces may be observed in stable patients, a
moderate hemothorax or any hemothorax in an unstable or symptomatic patient
requires tube thoracostomy. Severe or persistent hemorrhage requires thoracotomy.
322

1074. The child at moderate risk for intracranial injury would be the one with:
a. loss of consciousness for less than 5 minutes.
b. headache.
c. impact seizure.
d. vomiting.
e. seizure more than 20 minutes after impact.

1075. Concerning the Emergency Department management of facial lacerations:


a. beveled lacerations should be debrided parallel to the lacerated edges to preserve orientation with
the opposite side and allow for improved closure.
b. debris embedded in a traumatic abrasion should be removed by a consulting plastic surgeon 3 to 4
days after the accident to allow easier removal and facilitate a better cosmetic outcome.
c. dog bite puncture wounds to the face should be copiously irrigated, explored for deep tissue
injury, and closed primarily.
d. relatively clean facial wounds may be repaired up to 24 hours after injury.
e. cosmetic quality of the final result is apparent at 3 months.

1076. Loss of cerebral autoregulation leads to:


a. cerebral blood flow which varies with arterial pressure
b. compensatory hyperventilation
c. drastically decreased intracranial pressure
d. epidural hematoma
e. profound constriction of cerebral resistance vessels

1077. The most common CT scan abnormality found after severe closed head injury is:
a. cerebral contusion.
b. epidural hematoma.
c. intracerebral hemorrhage.
d. subdural hematoma.
e. traumatic subarachnoid hemorrhage.

1078. In children, the most commonly injured intra-abdominal structure is:


a. pancreas.
b. liver.
c. small bowel.
d. spleen.
e. kidney.

1079. Maternal death is the most common cause of fetal death in pregnant trauma patients.
What is the next most common cause?
a. placenta previa.
b. placental abruption.
c. fetal head injury.
d. fetal cardiac injury.
e. fetal organ rupture.
323

1074. e The prognostic significance of vomiting after pediatric head trauma presents the
th
Rosen 5 , clinician with many challenges. There is no adequate study defining an acceptable
Chapter 32, time frame in which vomiting after head injury is benign in nature. The development
p. 272 of seizures after head trauma, in contrast to vomiting, has been well studied. A brief
seizure that occurs immediately after the insult (with rapid return of normal level of
consciousness) is commonly called "an impact seizure" and is usually unassociated
with intracranial parenchymal injury and in no way mandates the institution of
anticonvulsant therapy. Seizures that occur later (longer than 20 minutes after the
insult) portend the greater possibility of both internal injury and the development of
seizures at a later date. Patients who experience seizures later in the course of the
posttraumatic event are best evaluated by the neurosurgical service.
1075. d Beveled lacerations should have their edges debrided to convert to perpendicular
Rosen 5th, edges. Any embedded material in a traumatic abrasion should be carefully removed.
Chapter 35, A delay in treatment may cause permanent discoloration (tattoo). Puncture wounds
p. 318, 319 from any species should not be closed. A wound up to 24 hours old may be closed on
Chapter 54, the face, but this must be tempered by the mechanism of injury (e.g., severe crushing)
p. 778 and the degree of contamination (e.g., animal bite). Patients with facial wounds should
be told that the final result of treatment cannot be evaluated until 6 to 12 months after
the injury, when maximum scar resorption and softening will have occurred.
1076. a With the loss of autoregulation, massive cerebral vasodilation occurs. Systemic
Rosen 5th, pressure is transmitted to the capillaries, and the outpouring of fluids into the
Chapter 34, extravascular space can contribute to vasogenic edema and thus further increase ICP.
p. 289
1077. e Traumatic subarachnoid hemorrhage (TSAH) is defined as blood within the CSF and
th
Rosen 5 , meningeal intima and probably results from tears of small subarachnoid vessels.
Chapter 34, TSAH is detected on the first CT scan in up to 33% of patients with severe traumatic
p. 310 brain injury and has an incidence of 44% in all cases of severe head trauma. It is
therefore the most common CT scan abnormality seen after head injury.
1078. d Injuries to the spleen are in the largest proportion of pediatric abdominal trauma. The
Rosen 5th, liver is the second most commonly injured solid organ in the pediatric patient with
Chapter 32, abdominal trauma.
p. 279
1079. b The leading causes of traumatic fetal death include maternal death, maternal shock and
th
Rosen 5 , hypoxia, placental abruption, and direct fetal injury. Abruption occurs in 2% to 4% of
Chapter 31, minor accidents and in up to 38% of major injuries. Placental position does not appear
p. 258 to affect the incidence of abruption. Fetal mortality in abruption cases approaches
54%. Classic clinical findings of abruption may include vaginal bleeding, abdominal
cramps, uterine tenderness, amniotic fluid leakage, maternal hypovolemia (up to 2 L of
blood can accumulate in the gravid uterus), or a change in the fetal heart rate. The
most sensitive indicator of placental abruption is fetal distress. Hence, prompt fetal
monitoring is very important in trauma during pregnancy. There is also a close linkage
of abruption to uterine activity. Pearlman et al find if there are 12 or more contractions
in any hour of a 4-hour cardiotocographic monitoring period, the risk of abruption is
14%.
324

1080. A 20-year-old cyclist hit a bump and was thrown over his handlebars at a speed of
approximately 20 mph. He has gross hematuria, severe abdominal pain, an elevated
lipase, and absent bowel sounds. The best approach to diagnosing his injuries is:
a. FAST exam.
b. diagnostic peritoneal lavage.
c. serial physical exams.
d. IVP.
e. CT with contrast.

1081. A 34-year-old man was the unrestrained driver in a high-speed rapid deceleration car
crash. He is transported with cervical spine precautions by a Basic Life Support unit. His
Glasgow Coma Scale score is 7, and there are no obvious signs of trauma. His breathing
is shallow, his pulse is thready, and his skin is cool. Prior to intubation, you should:
a. clear the lateral cervical spine x-ray
b. do a brief neurologic examination, including reflexes and Babinski.
c. give four good breaths with 100% oxygen using the bag-valve mask device.
d. obtain a complete set of vital signs.
e. perform chin lift to open the airway.

1082. A 19-year-old male was shot in the back, just lateral to the fifth thoracic vertebra, with a
small caliber handgun. Paramedics phone you with a blood pressure palpable at 90
mmHg, heart rate of 110, and a respiratory rate of 30 per minute. On arrival in your ED,
he has received 800cc of 0.9% saline, but his vital signs are unchanged. He is agitated
and sweaty. His neck veins are not distended, but youre not certain about his heart tones
because of noise. Soon after arrival, his blood pressure starts to fall and his breathing is
more labored, but he is still conscious and oriented. Your next step should be:
a. bedside cardiac ultrasound.
b. blood transfusion.
c. needle decompression of the left chest.
d. pericardiocentesis.
e. thoracotomy.

1083. Select the true statement:


a. Motor vehicle collisions account for most pediatric head injuries.
b. Because of their smaller body surface are, hypothermia is less common in children than in adults.
c. Head injury is the most frequent cause of traumatic death in children.
d. Initial assessment and management of an injured child differs from that of an adult.
e. Maintenance requirement for .

1084. A distraught mother brings her 3-month-old for evaluation after she accidentally dropped
him from his changing table onto the wooden floor. You would be most concerned about
major head injury if you found that the child:
a. had a tonic-clonic seizure lasting 20 seconds immediately after the fall.
b. has been lethargic since the fall.
c. is hypotensive.
d. threw up once almost immediately after the fall.
e. was pale and sweaty for a few minutes after the fall.
325

1080. e Compared with other modalities, numerous advantages of CT are cited. It is


th
Rosen 5 , noninvasive and has the potential to define the injured organ and the extent of the
Chapter pathologic condition. It is most accurate for solid visceral pathology and is often
capable of discerning the presence, source, and approximate quantity of intraperitoneal
hemorrhage. It evaluates the retroperitoneum, an area not sampled by DPL, as well as
the vertebral column and can be readily extended above or below the abdomen to
visualize the thorax or pelvis. It is helpful in the evaluation of hematuria and, if used
early enough, in determining renal artery injury. The primary role of FAST is
detecting free intraperitoneal blood after blunt trauma. IVP only visualizes the renal
parenchyma, and serial exams only suggest an improving or worsening clinical
situation.
1081. c This patient requires immediate intubation and ventilation. Securing the airway,
Rosen 5th, maintaining ventilation, controlling hemorrhage, and treating shock are first priorities
Chapter 30, because of their crucial importance for survival. The evaluation of neurologic
p. 246, 248 function, the cardiac examination, and the status of the abdomen and musculoskeletal
systems are in the second echelon of priorities and should be addressed only after the
more critical ones. For patients who need immediate intubation, head-stabilized oral
intubation with rapid sequence induction is recommended before obtaining cervical
spine radiographs.
1082. a It is unclear whether this patient has pericardial tamponade, a tension pneumothorax,
Rosen 5th, or hemorrhagic shock. The physical findings of pericardial tamponade are
Chapter 38, hypotension, distended neck veins, and, rarely, distant or muffled heart tones. This so
p. 398 called Beck's triad is sometimes difficult to demonstrate clinically, especially in the
midst of a major resuscitation with concomitant hypovolemia. The most reliable signs
of pericardial tamponade are an elevated CVP (>15 cm H2O) with hypotension and
tachycardia. When this triad is present, the diagnoses of acute pericardial tamponade
and tension pneumothorax, should be considered. Ultrasound enables rapid, accurate,
and noninvasive diagnosis of pericardial tamponade. It can be performed at the
bedside in the ED during the initial resuscitation of the patient. The sonographic
definition of tamponade is the simultaneous presence of pericardial fluid and diastolic
collapse of the right ventricle or atrium.
1083. c Falls account for 37% and MVCs for 18% of pediatric head trauma. The
Rosen 5th, proportionately large surface area of an infant or child relative to weight predisposes
Chapter 32, them to large amounts of heat loss as a result of evaporation. Head trauma is the
pp. 268, 272, leading cause of death among injured children and is responsible for 80% of all trauma
267 deaths. The initial assessment and management of children is similar to adults.
Trauma accounts for approximately 30% of infant deaths.
1084. c Many children experience a brief impact seizure at the time of relatively minor head
Rosen 5th, injury. By the time the child is evaluated, he or she is at baseline neurologic function.
Chapter 32, Following minor head trauma, children may demonstrate anorexia, lethargy, vomiting,
p. 274 or pallor soon after the insult followed by a rapid recovery to baseline. Hypotension
Chapter 34, from intracranial bleeding can occur in children less than 1 year of age with a large
p. 300 linear skull fracture and an underlying large epidural hematoma, which is associated
with major head injury.
326

1085. You are evaluating a 22-year-old woman who was the restrained driver in a head-on
motor vehicle collision. She is immobilized on a long board with a hard collar in place.
You know that:
a. in a neurologically normal patient, lack of midline tenderness can safely exclude a cervical spine
injury
b. a lateral cervical spine film is adequate when all 7 vertebrae are visualized
c. approximately 25% of patients with head injuries also have spinal injuries.
d. sedation of the patient may be necessary because of the discomfort of immobilization.
e. if you decide to order cervical spine x-rays, you must see a lateral, odontoid, and AP view before
you can radiographically clear the cervical spine.

1086. People over age 65 years:


a. are more cautious about their balance and therefore fall less frequently than younger people.
b. are more likely to die in a car crash than from any other traumatic injury.
c. have generally been spared from the increased occurrence in assaults and other person-on-person
crimes.
d. rarely are injured in falls which occur where someone is nearby, such as a nursing home.
e. usually sustain multiple orthopedic fractures after a fall.

1087. A 75-year-old man comes to the ED with his son, who tells you
the patient is not acting right. A head CT shows a large
collection of blood, with slight midline shift. You know that:
a. brain atrophy should make him less susceptible to this type of
bleed.
b. if the injury is acute, the possibility of the patient leaving the
hospital alive is less than 25%.
c. mortality from this injury in the elderly is less than half that in
younger patients.
d. the patient is probably a victim of abuse, as this is an uncommon
injury without major trauma.
e. this injury is usually caused by tearing of the middle meningeal
artery.

1088. An 82-year-old man complains of neck pain after falling and striking his head on the
refrigerator as he was bending over to retrieve his glasses from the floor. You have
ordered cervical spine radiographs, knowing the most common injury in this age group is:
a. Jeffersons fracture.
b. odontoid fracture, Type II.
c. rotary subluxation.
d. spinal cord injury without radiographic abnormality (SCIWORA).
e. unilateral facet subluxation.

1089. Which of the following is true regarding metatarsal fractures:


a. metatarsal fractures are 5-10% of foot fractures
b. nondisplaced first metatarsal fractures should be treated with casting for 4 to 6 weeks
c. the most common metatarsal base fractures occur at the first metatarsal.
d. fifth metatarsal shaft stress fractures are uncommon.
e. metatarsal shaft fractures are rarely unstable
327

1085. e A spinal injury can be safely excluded only when the patient has normal mental status,
th
Rosen 5 , absence of neck pain or tenderness, lack of new neurologic signs or symptoms, and
Chapter 30, lack of a distracting painful injury. A cervical spine film is not adequate unless all
p. 250 seven cervical vertebrae and the top of the first thoracic vertebra are visualized.
Chapter 36, Significant forms of head or facial trauma have a 5% to 10% incidence of associated
p. 346 cervical spine injuries. Sedation is reserved for patients with known or probable spinal
injuries who pose a danger to themselves because of excessive movement.
1086. b Falls are the most common mechanism of injury in the elderly, accounting for 40% of
Rosen 5th, trauma in patients older than 65 years of age. Most falls occur at home and are
Chapter 33, ground-level falls. The most common injuries sustained in falls are fractures,
p. 281 occurring in 5% of fall victims. Overall, the mortality rate from falls in elderly patients
approaches 12%. While MVCs are the second most common cause of trauma in the
elderly, the overall fatality rate among elderly MVC victims is as high as 21%.
1087. b Head injuries are the most common cause of mortality in elderly trauma patients. The
Rosen 5th, most common mechanism of significant head injury in the elderly is falls. Epidural
Chapter 33, hematomas are rare because of the adherence of the dura mater to the inside of the
p. 283 skull. Cerebral contusions, however, occur in up to one third of head-injured elderly
patients. Subdural hematomas become more common with age because of the
stretching of the fragile bridging veins as the brain atrophies. The mortality from
subdural hematoma in the elderly is 4 times higher than in younger patients. In acute
subdural hematomas only 1 in 5 elderly patients will survive to discharge.
1088. b The most common mechanism likely to cause a spinal injury in an older person is a
Rosen 5th, fall. Because of the relative immobility of the cervical spine related to DJD, the most
Chapter 33, common level of cervical spine injury in the elderly occurs at C1 to C3, higher than in
p. 283 younger patients. The most common fracture of the cervical spine in the elderly is a
type 2 odontoid fracture, which necessitates adequate visualization of this area of the
cervical spine and may require CT scans with reconstructions.
1089. b Phalangeal fractures are the most common forefoot fracture. Most undisplaced
Rosen 5th, metatarsal shaft fractures of the second through fifth metatarsal may be treated in a
Chapter 52, below-knee walking cast for 2 to 4 weeks. The great toe metatarsal requires more
p. 725, 728, aggressive management because of its biomechanical role and the stresses imposed on
729 it during gait. Nondisplaced first metatarsal fractures should be treated with casting for
4 to 6 weeks. Atleast the first 3 weeks of this immobilization, if not the entire period,
should be non-weight bearing. The most commonly encountered metatarsal base
fractures occur at the fifth metatarsal. The third metatarsal is the most commonly
fractured, and metatarsal shaft stress fractures are common. Navicular fractures,
although rare, are the most common midfoot fracture.
328

1090. Which of the following measurements of Boehlers angle would indicate a compression
fracture of the calcaneus?
a. 45 degrees
b. 40 degrees
c. 30 degrees
d. 25 degrees
e. 15 degrees

1091. Identify the fracture on the accompanying radiograph and choose the
correct emergency management.
a. bimalleolar fracture; cast and crutches in ED; referral to ortho in 48
hours
b. bimalleolar fracture; posterior splint in ED; referral to ortho in 48 hours
c. bimalleolar fracture; posterior splint; orthopedic consult in ED; admit to
ortho
d. trimalleolar fracture; cast and crutches in ED; referral to ortho in 48 hours
e. tibiotalar dislocation; posterior splint in ED; ortho consult in ED; admit to
ortho

1092. According to the Ottawa ankle guidelines, which patient does not need an ankle
radiograph?
a. 25-year-old man, inversion mechanism, could not bear weight at the scene of injury; point
tenderness at tip of lateral malleolus; could not bear weight in the ED
b. 35-year-old runner, inversion injury; finished last mile of his normal 3 mile run, then came to ED
with swollen, tender ankle.
c. 22-year-old woman, uncertain mechanism, possible eversion of ankle playing basketball, carried
off court due to inability to bear weight; point tenderness over medial malleolus.
d. 12-year-old Little League baseball player, turned ankle sliding into second base, walked off field;
pain over both malleoli.
e. 50-year-old construction worker, jumped off scaffold, landed on irregular dirt pile, turned ankle,
carried from site to ambulance; marked bilateral malleolar tenderness, swelling.

1093. Radiographs indicate a spiral fracture of the distal 1/3 of the tibia in a young athlete who
was injured when he made a sharp turn while wearing cleated shoes in a football game.
An associated injury that should be sought is a fracture of the:
a. contralateral tibia
b. ipsilateral 5th metatarsal
c. ipsilateral femur
d. ipsilateral proximal fibula
e. patella

1094. The typical physical finding in intertrochanteric hip fracture is a(n)


a. shortened, internally rotated lower extremity
b. flexed, abducted lower extremity
c. shortened, externally rotated lower extremity
d. externally rotated lower extremity of equal length with the opposite leg
e. lower extremity that is in the normal anatomic position
329

1090. e Boehlers angle is measured on the lateral view of the


th
Rosen 5 , angle between two lines: one between the posterior
Chapter 51, tuberosity and the apex of the posterior facet, and the
p. 724, 725 other between the apex of the posterior facet and the
apex of the anterior process. An angle of less than 20
degrees suggests a compression fracture, and comparison
measurement of the uninjured side is helpful in
questionable cases. Fig. 51-22
1091. c Bimalleolar fractures involve the disruption of at least two elements of the ankle ring
Rosen 5th, and therefore are unstable. These fractures result from adduction or abduction forces,
Chapter 51, although the latter is more common. Rotational injuries also can cause bimalleolar
p. 712 fractures as well as trimalleolar fractures if the posterior malleolus is involved. All
bimalleolar fractures require orthopedic consultation in the ED. Controversy exists
about whether such injuries should be treated closed or surgically.
1092. b The Ottawa Ankle Rules (OAR) state that an ankle radiographic series is required if
Rosen 5th, there is pain in the malleolar region and any one of the following findings: bone
Chapter 51, tenderness at the posterior edge of the distal 6 cm of the lateral malleolus, at posterior
p. 709 edge of the distal 6 cm of the medial malleolus, at the navicular bone, at the fifth
metatarsal, or if the patient is unable to bear weight for at least four steps both
immediately after the injury and at the time of evaluation. The OAR have a sensitivity
approaching 100% in detecting malleolar zone ankle fractures and midfoot zone
fractures. The OAR do not apply for subacute or chronic injuries.
1093. d This is a Maisonneuve's fracture, which is a proximal fibula fracture with an associated
Rosen 5th, ankle fracture or deltoid ligament tear. The mechanism resulting in the Maisonneuve's
Chapter 50, fracture is often an external rotatory force applied to the ankle that results in partial or
p. 702 complete syndesmotic disruption. Palpation of the proximal fibula should be
performed whenever significant ankle injury is present to assess for a Maisonneuve's
fracture.
1094. c The fracture line of intertrochanteric fractures extends between the greater and lesser
Rosen 5th, trochanter of the femur. They are considered extracapsular fractures. The fracture line
Chapter 49, extends through cancellous bone and has an excellent blood supply. The hip's short
p. 656 external rotators remain attached to the proximal femoral neck and the internal rotators
are attached to the distal femur, explaining the position the leg assumes with this
fracture.
330

1095. The abnormality depicted in this radiograph is


a. fifth metatarsal base fracture
b. tarsometatarsal joint disruption (Lisfranc)
c. tibio-talar dislocation
d. talar neck fracture
e. fracture first proximal phalanx

1096. Ankle radiographs of a young athlete who landed on a plantar flexed foot reveal anterior
displacement of the talus without a corresponding fracture. Choose the correct
emergency management of this dislocation.
a. Air Cast, discharge with instructions on partial weight-bearing, orthopedic referral in 48 hours
b. stirrup splint, crutches, referral to orthopedics in 48 hours
c. procedural sedation / analgesia, reduction in ED, observation for neurovascular compromise
d. procedural sedation / analgesia, reduction in ED, post-reduction radiographs, observation for
neurovascular compromise
e. posterior splint, arrangements for orthopedics to take patient to OR for open reduction

1097. The most appropriate Emergency Department management of an acute Achilles tendon
rupture is:
a. Ace bandage to ankle, referral to orthopedics
b. posterior splint in plantar flexion, crutches, referral to orthopedics in 1 week
c. posterior splint in dorsiflexion, crutches, referral to orthopedics in 1 week
d. immediate circular casting by the ED physician, crutches, orthopedic referral
e. emergency orthopedic consultation

1098. You are monitoring a woman who is in her third trimester of pregnancy and was involved
in a minor fender-bender. You have cleared the mother medically and have attached her
to a fetal monitor so you can check the fetus. You know that:
a. beat-to-beat variability of fetal heart rate during a uterine contraction is abnormal and ominous.
b. fetal heart rate should not vary at all during a contraction after trauma.
c. late decelerations are a sign of fetal hypoxia.
d. late heart rate decelerations after trauma are a favorable physiologic response to stress.
e. variable decelerations are normal.

1099. The term greenstick fracture refers to:


a. avulsion of ischial tuberosity in an athlete.
b. buckle fracture of a long bone in a child.
c. fracture that is not visible on standard radiographs, but visible on bone scan.
d. incomplete angulated fracture of a long bone in a child.
e. stress fracture in a child.

1100. A 10 year old has suffered a supracondylar humerus fracture. During exam you find that
the child cannot extend his fingers after wrist extension. The nerve which has probably
been injured is the:
a. anterior interosseous.
b. brachioradialis.
c. median.
d. radial.
e. ulnar.
331

1095. b This is a Lisfranc injury diagnosed by stepoff at the base of the second metatarsal and
th
Rosen 5 , middle cuneiform and widening between the bases of the first and second metatarsals.
Chapter 51, In homolateral injuries, all five metatarsals are displaced in the same direction. In
p. 726, 727 divergent injuries, the metatarsals are splayed outward in both the medial and lateral
directions. These usually occur between the first and second metatarsals because of
their lack of an intermetatarsal connection. The clinical presentation varies with the
extent of injury and displacement. Severe pain in the midfoot and inability to bear
weight, particularly on the toes, usually occur. Paresthesias are occasionally present,
and examination usually reveals edema and ecchymosis.
1096. d The treatment of major talar fractures is controversial. Any significantly displaced
Rosen 5th, fracture, particularly if associated with neurovascular or cutaneous compromise, should
Chapter 51, have an early attempt at closed reduction in the ED. With neurovascular or cutaneous
p. 722 compromise, reduction should not be delayed by waiting for radiographs or
consultation. After appropriate analgesia or conscious sedation, reduction is performed
by grasping the hindfoot and midfoot and applying longitudinal traction with plantar
flexion. This is followed by realignment of the foot as reduction is achieved. Posterior
slab immobilization and postreduction radiographs should then follow.
1097. e The choice of operative versus nonoperative management is controversial. Surgical
Rosen 5th, repair has a lower incidence of rerupture (1.4% versus 13.4% in nonoperative
Chapter 51, management), less muscle atrophy, and earlier resumption of activities. Nonoperative
p. 716 therapy, which involves a series of casts changed in 2-to 4-week intervals, avoids
operative complications and decreases hospitalization and sick leave time. Emergency
orthopedic referral of patients with Achilles tendon rupture is recommended.
1098. c The loss of beat-to-beat and long-term variability warns of fetal central nervous system
Rosen 5th, depression and reduced fetal movement caused by fetal distress. Late decelerations are
Chapter 31, an indication of fetal hypoxia. These decelerations are relatively small in amplitude
p. 262 and occur after the peak or conclusion of a uterine contraction. By comparison, early
decelerations are larger, occur with the contraction, and recover to baseline
immediately after the contraction. Early decelerations may be vagally mediated when
uterine contractions squeeze the fetal head, stretch the neck, or compress the umbilical
cord. Variable decelerations are large, occur at any time, and are possibly caused by
umbilical cord compression.
1099. d Bones of children are necessarily soft and resilient and therefore sustain a number of
Rosen 5th, incomplete fractures. Greenstick fractures are incomplete angulated fractures of long
Chapter 42, bones. The periosteum and and cortex are disrupted on the convex side; the
p. 169 periosteum on the fracture's compression side remains intact. This fracture may need
to be completed to achieve an anatomic reduction.
1100. d In all cases of supracondylar fractures, it is important to carefully assess the distal
Rosen 5th, neurovascular status. Nerve injuries occurred in 7% of 4520 fractures compiled from
Chapter 169, 31 major reported series. The radial, median, and ulnar nerves are all commonly
p. 2374 involved. Most deficits seen at the time of injury are neurapraxias that resolve with
Chapter 45, conservative management. Motor function returns within 7 to 12 weeks, whereas
p. 566 recovery of sensation may take more than 6 months. The radial nerve controls
Chapter 101, extension of the fingers, thumb, wrist, and elbow.
p. 1511
332

1101. An 8-year-old girl fell on her outstretched hands at the local playground. Her exam shows no
swelling or deformity, but she is tender when you palpate her distal radius. Her x-ray shows a
small buckle in the radial cortex, with no angulation. The correct ED management includes:
a. Ace bandage, ice / elevation, early mobilization.
b. immediate orthopedic consult for casting.
c. none films are normal.
d. preparation for immediate operative management.
e. volar or sugar-tong splint, ice / elevation, referral to family doctor or orthopedics.

1102. A 25-year-old male arrives with several softball teammates after he injured his shoulder during a
headfirst slide into third base. His hand is behind his head, and he is in severe pain. His humerus is
fully abducted and his elbow is flexed. He has probably incurred:
a. anterior glenohumeral dislocation
b. distal clavicle fracture
c. inferior shoulder dislocation (luxatio erecta)
d. posterior glenohumeral dislocation
e. proximal humerus fracture

1103. Which statement regarding pediatric trauma is correct?


a. Splenic lacerations are usually treated operatively.
b. Abdominal trauma is the most common cause of unrecognized fatal injury in children..
c. Blunt force mechanisms cause 70% of pediatric abdominal trauma.
d. Chance fractures occur in about 25% of restrained children involved in MVCs.
e. Children have proportionally smaller organs than adults.

1104. A patient with a C8 cervical radiculopathy will have difficulty with:


a. arm abduction.
b. elbow extension.
c. elbow flexion.
d. hand grasp.
e. wrist extension.

1105. An adult patient has a full thickness burn of the left arm and leg, with a complete facial burn,
sparing the posterior head. Genitalia and right arm and leg are spared. Half of the torso anteriorly
is burned. The estimated percentage of burn is:
a. 36%.
b. 41%.
c. 45%.
d. 51%.
e. 54%.

1106. With injuries caused by a fall, the Emergency Physician should be aware of the correlation between
the severity of an injury and the height from which the fall took place. Using 12 feet as an average
floor or story, the estimated heights lethal to 50% of the population is:
a. 4 floors.
b. 5 floors.
c. 6 floors.
d. 7 floors.
e. 9 floors.
333

1101. e There are three types of fractures of the radial metaphysis. This patient has a torus
th
Rosen 5 , fracture which results in a buckling of the radial cortex without cortical disruption. Due
Chapter 169, to the strong periosteum, these fractures are not typically associated with severe
p. 547, 2370 angulation, displacement, or rotational abnormalities, and the emergency physician can
competently manage most. Reduction techniques are rarely, if ever, necessary. These
fractures are initially treated with immobilization in a volar or sugar-tong splint with
conversion to a short arm cast on orthopedic follow-up. Healing usually occurs over 2
to 3 weeks, and complications are rarely seen. Analgesia requirements in the immediate
days after the injury are usually minimal after proper immobilization.
1102. c Although much less common than anterior and posterior shoulder dislocations, most
Rosen 5th, Emergency Physicians will see an inferior dislocation during their careers. Mechanism
Chapter 46, of injury usually involves a hyperabduction force that levers the humeral neck against
p. 595-599 the acromion. Then, the inferior capsule tears and the humeral head dislocates
inferiorly. Clinically the patient has the arm locked overhead in 110 to 160 degrees of
abduction. The elbow is usually flexed, and the forearm typically rests on top of the
head. The shoulder is fixed in this position, and any attempts at movement will result in
significant pain. The inferiorly displaced humeral head may be palpable along the
lateral chest wall. A thorough neurovascular examination is essential, and if possible,
orthopedic consultation should be obtained before attempting closed reduction. ED
reduction involves upward and outward traction, accentuating the long axis in which the
humerus is already found. A second person applies countertraction. Brachial plexus
injuries are very common and rotator cuff injuries are almost always associated with this
injury. Thrombosis of the axillary artery has also been associated with luxatio erecta;
adhesive capsulitis is a common long-term complication.
1103. b Because of the desire to maintain immunocompetency in childhood, the spleen is often
Rosen 5th, left in place as long as the patient can be adequately resuscitated with crystalloid and
Chapter 32, blood products. Abdominal trauma is the most common cause of unrecognized fatal
p. 276-279 injury in children. Pediatric abdominal trauma results from blunt causes in 85% of
cases, and penetrating trauma accounts for the remaining 15%. Blunt trauma related to
MVCs causes more than 50% of the abdominal injuries in children, and lap-belt injury,
including small bowel injury and Chance fractures occurs, in approximately 5% to 10%
of restrained children involved in MVCs. Children have proportionally larger solid
organs, less subcutaneous fat, and less protective abdominal musculature than adults,
and therefore relatively more solid organ injuries from both blunt and penetrating
mechanisms.
1104. b Cervical Nerve Root Motor Findings
Rosen 5th, Decreased forearm flexion and hand extension, decreased
C6
Chapter 101, shoulder adduction, weak biceps reflex
p. 609 C7 Weak triceps and finger extension; weak triceps reflex
C8 Weak hypothenar and hand flexors; weak elbow extension
1105. b 9% for arm
Rosen 5th, 18% for leg
Chapter 56, 5% for face
p. 806-807 9% for torso
1106. e With injuries caused by a fall, the Emergency Physician should be aware of the
Rosen 5th, correlation between the severity of an injury and the height from which the fall took
Chapter 30, place. Using 12 feet as an average floor or story, the estimated heights lethal to 50%
p. 244 and 90% of the population (LD50 and LD90) for falls are four and seven floors,
respectively.
334

1107. Criteria for transport to a trauma center include:


a. respiratory rate of 32 per minute.
b. fall from greater than twice patients height.
c. gunshot to extremity distal to elbow or knee.
d. motor vehicle crash with deformity of 12 18 inches.
e. motor vehicle crash with intrusion into passenger compartment of 8 10 inches.

1108. Estimated fluid and blood loss is made based upon certain physiologic criteria. Assuming that the
patient is a healthy, non-athletic, 40-year-old male, who takes no medication, has no pacemaker,
and has no other complicating medical issues, the findings or vital signs most consistent with a
class III hemorrhage would be:
a. normal blood pressure.
b. pulse rate of 110.
c. increased pulse pressure.
d. measured blood loss of 1750 cc.
e. pulse rate of 152.

1109. A physical finding which might help differentiate cardiac tamponade from a tension pneumothorax
would be:
a. absent breath sounds.
b. agitation.
c. distended neck veins.
d. hypotension.
e. muffled heart sounds.

1110. Match the Glasgow Coma Scale finding with its correct numeric value.
a. abnormal extension: 3
b. abnormal flexion: 4
c. disoriented but converses: 3
d. eye opening to pain: 1
e. eye opening to verbal command: 3

1111. The leading cause of traumatic death in children over 1 year of age is:
a. burns.
b. chest trauma.
c. head trauma.
d. infection after trauma.
e. multi-system failure.

1112. Which of the following is correct concering pediatric head trauma?


a. Sutures provide pliability to the pediatric skull that protects not only the skull, but also the brain
parenchyma from injury.
b. The pediatric brain is well myelinated.
c. Seizures that occur later (longer than 20 minutes) after the insult) portend the greater possibility
of both internal injury and the development of seizures at a later date.
d. Impact seizures are associated with intracranial parenchymal injury.
e. Subdural hematomas in children are almost always associated with the presence of overlying
fractures.
335

f.
1107. a Abnormal vital signs (GCS < 14 or systolic BP < 90) (respiratory rate < 10 or >29)
th
Rosen 5 , Multiple-system trauma
Chapter 30, Penetrating wound to: 1) Head, neck, or torso; 2) Gunshot wound(s) to extremities
p. 243-244 proximal to elbow and knee; 3) An extremity with neurovascular compromise
CNS injury (head, spine)
Suspected pelvic fracture
Mechanism of injury: Intrusion into passenger compartment greater than 12 inches or
major vehicular deformity greater than 20 inches
Ejection
Entrapment
Falls greater than three times the patients height
Fatality in same passenger compartment
Rapid deceleration
Auto-pedestrian / auto-bicycle injury with impact >5 mph
Vehicular rollover
Exposure to blast / explosion
1108. d Class I Class II Class III Class IV
Rosen 5th, Blood Loss (mL) Up to 750 750 1500 1500 2000 >2000
Chapter 30, Blood loss (%) Up to 15% 15 30% 30 40% >40%
Pulse rate <100 100 120 120 140 >140
Blood pressure Normal Normal Decreased Decreased
Pulse pressure Normal or increased Decreased Decreased Decreased
1109. a Beck's triad for pericardial tamponade (hypotension, distended neck veins, and, rarely,
Rosen 5th, distant or muffled heart tones) is sometimes difficult to demonstrate clinically,
Chapter 38, especially in the midst of a major resuscitation with concomitant hypovolemia. The
p. 389-390 most reliable signs of pericardial tamponade are an elevated CVP (15 cm H2O) in
association with hypotension and tachycardia. Pulsus paradoxus may also be present.
The cardinal signs of tension pneumothorax are tachycardia, jugular venous distention,
and absent breath sounds on the ipsilateral side. They are dyspneic, agitated, restless,
cyanotic, tachycardic, and hypotensive and display decreasing mental activity. Absent
breath sounds are the key to clinically differentiating these two conditions.
1110. e abnormal extension: 2
Rosen 5th, abnormal flexion: 3
Chapter 34, disoriented but converses: 4
p. 292 eye opening to pain: 2
1111. c Head injury is the most frequent cause of trauma death in children over age 1 year.
There are some important anatomical differences between children and adults that play a
1112. e role. The child's cranial vault is larger and heavier in proportion to the total body mass.
Rosen 5th,
This predisposes the child to high degrees of torque generated by any forces along the
Chapter 32,
cervical spine. The sutures in the child's skull are both protective and detrimental
p. 272-274
regarding head injury in these patients. The cranium is more pliable and resistant to
skull fractures, but forces are generated internally that can damage the brain
parenchyma without fracturing the skull. The pediatric brain is less myelinated,
predisposing it to shearing forces and further injury. A brief seizure that occurs
immediately after the insult (with rapid return of normal level of consciousness) is
commonly called an impact seizure and is unassociated with intracranial parenchymal
injury. However, seizures that occur longer than 20 minutes after the insult indicate
greater possibility of both internal injury and the development of seizures at a later date.
336

1113. An 8 month old infant is brought to you for evaluation after falling off the bed. The child will open
her eyes when you shout, withdraws and grunts to pain. What is the GCS score?
a. 8
b. 9
c. 10
d. 11
e. 12

1114. The most common form of shock in children is:


a. cardiogenic.
b. hypovolemic.
c. hypoxic.
d. neurogenic.
e. septic.

1115. In children, one may need a GCS system that is age oriented. A simple modification includes the
AVPU system. The correct matching for the AVPU system is:
a. A - the child is able to Ask for something
b. V - the child is able to Visually track the questioner
c. P - the child is able to Point to specific people
d. U - the child is Uncooperative when asked to move an extremity
e. P - the child responds to Painful stimuli

1116. After head trauma in a child, a high-risk finding which mandates CT evaluation is:
a. alcohol / drug intoxication.
b. dizziness.
c. focal neurologic findings.
d. headache.
e. persistent vomiting 8 hours after injury.

1117. The most common cause of lethal hemorrhage in the pediatric population is injury to the:
a. Spleen.
b. Liver.
c. Kidney.
d. Small intestine.
e. Large intestine.

1118. Which of the following children should be referred to a pediatric burn center?
a. 12-year-old girl with chemical burns of her chest
b. 10-year-old boy with second degree circumferential burns of his left thigh
c. 6-year-old boy with first degree burns of his hands, feet, or genitalia
d. 10-year-old girl with 8% TBSA second degree burns
e. 12-year-old boy with 5% TBSA third degree burns of his arm
337

1113. b The infant scores 3 for eye opening to verbal command, 4 for withdrawing to pain, and
th
Rosen 5 , 2 for grunting to pain for a total of 9. (Table 32-2)
Chapter 32,
p. 269
1114. b Hypovolemic shock is the most common form of shock in children. Recognizing the
th
Rosen 5 , subtle early signs of shock can be daunting even for experienced emergency
Chapter 32, physicians. Children can maintain an adequate blood pressure, even in the face of
p. 269-270 severe blood loss, but other signs of shock will be apparent.
1115. e In the AVPU system the childs consciousness is rated as: Alert, responds to Verbal
th
Rosen 5 , stimuli, responds to Painful stimuli, or Unresponsive. (Box 32-2)
Chapter 32,
p. 268
1116. c CT scan should be obtained for the presence of neurologic deficits, GCS 14 or lower,
th
Rosen 5 , or injury patterns that are the result of major forcible insults. For children less than
Chapter 32, one year of age, CT scan is recommended for any loss of consciousness, protracted
p. 274, 298, vomiting, irritability, poor feeding or suspicion of abuse.
302
1117. b The liver is the second most commonly injured solid organ in the pediatric patient with
th
Rosen 5 , abdominal trauma, after the spleen. It is, however, the most common cause of lethal
Chapter 32, hemorrhage, carrying a mortality rate of 10% to 20% in severe liver injury.
p. 279
1118. b Burn center referral criteria include 2/3 burns more than 20% BSA (or >10% BSA in
th
Rosen 5 , child under 10 years of age or involving face, hands, feet, or genitalia), 3 burns over
Chapter 56, 10% BSA, 2/3 circumferential burns, electrical/lightning/inhalational burns,
p. 812, Box chemical burns on face, feet, hands or genitalia, patients with burns and pre-existing
56-2 medical disorders that affect burn management, or inability/inexperience with pediatric
burn management at referring center.
338

1119. The best way to manage a patient whose foreskin is caught in a zipper is:
a. stat consult urology.
b. decompress the swollen tissue with an ice pack and wiggle the skin free of the zipper.
c. split the median zipper bar with a wire cutter.
d. use local anesthetic with epinephrine, then excise the skin and refer for a patch skin graft.
e. use a ring cutter on the zipper teeth at the level of attachment.

1120. The most common source of blunt abdominal injury in pregnant women is:
a. aggravated assault.
b. falls.
c. motor vehicle collisions.
d. self-inflicted.
e. spousal violence.

1121. Which physiologic changes of pregnancy can affect interpretation of vital signs in the pregnant
victim of trauma?
a. Blood pressure normally increases by 10 15 mmHg in the 3rd trimester.
b. Blood volume begins to expand at week 10, and may increase by 50%.
c. Cardiac output begins to decrease at week 10, and may reduce by 25%.
d. Heart rate decreases by 10 20 beats per minute during the 2nd trimester.
e. The diaphragm is lowered by as much as 4 cm and tidal volume increases by 40% as residual
volume increases by 25%.

1122. In the injured pregnant patient, a test which may help verify mixing of fetal and maternal blood
cells is the:
a. Bowling-Greene pathway.
b. Fels-Naphtha assessment of pH for ferning.
c. Hurling index.
d. Kleihauer-Betke assay.
e. Rimsky-Korsakov suite.

1123. Of listed radiographic studies, the one giving greatest radiation exposure to a viable fetus is:
a. abdominal film.
b. cervical spine film.
c. head CT scan.
d. lumbar spine film.
e. pelvis film.

1124. Which of these represents a correct chance of survival for perimortem cesarean delivery?
a. Gestational age 22 weeks, survival 8%
b. Gestational age 23 weeks, survival 12%
c. Gestational age 24 weeks, survival 40%
d. Gestational age 25 weeks, survival 50%
e. Gestational age 26 weeks, survival 75%
339

1119. d Zipper entrapment of the foreskin can occur in children, especially those between 2
th
Rosen 5 , and 6 years of age. The zipper can be removed with bone or metal cutters to cut the
Chapter 167, median bar of the zipper (see Figure 167-2)l The zipper falls apart and the foreskin is
p. 2328 freed.
1120. c The most common cause of blunt abdominal trauma in pregnant women is the motor
th
Rosen 5 , vehicle crash, which accounts for up to 70% of acute injuries. The major problem
Chapter 31, centers on one third to one half of all pregnant women not using seat belts or not using
p. 257 them properly. The reasons most often cited include discomfort, inconvenience, a
habit of never using seat belts, or fear of hurting the baby. However, unbelted women
are 2.3 times more likely to give birth within 48 hours of a crash and 4.1 times more
likely to experience fetal death. Improper placement of the lap belt over the pregnant
abdomen causes a threefold to fourfold increase in force transmission through the
uterus. Pregnant crash test dummy tests show that the lowest force transmission
readings through the uterus occur when a three-point seatbelt is used properly.
1121. b Maternal blood volume begins to expand at approximately week 10 of gestation and
Rosen 5th, peaks at 45 to 50% increase from baseline at week 28, resulting in a state of
Chapter 31, hypervolemia. Red cell mass increases to a lesser extent, leading to the relative
p. 256 physiologic anemia of pregnancy. Cardiac output is increased by 1.0 to 1.5 L/min at
week 10 of pregnancy and remains elevated until the end of pregnancy. Heart rate in
the mother is generally increased by 10 to 20 beats per minute in the second trimester,
accompanied by decreases in systolic and diastolic blood pressures of 10 to 15 mmHg.
The diaphragm is raised by as much as 4 cm and tidal volume increases by 40% as
residual volume diminishes by 25%.
1122. d The Kleihauer-Betke test of maternal blood has been used to detect fetal cells in the
Rosen 5th, maternal circulation. Unfortunately, the test is difficult to perform, not immediately
Chapter 31, available in most emergency labs, and only sensitive enough to detect 5 ml of fetal
p. 260 cells in the maternal circulation. Because only 0.1 ml of fetal cells is required to
sensitize the mother, routine immune globulin administration has been recommended
in situations likely to result in sensitization. In instances of significant blunt trauma to
the uterus, the Kleihauer-Betke test should be ordered to detect the rare large fetal
transfusions that may require specific fetal blood therapy or administration of
additional immune globulin to the mother. The standard dose of anti-D immune
globulin (RhoGAM 300 mcg) is sufficient to prevent maternal immunization for fetal
transfusions of up to 15 ml of red blood cells or 30 ml of whole blood.
1123. d Radiation exposure from various procedures:
Rosen 5th, X-ray lumbar spine: 204-1260 mrad
Chapter 31, X-ray abdomen and pelvis: 190-357 mrad
p. 259, Table X-ray cervical spine: <1 mrad
31-2, 31-3 CT head (1 cm slices): <50 mrad
1124. e Estimated perimortem cesarean delivery survival rates:
th
Rosen 5 , 22 weeks 0 % survival
Chapter 31, 23 weeks 15% survival
p. 262, Table 24 weeks 56% survival
31-5 25 weeks 79% survival
340

1125. More than 1.5 million traumatic brain injuries (TBI) occur annually in the US, with about 25%
hospitalization rate, and over 3% fatality rate. Which of the following statements concerning brain
injuries is true?
a. Ethanol-intoxicated individuals have at least a 50% greater chance of sustaining a head injury
than sober individuals.
b. The leading cause of traumatic brain injury death in the over-65 age group is motor vehicle
crashes.
c. The leading cause of TBI deaths in the 15 24 year old age group is motor vehicle crashes.
d. The group most commonly incurring TBI is young adult males.
e. The estimated costs for both acute and chronic TBI is upwards of $40 billion annually, in the
United States.

1126. The cerebral perfusion pressure (CPP) is a pressure gradient against which cerebral perfusion must
work, and is the result of the mean arterial pressure minus the intracranial pressure. Under normal
conditions, ICP is 0 to 10 mmHg. Patients with traumatic brain injury lose the ability to
autoregulate CPP. The true and pertinent statement is:
a. the MAP should be maintained at 120 mmHg or greater
b. the ICP should be kept at 28 mmHg or greater
c. hyperventilation should be used prophylactically to keep pCO2 below 28 mmHg.
d. if despite appropriate ICP and MAP determinations the patient continues to deteriorate, CSF
drainage by ventriculostomy is appropriate.
e. CPP should be maintained at 65 mmHg or lower.

1127. Which of the following is true regarding patients with severe traumatic brain injury?
a. The overall mortality is almost 25%.
b. Of the adults who survive severe head trauma, 30% will have moderate disability or good
outcome.
c. Field intubation has been shown to improve survival to the hospital.
d. Children have a poorer prognosis compared with adults.
e. Steroids administered within 6 hours of head injury improve outcome.:

1128. The most commonly seen brain herniation after traumatic brain injury is:
a. anti herniation of the parietal lobe through the tentorium
b. central transtentorial herniation involving frontal or occipital lobes
c. cerebellotonsillar herniation through the foramen magnum
d. uncal herniation of the temporal lobe through the tentorium
e. upward transtentorial herniation through the posterior fossa.

1129. Common findings in uncal herniation include:


a. bilateral pupillary dilatation.
b. compression of the oculomotor nerve, usually causing ipsilateral dilated and fixed pupil (may be
contralateral at times).
c. conjugate downward gaze and pinpoint pupils.
d. initial bilateral pinpoint pupils, followed by fixed midpoint pupils.
e. rotatory nystagmus.
341

1125. d The costs for treatment of both acute and chronic TBI have been estimated to be $4
th
Rosen 5 , billion dollars annually. The group most commonly incurring TBI is young adult
Chapter 34, males. The elderly and young children are also at greater risk because of underlying
p. 286 anatomic and physiologic factors. In addition, alcoholics are at an increased risk for
TBI; ethanol-intoxicated individuals have a 40% greater chance of sustaining a head
injury than sober individuals. The causal agent for TBI mortality varies greatly by age
and other demographic factors. For example, the leading cause of TBI-related deaths
in the 15- to 24-year-old age group is gunshot wounds, while for those over 65 years it
is falls.
1126. d CPP should be maintained at 70 mmHg or greater. To achieve this, two things must
Rosen 5th, occur: the MAP must be 90 mmHg or greater (which would mean a SBP of 120 to 140
Chapter 34, mmHg) and the ICP no greater than 20 to 25 mmHg. Under these guidelines, any
p. 288 patient with severe TBI must have a MAP >90 mmHg maintained. If this is achieved
and the patient exhibits progressive neurologic deterioration or evidence of herniation
not attributable to extracranial sources, then the ICP is most likely elevated and CPP
decreased. In this situation, CSF drainage by ventriculostomy is the best intervention,
followed by mannitol given as a bolus. Hyperventilation should not be used
prophylactically due to potentially serious side effects from decreased CBF. If it is
used prophylactically, then a PaCO2 of 30 to 35 mmHg should be maintained.
1127. e Up to 25% of patients with major head trauma patients will have lesions requiring
Rosen 5th, neurosurgical evacuation, and the overall mortality of severe head trauma is almost
Chapter 34, 40%. Mortality for children is lower. Adult survivors of severe head trauma are
p. 294 usually severely disabled; currently, only 7% will have moderate disability or good
outcome. Children older than 1 year who survive a severe closed head injury have a
better outcome than adults. Field intubation in patients with severe head injury has
been shown to improve survival to the hospital. Despite their popularity in the past; no
evidence indicates that steroids are of benefit in head injury. They do not lower ICP or
improve outcome and are therefore no longer recommended.
1128. d There are four major brain herniation syndromes: uncal, central transtentorial,
Rosen 5th, cerebellotonsillar and upward posterior fossa. The most common site of herniation
Chapter 34 occurs when the uncus of the temporal lobe is displaced inferiorly through the medial
edge of the tentorium. This is usually due to an expanding lesion in the temporal lobe
or lateral middle fossa. Uncal transtentorial herniation leads to compression of the
third (oculomotor) nerve, causing an ipsilateral fixed and dilated pupil. Further
herniation compresses the pyramidal tract leading to contralateral motor paralysis. In
1129. b up to 25% of patients the contralateral cerebral peduncle is forced against the opposite
Rosen 5th, edge of the tentorial hiatus. Hemiparesis is then detected ipsilateral to the dilated pupil
Chapter 34 and the mass lesion. This is termed Kernohan's notch syndrome and causes false-
localizing motor findings.
342

1130. Mild traumatic brain injury traditionally includes patients with a Glasgow Coma Scale (GCS) of 13
or greater. Which statement is correct?
a. More than 75% of patients with traumatic brain injury and a GCS of 13 have abnormalities on CT
scan.
b. In patients with minor head trauma, those with clinical signs of a basilar skull fracture have a
30% increased incidence of intracranial lesions.
c. About 6-9% of patients with GCS of 15 and history of LOC will have an intracranial lesion.
d. Mild traumatic brain injury represents more than 95% of head injured patients seen in the ED.
e. At least 10% of patients with mild traumatic brain injury may talk and deteriorate within 48
hours of their injury.

1131. Moderate traumatic brain injury, defined as an initial GCS of 9 to 13, is associated with:
a. a mortality rate >25%.
b. a morbidity rate of >45%.
c. a positive CT scan in >50%.
d. a higher neurosurgical intervention rate than mild TBI.
e. an admission rate ten-fold higher than in mild traumatic brain injury.

1132. Severe traumatic brain injury, defined as an initial GCS < 9, is associated with:
a. a surprisingly low need for operative intervention.
b. a mortality approaching 40%, with death usually occurring within 48 hours
c. a relative lack of co-morbidities and other injuries.
d. an intracranial bleed rate of less than 20%.
e. less than 20% long-term disability.

1133. Which dose is considered correct when performing rapid sequence intubation:
a. fentanyl 6 12 mcg/kg.
b. lidocaine 5 mg/kg.
c. succinylcholine 2.5 mg/kg.
d. thiopental 3 mg/kg (if normotensive).
e. vecuronium or pancuronium 1 mg/kg.

1134. The agent implicated in development of truncal and jaw rigidity is:
a. fentanyl.
b. lidocaine.
c. succinylcholine.
d. thiopental.
e. vecuronium.

1135. An agent which can precipitate bronchospasm in a patient with reactive airway disease is:
a. midazolam.
b. etomidate.
c. ketamine.
d. propofol.
e. methohexital.
343

1130. c Most authors now include patients with a GCS of 13 in the moderate traumatic brain
th
Rosen 5 , injury (TBI) group rather than the mild group. Mild TBI accounts for 75 to 80% of all
Chapter 34, patients with head injuries seen in the emergency department. About 3% of patients
p. 297-299 with mild traumatic brain injury may talk and deteriorate; unexpectedly. Less than 1%
will have surgically significant lesions. Patients with clinical signs of basilar skull
fracture have a 50-90% increased incidence of intracranial lesions associated with their
minor head trauma. About 6-9% of patients with GCS of 15 and history of LOC will
have an intracranial lesion. Thus, these patients should be considered high-risk minor
head trauma patients.
1131. b Moderate TBI accounts for approximately 10% of patients with head injuries.
Rosen 5th, Mortality rates for patients with isolated moderate TBI is less than 20%, but long-term
Chapter 34, disability is as high as 50%. Overall, 40% of moderate TBI patients have a positive
p. 297-299 CT scan and 8% require neurosurgical intervention, the same percentage as in mild
TBI. Approximately 10% of all moderate TBI patients will deteriorate due to
secondary brain injury and progress to severe TBI. Most patients with moderate TBI
should be admitted because of the potential for deterioration. Those with an initial
GCS of 13 who return to normal, who remain intact after a 6-12 hour observation
period, and who have a normal CT, can be discharged to family.
1132. b Severe TBI accounts for approximately 10% of all emergency department patients with
Rosen 5th, TBI. The mortality of severe TBI approaches 40% with deaths usually occurring
Chapter 34, within 48 hours. Long-term disability is common, with only about 10% making even a
p. 293 moderate recovery. There are frequently associated injuries, metabolic problems, and
co-morbid factors. More than 80% have CT evidence of bleeding, (1% epidural, 30%
subdural, 30 40% traumatic subarachnoid hemorrhage, 20 30% contusion), and
emergent neurosurgical intervention can be life saving in epidural, subdural, and
subarachnoid hemorrhage.
1133. d succinylcholine 1 - 1.5 mg/kg
Rosen 5th, lidocaine 1.5 mg/kg
Chapter 1, fentanyl 3 5 micrograms/kg
p. 15 vecuronium or pancuronium 0.01 mg/kg
1134. a Truncal and jaw muscle rigidity are rare side effects of fentanyl that can lead to
Rosen 5th, impaired ventilation. This rigidity most often occurs at high doses >10-15 g/kg) and
Chapter 183, may be reversed by parenteral naloxone. If naloxone is unsuccessful, paralysis and
p. 2580 endotracheal intubation may be necessary.
1135. e Methohexital is an ultra-short-acting barbiturate that provides sedation and amnesia for
th
Rosen 5 , short, invasive procedures. Barbiturate administration may precipitate bronchospasm
Chapter 183, in patients with moderate to severe reactive airway disease, thus limiting its use in
p. 2581 those patients. There is some evidence that ketamine may have a mild, transient
bronchodilatory effect.
344

1136. A 32-year-old man stepped on a large piece of broken glass while walking barefoot. He has a 10
cm cut on the medial plantar right foot. Prior to repair, you would want to do a peripheral block of
the:
a. deep peroneal nerve.
b. posterior tibial nerve.
c. saphenous nerve.
d. superficial peroneal nerve.
e. sural nerve.

1137. A 17-year-old softball player takes a bad bounce in her face and has a complex stellate cut on her
lower lip that crosses the vermilion border. Anesthesia is best achieved by performing:
a. inferior alveolar nerve block.
b. infraorbital nerve block.
c. local infiltration with 1% lidocaine with epinephrine.
d. local infiltration with 1% lidocaine without epinephrine.
e. mental nerve block.

1138. You are giving medical command to a prehospital team, which is at the scene of a bus turnover.
The driver has an obvious femur fracture and the paramedics wish to apply a femoral traction splint
(Hare traction). A contraindication to the use of this device is:
a. open femur fracture.
b. angulated femur fracture.
c. prior hip surgery.
d. severe head trauma.
e. shock.

1139. You have just delivered a premature infant who weighs 1150 grams. You do not have inpatient
neonatal capabilities at your hospital, so arrange for transfer to a specialty care center. The
appropriate intravenous fluid to use is:
a. lactated Ringers.
b. 5% dextrose in water.
c. 10% dextrose in water.
d. 0.9% saline.
e. 0.45% saline.

1140. The cornerstone of aviation physiology is:


a. Daltons law.
b. Henrys law.
c. Coles law.
d. Boyles law.
e. Ellay law.

1141. Which of the following is true regarding injury to the kidney?:


a. Renal pedicle injuries typically have a salvage rate of 30-40%.
b. Major renal lacerations represent 25% of all renal injuries.
c. Minor renal lacerations are the most common injuries to the kidney.
d. Patients will typically have more life threatening blood loss from renal vein injury than renal
artery injury.
e. Even in the absence of extension to the renal medulla or collecting system and no evidence of
urine extravasation, most renal lacerations require operative repair
345

1136. b The posterior tibial nerve innervates the sole of the foot. To perform a peroneal nerve
th
Rosen 5 , block, inject 1% lidocaine into the subcutaneous tissue lateral to the posterior tibial
Chapter 51, artery at the upper border of the medial malleolus. None of the other nerves listed
p. 719 supply the plantar surface of the foot. The saphenous nerve provides sensation to the
skin over the medial malleolus. The sural nerve supplies the lateral foot and fifth toe.
The superficial peroneal nerve innervates the dorsum of the foot and the other toes,
except the adjacent sides of the first and second toes, which derive sensation from the
deep peroneal nerve.
1137. e A regional block is preferred for a complex lower lip laceration because it preserves
Rosen 5th, tissue planes and landmarks, facilitating anatomically correct repair. The mental nerve
Chapter 35, supplies the skin and mucus membranes of the lower lip. The mental foramen is
p. 316 located inside the lower lip at its junction with the lower gum, just posterior to the first
premolar tooth. To avoid nerve injury, 1% lidocaine with epinephrine is injected close
to, but not into, the mental foramen. The inferior alveolar and lingular nerves do not
supply the lower lip and thus would not be effective in this patient.
1138. a The femoral traction splint is the preferred device for immobilization of femur
Rosen 5th, fractures. Traction is applied by using a hitch on the ankle that encounters resistance
Chapter 49, when the splint impinges proximally on the pelvis. Contraindications to using this
p. 487, 654 device include open fracture with exposed bone or suggestion of neurologic
involvement.
1139. c Because of the risk of hypoglycemia, all neonates should receive glucose-containing
Rosen 5th, fluids in preparation for and during transport. D10% should be used in infants with a
Chapter 9, p. birth weight greater than 1000 g. D5% is safer in smaller infants because of the risk of
hyperglycemia with the more concentrated solution.
1140. b The cornerstone of aviation physiology is Boyles law, which states that the volume of
Rosen 5th, a unit of gas (a specific number of molecules) is inversely proportional to the pressure
Chapter 187, on it. In concrete terms, Boyles law means that as altitude increases (and atmospheric
p. 2625 pressure decreases), air volume expands. With descent (increasing atmospheric
pressure), air volume contracts.
1141. d Renal vein injuries are more common than renal artery avulsions or intimal tears. Both
Rosen 5th, injuries are often associated with rapid deceleration events. As expected, a venous
Chapter 40, injury can potentially contribute more to a patient's unstable hemodynamic status than
p. 450 an arterial injury. The protective secondary vasospasm following arterial disruption
does not occur with venous injuries. Pedicle injuries rarely occur alone. Most often
they are associated with other life-threatening, non-urologic injuries that require
immediate surgical intervention. The salvage rate at best for a life-sustaining
functioning kidney approaches only 15% to 20%. An IV contrast-enhanced helical CT
scan will identify most renal artery disruptions, whereas renal vein injuries must be
indirectly diagnosed by the presence of a normal-appearing kidney in association with
a large hematoma disproportionate to the rest of the radiographic study. Major renal
lacerations represent approximately 2% to 4% of all renal injuries, and by definition
are associated with renal fractures extending deep into the renal medulla and collecting
system. They also are readily diagnosed by an IV contrast-enhanced helical CT scan.
This injury is in sharp contrast to minor renal lacerations (8% to 15%) and renal
contusions (85% to 92%) that do not extend into the renal medulla or collecting
system, are not associated with extravasation of urine, and heal spontaneously. These
latter two injuries can be managed expectantly and rarely, if ever, require initial or
subsequent operative intervention.
346

1142. The anterior humeral line is most helpful in detecting:


a. elbow dislocation.
b. olecranon fracture.
c. radial head fracture.
d. radioulnar dislocation.
e. supracondylar fracture.

1143. Radial head subluxation:


a. has a long recovery period.
b. has a typical presentation of the arm held in flexion and pronation.
c. is common among high-school athletes.
d. is diagnosed radiographically by a posterior fat pad sign.
e. is usually caused by fall on an outstretched hand.

1144. A 34-year-old woman complains of shortness of breath after a moderate-speed motor vehicle crash
in which she was a restrained front seat passenger. Her upright anteroposterior chest x-ray is
normal, but you detect a 2-mm pneumothorax on chest CT. You would place a thoracostomy tube
if she:
a. developed symptoms of cardiac contusion.
b. developed symptoms of pulmonary contusion.
c. had a paO2 <80 mmHg.
d. had one or more rib fractures.
e. required intubation and mechanical ventilation.

1145. Concerning trauma to the pancreas:


a. complications are infrequent and of little consequence.
b. concomitant injuries are rare.
c. diagnostic peritoneal lavage (DPL) is highly sensitive.
d. penetrating injuries are more common than blunt injuries.
e. serum amylase is almost always elevated.

1146. Which of the following is true of clinicians regarding forensics:


a. Clinicians should describe gunshot wounds as entrance or exit wounds.
b. The ED physician should make an educated speculation regarding the caliber of the bullet.
c. Wounds should be described according to standard anatomic position with the arms to the side
and palms up.
d. The term 'powder burn' should be used to describe the carbonaceous material associated with
close-range wounds. .
e. The physician must record the manner of a gunshot wound victim's death in the medical
record as homicide, suicide, or accidental.
347

1142. e The anterior humeral line is helpful in diagnosing supracondylar fractures.


th
Rosen 5 , Hyperextension forces that occur with a fall onto the arm typically displace the distal
Chapter 169, fracture fragment posteriorly. The anterior humeral line is drawn down the anterior
p. 559 aspect of the humerus on the lateral elbow film. Normally this line will transect the
middle of the capitellum. With a supracondylar fracture, the line usually passes
anterior to the capitellum or through the anterior third.
1143. b Radial head subluxation (AKA nursemaid's elbow) is often caused by traction on a
Rosen 5th, young child's arm, usually under the age of 4 years. The most common position to
Chapter 169, find the child in is in flexion and pronation. It is not associated with abnormal x-rays.
p. 573 The child will often start using the extremity within minutes of reduction.
1144. e Treatment of a simple pneumothorax depends on its cause and size. Most advocate
th
Rosen 5 , treating a traumatic pneumothorax with a chest tube to correct any respiratory
Chapter 72, compromise and because treatment with a chest tube is generally thought to be safer
p. 1004 than observation in these patients. Small pneumothoraces, whether spontaneous or
traumatic, have been treated with hospitalization and careful observation if the patient
is otherwise healthy, symptom free, does not need anesthesia or positive pressure
ventilation, and the size of the pneumothorax is not increasing.
1145. d Pancreatic injury is more common with penetrating trauma. It may also occur as a
Rosen 5th, result of a crushing injury that divides the pancreas over the vertebral column. It may
Chapter 39, also occur as a result of a crushing injury that divides the pancreas over the vertebral
p. 420 column. Examples include steering wheel or bicycle-handlebar injuries.
Unrecognized, this injury has considerable morbidity and mortality. DPL is usually
negative, and the serum amylase is usually normal. Neither serum amylase nor lipase
is useful when obtained routinely. Normal levels do not exclude a major pancreatic
injury, and elevated values may be caused by any of an assortment of reasons in
addition to an injured pancreas. The use of serum amylase isoenzymes has not
appreciably improved accuracy. Non-traumatic causes of hyperamylasemia include
several diseases and the use of alcohol, narcotics, and various other drugs. Amylase or
lipase may also be elevated with pancreatic ischemia produced by the systemic
hypotension that often accompanies trauma. Clearly these enzymes are neither highly
specific nor sensitive for pancreatic injuries. Elevated lavage amylase levels are less
specific and sensitive for pancreatic trauma. Rising levels in serial lavages and clinical
correlation may provide more suggestive evidence of a pancreatic injury.
1146. c Documentation of gunshot wounds should include the location, size, shape, and
Rosen 5th, characteristics of the wound. Clinicians should not describe wounds as entrance or
Chapter 59, exit, but should document a detailed description of the appearance and location of a
p. 828-829 wound without speculating on an interpretation or the caliber of the bullet. Exit
wounds are not always larger than the entrance wound. The term soot rather than
powder burns, should be used to describe the carbonaceous material associated with
close-range wounds. Powder burns are literally the burns associated with the
coincidental ignition of clothing by the flaming black powder used in muzzleloaders,
antique weapons, and blank cartridges. This does not occur with the smokeless
powder used in modern commercial ammunition. Powder burns, therefore, is an
obsolete and potentially misleading expression. It is unnecessary to write in the
medical record, the manner of a gunshot victim's death. The determination of whether
a death is accidental, suicidal, or homicidal is the responsibility of the coroner or
medical examiner and only after a detailed investigation of the scene and
circumstances of the incident.
348

1147. The amount of blood contained in an acute pericardial tamponade after penetrating cardiac injury is
approximately:
a. 10 mL.
b. 50 mL.
c. 200 mL.
d. 500 mL.
e. 1000 mL.

1148. Hamman's sign suggests:


a. pericarditis.
b. pneumomediastinum.
c. pneumothorax.
d. pulmonary embolism.
e. status asthmaticus.

1149. Regarding blunt tracheobronchial injuries:


a. Blunt cervical tracheal injuries usually occur proximal to the tracheal cartilage.
b. Common signs and symptoms include dyspnea, hemoptysis, subcutaneous emphysema,
Hamman's sign, and sternal tenderness.
c. Concurrent esophageal injuries occur in fewer than 5% of cases.
d. Injuries to the major bronchi are usually caused by elevated intraabdominal pressure.
e. Most lower tracheobronchial injuries occur 5 cm or more from the carina.

1150. The most commonly injured structure in the genitourinary (GU) tract is the:
a. bladder.
b. kidney.
c. penis.
d. ureter.
e. urethra.
349

1147. c In a previously normal pericardium, 200 mL of blood will create sufficient


th
Rosen 5 , intrapericardial pressure to restrict venous filling of the heart and cause tamponade.
Chapter 38, Although it may be detected by bedside ultrasound, this amount is too small to be
p. 398-401 reliably visible on chest x-ray. Removal of as little as 5 to 10 mL of blood can be life
saving by augmenting venous filling and dramatically improving stroke volume.
1148. b With pneumomediastinum, a crunching sound known as Hamman's sign can be heard
Rosen 5th, during systole secondary to mediastinal air surrounding the heart. The diagnosis of
Chapter 38, pneumomediastinum is most easily confirmed on CT but can sometimes be made with
p. 408, 1137, plain films. Subcutaneous emphysema in the neck is another suggestive physical
1237 finding.
1149. b Cervical tracheal injuries occur most frequently at the junction of the cricoid cartilage
th
Rosen 5 , and the trachea, usually after striking the anterior neck against the dash of a car. Most
Chapter 38, lower tracheobronchial injuries occur within 2 cm of the carina. Lower
p. 375 tracheobronchial injuries are caused by rapid deceleration and shearing of mobile
bronchi from fixed structures. Concurrent esophageal injuries occur in 25% of
patients.
1150. b In one case series, 67% of all GU injuries were to the kidney. The bladder is the
Rosen 5th, second most commonly injured structure. Kidney injuries include contusions,
Chapter 40, lacerations, ruptures, pedicle injuries, and renal pelvis ruptures.
p. 449
350

Pharmacology
1151. A 32 year old male with an unknown past medical history is brought to the ED after successful
prehospital resuscitation. He is intubated and normotensive but remains poorly perfused. The most
appropriate pressor agent to use in this situation is:
a. epinephrine at 0.05 to 1.0 mcg/kg/min.
b. dobutamine at 5 to 20 mcg/kg/min.
c. dopamine at 5 to 10 mcg/kg/min.
d. dopamine at 2 to 5 mcg/kg/min.
e. norepinephrine at 4 mcg/kg/min.

1152. While using ketamine for adult patients requiring PSAA (Procedural Sedation and Analgesia) in the
Emergency Department, you can minimize the emergence phenomena of hallucinations and
nightmares by also treating with:
a. midazolam.
b. morphine sulfate.
c. fentanyl.
d. intravenous ketorolac
e. nitrous oxide.

1153. Probably the best indication for antibody (Fab) treatment in a patient with digitalis toxicity is:
a. tachydysrhythmias.
b. severe bradycardia.
c. advance AV block.
d. intractable nausea and vomiting.
e. hyperkalemia.

1154. A 27 year old female was found unconscious by her mother, who does not know how long she may
have been this way. The paramedics recovered a suicide note and an empty bottle of the mothers
high blood pressure medications. The patient is profoundly hypotensive with a heart rate of 27.
The medication which she most likely ingested is:
a. hydrochlorothiazide.
b. enalapril.
c. irbesartan.
d. metoprolol.
e. verapamil.

1155. You are treating a 37 year-old woman who has an oral infection. She tells you that she has a
history of myasthenia gravis. You can safely use:
a. tetracycline
b. clindamycin.
c. doxycycline.
d. amoxicillin/clavulanate.
e. a topical preparation containing polymyxin.
351

1051. a Dobutamine may be an effective agent in the normotensive post-arrest patient who
th
Rosen 5 , remains poorly perfused. Dobutamine tends to decrease systemic vascular resistance
Chapter 8, (SVR), which is not helpful in the hypotensive patient.
p. 98
1052. a Emergence phenomena manifested by hallucinations and nightmares occur in up to
th
Rosen 5 , 50% of adults and up to 10% of children receiving ketamine. A study of adult ED
Chapter 183, patients requiring PSAA to facilitate a painful procedure demonstrated that a
p. 2582 combination of midazolam and ketamine produces safe and effective sedation and
analgesia with a low incidence of minor emergence phenomena.
1053. e The chief disadvantage of Fab fragment treatment is expense. Because the average
Rosen 5th, charge to the patient per vial is about $600, it is recommended for life-threatening
Chapter 146, toxicity rather than for routine or prophylactic treatment but certainly before pacing,
p. 2106 which carries significant risk. Probably the best indication for antibody treatment in
acute poisoning I hyperkalemia.
1054. e Calcium channel antagonists block the slow calcium channels in the myocardium and
Rosen 5th, vascular smooth muscle, leading to coronary and peripheral vasodilation. They also
Chapter 146, reduce cardiac contractility, depress SA nodal activity, and slow AV conduction. In
p. 2111 overdose, verapamil has the deadliest profile, combining both severe myocardial
depression with peripheral vasodilation.
1055. d Drugs that may exacerbate myasthenia gravis:
Rosen 5th, Cardiovascular: beta-blockers, calcium channel blockers, quinidine, lidocaine,
Chapter 102, procainamide
p. 1523, Box Antibiotics: aminoglycosides, tetracyclines, clindamycin, lincomycin, polymyxin B,
102-2 colistin
Other: phenytoin, neuromuscular blockers, thyroid replacement
352

1156. Prophylaxis is required for patients with cat bites, as they tend to be deep puncture wounds that are
difficult to irrigate adequately. The antibiotic of choice is:
a. cephalexin.
b. amoxicillin with clavulanate.
c. dicloxacillin.
d. clindamycin.
e. erythromycin.

1157. A 67 year old woman with a long-standing history of asthma presents in severe respiratory distress.
She is anxious, hyperactive and diaphoretic. She also has a history of hypertension and her BP is
210/140 mm Hg. Her heart beat is 190 / minute. It is apparent she will require rapid sequence
intubation soon, so you begin gathering the appropriate medications, including:
a. lidocaine 1.5 mg/kg IV.
b. rapacuronium 1.5 mg/kg IV.
c. etomidate 0.3 mg/kg IV.
d. rocuronium 1.0 mg/kg IV.
e. fentanyl 3 mcg/kg IV.

1158. A 69 year old female complains of fever; crampy abdominal pain; and explosive severe watery /
bloody diarrhea. She recently took clindamycin for an intraoral infection. She is sick enough to
require admission to the hospital, and your first drug of choice to treat he should be:
a. Lomotil 1 tablet after each loose bowel movement.
b. metronidazole 250 mg po daily for 7 days
c. metronidazole 500 mg IV tid for 10 to 14 days.
d. vancomycin 125 mg po qid for 10 to 14 days
e. vancomycin 250 mg IV bid for 7 to 10 days

1159. A 62 year old male with history of Zollinger-Ellison syndrome presents with severe epigastric pain,
vomiting with coffee-ground emesis. He has been doing relatively well on H2 blockers. The
appropriate dosage of proton pump inhibitor to treat this condition is:
a. omeprazole 60 mg/day.
b. rabeprazole 20 mg/day.
c. lansoprazole 15 mg/day.
d. pantoprazole 40 mg/day.
e. omeprazole 20 mg/day.

1160. A 27 year old female complains of a bright red rash on both of her cheeks; she also says her joints
are sore. When you examine the list of medications she is presently taking, you see that these
symptoms could be explained by her use of:
a. depo shots.
b. fluoxetine.
c. isoniazid.
d. albuterol.
e. naproxen.

1161. A commonly used, effective yet inexpensive topical ophthalmic anesthetic is:
a. tetracaine (Pontocaine).
b. adrenaline.
c. lidocaine.
d. bupivicaine.
e. benzocaine.
353

1056. b The organisms found in cat bites include Staphylococcus species, and most often,
th
Rosen 5 , Pasteurella multocida. P. multocida is usually found in cat bite wounds and is present
Chapter 52, in the normal oral flora of up to 70% of all cats. P. multocida is still sensitive to
p. 748 penicillin, but the infection is often polymicrobial. In addition, P. multocida is
resistant to dicloxacillin, cephalexin, and clindamycin, with many erythromycin-
resistant strains. Amoxicillin with clavulanate is the current recommendation for
antibiotic prophylaxis for cat bites.
1057. e In patients at risk from acute blood pressure elevation, administration of fentanyl 3
Rosen 5th, mcg/kg, during the pretreatment phase of RSI attenuates the HR and BP rise. The full
Chapter 1, sympatholytic dose of fentanyl; is 5 or 6 mcg/kg, but if this dosage is administered as a
p. 14 single pretreatment bolus, hypoventilation or apnea can occur. The administration of 3
mcg/kg is safer and can be supplemented with an additional 3 mcg/kg immediately
after the succinylcholine if full sympathetic blockade is desired or if hypertension and
tachycardia ensue.
1058. c Vancomycin is generally not effective if given intravenously because it does not reach
Rosen 5th, effective intraluminal concentrations. Because vancomycin is much more expensive
Chapter 89, than metronidazole and both seem to be equally effective, oral vancomycin is reserved
p. for patients who do not respond to metronidazole therapy or for those who are
extremely ill at the time of presentation.
1059. a Proton pump inhibitors are indicated for the short-term treatment of active duodenal
Rosen 5th, ulcer and severe erosive esophagitis and in the treatment of symptomatic GERD that
Table 84-2, has not responded to therapy with H2 blockers. They are used in significantly higher
p. 1248 dosages in patients with Zollinger-Ellison syndrome. Omeprazole 60 mg/day and
lansoprazole 60 mg/day are the two currently recommended therapies.
1060. c Numerous medications can cause a lupus-like syndrome.
Rosen 5th, High risk: procainamide, quinidine, hydralazine, methyldopa, reserpine, D-
Table 112-1, penicillamine
p. 1611 Moderate risk: isoniazid, penicillin, sulfonamides, streptomycin, tetracycline,
nitrofurantoin, phenytoin, ethosuximide, and primidone
Low risk: propylthiouracil, chlorpromazine, lithium carbonate
1061. a Tetracaine is a potent ester used for surface anesthesia of the cornea and is a
Rosen 5th, component of TAC solution.
Chapter 182,
p. 2574
354

1162. A 62 year old female with a longstanding history of rheumatoid arthritis takes several nonopioid
analgesics; she was recently diagnosed with essential hypertension. The most appropriate anti-
hypertensive medication for her is probably:
a. hydralazine.
b. atenolol.
c. nifedipine.
d. clonidine.
e. hydrochlorothiazide.

1163. The generally acknowledged opioid of choice for Procedural Sedation and Analgesia (PSAA) in the
Emergency Department is:
a. meperidine.
b. morphine.
c. fentanyl.
d. hydromorphone.
e. ketorolac.

1164. A 66 year old female presents to the ED with new onset of symptoms highly suggestive for
uncomplicated gastritis. She is has been taking NSAIDs for a number of years. Your initial
therapy should begin with:
a. an H2 antagonists.
b. a proton pump inhibitor.
c. bismuth subsalicylate.
d. misoprostol.
e. an aluminum-containing antacid

1165. The antidysrhythmic which has characteristics of all four Vaughan-Williams classes is:
a. lidocaine.
b. procainamide.
c. quinidine.
d. amiodarone.
e. bretylium.

1166. Hypoxia and ischemia can lead to interstitial accumulation of adenosine in the myocardium. A
competitive antagonist of adenosine is:
a. atropine.
b. magnesium sulfate.
c. aminophylline.
d. bretylium.
e. epinephrine.

1167. Calcium chloride 4 mg/kg (0.04 ml/kg of 10% solution) may be beneficial in attempting to
resuscitate a patient known to have
a. hyperkalemia
b. hypernatremia.
c. hypophosphatemia.
d. hypermagnesemia.
e. cyanide toxicity.
355

1162. c Nonopiod analgesic agents may interfere with the antihypertensive actions of
th
Rosen 5 , numerous drugs, and calcium channel blockers may be the antihypertensive agents of
Chapter 182, choice when used with a NNA.
p. 2569
1163. c When given IV, fentanyl rapidly crosses the blood-brain barrier, producing analgesia in
th
Rosen 5 , as little as 90 seconds. Serum fentanyl levels rapidly decline from peak concentrations
Chapter 183, because of extensive tissue uptake, producing a serum half-life of 90 minutes and
p. 2563 duration of action of 30 to 40 minutes. The combination of potency and short half-life
permits the administration of multiple small doses that can be titrated easily to the
desired clinical effect. It is an ideal agent for brief procedures such as incision and
drainage of abscesses or reduction of joint dislocations.
1164. a Treatment of presumptive gastritis can be started in the ED. H2 antagonists have been
Rosen 5th, shown to improve symptoms of dyspepsia in patients taking NSAIDs. Dosages should
Chapter 84, begin at the low end and be tapered up as necessary. Patients with persistent
p. 1241 symptoms should be referred to a gastroenterologist for further diagnostic evaluation.
1165. d Amiodarone is considered a class III antidysrhythmic agent but has characteristics of
th
Rosen 5 , all four Vaughan-Williams classes.
Chapter 7,
p. 79
1166. c Hypoxia and ischemia lead to the interstitial accumulation of adenosine in the
th
Rosen 5 , myocardium. Adenosine is known to depress automaticity of the sinus node. AV
Chapter 7, junction. His bundle, and Purkinje fibers, as well as conduction through the AV node.
p. 79-80 Aminophylline is a competitive antagonist of adenosine. Aminophylline (250 mg IV)
given to patients who failed ACLS with more than 2 mg of epinephrine and 2 mg of
atropine resulted in ROSC in 11 of 15 patients.
1167. d Calcium administration is likely to be beneficial in cases of hyperkalemia,
Rosen 5th, hypocalcemia, or calcium channel blocker toxicity. If required, 4 mg/kg of calcium
Chapter 7, chloride (0.04 ml/kg of 10% solution may be administered every 10 minutes.
p. 80
356

1168. Although SaO2 is maintained for up to ___ in untreated ventricular fibrillation, this rapidly falls to
hypoxic range within two minutes of chest compressions without ventilation.
a. 5 minutes
b. 10 minutes
c. 15 minutes
d. 30 minutes
e. 45 minutes

1169. The single most useful drug currently available for the treatment of cardiac arrest is:
a. epinephrine.
b. amiodarone.
c. lidocaine.
d. atropine.
e. calcium.

1170. A 68 year old female with a history of end-stage renal disease requires rapid sequence intubation.
You wish to use a neuromuscular blocking agent which is not excreted by the kidneys, so choose:
a. atracurium.
b. d-tubocurarine.
c. vecuronium.
d. pancuronium.
e. cisatracurium.

1171. A 35 year old woman presented to the Emergency Department in status asthmaticus and required
rapid sequence intubation. She is now starting to waken; she is restless and bucking the
ventilator. An appropriate drug, dose, and route to use is:
a. diazepam 0.2 mg/kg IV.
b. lorazepam 1 mg/kg IV.
c. phenobarbital 120 mg IM.
d. ketamine 1 to 2 mg/kg IM.
e. etomidate 0.3 mg/kg IV.

1172. The preferred induction agent for use in a patient with status asthmaticus or a head-injured patient
with normal blood pressure is:
a. diazepam 0.2 mg/kg IV.
b. lorazepam 1 mg/kg IV.
c. phenobarbital 120 mg IM.
d. ketamine 1 to 2 mg/kg IM.
e. etomidate 0.3 mg/kg IV.

1173. A 57 year old woman presents to the ED in apparent septic shock. She has no history of renal
insufficiency, but her urine output remains low despite a 20 ml/kg of normal saline solution and a
dopamine intravenous drip at 15 mcg/kg/min. Your next step should be to:
a. increase the dopamine to 20 mcg/kg/min.
b. start dobutamine at 5 mcg/kg/min; increase as needed to 20 mcg/kg/min to maintain urine output.
c. start norepinephrine at 0.1 mg/kg/min.
d. start milrinone at 5 to 10 mcg/kg/min.
e. start amrinone at 0.5 mcg/kg/min.
357

1168. d Although SaO2 is maintained for up to 30 minutes in untreated ventricular fibrillation,


th
Rosen 5 , this rapidly falls to hypoxic range within two minutes of chest compressions without
Chapter 7, ventilation.
p. 80
1169. a Epinephrine is the single most useful drug currently available for the treatment of
th
Rosen 5 , cardiac arrest. The most important mechanism of action of epinephrine during CPR is
Chapter 7, to increase peripheral vascular resistance through direct alpha-adrenergic activity on
p. 78 arteriolar smooth muscle.
1170. a Atracurium is the competitive neuromuscular blocking agent for use in patients with
th
Rosen 5 , renal failure because its excretion is completely independent of renal function.
Chapter 1,
p. 12
1171. a After intubation, longer paralysis is usually desired for patient control and to permit
th
Rosen 5 , mechanical ventilation. In most cases, one agent is comparable to another, and cost
Chapter 1, may be a consideration. Longer term neuromuscular blockade must not be undertaken
p. 13 without attention to appropriate sedation of the patient. An adequate dose of a
benzodiazepine, such as diazepam 0.2 mg/kg IV, is often the best initial choice for
sedation accompanying use of longer-acting neuromuscular blocking agents,
1172. d The principle use of ketamine in emergency airway management is for the induction of
Rosen 5th, patients with asthma and hemodynamically unstable trauma patients without head
Chapter 1, injury.
p. 13
1173. b Septic shock refractory to volume restoration requires vasopressor support. Dopamine
th
Rosen 5 , is a rational first-line therapy at 5 to 15 mcg/kg/min and titrated to urine output greater
Chapter 4. than 1 ml/kg/hr. If urine output remains low with high doses of dopamine, dobutamine
p. 43 should be started at 5 mcg/kg/min to increase cardiac output and increased to 20
mcg/kg/min to maintain urine output.
358

1174. A 57 year old man presents to the ED in cardiogenic shock with an extensive acute anterior wall
myocardial infarction. Despite maximal doses of dobutamine and dopamine with incremental fluid
challenges, he remains hypotensive. Your next step should be to add:
a. epinephrine at 0.1 mg/kg/hr IV.
b. amiodarone at 25 mg/kg/hr IV after a 300 mg bolus.
c. norepinephrine at 0.1 mg/kg/min.
d. milrinone at 5 to 10 mcg/kg/min after an IV bolus of 0.75 mg/kg.
e. amrinone at 5 to 10 mcg/kg/min after an IV bolus of 0.75 mg/kg.

1175. A 71 year-old gardener presents in apparent anaphylactic shock after being stung by several yellow
jackets. He has a Medic Alert bracelet which states Allergy to Bees; Heart Condition. Your
initial therapy should include:
a. dobutamine 5 mcg/kg/min IV.
b. ketamine 0.1 mg/kg IV.
c. epinephrine 1 ml of 1:10,000 (100 mcg) slow IV.
d. diphenhydramine 5 mg/kg IV.
e. hydrocortisone 30 mg/kg IV.

1176. A 3 week old female presents lethargic, somnolent, ashen and with a very poor suck. You suspect
acute bacterial meningitis and begin empiric therapy with:
a. ampicillin 100 mg/kg IV plus cefotaxime 50 mg/kg IV or ceftriaxone 100 mg/kg IV.
b. ampicillin 100 mg/kg IV plus cefotaxime 50 mg/kg IV or gentamicin 2.5 mg/kg IV.
c. cefotaxime 50 mg/kg IV or ceftriaxone 100 mg/kg IV and vancomycin 15 mg/kg IV.
d. nafcillin 25 mg/kg IV.
e. ticarcillin 50 mg/kg IV.

1177. Of the benzodiazepines, the one best suited as an induction agent is.
a. alprazolam.
b. diazepam.
c. lorazepam.
d. midazolam.
e. triazolam.

1178. Naloxone is a pure opioid antagonist. It is the most commonly used agent for reversal of opioid-
induced sedation and respiratory depression. The dose for complete reversal is:
a. 0.1 to 2 mg IV.
b. 0.8 mg IV.
c. at least 2 mg.
d. 0.16 mg.
e. 0.01 mg/kg.

1179. The sedative-hypnotic which must be given in a continuous intravenous infusion is:
a. propofol.
b. ketamine.
c. midazolam.
d. alprazolam.
e. phenobarbital.
359

1174. e To improve myocardial contractility, dobutamine and dopamine are agents of choice in
th
Rosen 5 , cardiogenic shock. For refractory hypotension and shock, amrinone or milrinone may
Chapter 4, improve cardiac output. These drugs exhibit little tachyphylaxis with no measurable
p. 45 increase in myocardial oxygen consumption. A loading dose of 0.75 mg/kg for
amrinone or 50 mcg/kg for milrinone is necessary, followed by a titrated constant
infusion for either drug (5 to 10 mcg/kg/min for amrinone and 0.5 mcg/kg/min for
milrinone).
1175. c In the treatment of anaphylactic shock, epinephrine should be administered
Rosen 5th, intravenously in patients with hypotension, even in the presence of coronary artery
Chapter 4, disease. Initially 1 ml of 1:10,000 epinephrine (100 mcg) can be injected slowly and
p. 45 the response monitored. Afterward 5 mg of epinephrine can be diluted in 500 ml of
saline, with a starting infusion rate of 10 ml/hr (about 0.02 mcg/kg/min) and titrated to
maintain perfusion.
1176. b Empiric antibiotics for acute bacterial meningitis
Rosen 5th, 0 4 weeks: ampicillin 50-100 mg/kg IV plus gentamicin 2.5 mg/kg or cefotaxime 50
Table 168-3, mg/kg IV
p. 2350 4 weeks 3 months: ampicillin 50-100 mg/kg IV plus cefotaxime 50 mg/kg IV or
ceftriaxone 100 mg/kg IV or chloramphenicol 25 mg/kg IV
>3 months: cefotaxime 50 mg/kg IV or ceftriaxone 100 mg/kg IV and vancomycin 15
mg/kg IV.
1177. d Of the benzodiazepines, only midazolam is well suited to use as an induction agent,
Rosen 5th, with a recommended dose of 0.1 to 0.3 mg/kg IV. In a dose of 0.2 mg/kg IV,
Chapter 1, midazolam produces loss of consciousness in about 30 seconds and has a clinical
p. 13 duration of 15 to 20 minutes.
1178. c The dose of naloxone for adults is 0.1 to 2 mg IV; for complete reversal, at least 2 mg
th
Rosen 5 , should be given. If the procedure has not been completed, but the patient becomes
Chapter 183, overly sedated or experiences respiratory depression, partial reversal may be achieved
p. 2583 by using incremental doses of 0.1 to 0.4 mg.
1179. a Propofol is an ultra-short-acting sedative hypnotic that produces dose-dependent levels
th
Rosen 5 , of sedation ranging from light sedation to general anesthesia. Its short duration of
Chapter 183, action makes titration simple, but also requires the drug to be given via continuous
p. 2582 intravenous infusion. Sedation is usually achieved by administering a constant infusion
of 25 to 125 mcg/kg/min.
360

1180. An 8 year old male complains of severe pain and gross deformity of his elbow after falling from the
monkey bars at a local playground. He denies any other injury. His x-ray shows posterior elbow
dislocation. Your orthopedic surgeon will be occupied in the operating room for the next few
hours, so you wish to reduce this as soon as possible. You want to use an agent with rapid onset of
sedation with a short duration of unconsciousness, so you choose:
a. thiopental.
b. chloral hydrate.
c. methohexital.
d. secobarbital.
e. phenobarbital.

1181. Emergence phenomena are a common side effect of ketamine, occurring in up to 50% of adults and
10% of children. Risk factors include:
a. age less than 10 years.
b. male gender.
c. intramuscular administration.
d. simultaneous use of nitrous oxide.
e. a patient who normally has frequent dreams.

1182. The dose of ketamine to obtain the dissociative state is:


a. 1 to 2 mg/kg IM.
b. 4 to 5 mg/kg IM.
c. 1 mg/kg single IV dose.
d. 2 mg/kg single IV dose.
e. 0.05 to 0.1 mg/kg/min IV.

1183. A 6 year old male has an obviously deformed left forearm after falling off a bed while jumping with
a sibling. He is anxious and apprehensive, and the parents are adamant about not causing any more
pain with an intravenous line. The last time the child received ketamine, he developed severe
laryngospasm. An appropriate drug which can be administered rectally is:
a. nitrous oxide.
b. ketamine.
c. morphine sulfate.
d. methohexital.
e. lorazepam.

1184. The competitive neuromuscular blocking agent that neither releases histamine nor exhibits cardiac
muscarinic blockade is:
a. vecuronium.
b. pancuronium.
c. d-tubocurarine.
d. rocuronium.
e. metocurine.
361

1180. c Methohexital has a rapid onset of action, producing unconsciousness, sleeplike state in
th
Rosen 5 , less than 1 minute. It also has a brief duration of action, with most patients awake in
Chapter 183, less than 10 minutes. The elimination half-life is 3.9 hours. Rapid onset and short
p. 2581 duration of unconsciousness make methohexital an excellent agent for short procedures
such as cardioversion, bone or joint reduction, and CT scan in children. It should be
noted that although methohexital causes profound amnesia, it does not provide
significant analgesia. Consequently, some physicians coadminister small doses of an
opioid for extremely painful procedures.
1181. e Risk factors for emergence phenomena associated with ketamine use include age
Rosen 5th, greater than 10 years, female gender, intravenous administration (especially when
Chapter 183, given rapidly), prior personality disorders, and patients who normally have frequent
p. 2582 dreams.
1182. b The IV dose of ketamine to obtain the dissociative state is 1 to 2 mg/kg given at a rate
th
Rosen 5 , of 0.5 mg/kg/min. For short procedures, particularly in children, the IM route is simple
Chapter 183, and effective. A dose of 4 to 5 mg/kg IM is required to consistently achieve the
p. 2582 dissociative state; larger doses offer no additional benefit.
1183. d When IV access is not possible or not required, the use of rectal methohexital offers
th
Rosen 5 , high success rates, easy administration, and high patient and parent acceptance for
Chapter 183, children requiring sedation for procedures.
p. 2585-2586
1184. a Vecuronium neither releases histamine nor exhibits cardiac muscarinic blockade, so it
th
Rosen 5 , has become popular despite its higher cost and the inconvenience of reconstituting it
Chapter 1, from powder with each use.
p. 12
362

1185. A 27 year male lost control of his motorcycle and collided with an 18-wheeler at approximately
60 mph. He has a severe open head injury and significant blunt chest and abdominal trauma. The
paramedics were unable to establish an airway. The patient is posturing with severe teeth
clenching. His respirations are shallow at a rate of 8 / minute, his blood pressure is 60/p and his
heart rate is 170 / minute. Your most appropriate induction agent is:
a. thiopental.
b. etomidate.
c. lorazepam.
d. ketamine.
e. methohexital.

1186. A 37 year old male with a long-standing history of intravenous drug use is brought by a friend; the
patient is unconscious, diaphoretic, and has shallow respirations at 6 / minute. The friend says that
the patient had been hoarding his methadone and took an 8-day supply about 30 minutes ago. An
appropriate antidote and dosage would be:
a. nalmefene 0.5 1.5 mg IV.
b. naloxone 0.2 mg IV.
c. flumazenil 0.5 mg/kg IV.
d. pentazocin 25 mg IV.
e. clonidine patch.

1187. A synthetic opioid which is highly lipid soluble and easily absorbed through mucous membranes is:
a. fentanyl.
b. hydrocodone.
c. hydromorphone.
d. meperidine.
e. morphine.

1188. 72 year old female inadvertently took several doses of her digoxin and complains of palpitations;
her ECG shows 2:1 atrial flutter. An antidysrhythmic though to be safe in this setting is:
a. bretylium.
b. isoproterenol.
c. phenytoin.
d. procainamide.
e. quinidine.

1189. In patients who have suffered from Clostridium difficile colitis, a novel therapy which has been
shown to decrease recurrence is:
a. amoxicillin 500 mg tid for 14 days.
b. clarithromycin 500 mg bid for 7 days.
c. minocycline 250 mg tid for 10 days.
d. nitazoxanide 100 mg daily for 5 days.
e. saccharomyces boulardii 500 mg bid for 4 weeks.

1190. The local anesthetic you should avoid when treating a patient with coronary artery disease is:
a. benzocaine.
b. tetracaine.
c. lidocaine.
d. cocaine.
e. chirocaine.
363

1185. b Because etomidate is able to decrease intracerebral pressure, cerebral blood flow, and
th
Rosen 5 , cerebral metabolic rate without adversely affecting systemic mean arterial blood
Chapter 1, pressure and thus cerebral perfusion pressure, it is an excellent induction agent for
p. 13 patients with elevated intracerebral pressure, even with hemodynamic instability. The
induction dose of etomidate is 0.3 mg/kg IV.
1186. a In contrast to naloxone, nalmefene, another opioid antagonist, has a long half-life (8 to
Rosen 5th, 11 hours) and duration of clinical effect. The initial intravenous dose is 0.5 to 1.5 mg.
Chapter 156, Once a clinical response has been achieved, repeated doses or continuous infusions are
p. 2185 generally not required.
1187. a In children, oral transmucosal fentanyl citrate may be used when IV access is not
th
Rosen 5 , feasible. A fentanyl-impregnated, sweetened matrix in lozenge form on a holder
Chapter 182, (fentanyl lollipop) allows transmucosal delivery and a rapid onset of action because
p. 2563 direct absorption into the systemic circulation avoids first-pass metabolism by the
liver. It has been shown to produce sedation and relieve pain 12 to 30 minutes after
administration.
1188. c Phenytoin and lidocaine are believed to be the safest of the antidysrhythmic drugs for
Rosen 5th, controlling tachydysrhythmias in the setting of digitalis intoxication. Most other
Chapter 146, cardiac drugs, such as isoproterenol, procainamide, and bretylium, may worsen
p. 2106 dysrhythmias or depress AV conduction.
1189. e Adding the yeast Saccharomyces boulardii, 500 mg PO bid for 4 weeks, to antibiotic
Rosen 5th, treatment dramatically decreases the number of recurrences of C. difficile-associated
Chapter 89, disease in patients with previous episodes. However, no benefit results when S.
p. 1313 boulardii is given to patients with an initial episode. No serious adverse reactions have
occurred with the use of S. boulardii.
1190. d Cocaine should not be used in patients with known coronary artery disease because it
Rosen 5th, may cause coronary artery vasoconstriction. Cocaine is also a component in TAC
Chapter 182, (tetracaine, epinephrine, and cocaine).
p. 2573
364

1191. A 10 year old boy fell while riding his skateboard and chewing on a Popsicle stick. He sustained a
complex intraoral laceration. You repair the wound and decide to start him on:
a. azithromycin.
b. clindamycin.
c. erythromycin.
d. penicillin.
e. trimethorpim-sulfamethoxazole.

1192. A 42 year old man fell while playing basketball and appears to have a fracture-dislocation of his
elbow. You wish to perform regional anesthesia, but cannot find any lidocaine without
epinephrine. A safe alternative would be:
a. benzocaine.
b. cocaine.
c. dubicaine.
d. prilocaine.
e. tetracaine.

1193. The best method to achieve effective dosing for relief of moderate to severe pain is:
a. intramuscular bolus followed by intravenous maintenance.
b. intravenous bolus followed by oral maintenance.
c. repeat intravenous bolus.
d. repeat oral administration.
e. slow intravenous titration.

1194. An 87 year old man complains of severe weakness and dizziness. He is bradycardic and
hypotensive. The medicine most likely causing his symptoms is:
a. atenolol.
b. clopidogrel.
c. HCTZ.
d. isosorbide.
e. metformin.

1195. A 27 year woman complains of severe diarrhea. Yesterday she eloped from another local hospital
where she had been treated for diarrhea and dehydration. You call the hospitals lab and determine
that stool cultures were positive for salmonella. Your therapy should start with:
a. ciprofloxacin or trimethoprim/sulfamethoxazole.
b. ivermectin.
c. metronidazole.
d. tetracycline or doxycycline.
e. vancomycin.

1196. A first line agent for both streptococcal and staphylococcal toxic shock syndrome is:
a. clindamycin.
b. ticarcillin.
c. doxycycline.
d. erythromycin.
e. amoxicillin.
365

1191. d Lacerations of the oral mucosa involve bacteria-rich oral secretions and may become
th
Rosen 5 , infected slightly more than other wounds. Although little data suggest a clear
Chapter 52, indication for prophylactic antibiotics, one study showed that patients benefit from
p. 748 antibiotics if they are compliant with their regimen. Rates of infection for complex
through-and-through lacerations may be twice those for simple mucosal lacerations. It
may be reasonable to limit antibiotic use to complex lacerations. Penicillin is an
appropriate choice of antibiotic.
1192. a The intravenous regional anesthesia procedure, also called a Bier block, is an effective
Rosen 5th, and rapid technique to anesthetize extremities for fracture reduction or repair of
Chapter 182, extensive wounds. The method involves the IV injection of a local anesthetic agent
p. 2574 into a previously exsanguinated limb. This procedure has been adapted for use in the
ED in the form of a minidose of 100 mg of lidocaine. A safe alternative is to use the
relatively nontoxic local anesthetic agent, prilocaine.
1193. e The proper way to achieve the effective dosage for moderate to severe pain is to use a
Rosen 5th, slow IV titration. Some patients require greater than 25 mg of morphine for relief,
Chapter 182, whereas others become comfortable at substantially lower doses.
p. 2561
1194. a The most common initial sign of beta-blocker ingestion remains bradycardia, which
th
Rosen 5 , should draw attention to the possibility of cardiac drug overdose. Hypotension and
Chapter 146, unconsciousness are the second and third most common signs, respectively.
p. 2109
1195. a The following antibiotics are the drugs of choice in the pathogens listed below:
th
Rosen 5 , Salmonella and Shigella: ciprofloxacin, TMP/SMX
Table 89-2, Vibrio parahaemolyticus: tetracycline or doxycycline
p. 1303 Clostridium difficile: metronidazole, vancomycin
Giardia lamblia: metronidazole, furazolidone
Strongyloides stercoralis: ivermectin
1196. a For both staphylococcal and streptococcal toxic shock syndrome, most clinicians
Rosen 5th, recommend clindamycin as a first-line agent. Clindamycin is a potent suppressor of
Chapter 123, bacterial toxin synthesis; it also facilitates phagocytosis streptococci and has a longer
p. 1809 postantibiotic effect than the beta-lactams.
366

1197. A 29 year old woman who is seven months pregnant went camping about 3 weeks ago. Shortly
after returning she developed generalized weakness, lethargy, and a temperature of 40oC which
lasted about 3 days. Her family doctor and obstetrician both felt she had a viral illness. Now she
again feels weak and feverish, but not as bad as the first time. She should be treated with:
a. oral tetracycline 500 mg qid for 5 to 10 days.
b. intravenous ticarcillin 1 mg intravenous.
c. oral doxycycline 100 mg bid for 7 days.
d. oral erythromycin 500 mg qid for 5 to 10 days.
e. oral ampicillin 1 gram daily for 10 days.

1198. In patients with tick-induced relapsing fever, as many as one third of patients may experience a
Jarisch-Herxheimer-type reaction characterized by an increased temperature, severe rigors, a drop
in leukocyte and platelet counts and hypotension. Suggested treatment for this reaction includes
intravenous volume expansion and:
a. dopamine.
b. vitamin B6.
c. diphenhydramine.
d. naloxone.
e. meptazinol.

1199. The drug of choice to treat Lyme disease in men, nonpregnant and nonlactating women, and
children older than 8 years of age is:
a. amoxicillin.
b. clindamycin.
c. doxycycline.
d. erythromycin.
e. ticarcillin.

1200. The drug of choice to treat Lyme disease in pregnant women, and children younger than 8 years of
age is:
a. amoxicillin.
b. clindamycin.
c. doxycycline.
d. erythromycin.
e. ticarcillin.

1201. A 3 year old girl presents with frequent paroxysms of coughing. The mother says the child coughs
at least 50 times a day. She is noted to cough repeatedly in short exhalations, and after 10 to 15
coughs, he inhales forcibly. She is nontoxic appearing and has a low-grade temperature. With
strict isolation for 7 days and erythromycin at 50 mg/kg/day in four divided doses she can be
managed on an outpatient basis, but close contacts should be treated with:
a. amoxicillin.
b. ciprofloxacin.
c. doxycycline.
d. erythromycin.
e. rifampin.
367

1197. d Relapsing fever is treated effectively with tetracycline or erythromycin. Tetracycline


th
Rosen 5 , should be avoided in children younger than 8 years old and in pregnant women.
Chapter 128,
p. 1890
1198. c As many as one third of patients may experience a Jarisch-Herxheimer-type reaction
th
Rosen 5 , after treatment with antibiotics. The reaction may be severe, especially with louse-
Chapter 128, borne relapsing fever. This phenomenon may be related to administration of cytokine
p. 1890 intermediaries or endogenous opioids. Approximately 4 hours after antibiotic
treatment and coinciding with the clearance of spirochetes from the blood, the patient
usually experiences an increase in temperature, severe rigors, a drop in leukocyte and
platelet counts, and hypotension. Anticipation of the reaction is crucial because IV
volume expansion with saline solution may be required to maintain the blood pressure;
the reaction may be more threatening than the disease itself. Meptazinol, an opioid
antagonist with agonist properties, has been proposed for use in treatment of this
reaction.
1199. c Prompt antibiotic therapy is essential in early Lyme disease because it generally
Rosen 5th, shortens the duration of the rash and associated symptoms and, more important,
Chapter 128, prevents later illness in most patients. Some patients with severe early disease,
p. 1887 however, develop later stages despite courses of antibiotics. The drug of choice for
men, nonpregnant and nonlactating women and children older than 8 years of age is
1200. a doxycycline. Pregnant or lactating women and children less than 8 years old should
Rosen 5th, receive amoxicillin. For patients who cannot tolerate tetracyclines and are allergic to
Chapter 128, penicillins, erythromycin is recommended but is probably less effective.
p. 1887
1201. d Pertussis patients without significant complications may be managed as outpatients and
th
Rosen 5 , advised of the potential complications. The patient should be strictly isolated for 7
Chapter 123, days and erythromycin prescribed to limit the spread of infection. Erythromycin
p. 1790 should also be prescribed for any unimmunized person or partially immunized infant
with a history of significant exposure to the index case. Adults who will come in close
contact with susceptible children should also take erythromycin prophylaxis.

S-ar putea să vă placă și